Activados-Matemática-5

May 25, 2018 | Author: Jose Adan Duarte Urbina | Category: Exponentiation, Equations, Sequence, Publishing, Mathematics
Share Embed Donate


Short Description

Descripción: ejercicios matematicos...

Description

MATEMÁTICA

Gerente editorial Daniel Arroyo Jefa de contenidos editoriales Verónica Lombardo Jefe del área de Matemática Gabriel H. Lagoa Editing Belén Boscaroli Autores Aperturas: Pablo Amster ¿Para qué sirve?: Laura Pezzatti

Coordinadora del área de Marcas y derechos Amorina Scalercio Jefe del departamento de Arte y diseño Lucas Frontera Schällibaum Diseñadoras de maqueta Patricia Cabezas Laura Porta Diagramación Olifant – Florencia Galeano & Valeria Miguel Villar – Ilustrador Pablo Zerda

Roxana Abálsamo Adriana Berio Silvana Mastucci Nora Quirós Fernando De Rossi

Fotografías Archivo de imágenes de Grupo Macmillan Thinkstock Wikimedia commons

Corrector de estilo Gabriel Valeiras

Gerente de Prerensa y Producción Editorial Carlos Rodríguez

Matemática 5 ¿para qué sirve? / Roxana Abálsamo ... [et.al.]. - 1a ed. -Boulogne: Puerto de Palos, 2013. 256 p. : il. ; 28x20 cm. - (Activados ) ISBN 978-987-547-590-8 1. Matemática. 2. Enseñanza Secundaria. I. Abálsamo, Roxana CDD 510.712

© Editorial Puerto de Palos S.A., 2013. Editorial Puerto de Palos S.A. forma parte del Grupo Macmillan. Av. Blanco Encalada 104, San Isidro, provincia de Buenos Aires, Argentina. Internet: www.puertodepalos.com.ar Queda hecho el depósito que dispone la Ley 11.723. Impreso en Argentina. Printed in Argentina. ISBN 978-987-547-590-8 La presente obra se ha elaborado teniendo en cuenta los aportes surgidos de los encuentros organizados por el “Instituto Nacional contra la Discriminación, la Xenofobia y el Racismo” (INADI) con los editores de texto. No se permite la reproducción parcial o total, el almacenamiento, el alquiler, la transmisión o la transformación de este libro, en cualquier forma o por cualquier medio, sea electrónico o mecánico, mediante fotocopias, digitalización y otros métodos, sin el permiso previo del editor. Su infracción está penada por las leyes 11.723 y 25.446. Primera edición. Esta obra se terminó de imprimir en noviembre de 2013, en los talleres de F P Compañía Impresora, Beruti 1560, Florida, provincia de Buenos Aires, Argentina.

MATEMÁTICA

MATEMÁTICA

Es una nueva propuesta que facilita el aprendizaje de la matemática a través de 691 actividades que favorecen la comprensión de los distintos temas. En formato binarizado, la sección ¿Para qué sirve? conecta la matemática con la vida cotidiana a través de una pregunta que surge constantemente en el aula.

capítulo

LOS CAPÍTULOS INCLUYEN LAS SIGUIENTES SECCIONES Y PLAQUETAS:

2

Sucesiones Contenidos 8. Sucesiones. 9. Sucesiones aritméticas. 10. Sucesiones geométricas. 11. Análisis de sucesiones. 12. Clasificación de sucesiones.

Apertura: en esta sección, Pablo

una supuesta carrera con un El filósofo griego Zenón de Elea imaginó resultado muy curioso. por un evento deportivo fuera de “El mundo antiguo se ve alborotado el más veloz de los hombres, y una lo común: juegan una carrera Aquiles, héroe concede al animalito una ventortuga. Nobleza obliga: el glorioso ante la sorpresa de todos, y, correr... a taja inicial antes de comenzar la Aquiles llega al punto del que parte nunca lo alcanza. Es que cuando se ha movido un poco; nuevamente tortuga, ella, lenta pero persistente, más, la tortuga se ha movido un poco Aquiles intenta llegar a ella, pero

Amster, especialista en el área de la matemática, ofrece textos relacionados con la historia y evolución del pensamiento matemático.

En el cuadro de contenidos aparecen los temas numerados para su fácil identificación.

y así sucesivamente.” deportivas, sino más bien en refuZenón no estaba interesado en lides época. No se trata de negar el hecho tar el pensamiento filosófico de la alcanza a la tortuga; lo que está “evidente” de que Aquiles efectivamente infinitamente el espacio y el tiempo. en juego es la posibilidad de dividir pero motivaron siglos de una discuLos argumentos parecen sencillos, con la aparición del moderno consión que recién empezó a resolverse cepto de límite.

1. Lean atentamente y respondan.

más rápido Borges, Aquiles corre diez veces a. En un texto del escritor Jorge Luis alcanzarla? atrás de ella. ¿Cuánto recorre para que la tortuga y parte diez metros ¿No es se discutió durante muchos siglos? b. ¿Por qué creen que este problema tortuga? evidente que Aquiles alcanza a la

Conector: invita a repasar 39

40

41

42

43

44

45

46

47

48

conceptos explicados en páginas anteriores.

49

Ecuaciones exponenciales

InfoActiva: presenta

INFOACTIVA Una ecuación exponencial es aquella ecuación en la que la incógnita aparece en el exponente. Para resolver una ecuación exponencial vamos a tener en cuenta: 1. ax ⇒ a > 0 ∧ a ≠ 1 En la página 20 2. ax = ax ⇒ x = x pueden repasar 1 2 las propiedades de 3. Las propiedades de las potencias. la potenciación.

definiciones, clasificaciones, procedimientos básicos y ejemplos de cada contenido que facilitan la comprensión.

1

2

Resuelvan las siguientes ecuaciones

exponenciales.

a. 32x+1 = 81 32x+1 = 34 ⇒ 2x + 1 = 4 ⇒ x = __ 3 2 b.

2x+1

x+2 _____

____

__

32x+2 = 38 3 __

x+2 3⇒ 22x+1 = 22 ⇒ _____ 1 = __ x = __ 2x+1

2

4

7

c. 4x–2 + 4x + 4x+1 = 324 4x + x ___ 1 4 + 4x . 4 = 324 ⇒ 4x . ___ 42 16 + 1 + 4 81 x 3 4x . ___ 16 = 324 ⇒ 4 = 4 ⇒ x = 3

(

) = 324

d. Sn = 1 + 3 + 9 + 27 + … + 3x = 3 280 Se utiliza la fórmula de la suma de n términos x 3 .3–1 ________ 3–1 =

8

. r – a1 n de una sucesión: S = a________ n r–1

17

¿Para qué sirve? PÁGINA 3

del otro. ordenado de números, uno a continuación Una sucesión es un conjunto orden elementos. naturale es una sucesión de infinitos El conjunto de los números naturales d los elementos de la sucesión. no a cada uno de Se denomina término

⇒ 3x = 2 187 ⇒ 3x = 37 ⇒ x = 7

e. 32x+1 – 2 . 3x – 1 = 0 3 . 32x – 2 . 3x – 1 = 0 Se usa una variable t = 3x ⇒ 32x = (3x 2 2 ) =t

{

16

15

14

13

12

11

1100

INFOACTIVA

3 280

3x . 3 – 1 = 6 560 ⇒ 3x . 3 = 6 561

9

Sucesiones

64;

125;

216;

1;

8;

27;













a1

a2

a3

a4

a5

a6

encontrar un término general ones se puede enc En algunas sucesiones del lugar que ocupa. iera en función de de un término cualquiera

216;…, En la sucesión 1; 8; 27; 64; 125; 2

x 3t2 – 2t – 1 = 0 t1 = 1 ⇒ 3 = 1 ⇒ x1 = 0



an

...

an (término enésimo), que es la fórmula

= n3. el término general de la sucesión es an

misma, reemsucesión, o cualquier término de la érmino general, se puede hallar la Si se conoce el término general. naturales en el valor n del término

1 x 1 __ t2 = – __ 3 ⇒ 3 = – 3 ⇒ x2 no es solución.

nú números Cuando la incógnita está en el exponente, plazando en forma consecutiva los se puede despejar aplicando en cada 1 ;... 1 ;... ; __ 1 ; __ 1 ; __ 1 ; __ miembro el logaritmo 1 ; __ cuya base es la base de la potencia. 1 n será: 1; __ __ 2 3 4 5 6 sucesión es an = n , entonces la sucesión Si el término general de una suce ax = b loga ax = log b ⇒ x . log a = log natural un número real. f: → b ⇒ x = log b a función que le asigna a todo número a a Por lo tanto una sucesión es una fu a

Hallen el valor de x. 10x–2 = 8 log 10x–2 = log 8 ⇒ (x – 2) . log 10 =

ticas méétic Sucesiones aritméticas

log 8 ⇒ x = log 8 + 2 ⇒ x = 2,903

obtiene sumando cual cada término de la misma se ucesión aritmética a aquella en la Se denomina sucesión razón aritmética. llam mero constante r llamado al anterior un número

160

4+8

2+8 12

36 ...

28

20

12

4

20 + 8

Sucesión aritmética con r = 8.

28 + 8

debe verificarse a ucesión sea aritmética, suc una sucesión ue una que ra que Para

que: a2 – a1 = a3 – a2 = … = an – an–1 = r

eométricas eométric geomé geométricas i nes g S Sucesiones

término de la misma se obtiene multigeométrica a aquella en la cual cada Se denomina sucesión geométri constante q llamado razón geométrica. plicando el anterior por un número

–9

3 3 . (–3)

27

–81

243 ...

34

Sucesión geométrica con q = –3.

–9 . (–3) 27 . (–3) 81 . (–3)

Para que una sucesión sea geométrica, debe

a3 a2 __ __

an ___

verificarse que: a1 = a2 = … = an–1 = q ⇔ a1

Conexión con

¿Para qué sirve?

n3

...

≠0

de comprensión

Test

1. Respondan y expliquen las respuestas.

a = 12 y a3 = 0, ¿cuál es la razón? a. En una sucesión aritmética, si 1 de x e y de una sucesión aritmética en función b. ¿Cómo se puede expresar la razón con a = x y a8 = y?

Test de comprensión: incluye preguntas básicas que permiten evaluar la comprensión de la teoría y revisar errores comunes.

7

Sucesiones aritméticas 9 ACTIVIDADES

ACTIVIDADES 12 Clasificación de sucesiones

caso. 5. Completen con el dato que falta en cada

83 ___ c. an = 153,32; a1 = 25; r = 25 24. Tengan en cuenta la siguiente sucesión definida por recurrencia y resuelvan. a =3 an = a1 = 2an–1 + 1 para n ≠ 1 n a. Hallen los 10 primeros términos de esta sucesión.

b. a120 = 1 345; r = –9

12 ___ a. a1 = – 5 ; r = 5

{

menteACTIVA: propone situaciones problemáticas con un mayor nivel de complejidad.

b. ¿Es una sucesión monótona creciente? ¿Por qué? n=

a1 =

a12 =

c. Calculen la suma de los términos calculados.

términos. 6. Calculen la suma de los 30 primeros

ral es: an = –12 + 5n. término general a. Dada la sucesión aritmética cuyo

25. Tengan en cuenta los primeros siete términos de esta sucesión y hallen una fórmula definida por recurrencia. –23, … b. Dada la sucesión: –4, –10, –16,

3; 16; 82; 412; 2 062; 10 312; 51 562;…

–6. aritmética a1 + a3 = 18 y a5 – a2 = sión ar una sucesión 26. Resuelvan. 7. Resuelvan teniendo en cuenta que en Fibonacci, matemático italiano del siglo XIII, descubrió una sucesión que tiene numerosas aplicaciones en biología, en ciencias de la computación, en matemática y en la teoría de juegos. La sucesión es: 1; 1; 2; 3; 5; 8; 13; 21; 34; 55; 89; 144; 233; 377; 610; 987; 1 597; 2 584; 4 181; 6 765;… a. Hallen una fórmula por recurrencia para la sucesión de Fibonacci.

a. ¿Cuál es la razón?

b. Calculen a1, a2 y a3.

Actividades: para cada tema

b. Construyan una nueva sucesión donde cada término esté formado por cocientes de términos consecutivos de la sucesión de Fibonacci.

8. Tengan en cuenta los datos y resuelvan. a = x; r = x – 3 con x D 1

a S10? a. ¿Cuál es la expresión correspondiente 9x – 135 (9x + 27) . 5

c. La sucesión anterior ¿es convergente 55x – 135 o divergente? Si es convergente, indiquen a qué número se acercan sus términos. es 13. co os consecutivos dos términos b. Calculen S10 si la diferencia entre

se proponen distintas actividades que están organizadas de manera secuencial.

27. Tengan en cuenta que el término general de una sucesión aritmética es a = 2 + 3n y resuelvan. Escriban los 5 primeros términos n $1 000 y cada y hallen una fórmula de la misma sucesión, pero por recurrencia. notebook. Si empezó reservando comprarse una n Franco decidió ahorrar dinero para d un año? és de tendrá después mes agrega $260, ¿cuánto dinero

9. Lean atentamente y resuelvan.

37

mente ACTIVA Una ciclista tarda 40 segundos en dar la primera vuelta a una pista; por los efectos del cansancio, en cada vuelta tarda 6 segundos más que en la anterior. a. Calculen los seis primeros términos de la sucesión que representa los segundos que tarda por cada vuelta. b. Definan la sucesión por recurrencia. ¿Qué tipo de sucesión es?

45

capítulo

2

CONTENIDOS

8*9*10*11*12

INTEGRACIÓN 28. Escriban el término general de las sucesiones.

37. Clasifiquen las sucesiones en convergentes, 33. Resuelvan.

y hallen Indiquen si son aritméticas o geométricas la razón. 0; 5; __12; 21;…__ a. –3; __ __ b. 33 ; 2 . 33 ; 3 . 33 ; 4 . 33 ;... c. 0,2; 0,02; 0,002; 0,0002... d. –2; 3; 8; 13;… e. 2; 6; 18; 54; 162; 486;...

29. Las siguientes sucesiones no son aritméticas pedidos. ni geométricas. Hallen los términos a. 1; 2; 3; 9; 8; 7; 1; 2; 3; 9; 8; 7;... a14 =

;

a120 =

en sucea. Los ángulos de un triángulo están Calculen sión aritmética de razón 30 grados. cada ángulo. están en b. Los ángulos de un cuadrilátero 4 veces el sucesión geométrica y el último es segundo. Calculen dichos ángulos.

si son divergentes u oscilantes. Luego, indiquen y si son monótonas crecientes o decrecientes acotadas inferior o superiormente. –n 4 _____ a. an = n + 1

n+1 c. cn = 3 . (–1)

;

u oscilantes. siones en convergentes, divergentes a. y

20? ¿Por a. ¿Puede tener un término que valga qué? términos b. ¿Para cuál valor de n uno de los vale 2,4?

31. Resuelvan.

genea. Una sucesión aritmética tiene término a y el ral an = –5 + 4n. Calculen la razón, 395 orden del término de valor 2 011. = 4 y a2 = 2. b. Una sucesión geométrica tiene a1 y a10. Calculen la razón, el término general a = 3, q = 2 c. Una sucesión geométrica tiene 1 k. y Sk = 1 533. Calculen el valor de q = –0,5 y d. Una sucesión geométrica tiene 3 a = __. Calculen S15. 4

de 32. Los números 2 y 20 son los extremos

Hallen una sucesión aritmética de siete términos. los números que completan la sucesión.

b.

crecientes o decrecientes. a. 1,2; 1,23; 1,234; 1,2345; 1,23456... b. 0; 3; 5; 0; 3; 5; 0;… c. 6,7; 6,07; 6,007; 6,0007;…

4

3

2

1

x

y 3

y 36. Tengan en cuenta la siguiente sucesión

2

resuelvan.

5; 5; 4; 4; 3; 3; 2; 2;... a. ¿Es una sucesión monótona decreciente? b. ¿Está acotada superior o inferiormente? superiores y c. Hallen, si es posible, tres cotas tres cotas inferiores. oscilante? d. ¿Es convergente, divergente u

1 0

1

2

3

4

5

x

-1

270°

90°

–cos 30°

cos 60°

cos 210°

c. sen ^ _= sen (2/+ ^ _) d. cos ^ _= cos (2/+ ^ _ 

52. ¿En qué intervalo tiene exactamente tres soluciones la siguiente ecuación? 2 . sen2 ^ _+ 2 . sen ^ _= 0

a. [0;/]

[

3 b. 0;__ 2/

]

c. [0;2/]

d. [–/;/]

53. ¿Cuáles son las soluciones de la siguiente ecuación en el intervalo [0;/]? 2 . cos 3^ _ = 1. 5 a. __ 9 /

si n * 2

/ b. __ 9

5 c. – __ 9 /

2 d. __ 9 /

54. Lean atentamente y resuelvan.

y 42. Tengan en cuenta la actividad anterior marquen las opciones correctas. de la sucesión a. ¿Cuáles son las características de término general an? Es geométrica. Es aritmética. Tiene razón 12. Tiene razón 5. Es monótona creciente. Es monótona decreciente. – 7. Se la puede definir con an = 12n 7. Se la puede definir con an = 5n + de la sucesión b. ¿Cuáles son las características de término general bn? Es convergente a 0.

y 2 0

c.

cos 30°

a. sen ^ _ = –sen (2/ – ^ _) b. cos ^ _= cos (2/ – ^ _)

b1 = –2 bn–1 bn = ___ 2n

360°

51. ¿Cuáles son las igualdades correctas?

{

{

a las dadas?

b. cos 150°

__ una sucesión es an = 4 – n y resuelvan. y represena. Escriban los 4 primeros términos ten en un sistema cartesiano. b. ¿Es monótona creciente o decreciente? c. ¿Está acotada superior o inferiormente? y/o supremo d. Si es posible, hallen el ínfimo de la sucesión. e. Clasifiquen la sucesión en convergente,

b. bn =

-2 -4 -6 -8 -10 -12 -14 -16

35. Indiquen si estas sucesiones son monótonas

180°

tes sucesiones.5 a1 = a. an = a = an–1 + 12 si n * 2 n

-1

de la a. Calculen los primeros cuatro términos de cada sucesión formada por el perímetro cuadrado que se va formando. b. La sucesión ¿es aritmética o geométrica? sucesión. c. Escriban el término general de la decreciente? o creciente monótona d. ¿Es

a. /

siguien41. Hallen los términos 2, 5 y 8 de las x

5

4

3

2

1

0 -0,5

– 16 y respondan. 4n _______ general an = n

8

Marquen las opciones correctas 50. ¿Cuáles son las expresiones equivalentes

divergente u oscilante.

1 0,5

b50 =

30. Tengan en cuenta la sucesión de término

2

1 __ par n si n es 1 si n es impar ___ 2n

las suce38. Observen los gráficos y clasifiquen

b. 1; 2; 3; 4; 5; 11; 12; 13;… b21 =

{

d. dn = los puntos En un cuadrado de lado 1 se unieron otro cuadramedios de sus lados determinando en do en su interior. Se repitió el procedimiento como el segundo cuadrado y en los sucesivos se ve en la figura.

1 __ negatia. La sucesión an = n2 ¿tiene términos vos? ¿Es convergente? a infinito b. Escriban una sucesión divergente negativo y una oscilante.

de 40. Tengan en cuenta que2 el término general

3 –n n ______ b. bn = 2n

34. Lean atentamente y resuelvan.

capítulo

AUTOEVALUACIÓN

39. Resuelvan.

Una persona observa la terraza de un edificio con un ángulo de elevación de 55° y la de otro edificio con un ángulo de elevación de 72°. Se encuentra a 80 m del primer edificio y a 60 m del segundo. ¿Qué altura tiene cada edificio, sabiendo que la persona observa desde una altura de 1,70 m?

edificio I

55°

72°

80 m

60 m

edificio II

a. 114,25 m y 184,66 m

c. 115,95 m y 186,36 m

b. 114,25 m y 186,36 m

d. 115,95 m y 184,66 m

55. ¿Qué fórmula conviene utilizar para resolver el siguiente triángulo oblicuángulo? ___ __ c

ab ac ______ a. ______ ^ = sen c

20°

Es divergente. No tiene supremo. Tiene supremo. Está acotada. No está acotada.

___ 10 cm

a

8 cm

^ sen b __

ab bc ______ b. ______ ^ = sen c

sen ^ a

__

___

__

___

__

__

___

__ c. ac 2 = ab 2 + bc 2 – 2 . ab . bc . cos ^ b __

__ d. ab 2 = ac 2 + bc 2 – 2 . ac . bc . cos ^ c

b

47 196 46

Integración: incluye más actividades para resolver en el cuaderno.

Autoevaluación: propone más actividades para que cada alumno pueda evaluar los conocimientos adquiridos durante el capítulo.

¿Para qué sirve? ¿Para qué sirve?: en esta sección, Laura Pezzatti, especialista en el área de la matemática, ofrece una serie de textos que conectan los contenidos de los capítulos con la vida cotidiana y otras disciplinas con el objetivo de responder a la pregunta inicial que se plantea.

...

ve? sirve quéésir raqu Para ¿¿Pa capítulo 7 contenido

39

icas Funciones exponenciales y logarítm

describir utilizando fenómenos. Muchos de estos se pueden Las funciones nos permiten modelizar Veamos algunos ejemplos. funciones exponenciales y logarítmicas. elementos inestables capaces de físico que ocurre en el núcleo de La radiactividad es un fenómeno produciendo radiación. Este núcleos de elementos más estables transformarse espontáneamente en y en aplicaciones de energía nuclear, se usa en medicina fenómeno se aprovecha para la obtención que no tienen un balan-industriales. En general, las sustancias son radiactivas. ce correcto entre protones y neutrones se puede modelizar Resulta que la desintegración radiactiva –h.t n(t) es la cantidad de con la función n(t) = n0 . e donde tiempo t, n0 es la cantidad radionucleidos en un instante de es la llamada constante de existente en el instante t = 0 y h de cada elemento. Codesintegración radiactiva y dependerá y predecir el compornociendo esta función se puede analizar sustancias, así como su tamiento radiactivo de las diferentes velocidad de desintegración. logarítmicas. Un ejemplo sentidos tienen que ver con las funciones Por otro lado, varios de nuestros magnitud de un que establece la relación entre la conocido, es el llamado ley de Weber-Fechner, propusieron que la relación lo percibimos (P). Weber y Fechner estímulo físico (S) y la forma en que por debajo suyo no se perciS __ S es el nivel de estímulo en el que está dada por P = k . ln S0 donde 0 caso. cada de dependerá que be sensación y k es una constante

( )

experimentado es cuando el Una que seguramente todos hemos Esta relación nos dice varias cosas. pesa 100 o 102 kilos, es difícil que podamos percibir si algo estímulo físico es el peso. Por ejemplo, pesa 100 y algo que percibir algún cambio entre algo que sin embargo es probable que podamos percibir una nos dice que probablemente no podamos pesa 110 kilos. Pero esta misma relación justamente la relación no es y algo que pesa 1 010 kilos, pues diferencia entre algo que pesa 1 000 en la magnitud del exponenciales cambios los relación es que lineal. Lo que si nos garantiza esta estímulo los podremos percibir linealmente.

Actividades exponenciales. pueden modelizar utilizando funciones 1. Investiguen qué otros fenómenos se logarítmicas. pueden modelizar utilizando funciones 2. Investiguen qué otros fenómenos se la siguiente experiencia. 3. Reúnanse con un compañero y realicen alguna cosa con determinado peso;

capítulo 8 contenido

47

Trigonometría

de un triángulo rectángulo que establece una relación entre los lados Así como el teorema de Pitágoras medida de los otros dos, los tercer lado una vez que conozco la nos permite conocer la medida del ángulo o un lado conociendo permiten conocer la medida de un teoremas del seno y del coseno nos otros datos. de Pitágoras para cualquier que es una generalización del teorema Por ejemplo, el teorema del coseno, ^ A, B y C son las medidas C2 = A2 + B2 – 2 . A . B . cos a donde triángulo, establece la siguiente relación: medidas A y B. Esta relación ^ el ángulo formado por los lados de de los lados de un triángulo y a es de un triángulo, encontrar la medida de dos lados y un ángulo nos permite, por ejemplo, si conocemos una relación entre establece parte, otra del teorema del seno, por la medida del tercer lado. El caso la conocemos si ejemplo, por ángulos y lados que nos permite, de un triángulo, encontrar la medida de dos ángulos y un lado medida de los otros dos lados. sirve para calcular distancias. Como vemos, la trigonometría nos sentido es el que le permitió Uno de los principales logros en este los diferentes países. Uno de a los cartógrafos realizar mapas de sentido fue el denominado los proyectos más conocidos en este por el siglo XIX y que, entre allá Trigonométrica Gran Planimetría permitió descubrir el punto otras cosas, cuenta la leyenda que ese momento denominado más alto sobre la Tierra en 1852, en Everest, como hoy lo conocePico XV y luego, en 1856, llamado de este proyecto. mos, en honor a uno de los inspectores en áreas como la astronomía, la cartografía La trigonometría, además de ser utilizada otras “medir cosas”, también se aplica en y la arquitectura principalmente para y la biología para estudiar disciplinas como la economía, la meteorología y periódicos. También los oceanógrafos objetos que tienen comportamientos los músicos hacen uso de ella.

Actividades el teorema 1. ¿Cómo les parece que podemos deducir

del coseno? de Pitágoras utilizando el teorema calcular medidas 2. ¿Podríamos utilizar estos teoremas para de lados de otros polígonos? ¿Cómo?

vendados y sostener -Uno de los dos debe tener los ojos ver, debe avisar cuancosas de igual peso. El que no puede el otro irá agregándole, de a una, y se anota el peso de inicio y el final. do percibe un cambio en el peso agregando cosas que pesen algo que pese unos 100 g y se van -Por ejemplo, pueden empezar con peso. ojos vendados perciba un cambio de unos 10 g hasta que el que tiene los de distintos pesos. cosas con empezando -Repitan varias veces la experiencia ? obtenidos con la ley de Weber-Fechner ¿Cómo pueden relacionar los resultados 13

12

Índice general Capítulo 1: NÚMEROS REALES ..................... 9 1. Números reales. Intervalos ................ 2. Módulo de un número real. ............... 3. Ecuaciones e inecuaciones con módulo. .............................................. Integración .......................................... 4. Radicales. ........................................... 5. Operaciones con radicales. ................ 6. Operaciones combinadas. .................. 7. Racionalización de denominadores. ... Integración .......................................... Autoevaluación ....................................

10 12 14 18 20 22 26 28 30 32

Capítulo 2: SUCESIONES ........................... 33 8. 9. 10. 11. 12.

Sucesiones. ......................................... Sucesiones aritméticas. ...................... Sucesiones geométricas. .................... Análisis de sucesiones. ........................ Clasificación de sucesiones. ............... Integración .......................................... Autoevaluación ....................................

34 36 38 40 42 46 48

Capítulo 3: NÚMEROS COMPLEJOS ........... 49 13. El conjunto de los números complejos. .......................................... 14. Módulo de un complejo. Complejos conjugados. ........................................ 15. Adición y sustracción. ........................ Integración .......................................... 16. Potencias de la unidad imaginaria. Cuadrado y cubo de un complejo. .... 17. Multiplicación y división. ................... 18. Operaciones combinadas. .................. 19. Ecuaciones. ......................................... Integración .......................................... Autoevaluación ....................................

50 52 54 56 58 60 64 68 72 74

Capítulo 4: CÓNICAS .................................. 75 20. 21. 22. 23.

Circunferencia. .................................... Elipse. .............................................. Parábola. ............................................ Hipérbola. ........................................... Integración .......................................... Autoevaluación ....................................

76 78 82 84 88 90

Capítulo 5: FUNCIONES ............................. 91 Funciones. ........................................... 92 Función inversa. ................................. 96 Interpretación y análisis de gráficos. . 98 Función lineal. .................................. 100 Función cuadrática. .......................... 104 Integración ........................................ 108 29. Ecuaciones cuadráticas. .................... 110 30. Sistemas de ecuaciones lineales. ..... 112 31. Sistemas de ecuaciones mixtos. ....... 116 Integración ........................................ 120 Autoevaluación .................................. 122 24. 25. 26. 27. 28.

Capítulo 6: FUNCIONES POLINÓMICAS Y RACIONALES .................................... 32. Función polinómica. ......................... 33. Análisis de la función polinómica. ... Integración ........................................ 34. Función racional. .............................. 35. Representación gráfica de funciones racionales. ......................................... 36. Función homográfica. ......................... Integración ........................................ Autoevaluación ..................................

Capítulo 8: TRIGONOMETRÍA .................... 171 123 124 126 130 132 134 138 142 144

Capítulo 7: FUNCIONES EXPONENCIALES Y LOGARÍTMICAS ................................. 37. Función exponencial. ........................ 38. Logaritmos. ....................................... 39. Función logarítmica. ......................... Integración ........................................ 40. Ecuaciones exponenciales. ............... 41. Ecuaciones logarítmicas. .................. Integración ........................................ Autoevaluación ..................................

Sistema de medición de ángulos. ... Razones trigonométricas. .................... Valores exactos y aproximados. ...... Ecuaciones trigonométricas. ............. Integración ........................................ 46. Triángulos rectángulos. ....................... 47. Teoremas del seno y del coseno. .... 48. Triángulos oblicuángulos. ................ Integración ........................................ Autoevaluación ..................................

42. 43. 44. 45.

172 174 178 180 182 184 188 190 194 196

Capítulo 9: ESTADÍSTICA 145 146 150 154 158 160 164 168 170

Y PROBABILIDAD ......................................... 49. Estadística. ....................................... 50. Intervalos de clase. .......................... 51. Parámetros de posición. .................. 52. Parámetros de dispersión. ............... Integración ........................................ 53. Combinatoria. ................................... 54. Permutaciones, variaciones y combinaciones. ................................. 55. Probabilidad. .................................... 56. Sucesos y probabilidad condicional. Integración ........................................ Autoevaluación ..................................

197 198 200 202 206 210 212 214 218 220 224 226

Control de resultados ............................... 227

¿Para qué sirve?

Contenidos

1

1. Números reales. Intervalos. 2. Módulo de un número real. 3. Ecuaciones e inecuaciones con módulo. 4. Radicales. 5. Operaciones con radicales. 6. Operaciones combinadas. 7. Racionalización de denominadores.

De todos los conceptos matemáticos, hay uno que debería ser el más sencillo de todos y sin embargo llevó más de veinte siglos entenderlo. Casi toda la matemática, y con ella la ciencia, se apoya en el número, principio fundamental de toda medida. Los pueblos más antiguos emplearon fracciones de enteros y estaban felices con esto; según cuenta la historia, recién a los pitagóricos les cupo el trago amargo de encontrarse, cara a cara, con los irracionales, que son aquellos que no pueden expresarse como cociente de enteros. Pero su definición rigurosa no es sencilla: a la antigüedad siguió la Edad Media y luego los tiempos modernos, en los que Newton y Leibniz desarrollaron el denominado cálculo infinitesimal. Sin embargo, los irracionales seguían siendo, en algún sentido, un misterio. Hizo falta esperar al siglo XIX para encontrar, por fin, una definición apropiada de “número real”.

1. Lean atentamente y respondan. a. Los matemáticos ¿podían resolver problemas con números irracionales antes del siglo XIX? b. Si se quiere confeccionar un aro de 60 cm de diámetro, ¿se puede utilizar un número racional cercano a π para calcular la cantidad necesaria de material? a. Sí; porque la dificultad era teórica: definir los números reales. Esto no les impedía resolver cálculos. b. Sí, porque hay infinitos racionales tan cerca como se quiera de cualquier irracional. En este caso conviene usar un número mayor que π para que no falte material.

capítulo

Números reales

1

2

3

4

5

6

7

8

9

10

Números reales. Intervalos ¿Para qué sirve?

INFOACTIVA

PÁGINA 2

Los números reales ( ) están formados por los números racionales e irracionales. Los números irracionales ( ) son aquellos que no pueden ser expresados como un cociente entre dos números enteros. Los números irracionales tienen infinitas cifras decimales no periódicas. Al escribir __ 3 __ su expresión decimal, en todos los casos se está realizando una aproximación. 32 ; 39 y π son números irracionales. __

3

__

39 ~ 2,08008382…

32 ~ 1,41421356…

π ~ 3,14159265…

Densidad y continuidad Entre dos números reales siempre existe otro número real. Se dice entonces que es un conjunto denso. A cada número real le corresponde un punto en la recta y recíprocamente. Se dice entonces que es un conjunto continuo.

Intervalos reales Se denomina intervalo real a toda semirrecta o segmento de la recta real. Si se utiliza paréntesis, significa que el extremo no pertenece al intervalo (intervalo abierto). Si se utiliza corchete, significa que el extremo pertenece al intervalo (intervalo cerrado). Los números mayores que a y menores que b se representan de la siguiente manera: A: x ∈ ∧ a < x < b = (a;b)





a

b

Los números mayores o iguales que a y menores o iguales que b se representan de la siguiente manera. B: x ∈ ∧ a ≤ x ≤ b = [a;b]





a

b

Los números mayores que a y menores o iguales que b se representan de la siguiente manera: C: x ∈ ∧ a < x ≤ b = (a;b]





a

b

Los números mayores o iguales que a se representan de la siguiente manera: D: x ∈ ∧ x ≥ a = [a;+')

 a

Expresen de dos maneras distintas la siguiente expresión. Luego, grafiquen. Los números mayores que –5 y menores o iguales que 3. A: x D ∧ –5 < x ≤ 3 = (–5;3]



–5 10



3

Test

de comprensión

1. Respondan y expliquen las respuestas. ___ a. 310 y 1,414215 son números reales. ¿Cuál es racional y cuál, irracional? b. ¿Cuántos números hay entre 1,3 y 1,31? ___

a. 310 es un número irracional, ya que tiene infinitas cifras decimales no periódicas y 1,414215 es un número racional porque se puede expresar como fracción. b. Hay infinitos porque los números reales forman un conjunto denso.

ACTIVIDADES Números reales. Intervalos

1

1. Dados estos números reales, indiquen cuáles son racionales y cuáles, irracionales. a. 0,215215215...

e. –136

Racional

Racional

__ 3

b. 4,134441334444….

Irracional

f. 37

Irracional

c. __52 π

Irracional

g. π – __31 π

Irracional

Racional

h. 1,141414…

Racional

4

______

d. 30,0016

2. Completen los cifras de los siguientes números irracionales respetando su ley de formación. a. 3,1112131415161 7

1

8

1

9

2 …

b. –22,1871187711187 7

7

1

1

1

1 …

3. Escriban un número racional k que cumpla las condiciones pedidas en cada caso. __ a. 2 < k < 3

c. 2,449 < k < 36

5 Por ejemplo __ 2.

Por ejemplo 2,4493. __

b. 3 < k < π

__

d. – 38 < k < – 37

Por ejemplo 3,12.

Por ejemplo –2,5.

4. Escriban de dos formas distintas los números que están representados en las rectas, teniendo en __ cuenta que a = – __23 y b = 33 . a.







a

b

-b

a

__

A: x D

b.



[

__

2 ≤ x ≤ 3 = __ 2; 3 ∧ __ 3 3 33

]

__

B: x D

__

2 = – 3 ;– __ 2 ∧ 33 < x ≤ __ 3 3 3

(

]

5. Escriban como intervalo real y grafiquen en la recta numérica. a. Los números reales mayores que –3 y menores o iguales que π.

b. Los números reales mayores o iguales que __76 .

c. Todos los números reales positivos.

(–3;/]

[ __76;+')

(0;+')

11

2

1

3

4

5

6

7

8

9

10

11

Módulo de un número real INFOACTIVA El módulo o valor absoluto de un número real es su distancia al cero sobre la recta real. Para todo número real x, su módulo se expresa: |x|. x si x ≥ 0 ∀ x ∈ : | x | = –x si x < 0

{

|–3| = 3

Referencias ∀: para todo ⇒: entonces ‹: si y solo si ∪: unión ∧: y ∨: o ≠: es distinto a

|4| = 4

¨ « « © « « ª ¨ « © « ª –3

|4| = 4

0

4

| –3 | = –(–3) = 3

Propiedades del módulo 1. | x | * 0

3. | x + y | ) | x | + | y | | –2,1 + 1 | ≤ | –2,1 | + | 1 | ⇒ 1,1 ≤ 3,1 | –3,4 + (–2,1) | ≤ | –3,4 | + | –2,1 | ⇒ 5,5 ≤ 5,5

| –5,2 | = –(–5,2) = 5,2 ≥ 0 1≥0 1 = __ __ 5 5

| |

2. | x | = | –x |

4. | x . y | = | x | . | y |

| –3,2 | = –(–3,2) ∧ | –(–3,2) | = 3,2 2 = – –__ 2 = __ 2 = __ 2 ∧ –__ 2 __ 3 3 3 3 3

| |

| |

( )

| –4,7 . 5 | = | –4,7 | . | 5 | ⇒ 23,5 = 23,5

| –2,5 . (–4) | = | –2,5 | . | –4 | ⇒ 10 = 10

Para entender mejor las propiedades que siguen, se representan los siguientes intervalos reales.

 –a

0 –a < x < a

x < –a

a x>a

5. |x| > a ∧ a > 0 ⇒ x > a ∨ x < –a ⇒ x ∈ (–';–a) ∪ (a;+')

 –a

0 |x| > a

a

|x| > 3 ⇒ x > 3 ∨ x < –3 ⇒ x ∈ (–∞;–6) ∪ (6;+∞) |x| ≥ 1,8 ⇒ x ≥ 1,8 ∨ x ≤ –1,8 ⇒ x ∈ (–∞;–1,8] ∪ [1,8;+∞) 6. |x| < a ∧ a > 0 ⇒ –a < x < a ⇒ x ∈ (–a;a)



–a

0 |x| < a

|x| < 7 ⇒ –7 < x < 7 ⇒ x ∈ (–7;7) 3 ⇒ –__ 3 ≤ x ≤ __ 3 ⇒ x ∈ –__ 3 ;__ 3 |x| ≤ __ 4 4 4 44

[

12

]

a

Test

de comprensión

1. Respondan y expliquen las respuestas. a. ¿Es cierto que | a – b | ≤ | a | + | –b |? b. ¿Para cuáles valores de x se cumple | –x | = 2? a. Sí, porque a – b = a + (–b) ‰ | a + (–b) | ) | a | + | –b |. b. x = –2 ∨ x = 2.

2

ACTIVIDADES Módulo de un número real

6. Calculen los siguientes módulos. __

a. | 16,14 | =

d. | –3 + 311 | =

16,14 __

__

b. | – 35 | =

__

–3 + 311

e. Si b < 0, | –b | =

35

c. | 1– (–5) – 12 | =

–b

f. Si a > 0, | a – 2a + 1,25a | =

6

0,25a

7. Completen con >, < o = según corresponda en cada caso. a. | – __76 |

= __

__

| __76 |

b. | 2 + 37 |

d. | –4 . 39 | = __

= | 2 | + | 37 |

c. | 5 + (–12) |

<

| –5 |

e. | 3,5 – (–2,4) |

| –4 |

>

. |3| | 3,5 | – | –2,4 |

f. Si a = 0 y b = 0, | a + b | =

+ | –12 |

|

a| + |b|

8. Expresen los valores que pueden tomar las variables. Luego, represéntenlos en la recta numérica. a. | x | = 2

x = 2 ∨ x = –2 No tiene solución.

b. | x | = –5 El módulo no puede ser negativo. c. | x | + 3 = 10 x = 7 ∨ x = –7 13 x = 31 ∨ x = –31 d. __31 . | x | – 6 = ___ 3

e. | x | ) __41

[

1 ;__ 1 x D –__ 4 4

]

f. | y |> 3 y D (–';–3] F (3;+') __

__

__

g. | h | < 36 h D (–36 ;36 )

(

__

) (

__

)

2 2 y D –';– 3__ F 3__ ;+' 2 2 h. 2 . | y | – 32 * 0 __

9. Marquen las opciones correctas. ¿Cuáles son los valores que puede tomar la variable en cada caso? a. | x | = 4 Ž x * –2

x D [–2;+')

b. | y | * –2 Ž y < 1

y D [–2;1)

c. –3 . | h | > –9 Ž x & 1

h D (–3;1)

X x = 4

y D (–',–1] X h D (–3;3) – {1}

x D [–2;4] X y D (–',–2]

h D (1;3) 13

3

2

4

5

6

7

8

9

10

11

12

Ecuaciones e inecuaciones con módulo INFOACTIVA Para resolver una ecuación, se debe aplicar la definición de módulo. x si x ≥ 0 |x| = –x si x < 0

{

Resuelvan las siguientes ecuaciones con módulo. 4 . | 2x – 5 | – 8 = x + 1 ← se elimina el módulo aplicando la definición. ∨ Si 2x – 5 < 0 ⇒ 4 . ( –2x + 5 ) – 8 = x + 1 Si 2x – 5 ≥ 0 ⇒ 4 . ( 2x – 5 ) – 8 = x + 1 5 ⇒ 8x – 20 – 8 = x + 1 Si x ≥ __ 2



5 ⇒ –8x + 20 – 8 = x + 1 Si x < __ 2

5 ⇒ 7x = 29 Si x ≥ __ 2



5 ⇒ –9x = –11 Si x < __ 2

29 5 ⇒ x = ___ Si x ≥ __ 2 7



5 ⇒ x = ___ 11 Si x < __ 2 9

 0

1



2 5 __ 3 2

4 29 ___ 7

5

–2

–1

0

1 11 ___ 9

2 5 __ 3 2

4

29 ∨ x = ___ 11 . La solución es S: x = ___ 9 7

Una inecuación se resuelve como una ecuación, salvo en el caso en que se divida o multiplique a ambos miembros por un número negativo, lo que invierte el sentido de la desigualdad.

Resuelvan las siguientes inecuaciones con módulo. a. | 3x + 5 | < 4 ← se elimina el módulo aplicando la definición. Si 3x + 5 ≥ 0 ⇒ 3x + 5 < 4 ∨ Si 3x + 5 < 0 ⇒ 3x + 5 > –4 5 ⇒ 3x < –1 Si x ≥ – __ 3



5 ⇒ 3x > –9 Si x < – __ 3

5 ⇒ x < – __ 1 Si x ≥ – __ 3 3



5 ⇒ x > –3 Si x < – __ 3



[ – __53;– __13 ) 

–2

5 – __ 3

4 – __ 3

–1

2 – __ 3

( –3;– __53 )



1 – __ 3

 0

–4

–3



__–1 –2 – 5 3

0

1

5 ∪ – __ 5 ;– __ 1 = –3;– __ 1 La solución es la unión de los intervalos. S: ( –3;– __ 3) 3 3) ( 3)

[

b. | 2x – 7 | + 5 > 3 + x | 2x – 7 | > –2 + x ← se elimina el módulo aplicando la definición. Si 2x – 7 ≥ 0 ⇒ 2x – 7 > –2 + x ∨ Si 2x – 7 < 0 ⇒ 2x – 7 < 2 – x 7 ⇒ x>5 Si x ≥ __ 2



7 ⇒ 3x < 9 Si x < __ 2

7 ⇒ x>5 Si x ≥ __ 2



7 ⇒ x k, ¿qué valores puede tomar k para que tenga solución? a. k < 0, porque el módulo de un número no puede ser negativo. b. Siempre tendrá solución, pues si k > 0, | x | puede ser mayor que ese positivo y si k < 0, para todo x, | x | > k.

3

ACTIVIDADES Ecuaciones e inecuaciones con módulo

10. Resuelvan las siguientes ecuaciones con módulo. d. –5 + | 3 – 3x | = –2

a. | 2 + x | = 5 x = 3 ∨ x = –7

x=0∨x=2

2 . |x – 5| e. _________ –1=4 7

b. | 3 – x | = __61 19

45 25 ___ x = ___ 2 ∨x=– 2

17 x = ___ ∨ x = ___ 6 6

____

___

f. 7 . |x – 3125 | = 8 . | x – 325 | – 1 3

c. | 4 + 2x | = –10 No tiene solución, pues | 4 + 2x | * 0.

x=4∨x=6

11. Hallen los valores de a para que cada ecuación tenga la cantidad de soluciones pedidas. a. | x – 2a | = a + 2, que tenga solución única. Para que tenga solución única: a = –2.

b. | x – (–5) | = 9a – 2, que tenga dos soluciones distintas. 2. Para que haya dos soluciones distintas: a > __ 9

12. Lean atentamente y escriban V (Verdadero) o F (Falso) según corresponda. La distancia entre dos puntos (a y b) se expresa: D(a;b) = | a – b | a. Si | x – 4 | = 5 ‰ D(x;–4) = 5. F

c. Si | x + 4 | = 5 ‰ D(x;–4) = 5. V

b. Si | x – 4 | = 5 ‰ D(x;4) = 5.

d. Si | x + 4 | = 5 ‰D(x;4) = 5.

V

F

13. Planteen la ecuación y resuelvan. La distancia entre el doble de un número real y el opuesto de 10 es 18. La ecuación es | 2x + 10 | = 18 y las soluciones son x = 4 y x = –14.

15

3

ACTIVIDADES Ecuaciones e inecuaciones con módulo

14. Marquen las opciones correctas. 15 ¿Cuáles son las soluciones de la ecuación __21 + 3 . | –6 + 5x | = ___ 2 + 3? 15 15 ___ a. x = ___ 28 ∨ x = 8

15 28 ___ c. x = ___ 8 ∨ x = 8

28 8 ___ X b. x = ___ 15 ∨ x = 15

15. Resuelvan las ecuaciones y representen la solución en la recta. c. 2 – 3x = –10 + 3 . | 2 – 2x |

a. | 1 – x | = 2x + 1 x=0

x = –2

d. | 3 – __21 x | – 4x = | – __21 x + 3 | + 4

b. 4 . |x – 4 | + 4 = 5 – x No tiene solución.

x = –1

16. Marquen las opciones correctas. 2 a + 7 tenga como solución a [–6;12]? a. ¿Cuál es el valor de a para que | x – 3 | ) __ 5

X a = 5

a = –5

a = –25

b. ¿Cuál es el valor de b para que | 5x – b | > __21 b + 2 tenga como solución a –';– __51 F (1;+')?

(

)

X b = 2 b = –2 b=0 c __ | | c. ¿Cuál es el valor de c para que x + 1 )2 + 3 no tenga solución?

c = –6

X c < –6

c > –6

17. Escriban las inecuaciones que representan cada situación y resuelvan. a. Un número está a menos de 5 unidades de distancia con respecto a 2. |x

– 2 | < 5; S: (–3;7).

b. Un número está a una distancia no menor de 4,5 unidades con respecto a 8. |x

– 8 | * 4,5; S: (–';3,5] F [12,5;+').

c. El anterior de un número está a una distancia mayor de 4 unidades con respecto a 0. |x

16

– 1 | > 4; S: (–';–3) F (5;+').

3

ACTIVIDADES Ecuaciones e inecuaciones con módulo

18. Resuelvan las siguientes inecuaciones. Luego, representen la solución en la recta numérica. a. | x + 6 | ) 3

d. | 2 – 2x | > 2

S: [–9;–3]

S: (–';0) F (2;+')

b. | 5 + 2x | < 2

e. 4 – | z + 1 | < 7 + 2z

7 __ S: – __ ;– 3 2 2

S: (–2;+')

(

)

c. –14 . | __71 + x | > –7 9

5 ;___ S: – ___ 14 14

(

f. 1 – __21 . (3x + 4) ) 3 – | 1 + 2x |

[

10 S: – ___ 7 ;6

)

]

19. Escriban una inecuación con módulo que tenga el conjunto solución pedido en cada caso. a. S: [–2;4]

d. S:

|x

|x

– 1| ) 3

b. S: __21 ;__29

|

( )

+ 2 | *0

e. No tienen solución.

|

5 x – __ 2 

|x

+ 2 | 0

c. S: (–';–6] F [4;+')

f. S:

|x

|x

+ 1 | *5

– {4}

– 4 | 0

mente ACTIVA Observen la siguiente resolución y encuentren el error. No se aplicó correctamente la propiedad del módulo. | –3z + 6 | ) 21 [ –3z + 6 * 0 ⇒ –3z + 6 ) 21 ] ∨ z D [–5;9]

| –3z

+ 6| –3z + 6 –3z z

) ) ) *

21 21 15 –5 ‰ x D [–5;+')

[ –3z + 6 < 0 ⇒ –3z + 6 * –21 ]

17

INTEGRACIÓN 20. Escriban V (Verdadero) o F (Falso) según

23. Escriban las siguientes expresiones de dos

corresponda. Expliquen las respuestas.

maneras distintas. a. Los números reales mayores que 4 y meno(4;18]; x > 4 ∧ x ≤ 18 res e iguales que 18. b. Los números reales mayores que cero y (0;21); x > 0 ∧ x < 21 menores que 21. __ c. Los números reales menores que 38 . d. Los números reales menores o iguales que 3 3 ; x ≤ – __ . d. –';– __ – __43 . 4 4 e. Todos__ los números reales. __

V

a. –5 es un número racional. ____ 3

b. – 3729 es un número irracional.

F

c. 1,18 es un número racional. V __

V

d. 35 es un número real.

(

F

e. 2/ = 6,283

f. __03 es un número racional. F

]

c. (–';38 ); x < 38 ; e. (–';+');

g. Todo número real es irracional.

F

h. Todo número irracional es real.

V

i. Existen números reales que son racionales e

24. Escriban como intervalos los números que están representados en las __rectas. Tengan en cuenta que a = – __23 y b = 33 . a.

irracionales. F

pre es otro número irracional.

F













a

j. La suma de dos números irracionales siem-

b

b.

k. Un número decimal periódico es racional.

V

–a

b

l. Todo número real tiene su inverso multiplicativo.

c.

F

–b

21. Completen las cifras de los siguientes números irracionales respetando su ley de formación. a. –0,3691215

1

b. 78,235711 1 c. 1,24816 3

8

2

3 2

1

6

d. –3,51015202 5

1 7

4 3

2 1

1 0

3

4 9

2 5

a

d.







a

–a

… Solución a cargo del alumno.

… 4

25. Marquen las opciones correctas. …

22. Escriban las siguientes expresiones como intervalos. Luego, grafiquen en una recta. (–';0) a.  – f. | x – 3 | > __21 b. x * –1 [–1;+') g. | 2x – 1 | < 4 Ž x * 1 [0;+') c. + h. | x | > 2 Ž | x | ) 5

a. ¿Cuál es la inecuación cuyo conjunto solución es (–4;4)? X | –x | < 4

– |x| > 4

|

x| ) 4

b. ¿Cuál es la inecuación cuyo conjunto solución es (–';–4] F [4;+')?

0

i. | x – 1 | < 1 Ž | x |> 1 j. | x – 3 | ) 4 Ž | x | * 2 d. (–';3/) F (3/;+'); e. [–5;5];

d. – {3/} e. | x | ) 5

5 5 7 __ __ f. –';__ 2 F 2 ;+' ; g. 1; 2 ; h. [–5;–2) F (2;5]; i. (1;2); j. (–';–2] F [2;7]

(

18

] [

)

[ )

|

x| > 4

X – | x | ≤ –4

|

x| ) 4

capítulo

CONTENIDOS

1

1*2*3 26. Marquen las opciones correctas. ¿Cuál es la expresión que representa cada intervalo?

[

] [ ]

a. –3;– __23 F __23 ;3 |

X

x | ) 1,5 Ž | x |* 3

|

x |  1,5 Ž | x | * 3

|

x | * 1,5 Ž | x | ) 3

a. __56 . | 2,5x – 5 | ) 1 : __65 b. 9 – __47 . | 3 – x | > __41 c. | 5x + 4 | + 10 * 3 . | 5x d. __32 . | x + 4 | – __35 . | x + 4 | 1 x + __

S: [1,6;2,4] S: (–2;8)

[

8 __

d. S:

(

) (

)

e. S: (–';–1)

32. Planteen la inecuación y resuelvan.

__

__

X | y | & 4 Ž y * 1 Ž | y | * 33 __

y & 4 Ž y * 1 Ž | y |  33 __

y& 4 Ž y  1 Ž | y |  33

27. Escriban V (Verdadero) o F (Falso). Expliquen las respuestas. F

a. El triple de la distancia entre un número real y 2 es menor que el doble del consecutivo de 4. b. La quinta parte de la distancia entre un número real y 1 es mayor o igual a –2. 4 ___ 1 | | ; 16 ; b. __ a. 3 . | x – 2 | < 10; S: – __ 5 . x – 1 * –2; S: 3 3

(

)

33. Escriban en lenguaje coloquial las siguientes expresiones. a. 4 . | x + 9 | * 8 b. 1 ___ + | x + 10 | = 7 ___ 4 3 c. 316 – | x – 8 | < – 327 Solución a cargo del alumno.

b. | a | = | b | ‰ a = b

F

34. Resuelvan las ecuaciones e inecuaciones del

c. | z | > | w | ‰ z > w F

ejercicio anterior. a. S = (–';–11] F [–7;+');

d. | –4 + (–6,1) | ) | –4 | + | –6,1 | V

35. Marquen las opciones correctas.

__

__

e. | –3 . 37 | = 3 . 37

]

+ 4 | + 2 S: – 5 ;0 ) –5x + 4 . | x + 4 |

1 1 2 __ __ e. x2 – _____ 6 > x – 2 . x + 2

b. [ 33 ;+') – {4}

a. | a | > 0 ‰ a > 0

31. Resuelvan las siguientes inecuaciones.

V

28. Resuelvan las siguientes ecuaciones. 35 37 , x = ___ a. – __71 + 4 . | –x + 9 | = __76 x = ___ 4 4 1 __ b. | x – 2 | + 4,5 = –1,5 . | x – 2 | – 2 No tiene solución. c. 7x + | 1 – 3x | = 1 – __21 x x = 0 d. | x – a | – 2a = 2 . | x – a | – 3a Ž a * 0

x = 2a, x = 0

29. Hallen los valores de w para que la siguiente ecuación tenga la solución pedida en cada caso. | x – w | – 2w = 2 . | x – w | – 3w Ž w * 0 a. S: x = 0 b. S: x = 0 ∨ x = 6 a. w = 0; b. w = 3

30. Planteen la ecuación y resuelvan. a. La distancia entre el triple de un número y el opuesto de –5 es 7. b. La diferencia entre 4 y la distancia entre el doble de un número real y –1 es igual a –10. 2 a. | 3x – 5 | = 7 y S: x = 4 ∨ x = – __ 3 b. 4 – | 2x + 1 | = –10 y S: x = –7,5 ∨ x = 6,5

b. S = {–4;–16}; c. S = (–';3) F (13;+')

a. ¿Cuál es el conjunto solución de la ecuación __ 2 . 33 . | __83 + 2x | = 0? No

tiene solución.

3. X Su solución es x = – ___ 16 3. Su solución es x = ___ 16

b. Si | x – a | ) 3a tiene como conjunto solución a S: – __81 ;__41 , ¿cuál es el valor de a? (a > 0)

(

)

a = __81

a = __41

1 X a = ___ 16

c. Si b > 0, ¿cuál es la inecuación que tiene la siguiente solución? x < –14b – 5  x > 16b + 5 |x

+ b | – 5b > 10b + 5

X | x – b | – 5b > 10b + 5 |x

– b | – 5b > –10b – 5

19

4

3

5

6

7

8

9

10

11

12

13

Radicales INFOACTIVA Los números radicales son aquellas raíces que no tienen solución racional. Todos ellos son números irracionales. __ 3 __ __ 4 __ 32 ; 39 ; 33 ; 34 ; son radicales.

Propiedades de la potenciación a0 = 1 ⇔ a ≠ 0 1 ⇔a≠0 a–n = __ an n m (a ) = an . m an . am = an + m an ___ = an – m ⇔ a ≠ 0 am (a . b)n = an . bn n n ( __ba ) = __ban ⇔ b ≠ 0

ˆPotencia de exponente cero. ˆPotencia de exponente negativo. ˆPotencia de otra potencia. ˆProducto de potencias de igual base. ˆCociente de potencias de igual base. ˆDistributividad respecto de la multiplicación. ˆDistributividad respecto de la división.

Propiedades de la radicación

__

__1

La radicación se puede expresar como una potencia de exponente fraccionario: 3n a = an __

1 __ 2

3

37 = 7

__

1 __ 3

5

__ 4

4 __ 5

3x = x

36 = 6

__ 1 = x–__34 __ x3

3 4

Las propiedades de la radicación son análogas con las de la potenciación. ___ __

1 1 __ __

1 ____

__ 3a

ˆRaíz de raíz.

3 m3a = ( am )n = an.m =

ˆDistributividad respecto de la multiplicación.

3a . b = (a . b)n = an . bn = 3n a . 3b

ˆDistributividad respecto de la división.

n

n

n

_____

1 __

__

___

__

( )

3

___ n

___

12

___

1 __

1 __

n __

bn

3b

__

1 __

m:r ___ n:r

m __ n

__

n:r

___

12

ˆEliminación del radical.

3an = a ⇔ n es impar 3an = |a| ⇔ n es par ___ _____ ____ 3 ___ ___ 5 7 3 5

___

___

ˆAmplificación de índices. 2.2

____

4

___

__

n

332 = 325 = | 2 | = 2

336 = 362 = | 6 | = 6 __

3125 = 353 = 35

316 = 324 = 32 n

4

___

3 7 = 3 71.2 = 3 72 = 3 49

___

m.p ____

m __

__

_____

7 3–128 = 3 (–2)7 = –2

3125 = 353 = 5

n.p ____

3am = a n = a n.p = 3am.p ⇔ p ≠ 0 __ 3.3 ____ 9 ___ 9 ____ 3 n

__

an a 3 __ ____ = ___ 1 = n __

333 = 33

3

n

3am = a = a = 3am:r ⇔ r ≠ 0 ____ 15 ___ 5 __ 15

ˆSimplificación de índices. 6

1 __ n

a a __ = __ b b

m.n

3

__

3.2

___

6

__

3 x4 = 3 x4.2 = 3 x8

3 9 = 3 32.3 = 3 36 = 3 729

Extracción de factores de un radical Existen factores, dentro de un radical, que pueden ser extraídos si el exponente de los mismos es mayor o igual que el índice de la raíz. Para ello deben aplicarse las propiedades de la potenciación y de la radicación. ______

_________

____________

___

__

__

_____

___

3 3 3 3 4 3 3 3 3 3 48x6y4 = 3 2 . 3x6yy3 = 3 2 . 23 . 3x6yy3 = 323 . 3x6 . 3 y . 32 . 3y = 2x2y . 3 6y (x ≥ 0; y ≥ 0) 3____ ____ ______

__

20

32x4 _____ 81y6

3 4

4 4

4 4

__

2 x . 4 __ 2 . 2 x = 4 ____ 2x . 4 __ 2 = ___ 2 = 4 _______ 4 4 34y2y4 y2 3y y2

3

33 y 3

3

(x ≥ 0; y > 0)

Test

de comprensión

1. Respondan y expliquen sus respuestas.

_______

5 3 _____

a. ¿Qué propiedad de la radicación se puede aplicar a 3 32 . a6 ?_____ b. ¿Se puede cancelar el índice de la raíz con el exponente en 3(–7)2 ?

a. Se puede aplicar ___ producto __ 15 __de índices de radicales y propiedad distributiva de la radicación con respecto 15 15 al producto. 32a6 = 32 . 3a6 . _____ ___ b. No se puede cancelar la raíz con el exponente por tener base negativa. 3(–7)2 = 349 = 7.

ACTIVIDADES Radicales

4

36. Escriban V (Verdadero) o F (Falso) según corresponda. ___

__

3 __

a. b4 . b5 = b20 F

e. (a + b – c)2 = a2 + b2 – c2 F

i. 35a3 = 35 . a2 V

b. a10 : a4 = a6

f. (a . b4 . c–3)5 = a5 . b10 . c–5

j. 3x2 = x F

V

4

__ _____

d. (a : b)n = an : b

4 __

g. 3a7 = a7

c. d . d . d . d . d = 5d F

5

__

___ __

k. 336 a =

V __

__ 3a

11

F

__

h. 3a – b = 3a – 3b

F

__

F

l. 3a2 = | a | V

F

37. Reduzcan a la mínima expresión posible aplicando las propiedades de la potenciación. c. (x–3 . z4)5 : (x–2 . z6)–3 =

a. (a2 . a9 : a5)7 = a42

x–21 . z38 1 d. (v–3 . w5 . w4) : ( v5 . w–2 . __ w) =

b. (y5)–2 : (y3 . y4)3 = y–31

v–8 . w12

38. Reduzcan a la mínima expresión posible aplicando las______ propiedades de la radicación. ______ __ __ __ __ 5 4 10 6 2 4 5 3 2 a. 35 . 3x . 3x . 3x = c. 3x . y . 3x . y8 = 7 __

5x6

6 __

x5 . y5

______

x15 b. 6 ___ . z18 = y13

3 5 __

4

13 ___

39. Resuelvan aplicando propiedades. __ __ 54 a. __ + 6–2 – 33 . 33 + __41 52

–3

( )

3 097 = _____ 36

__

4 b. [ 32–1 : 2–2 ] 7 – 52 . 35 = 7 4 __

1 __

256 22 1 _____ __ __ –2 6 2 401 – 4 . 43 . 4 – 2

33

_______

3

_______

3(x + y)–1 . 3(x + y)4 =

__

___

18 3__8 . 3____ d. _________ – __41 32 . 3200

–3

( )

4

. 42 : __41

–5

( )

= –0,4

_______

e. 3625 . 34 . (–1)17 . (–2)5 = 480

______ _____

c.

12

(x + y)2

x2 . y– 6 . z3

___

_______

d. 3(x + y)3 .

–3

( )

164 = – ____ 7

__

__

_______

6 3 12 81 f. 395 . 338 . 52 : ( 2 . 334 . 524 ) = ___ 2

21

5

4

6

7

8

9

10

11

12

13

14

Operaciones con radicales INFOACTIVA Radicales semejantes Dos radicales son semejantes cuando tienen igual índice y el mismo radicando. ˆTérminos con__radicales __ semejantes. ˆTérminos con__radicales no __ semejantes. __ __ __ __ 3 3 3 2 . 35 y 5 . 35 7 . 38 y –38 2 . 35 y 2 . 35 8 . 37 y 7 . 38

Adición y sustracción de radicales Solo es__posible__sumar términos que contienen radicales semejantes. __ o restar __ __ 4 . 33 3 – 33 __ + 2 . 3__ __ = 33 . (4 __ + 32__– 1) = 5 . 33 __ __ __ 3 3 3 3 . 32 – 4 . 32 + 32 + 6 . 32 = 32 . (3 + 1) + 32 . (–4 + 6) = 4 . 32 + 2 . 32 Existen casos en ___ los cuales ___ ciertos ___ radicales son__semejantes de llevarlos ___ luego_____ ___ a su mínima expresión. __ 8 4 8 3 –4 . 33 + 5 . 381 – 3 . 312 + 327 = –4 . 3__ 3 + 5 . 3__ 3 – 3 . 3___ 22 . 3__+ 33___ __ 3 + 5 . 3__ 3 – 3 . 322 __ . 33 + 33__2 . 33 = –4 . 3__ 3 3 3 3 = –4 __ . 3 + 5 . 3 – 3 . 2 . 3 +__3 . 3 = 33 . (–4 + 5 – 6 + 3) = –2 . 33

Multiplicación y división de radicales Para efectuar cualquier multiplicación o división de radicales, estos deben tener el mismo índice. La operatoria con radicales cumple con las siguientes propiedades. ˆ Propiedad distributiva de la multiplicación y de la división respecto de la suma y de la resta. a . __ (b ± c) ± c) . a__= ab (b ___ ± c) : a__= b __: a ±___ c : a __ __ __ __ = (b___ __ ± ac __ ___ ( 318 – 38 ) : 32 = 318 33 . ( 33 + 327 ) = 3__ 3 . 33___ + 33 . 327 2 – 38 : 32 __ : 3__ = 39 – 34 = 3 – 2 = 1 = 39 + 381 = 3 + 9 = 12 ˆ Cuadrado de un binomio y diferencia de cuadrados. 2 2 2 (a ± __b)2 =__a2 ± 2ab (a __ + b) .__(a – b) __ = a__– b __ __ __ + b __ __ __ 2 2 2 2 ( ( ( ) ( ) ) ( 32 – 33 ) = ( 32 ) – 2 ( ) 3 3 3 3 3 3 3 3 3 . 2 . 3 + 3 7 + 5 . 7 – 5 = 7 – 5 )2 __ __ =7–5=2 = 2 – 2 . 36 + 3 = 5 – 2 . 36

Multiplicación y división de radicales de distinto índice Para que los índices de dos o más radicales sean iguales, se debe calcular el mcm de los índices de los __ radicales dados, obteniéndose así el mínimo común índice. __ 3 2 6 3__ a y3 x ___ ← mcm(3;6) = 6; ambos radicales deben tener índice 6. __ __ 3 3.2 6 6 3a2 = 3a2.2 = 3a4 y 3 x Para multiplicar o dividir radicales de distinto índice, se los debe reducir a mínimo común índice y luego aplicar las propiedades recíprocas de las distributivas de la radicación respecto de la multiplicación y división. __ ___________ __ n __ __ n __ n __ n n n __ a n 3a ___ ⇔b≠0 3a . 3b . 3c ... 3d = 3a . b . c ... d ∧ n __ = b

3

3b

__

3

__

2.3

____

33 . 33 = 33

1.3

22

3.2

____

. 33

1.2

6

___

6

___

6

______

6

___

= 33 . 33 = 33 . 3 = 33 3

2

3

2

5

___

____

___

__ 6.2 5.2 5 12 ___ 33 33 310 = 1233 ____ ___ = ______ ____ 4.3 3.3 = 9 4 3 3 33 33 6

3

de comprensión

Test

1. Respondan y expliquen las respuestas. __

__

__

__

a. Los términos 2 . 3v y –5 . 33 v ¿son semejantes? ¿Y__ 32 y__38 ? __ ______ ___ __ __ __ 30 7 7 7 7 2 4 b. ¿Qué propiedades de radicación se aplicaron en 3a . 3b3 . 3c4 = 3a2b3c__ ? ¿Y __en 3x .33 x . 35 x = 3x31 ? a. No __son semejantes, pues en uno la __raíz es cuadrada y en el otro, cúbica. 32 y 38 son semejantes, ya __ que 38 = 2 . 32 que es semejante a 32 . b. Propiedad recíproca de la distributiva de la radicación del mismo índice con respecto al producto. Propiedad de reducción a común índice de radicales distintos en el producto.

ACTIVIDADES Operaciones con radicales

5

40. Sumen y resten los términos con radicales semejantes. __

27 __ ___ . 2 4 3

__

__ a. 8 . 32 – 2 . 32 + __43 . 32 = __

__

__

__

b. –3 . 33 + 4 . 35 – __21 . 33 + 12 . 35 = ___

____

___

c. 0,5 . 312 + 4 . 375 – 0,3 . 3108 = ___

____

____

___

d. 316x – 325x + __23 . 336x – 381z = __

__

6

4

2

__

__

__

__

19 . 33 __

__

13 . 3x – 4 . 3z __

11 b . 3 b __ 3 4

__

9 e. 3b + 2b . 3b – __41 b . 3b3 = 3

__

7 – __ 2 . 33 + 16 . 35

( __52 c + __23 c

___

4 4 4 f. __52 . 3c5 + __23 . 3c9 – __71 . 3c13 =

2

__ – __71 c3 ) . 34 c

41. Resuelvan aplicando la propiedad distributiva o diferencia de cuadrado según corresponda. __

__

___

__

4 . 32

__

__

__

___

d. 33 . ( –324 + 36 ) + 398 =

–3 . 315 – 12 . 333

___

__

___

__

__

a. –3 . 33 . ( 35 + 4 . 311 ) =

__

b. ( 2 . 32 + 5 . 35 ) . ( 3 . 35 + 32 ) =

_____

_____

__

__

e. ( 3100a7 + 336b5 ) . ( 10a3 . 3a – 6b2 . 3b ) =

___

100a7 – 36b5

11 . 310 + 79

__

__

__

__

c. ( 36 – 37 ) . ( 36 + 37 ) =

5

_____

___

5

_____

f. 332x10 . 35 xy . 3x4 . y4 . (x – y) = 2x4 . y – 2x3 . y2

–1

42. Resuelvan aplicando el cuadrado del binomio. __

__

a. ( –2 . 33 + 3 . 32 )2 =

__

__

__

30 – 12 . 36

__

__

__

c. 2 . 33 . ( 33 + 32 )2 = __

10 . 33 + 12 . 32

____

2 b. ( 3a7 – 4 . 39a11 ) =

a7 – 24a9 + 144a11

__

___

___

4 d. ( 5 . 35 – 8 . 320 )2 – 3252 =

600

23

ACTIVIDADES Operaciones con radicales

5

43. Reduzcan a común índice los siguientes radicales. 4

__

3

__

__

12

12

___

__

___

__

65

60

___

260

__

21

__

__

___

60

60

___

___

___

60

60

___

3e24 y 3e18 y 3e70

__

__

91

c. 3c7 y 3c4 260

21

5

___

__

__

e. 3e2 ; 3e18 y 3e70

3b4 y 3b15

52

21

3d7 ; 3d3 y 3d

14

70

__

21

b. 3b2 y 3b3 70

__

7

d. 3d ; 3d y 3d

3a9 y 3a16

35

__

3

a. 3a3 y 3a4

77

__

__

143

f. 3f11 ; 3f13 y 3f7

__

1 001

___

3f 121 ;

3c35 y 3c16

1 001

___

3f 169 ;

1 001

___

3f 49 .

44. Resuelvan. ________ __ __

6

__

3

________ __ 4 __

a. 332 . 323 =

c. 352 . 3326 . 358 =

2

10

__

__

__

3 8 12 b. 334 : (335 . 337 ) = 8

3

___

12

__

9

__

d. 318 . 394 : 336 =

__

3

33

___

318

45. Escriban V (Verdadero) o F (Falso). Expliquen las respuestas donde escribieron F. __

__

__

18

___

a. 33 a . 36 a . 39 a = 3a36 ___ 18

3a11 4

___

__

12

__

___

___ ay3 3___ c. _____ = 36 ay V 3 3ay4

__

4

__

d. 33 xy . 3x2 . 3y5 = y10 . 12

y2 .

3 x__ 3x9 __ V b. 3____ = _______ 3 12 b . 3b4 3b4

24

F

_______

12

______

F

3x10 . y2

3x10 . y10

__

_____

_____

_____

3 2 3 x2 . y xy . 3x2 . y .y 3 xy _____ . 3 x_____ __________ e. ___ 5 = 2 . 100 10

3

3

3

___________ __________ 3

_____ 2 6 – 4x + 4 3x_______ f. 3_____________ = 3x – 2 6 2 3(x – 2)

F

1

F

3

___

xy . 3x2 _______ 10

ACTIVIDADES Operaciones con radicales

5

46. Resuelvan los siguientes cálculos hasta encontrar su mínima expresión. 3

___

5

12

327a _______

20

33–1 . a–2

__

3

8

3

__

3t . 3tu

3 __ 3u

__

__

3 5 . 3u___ :u 3t__ e. ___________ = 48 10 6 2

__ 3v

3

___

___3m . 3m

__

3v5

___

2

3m13

. 3v2 3v __ b. ________ = 4 3 12

___

m2___ . 3m5 : m d. 3_____________ = __ 4 3

39a ____ a. ______ = 4 2

___

6

7

3x2 . 32x3 34x . ____ c. ______________ = 3 2

____

3

__

b3d11 . ____ 3d5 3____ f. ___________ 3 2 7 3 8 =

327x 2 x . 6 __ __ 2 3 3

3b d . 3b d ___ 21 37 3d__ _____ 3 3b2

47. Marquen las opciones correctas.

__

__

__

__

8 4 4 8 a. ¿Cuál es la expresión simplificada de ( 3x5 – 3x5 ) . ( 3x5 + 3x5 )?

__

__

4 X x2 . 3x – x . 3 x

__

__

__

x . 3x – x2 . 34 x

__

x . 3x + x . 34 x ___

4x + 9y – 12 . 3xy ___ ___ ? b. ¿Cuál es la expresión simplificada de ___________________ 2 (a – b) . ( 34x – 39y ) 1 _____ a+b

1 X _____

a–b

a–b

5

__________

4

__________

10

__________

c. ¿Cuál es la expresión simplificada de 3a2 . b10 . c12 . 3a2 . b10 . c12 . 3a2 . b10 . c12 ? 20

_________

ab10c12 . 3a2 . b5 . c6

20

__________

20

X ab5c6 . 3a2 . b10 . c12 _____________

__________

ab5c . 3a2 . b10 . c12 ___

(3 )

d. ¿Cuál es la expresión simplificada de __21 . __32 . x2 . v . w3 . – __94 v ? 3

3

______

_________

________

xw 6 __ 2 5 3 X – ___ 3 . 3v w

xw 6 __ 2 2 3 3 ___ 3 . –3 x v w

xw 6 __ 2 2 3 3 ___ 3 . 3x v w

3

3

3

_________

__ 2 __ – b2 . a2 e. ¿Cuál es la expresión simplificada de a . 3a + a__________ – 3a3 ; con a > 0 y b & 1? 1 – b2 __ 3a

a

__ . 3a

3

X a

25

6

5

7

8

9

10

11

12

13

14

15

Operaciones combinadas INFOACTIVA Para resolver un cálculo combinando con radicales, se deben seguir estos pasos teniendo en cuenta la jerarquía de las operaciones y sus propiedades. 1. Se separa en términos. 2. Se escriben los radicales en su mínima expresión. 3. Se resuelven las potencias. 4. Se resuelven las multiplicaciones y divisiones. Si es necesario, se obtiene el mínimo común índice de los radicales para resolver. 5. Cuando sea posible, se reducen a su mínima expresión los radicales obtenidos en el paso anterior. 6. Se resuelven las sumas y restas entre los radicales semejantes.

ˆ Resuelvan los siguientes cálculos combinados. __

___

__

__

a. 33 . 33 . 327 ____ 3243 ___ 5 3 ___3 __ 334 . 33__ 3 32 . 3__ 9 . 33 __

3

___

___ __

___

___

___

6 6 6 35 . 336 : 310 – 351 : 317 = ____ 3.2 ____ _______ ___ 1 __ 2.3 1.3 6 6 35 . 362 . 2 : 310 – 351 : 17 =

b.

____

6

6

__

6

___

6

3125 . 36 : 310 – 6

__

33

___________

6

3125 . 6 : ___ 10 – 6 3 75 ______ – 6 3 25 . 3__ – ___ 6 6 325 . 33 –

= = = = ___ __ 6 6 = ( 325 – 1 ) . 3 3

__

3__ 3 6 33 __ 6 3__ 3 6 33

ˆ Hallen la mínima expresión posible. __

5

__

3______ 5___ . 35 4 325 2.10

____

20

___

20

___

3510___ . 354 __________ 20 10 35 10

4

5 .5 3_______ 5 10

20

__

__

2 . 36

__

__

__

___

+

3

– 2 . 324 + 8



2 . 36

+

11



+

11



+

11



__

__



2 . 36

2 . 322 . 6 ___

__

2 . 36

=

_____

__

354 – 5

__

__



___

35

__

3 + ( 33 )2 – 2 . 33 . 38 + ( 38 )2 = – 2 . 32 . 8 . __ 8

_______

20

__

__

3 + ( 33 – 38 )2 = – 2 . 32 . 38 . 3__ 8

_______

____

5.4

351.10____ . 351.4 _____________ 4.5 2.5 35

26

__

32 = + (4 . 3___ 6 – 3 . 310 ) .___ 312 – 3 . 3_____ + 4 . _____ 20 = + 4 . 3___ 22 . 3__ – 3 . 3___ 22 . 5__ = + 4 . 322 . 3__3 – 3 . 322 . 3__5 = 33 – 3 . 2 .__35 = + 4 . 2 . __ __ __ + 8 . 33 – 6 . 35 = 17 . 33 – 6 . 35

=

__

2 . 322 . 36 = __

4 . 36

5

__

__

= 35 – 6 . 36 + 11

de comprensión

Test

1. Respondan y expliquen las respuestas. a. ¿Es __ correcta __la resolución___ del siguiente cálculo? __ __ __ __ __ 9 . 33 – 6 . 33 . 32 + 2 . 354 = 9 . 33 – 6 . 36 + 2 . 3 . 36 = 9 . 33 __ __ 5 __ 15 __ 3 3 b. ¿En qué orden se deben resolver las operaciones del siguiente cálculo? 34 + 34 . 34 : 34

a. Sí, es correcto. b. Primero se debe resolver la multiplicación y la división de radicales, y luego, si se obtienen términos semejantes, se resuelve la suma.

ACTIVIDADES Operaciones combinadas

6

48. Resuelvan. __ __ __

3

____

3

__

4

3

__

4

6 + 3 . 33

__

____

______

1 + __ 3405 4 d. _________ . 35 . 81–1 = 2

a. 32 . 36 . 33 – 3–27 . 33 =

__

32

__

(35 + 3 . 35 ) : 6

__

__

__

__

2 2 b. (36 + 35 ) + ( 36 – 35 ) =

e.

____

__

__

( 3___814 – 32 + 5 . 30,02 ) . ( 32 + 1 ) 4

2

=

1 __ __ 2 – __ 2 . 32 – 3

22

__

____

____

____

____

3 3 3 3 3 c. ( 3500 – 34 + 3108 ) . ( 3256 – 3500 ) = 3

__ 2

_____

3

_______ __

f. ( 0,3 . 3(–2)4 + 35 – 34 4

__

) . ( 33 3

3

___

3

___

3

+ 324 + 381 ) =

__ 1 944 _____ 5 + 648 . 33

–7 . ( 34 )

49. Hallen la mínima expresión posible. __

__

__

_____

27 a. 3b4 + 2b . 3b2 – __31 . 3b3 . ______ = 512b16

(

3

6

9

1 __ b4

__

) 3 3

(

–2y

)

______

[

1 __

]

1 – __ 3

__

____ ____ y3 y5 3___ b. 3y – 3___ y + y2 . ( 325y – 349y ) = __

______

c. ( 39x + 9 – 34x + 4 ) : –3 . (x + 1)2 =

_____________ 3

2

_______

x + 25x 3x + 10 _____ d. _______________ – 3x2 + 5x = 5+x 3

0

27

7

6

8

9

10

11

12

13

14

15

16

Racionalización de denominadores INFOACTIVA Racionalizar el denominador de una fracción es transformarlo en un número racional; por lo tanto, siempre que en el mismo aparezcan radicales irracionales, se debe hallar una fracción equivalente a la dada con denominador racional.

ˆ Primer caso: en el denominador hay un único radical. 2 ___ __ 4 33

1 ___ __ 33 __

__

__

3__ 3__3 3 = ______ 1 = ___ 1 . ___ ___ __ __ ( 33 )2 33 33 33

___

3 3 ___

=3

___

___

___

4 3 4 4 2__ . 3 3 . 333 = 2 . 34___ 27 = 2_______ . 327 ____ ______ ______ ___ = 2 _____ = ___ 4 4 3 4 4 3 33 33 33 . 33 334

2__ ___ 4 33

Si en el cálculo aparecen letras, se procede de la misma forma. (x ≠ 0; y ≠ 0)

7 _____ ____ 5 3x2y4 7 _____ ____ 5 3x2y4

5

___

5

___

5

___

5

___

3 y 7______ . 3x3y 7 . ____ . 3x3y 3x___ 3x3y = 7_______ 7 . _____ ____ = _____ = ________ = _______ 5 3 5 2 4 5 2 4 3 5 5 5 xy

3x y

3x y

3x y x y

3x y

ˆ Segundo caso: el denominador es una suma o resta de uno o dos radicales de índice 2. Para racionalizar este tipo de expresiones, se debe aplicar el producto de una suma de dos términos por su diferencia: ( a + b ) . ( a – b ) = a2 – b2

Racionalicen y hallen la mínima expresión. __ 15__ a. ________

37 – 32

__

__

__

__

__

__

__

__

. ( 37 +__32 ) __ 37 + 15__. 32 15 . __ . 37 + 15 . 32 = 3__ 7 + 3__ 2 = ___________________ ________ _______________ __ 15__ = ________ __ 15__ . ________ __ 15 __ = 15 = _______________ 7–2 ( 37 )2 – ( 32 )2 37 – 32 37 – 32 37 + 32 ( 37 – 32 ) . ( 37 + 32 ) __ __ __ __ . 37 + 15 . 32 = 3 . 37 + 3 . 32 _______________ = 15 5 __

b.

33 +__ 2 _______ 5 – 35 __

__

__

__

__

33 + 2 ) . ( 5 + 35 ) 33 +__ 33 +__ 2 = _______ 2 . ______ 5 = (________________ 5 + 3__ _______ __ __ 5 – 35 5 – 35 5 + 35 ( 5 – 35 ) . ( 5 + 35 ) __

=

__

__

__

__

__

___

5 . 33 + 315 + 10 + 2 . 35 = _______________________ 5 . 33 + 315 + 10 + 2 . 35 = _______________________ 25 – 5 20

3___ 3

3____ 15

__

___

__

5 . 33 + 33 . 35__+ 2 . 5 + 2 . 35 = ___________________________ 52 – ( 35 )2

__

___

= 4+

Si en el cálculo aparecen letras, se procede de la misma forma.

Racionalicen y hallen la mínima expresión. (x > 0; y > 0) _______ __ 2 __ 3x + 3y __ __ __ __ __ __ __ __ 2 . ( 3x – 3y ) 2. 3x – 2 . 3y 2. 3x – 2 . 3y 3x – 3y _______ __ __ = _________________ __ __ __ __ = ____________ __ 2 __ = _______ __ 2 __ . _______ __ 2 __ 2 = ___________ x–y 3x + 3y 3x + 3y 3x – 3y ( 3x + 3y ) . ( 3x – 3y ) ( 3x ) – ( 3y )

28

__

5 1 + 3___ + __ 20 2 10

Test

de comprensión

1. Respondan y expliquen las respuestas. 3 __ a. ¿Cómo se racionaliza ____ ? 3π

2 __ b. ¿Por cuánto hay que multiplicar la siguiente expresión para racionalizarla? ______ 1 + 3a __ 1 – 3__ a a. No se puede racionalizar porque π es un número irracional; b. Por ______ . 1 – 3a

ACTIVIDADES Racionalización de denominadores

7

50. Racionalicen los cálculos que tienen un solo radical en el denominador y hallen la mínima expresión. __ 2 __ a. ____ =

2 . __ __ 11 311

9z ___ d. ____ 8 11 =

4 __ b. ______ = 3 . 33

4 __ __ . 3 9 3

–5______ . 3v4 w3 e. __________ = 332 v3 w

5 2 – __ 8 . 32vw

5__ c. ____ 5 3 =

352

5abc ______ = f. ________ 4 11 2

5 . 3ab2 c3 _________ a2

311

35

5

3z

8

9 . 3z5 _______ z

____

__

3a b c

_____

4

_____

51. Racionalicen los cálculos que tienen una suma o resta con radicales en el denominador y hallen la mínima expresión. __ 3 __ a. _______ = 37 – 2 2+ 7

_____

–x 35 _____ c. __________ = 1 – 35 + x

3

__ 3x __ = b. ________

32 + 35

__

__

x . 35 – x . 32

_____

_______

5 – x + 325 – x2 3________________ –4 – x

__ __ 9x – 4y____ ____ d. ___________ = __23 . 3x – 3y 336x + 316y

52. Hallen la mínima expresión de los siguientes cálculos. ______ __

2__ + 1 33______ a. ________ = 332 – 1

_________ __

33 + 2 . 32

__

___ ___ 3y . 33 a __ b. _______ + 33 ay = 4 . 33 ay 3 2 y 3

29

INTEGRACIÓN 53. Escriban V (Verdadero) o F (Falso).

57. Resuelvan aplicando propiedades de la

Expliquen las respuestas donde escribieron F.

potenciación y de la radicación.

3

4

–3

a. (2 . 2 : 2 ) = 2 b. [ (4 ) ] = 4 3 –2 –3

5 __

18

_____

345

__

__

___ 4

8

___

F

__ 4

_____ ___

4 6 __

F

h. 333 = 33 __

6

2

V

5 __ 3

10

___

__

___

__

1 __ 8 3

b. ( 3a ) : ( a c. e3 . ( e

1 2 __ 5

)

__

– __54

__

.e .e

: __e1

( )

= 3e27

b. 32 : 316 . 1 __ 2

c. ( 3

(

___

+ 327

___

3

d. 372 –

15

1 __ 8

1 __ 3

3

) –4 3( ____

5

–2

____

)

__ 3 89 ___ + 3 . 32 4

=

__

15

. 34 =

+ 3243 ) . 3 . 3

__

1 __ 4

. __31

( )

___

__

1 – 2 . 32 – __41

8 1 __ ___ 9 + 3200 . 25 . 372 =

3

16 ___ . 7 – 25 . 38 ; e. 26 . 321 – 129; f. –198 3 3__ –4 . 35 ; h. 307;

___

____

____

___

___

___

___

__

__

3

__

____

e. –a . 3b2 . (b . 33 a – a . 3b ) = – 33 a4b5 + a2b

( )

___

b. g.

___

__ 2 3 . 63 –2 a. 6______ + 2 – 2 . 3 + __41 3 4 6 ____ 5

__

___

___ __ 6 6 c. __32 . 3w3 – __21 . 3w3 + __61 . 3w9 = __5 . 3w3 – __21 . 3w 6 __ __ __ __ d. (9a . 3x + 5x . 3a ) . (9a . 3x – 5x . 3a ) =

56. Resuelvan.

–1

___

k. ( –3 . 3__3 + 6 . 3__12 )2 – 394___= 162

____

z9

___

___

a. 324y + 324z – 36y – 354z = 36y – 36z __ __ __ _ _ 8 4 8 4 b. 3t5 – 3t . 3t4 – __21 t . 3t2 = __21 t . 3t – 3t . 3t

___ 10

– __21

___ 3 45 . z3 . z5 d. z________ = 3z137 8 __

5

___

correspondan.

__

3 __

1 __

__

59. Resuelvan aplicando las propiedades que

5 = 3a

3 __ 5

__

__

__

5

)

__

2 3 3 3 3 j. ( –6 . 327 + 325 ) = 36 . 3214 + 3210 – 192

= 3x3

1 2 __ __ 3 2

__

__

__

radicales. __ 5

__

__

2 i. ( 4 . 35 – 5 . 36 ) = 230 – 40 . 330

55. Resuelvan. Expresen el resultado utilizando 6 __

___

g. 15 . 32 – 310 . (2 . 32 + 3 . 35 ) =

__

)

___

f. (2 . 311 – 11 . 32 ) . (2 . 311 + 11 . 32 ) = __

137 ____

– __31 5

__

h. (12 . 33 + 5 . 35 ) . (12 . 33 – 5 . 35 ) =

3256

1 2 __ __

__

____

__

__

45 8 . ____ 32 . 38 b. __________ =2 9

a. ( x5 )2 : ( x

__

d. –4 . 36 . (2 . 3__ 21 – 37 )__= –24.314 + 4 .342 __ __ e. (3 . 37 – 4 . 33 ) . (2 . 33 – 5 . 37 ) =

V

__

=

c. –0,25 . 3320 – 9 . 35 + 0,2 . 3405 = –9 . 35

exponentes fraccionarios. 149 __ 5 __ 7 __ __ ____ 4 a. 33 . 333 . 335 : 33 = a140 5

0

7

____

__

33

54. Resuelvan. Expresen el resultado utilizando __

__ : 3a

___

__

3

5 __ –4

49 a. –5 . 32 – 12 . 32 + __32 . 32 = – ___ . 32 __ __ __ __3 1 __ b. 5 . 37 – 8 . 38 + 3 . 37 – 17 . 38 =

____

i. 35 = 3125

__

10

58. Resuelvan los siguientes cálculos. __ __ __

3256 = 2 24

F

8 __ 5

a . 3a

325

____ 8

__

10

_______ __

d. [ a . ( 3a ) . ( 3a ) . ( 3a ) ]

V

f. 325 : 8 = 325 : 38 g. 3(–2)8 = –2

5

5 __ 2

3

e. 39 + 16 = 39 + 316 ______ 4

___

_____ __ 5

___

__

__

c. 3b . 3b . 3b3 . 3b – b . 3b3 =

d. 38 . 3 = 38 . 33 ______

___

__

3

F

__

__

8

–5 –3 3__4a . _____ 39a b. __________ + ( __a1 ) . a2 : ( __a1 ) = 3a . 3 100a

V

__

5

c. 43 = 343

____ ___ __

4 7 28 a. ( 333v ) . 3v3 . 3v4 = v2 . 3v9

210

F

2

= 117 8

__

__

____

__

f. 3x . 3y3 . (– 3y + 3y . 3x ) = __

__

__

–3xy2 + 9 . 3x3y5 __

2 4 12 a3 + 4 . 3a5 – 28a2 g. ( 7 . 3a9 – 2 . 3a5 ) = 49 . 3___ __ ___

__

____

3

12

2 4 3 32.3b h. ( –8 . 3b5 + 316b3 ) = 64 .3b __+ 4 .3b –___ 5 35 10

3

29

__ __ __ 2 3c2 + 4 . 3c17 14 7 i. ( –2 . 3c2 – 35 c ) – 4 . 3c8 =

d. 81a2x – 25x2a

60. Hallen el valor de a para que se cumplan las siguientes igualdades. _______ __ a–1 a. 32 + 2 = 33 _______ b. 5a + __41 = –2

3

a. a = 1; b. No existe valor de a que satisfaga la igualdad.

30

capítulo

CONTENIDOS

1

4*5*6*7 61. Escriban V (Verdadero) o F (Falso).

63. Tengan en cuenta el valor de c y calculen lo

Expliquen las respuestas donde escribieron F. __

__

___

__

a. 32 . ( 38 – 318 )2 = –2 . 32 F ___

__

__

32 – 1

________

______ 3 __1 __ __ . ( 36 a )2 + a2 . 3a = ______ ___ _____ __

3 )[

__

(

c. a . 33 a – __81 a4 3

____

__

]

3

__

a .33 a F

–2

d. ( 3128 : 32 ) . 3–1 + ( 3m5 : 3381m5 ) = 5 V 3

3

3

4

__

c. a . ( 33 a )2

__

3 + 3w

__ __

X Ninguna de las anteriores. 25w + 1 + 10 . 3w _________ ______ __

1 3a . 3__a . 3a – ____ b. ___________ 12 __ = 6 3a

3a

a3 . 3a–7 a

–a

11 ___ 12

a X Ninguna de las anteriores. a24 – ___ a 17 ___

__

1__ 3b – ___ b = 3 ________ __ c. 1 –__3b ______ 3b

31 024 c d __ 1 __+ 37 = c. _______ 37 – 1

____

5 5 3 4 __ 4 . 3c d

1 . (1 + __ __ 37 )2 6 ___ ____ ____ __ 1 ___ – 3500 + __ + 345 3125 ___ d. ___________________ = 62 . (25 + 35 ) 1 – 35 – 380

2 – 3__ 2 3_______ __ e. ________ = 32 – 1 32__+ 32 __ __ __ 2 y – 3 . 32 (____________ __ 3y – 3 . 32 ) __ = f. – 3__________ 3 . 32 + 3y 18 – y

__

__

2

(32 + __1 ) a. _________ – 5 . 32 + 7 = 4 – 8 . 32 1 – 32

__

__

__

22 . 33 – 10 3 .__36 + 32 __ b. ______________ = ___________ 13 1 1 __ __

32 + 3 . 36 2

¿Cuál es el resultado de cada una de las siguientes expresiones?

X Ninguna de las anteriores. __ –1 – 3b

__

5 c3 d2 _________ b. __________ = 5 12 6

66. Marquen las opciones correctas.

1 – 3b

+ 3b

3y

1 __

__

3a

__

y 4 __ = y2 . y3 a. ___ 4 3

__ ( 8__2 + 1 ) c. ________ = 11 – 6 . 32 (32 + 1)2

__

(

__

64. Racionalicen las siguientes expresiones.

4.

–1 + 3b

__

__

a. 17 + 12 . 32 ; b. 6; c. 11 + 12 . 32 ; d. 102 + 72 . 32

expresión. _____

11 ___ 12

32 __ __ d. ________

d. c2 . ( __c1 + c ) =

__

65. Resuelvan hasta reducir a su mínima

___ 12

__ . 3a

b. __c1 + c =

_______ __

3 – 3w

3

c. c2 – ( __c1 + c ) =

a. c2 =

3

62. Marquen las opciones correctas. ¿Cuál es la expresión equivalente a la dada en cada caso? __ __ a. (4 . 3w + 3w + 1)2 =

3

__

1__+ 32 c = _______

2 . 32

b. ( 345 + 35 ) . ( 3 . 35 – 3(–2)2 + 1 ) = 40 V ____

pedido en cada caso.

__

_______________ __

75 . (3 . 33 – 1)2 ___ a. 3_______________ 6 1 + 327

2

)

__

__

–25 . 33 + 30 __

2

2____________ . ( 3a – 3b ) ( a – b )2

X 25 . 33 – 30 ___

+ 2b – 4 . 3ab ________________ X 2a 2 (a – b)

Ninguna de las anteriores.

Ninguna de las anteriores. _______ __

1 __+ 35 b. 2 . ______

3

__

35 – 1 ______ __

36 + 32 . 35 _________ __

36 + 2 . 35

__

X Ninguna de las anteriores. 1 + 35

31

capítulo

1

AUTOEVALUACIÓN Marquen las opciones correctas en cuenta___los valores de x e y. Luego, respondan. 67. Tengan __ x = 2 . 311 + 1; y = 344 – 1 ¿A qué conjunto numérico pertenece x + y? X a. Irracional.

b. Racional.

c. Entero.

68. ¿Cuáles expresiones son verdaderas? a. Si a < 0. |

a| = a

X | a | = –a

|

X | –2,3 + b | ) | –2,3 | + | b |

| –2,3 + b | < | –2,3 | + | b |

a| < a

b. Si b < 0. X | –2,3 + b | = | –2,3 | + | b |

69. ¿Cuántas soluciones tiene la siguiente ecuación? | 2x

+ 1 | + x = 3x b. S: x = – __41 .

X a. No tiene solución.

c. S: x = __41 .

70. ¿Cuál__es el conjunto solución en _______ cada _____caso? 2 –2 | | a. 4 + 35 – (2x – 4 ) ) 2 – x + 33(5–2)–2 97 S: ___ 48 ;+'

[

X

97 33 ___ S: –';___ 48 F 16 ;+'

] [

(

) __

___

)

Ninguna de las anteriores.

___

b. –16 . | x – 4 | – 7 . 33 > 375 + 348 __

__

__

X S = (4 – 33 ;4 + 33 )

__

S = (–4 – 33 ;–4 + 33 )

Ninguna de las anteriores.

71. ¿Cuál es la expresión equivalente a cada una de las siguientes? ________

3 4 ______

a. 33 a6 . b23 = 12

___

__

__

__

5

5

__

7 5 __ 2 . 3h 3

12

__

__

X 4 . 3h – __1 . h . 3h 2 5

___________

___

a . 3b11

__

b. 3h – __21 . 3h6 + 3 . 3h = 5

___

X b . 3a . 123b11

ab . 3b11

_____

5

__

5

5

_____

c. 3a2 + 2ax + x2 . 3a + x : 36 a + x = 3

______

3

3a2 + x2 ____

___

______

X a + x

3a2 – x2 ___

___

3112 – 3448 +____ 375 – 312 d. _______________________ = 1 __ 3 ___ 72 + 3327 ___

–37 + 15 . 321 X ____________ 4

__

___

15 –37 + ___ 4 . 321

___

–37 – 15 . 321 ____________ 4

___

( 2 – 3x )2 + 316x – 2x __ e. ____________________ = 2 – 3x

__

X 2 + 3x

32

__

2 – 3x

__

4 + 3h – __21 . h . 3h

__

–2 + 3x

Contenidos

2

8. Sucesiones. 9. Sucesiones aritméticas. 10. Sucesiones geométricas. 11. Análisis de sucesiones. 12. Clasificación de sucesiones.

El filósofo griego Zenón de Elea imaginó una supuesta carrera con un resultado muy curioso. El mundo antiguo se ve alborotado por un evento deportivo fuera de lo común: juegan una carrera Aquiles, el más veloz de los hombres, y una tortuga. Nobleza obliga: el glorioso héroe concede al animalito una ventaja inicial antes de comenzar a correr... y, ante la sorpresa de todos, nunca lo alcanza. Es que cuando Aquiles llega al punto del que parte la tortuga, ella, lenta pero persistente, se ha movido un poco; nuevamente Aquiles intenta llegar a ella, pero la tortuga se ha movido un poco más, y así sucesivamente. Zenón no estaba interesado en lides deportivas, sino más bien en refutar el pensamiento filosófico de la época. No se trata de negar el hecho “evidente” de que Aquiles efectivamente alcanza a la tortuga; lo que está en juego es la posibilidad de dividir infinitamente el espacio y el tiempo. Los argumentos parecen sencillos, pero motivaron siglos de una discusión que recién empezó a resolverse con la aparición del moderno concepto de límite.

1. Lean atentamente y respondan. a. En un texto del escritor Jorge Luis Borges, Aquiles corre diez veces más rápido que la tortuga y parte diez metros atrás de ella. ¿Cuánto recorre para alcanzarla? b. ¿Por qué creen que este problema se discutió durante muchos siglos? ¿No es evidente que Aquiles alcanza a la tortuga? a. Se puede pensar como 10 + 1 + 0,1 + 0,01 + ... = 11,1111... b. El problema es “filosófico” y tiene que ver con la manera de pensar el tiempo y el espacio. Para los antiguos, el infinito matemático no era un tema fácil de pensar: pasó mucho tiempo hasta que se desarrollaron las ideas matemáticas necesarias para plantearlo en forma rigurosa.

capítulo

Sucesiones

8

7

9

10

11

12

13

14

15

16

17

Sucesiones ¿Para qué sirve?

INFOACTIVA

PÁGINA 3

Una sucesión es un conjunto ordenado de números, uno a continuación del otro. El conjunto de los números naturales es una sucesión de infinitos elementos. Se denomina término a cada uno de los elementos de la sucesión.

1;

8;

27;

64;

125;

216;













a1

a2

a3

a4

a5

a6

n3

...



...

an

En algunas sucesiones se puede encontrar un término general an (término enésimo), que es la fórmula de un término cualquiera en función del lugar que ocupa.

En la sucesión 1; 8; 27; 64; 125; 216;…, el término general de la sucesión es an = n3. Si se conoce el término general, se puede hallar la sucesión, o cualquier término de la misma, reemplazando en forma consecutiva los números naturales en el valor n del término general. 1 , entonces la sucesión será: 1; __ 1 ; __ 1 ; __ 1 ; __ 1 ; __ 1 ;... ; __ 1 ;... Si el término general de una sucesión es an = __ n n 2 3 4 5 6

Por lo tanto una sucesión es una función que le asigna a todo número natural un número real. f:



Sucesiones aritméticas Se denomina sucesión aritmética a aquella en la cual cada término de la misma se obtiene sumando al anterior un número constante r llamado razón aritmética.

4

12 4+8

12 + 8

20 20 + 8

28

36 ...

Sucesión aritmética con r = 8.

28 + 8

Para que una sucesión sea aritmética, debe verificarse que: a2 – a1 = a3 – a2 = … = an – an–1 = r

Sucesiones geométricas Se denomina sucesión geométrica a aquella en la cual cada término de la misma se obtiene multiplicando el anterior por un número constante q llamado razón geométrica.

3

–9 3 . (–3)

27

–81

243 ...

Sucesión geométrica con q = –3.

–9 . (–3) 27 . (–3) 81 . (–3) a

a

a

n 2 __3 ___ Para que una sucesión sea geométrica, debe verificarse que: __ a = a = … = a = q ⇔ a1 ≠ 0 1

34

2

n–1

Test

de comprensión

1. Respondan y expliquen las respuestas. a. ¿Se puede definir una sucesión numérica como una función f: → ? b. Una sucesión numérica ¿puede ser aritmética y geométrica simultáneamente? a. No, porque el primer conjunto determina el lugar que ocupa cada término, por lo tanto el conjunto debe ser los números naturales. b. Sí. Por ejemplo, la sucesión 0, 0, 0, 0, 0…

ACTIVIDADES Sucesiones

8

1. Escriban 3 términos más para cada sucesión y el término general cuando sea posible. __

__

__

__

__

__

a. 3, 9 ,15, 21, 27, 33 , 39 , 45 , …

d. 1, 32 , 33 , 2, 35 , 36 , 37 , 38 , 3 , …

Término general: an = –3 + 6n

Término general:

7 8 9 b. __32 , __43 , __54 , __65 , __76 , , , ,… 8 9 10

1 1 1 1 e. __21 , __41 , __81 , ___ , , ,… 16 , 32 64 128

__

n+1 an = _____ n+2

Término general:

c. 9, –9, 9, –9, 9, –9, 9

1 an = __ 2

( )

Término general:

f. 0, 3, –1, 4, –25, 1 000,

, –9 , 9 , …

Término general: an = 9 . (–1)

an = 3n

n–1

n

,

,

,…

Término general: No es posible.

2. Escriban los 5 primeros términos de estas sucesiones y también el término de orden 60. 89 7 __ 13 1, 1, __ , 5 , ___ ; a = _______ 108 000 9 8 25 60

–2 ______ a. an = 3n n2

b. bn = –n

–1, –8, –27, –64, –125; a60 = –216 000

3

4; 6; 4; 6; 4; a60 = 6

c. cn = 5 + (–1)n d. dn = 1 + __n1

(

n

)

9 ___ 625 _____ 61 2, __ , 64 , ____ , 7 776 ; a = ___ 4 27 64 3 125 60 60

( )

60

3. Tengan en cuenta la siguiente sucesión, grafiquen el término a6 y resuelvan.

1

3

6

10

15

Encuentren una forma sencilla de calcular la cantidad de elementos que tendrán los siguientes términos. Para encontrar cada término, se suma n al término anterior. Por ejemplo, a6 = 15 + 6 = 21.

4. Indiquen si las siguientes sucesiones son aritméticas o geométricas. Escriban la razón según corresponda. a. 2, 10, 18, 26, 34,... Aritmética; r = 8

b. 0,5; 0,25; 0,125;… Geométrica; q = 0,5

c. an = 9 – 5n Aritmética; r = –5

d. 3, 7, 10, 17, 27, 44,… Ninguna de ellas. n e. bn = 3 . __21 )

(

1 Geométrica; q = __ 2

f. 1, 2, 3, 4, 5, 6,... Aritmética; r = 1 35

9

8

10

11

12

13

14

15

16

17

18

Sucesiones aritméticas ¿Para qué sirve?

INFOACTIVA

PÁGINA 4

En una sucesión aritmética cada término se obtiene sumándole al anterior un valor constante r. a1 = a1 + 0r a2 = a1 + 1r a3 = a2 + r = a1 + r + r = a1 + 2r El término general an es: a4 = a3 + r = a1 + r + r + r = a1 + 3r an = a1 + (n – 1) . r an = an–1 + r = a1 + r + r + ... + r = a1 + (n – 1) . r n – 1 veces

ˆPara calcular un término determinado de una sucesión aritmética conociendo dos términos consecutivos, se deben seguir estos pasos.

Calculen a6 si a1 = –3 y a2 = 5. 1. Se halla la razón. r = a2 – a1 ⇒ r = 5 – (–3) ⇒ r = 5 + 3 ⇒ r = 8 a6 = a1 + (6 – 1) . r ⇒ a6 = –3 + (6 – 1) . 8 ⇒ a6 = –3 + 5 . 8 ⇒ a6 = 37 2. Se calcula cada término. La razón es igual a la diferencia entre dos términos consecutivos: r = ak – ak–1 ∧ k ∈

– {1}

ˆ Para calcular un término determinado de una sucesión aritmética conociendo otro término y la razón, se deben seguir estos pasos.

Calculen a3 si a9 = –28 y r = –9 Se considera a a3 como primer término (a3 → a1) y a a9, por lo tanto, como séptimo (a9 → a7). an = a1 + (n – 1) . r ⇒ a7 = a1 + (7 – 1) . r ⇒ –28 = a1 + 6 . (–9) ⇒ a1 = –28 + 54 ⇒ a1 = 26 → a3 = 26 ˆPara calcular el número de términos de una sucesión aritmética, se deben seguir estos pasos.

Calculen el número de términos de la sucesión sabiendo que a1 = 5; a2 = 19; …; an = 145. r = a2 – a1 ⇒ r = 19 – 5 ⇒ r = 14 an = a1 + (n – 1) . r ⇒ 145 = 5 + (n – 1) . 14 ⇒ 140 = (n – 1) . 14 ⇒ 10 = n – 1 ⇒ n =11 Suma de los términos de una sucesión aritmética La suma de los n primeros términos de una sucesión aritmética se obtiene de la siguiente manera. a1; a2; a3; ... an–2; an–1; an a1 + 2r + an – 2r = a1 + an a1 + r + an – r = a1 + an

La suma de los n primeros términos es: ( a1 + an ) . n Sn = ___________ 2

a1 + an ˆ Para calcular la suma de los términos de una sucesión aritmética se deben conocer el primer término, el último y la cantidad de términos.

Calculen la suma de todos los números pares comprendidos entre 46 y 112, inclusive. 112 = 46 + 2 . (n – 1) ⇒ 66 = 2 . (n – 1) ⇒ 33 = n – 1 ⇒ n = 34 (

)

46 + 112 . 34 ⇒ Sn = 158 . 17 ⇒ Sn = 2 686 Sn = ______________ 2

36

Test

de comprensión

1. Respondan y expliquen las respuestas. a. En una sucesión aritmética, si a1 = 12 y a3 = 0, ¿cuál es la razón? b. ¿Cómo se puede expresar la razón de una sucesión aritmética en función de x e y con a7 = x y a8 = y? a. r = –6; b. r = a8 – a7 = y – xy

9

ACTIVIDADES Sucesiones aritméticas

5. Completen con el dato que falta en cada caso. 12 a. a1 = – ___ 5; r = 5

a12 =

52,6

83 c. an = 153,32; a1 = ___ 25 ; r = 25

b. a120 = 1 345; r = –9

a1 =

2 416

n=

7

6. Calculen la suma de los 30 primeros términos. a. Dada la sucesión aritmética cuyo término general es: an = –12 + 5n. S30 = 1 965

b. Dada la sucesión: –4, –10, –16, –23, … S30 = –2 730

7. Resuelvan teniendo en cuenta que en una sucesión aritmética a1 + a3 = 18 y a5 – a2 = –6. a. ¿Cuál es la razón? r = –2

b. Calculen a1, a2 y a3. a1 = 11; a2 = 9; a3 = 7

8. Tengan en cuenta los datos y resuelvan. a1 = x; r = x – 3 con x D a. ¿Cuál es la expresión correspondiente a S10? (9x + 27) . 5

9x – 135

X 55x – 135

b. Calculen S10 si la diferencia entre dos términos consecutivos es 13. 745

9. Lean atentamente y resuelvan. Franco decidió ahorrar dinero para comprarse una notebook. Si empezó reservando $1 000 y cada mes agrega $260, ¿cuánto dinero tendrá después de un año? 1 000 + 12 . 260 = 1 000 + 3 120 = 4 120

Luego de un año tendrá $4 120. 37

10

9

11

12

13

14

15

16

17

18

19

Sucesiones geométricas INFOACTIVA En una sucesión geométrica cada término se obtiene multiplicando el anterior por un valor constante q. a1 = a1 . q0 a2 = a1 . q1 a3 = a2 . q = a1 . q . q = a1 . q2 El término general an es: a4 = a3 . q = a1 . q2 . q = a1 . q3 an = a1 . qn–1 an = an–1 . q = a1 . q . q . q . q ... q = a1 . qn–1 n – 1 veces

ˆ Para calcular un término determinado de una sucesión geométrica conociendo dos términos consecutivos, se deben seguir estos pasos.

Calculen a6 si a1 = 2 y a2 = 8. a2 8 __ q = __ a1 ⇒ q = 2 ⇒ q = 4 a6 = a1 . q6–1 ⇒ a6 = 2 . 45 ⇒ a6 = 2 . 1 024 ⇒ a6 = 2 048

1. Se halla la razón. 2. Se calcula cada término. a

k La razón es igual al cociente entre dos términos consecutivos: q = ___ ak–1 ∧ k ∈

– {1}

ˆ Para calcular un término determinado de una sucesión geométrica conociendo otro término y la razón, se deben seguir estos pasos.

Calculen a4 si a11 = 8748 y q = 3. Se considera a a4 como primer término (a4 → a1) y a a11, por lo tanto, como octavo (a11 → a8). 8 748 ⇒ a = 4 → a = 4 an = a1 . qn–1 ⇒ a8 = a1 . 38–1 ⇒ 8748 = a1 . 37 ⇒ 8748 = a1 . 2187 ⇒ a1 = ______ 1 4 2 187 ˆ Para calcular el número de términos de una sucesión geométrica, se deben seguir estos pasos. 3 ; a = __ 8. 1 ; ...; a = ___ Calculen el número de términos de la sucesión geométrica sabiendo que a1 = __ n 4 2 2 81 1 __

a

2 2 2 __ __ q =___ a ⇒q= 3 ⇒q=3 1

__ 4

3 . __ 8 = __ 2 an = a1 . qn–1 ⇒ ___ 81 4 ( 3 )

n–1

n–1

32 = __ 2 ⇒ ____ 243 ( 3 )

5

n–1

2 = __ ⇒ ( __ ( 23 ) 3)

⇒n–1=5⇒n=6

Suma de los términos de una sucesión geométrica La suma de los n primeros términos de una sucesión geométrica se obtiene de la siguiente manera: Sn = a1 + a1 . q + a1 . q2 + a1 . q3 + ... + a1 . qn–1 (1)



Sn . q = a1 . q + a1 . q2 + a1 . q3 + ... 2

3

Sn = a1 + a1 . q + a1 . q + a1 . q + ... Sn . q – Sn = –a1

+ a1 . qn–1 + a1 . qn (2) n–1

+ a1 . q

+ a1 . qn

Se multiplican ambos miembros de (1) por q.

(1)

Se resuelve (2) – (1) qn – 1

Sn . q – Sn = –a1 + a1 . qn ⇒ Sn . (q – 1) = a1 . (–1 + qn) ⇒ Sn = a1 . ______ q–1 ∧q≠1

38

Test

de comprensión

1. Respondan y expliquen las respuestas. a. Si en una sucesión geométrica a4 = 8 y a5 = 4, ¿es cierto que la razón es 2? b. Los términos de una sucesión geométrica de término general an = –2 . (–3)n–1, ¿son todos negativos? 1 1 a. No, la razón es __ 2 ; b. No, por ejemplo, para n = 2, a2 = –2 . (–3) = 6.

ACTIVIDADES Sucesiones geométricas.

10

10. Completen con el dato que falta en cada caso. a. a1 = –9; q = 2

a11 =

–9 216

b. a25 = 122; q = __31

c. an = 2 048; a1 = 4; q = 2

a1 = 3,45 . 10–13

n=

10

11. Calculen la suma de los 11 primeros términos. a. Sea la sucesión geométrica de término general an = 3 . 2n–1. 6 141

b. Sea la sucesión geométrica 3; 12; 48; 192; 768; … 4 194 303

12. Tengan en cuenta que las sucesiones son geométricas y resuelvan. a. Encuentren 4 términos entre el término de valor 2 y el término de valor 486 de una sucesión. ____

5 486 Serán 6 términos en total contando los del enunciado. Se calcula q = ____ 2 = 3; luego, los términos son 6, 18, 54, 162.

3

b. En una sucesión de razón 1,5, la suma de los dos primeros términos es 1,7. Calculen la suma de los 5 primeros términos. a1 + a2 = a1 + 1,5 . a1 = 1,7 Luego, a1 = 0,68 y s5 = 8,9675

c. El producto entre el segundo y el tercer término de una sucesión es 5 400. Calculen la razón y la suma de los 12 primeros términos, si el primer término es 5. a2 . a3 = ( a1 )2 . q3 = 5 400 luego q = 6 y la suma s12 = 612 – 1

13. Marquen las opciones correctas. a. Si en una sucesión geométrica el primer término es 2 y su razón __21 , ¿cuál es el producto a20 . a21? X 2 . ( __1 )39 2

2 . ( __41 )

39

4 . ( __21 )

39

b. Si en una sucesión geométrica a1 = x con x > 1 y q = a1, ¿cuál es la expresión de S5? 5 x_____ –1 x–1

6 –x X x_____

x–1

5 x_____ –x x2 – x

39

11

10

12

13

14

15

16

17

18

19

20

Análisis de sucesiones INFOACTIVA Existen muchas situaciones en las cuales puede aplicarse el concepto de sucesión para resolver diversos problemas de índole práctica.

Cotas superiores e inferiores Una sucesión está acotada superiormente, si ∃ k ∈ / ∀ n ∈ : k ≥ an. Se dice que k es cota superior en la sucesión. Una sucesión está acotada inferiormente, si ∃ k ∈ / ∀ n ∈ : k ≤ an. Se dice que k es cota inferior en la sucesión. El supremo es la menor de las cotas superiores de una sucesión y el ínfimo es la mayor de las cotas inferiores de una sucesión. Una sucesión es monótona creciente, si ∀ n ∈ : an ≥ an–1. Una sucesión es monótona decreciente, si ∀ n ∈ : an ≤ an–1.

Si el término general de una sucesión es 2 , entonces la sucesión será: an = __ n

y 2

2 ; __ 2 ; __ 2 ;... 1 ; __ 1 ; __ 2; 1; __ 3 2 5 3 7

1

En el gráfico de la derecha están representados los valores que toma la sucesión.

1

0

2

3

4

5

6

7

x

ˆEsta sucesión está acotada superior e inferiormente. ˆEn este ejemplo, 2, 4, 8, 100, e, π, son cotas superiores y 0, –1, –3, – __43 , son cotas inferiores. ˆEl supremo es 2 y el ínfimo es 0. ˆTambién se afirma que esta sucesión es monótona decreciente, al ser cada término menor que el anterior. y

Si el término general de una sucesión es an = 2 – (–1)n, entonces la sucesión será:

3 2

3; 1; 3; 1; 3; 1; 3; 1; 3;…

1

En el gráfico de la derecha están representados los valores que toma la sucesión.

0

1

2

3

ˆEsta sucesión está acotada; admite cotas superiores y cotas inferiores. ˆEl supremo es 3 y el ínfimo es 1. ˆEsta sucesión no es ni monótona decreciente ni monótona creciente.

{

3 si n es impar

ˆOtra manera de definir esta sucesión es an = 1 si n es par 40

4

5

6

7

8

x

Test

de comprensión

1. Respondan y expliquen las respuestas. a. La sucesión 3, 3, 3, 3, 3,… ¿es monótona creciente o decreciente? b. Para que una sucesión tenga ínfimo, pero no supremo, ¿qué condición debe cumplir? a. Las dos, porque en ambas definiciones se pide que cada término sea menor o igual, o mayor o igual; en ambos casos cumple con la igualdad. b. Tiene que estar acotada inferiormente y no acotada superiormente.

11

ACTIVIDADES Análisis de sucesiones

14. Indiquen si las sucesiones son monótonas crecientes o decrecientes. Expliquen las respuestas. –1 a. an = n_____ n

c. an = (–1)n . n

1 __ 2 __ 3 Monótona creciente, ya que 0 ) __ 2 ) 3 ) 4 ...

Ninguna de ellas, porque la sucesión es –1; 2; –3; 4; –5;...

d. a1 = 2 y an = a1 – an–1 para n ≥ 2.

b. an = –2n Monótona decreciente, ya que –2 * –4 * –6 * –8...

Ninguna de ellas, porque la sucesión es 2; 0; 2; 0; 2; 0;...

15. Indiquen si las sucesiones están acotadas. Escriban tres cotas inferiores y/o tres superiores, el ínfimo y el supremo, cuando sea posible. Pueden ayudarse realizando un gráfico en sus carpetas. a. 1; 0,8 ; 0,5 ; 0,1 ; 0,01 ; 0,001; 0,0001,… Está acotada entre 0 y 1. Por ejemplo: 0, –1, –10 son cotas inferiores y 1, 2, 10 000 son cotas superiores. El 0 es ínfimo y el 1 es supremo.

b. 4, 0, –4, –8, –12, –16, –20… Está acotada superiormente. Por ejemplo: 4, 17, 22,5 son cotas superiores. El 4 es supremo. No tiene ínfimo.

c. 5, –5, 5, –5, 5, –5… Está acotada entre –5 y 5. Por ejemplo: –6, –80, –100 000 son cotas inferiores y 5, 7, 12 340 000 son cotas superiores. El –5 es ínfimo y el 5 es supremo. 1

16. Tengan en cuenta la sucesión definida por la fórmula bn = (1 + n)__n, completen la tabla y respondan. b1 2

b2 1,732

b3

b4

b10

b101

b1 000

b10 002

1,587

1,495

1,271

1,047

1,007

1,001

a. La sucesión ¿es monótona creciente, monótona decreciente o ninguna de ellas? La sucesión es monótona decreciente.

b. Si n crece infinitamente, los términos de esta sucesión ¿a qué número se aproximan? Los términos se aproximan a 1.

c. ¿Está acotada? ¿Entre qué valores están comprendidos todos los términos de la sucesión? Sí; la sucesión está acotada y para todo n, 1 ) bn ) 2.

41

12

11

13

14

15

16

17

18

19

20

21

Clasificación de sucesiones INFOACTIVA Sucesiones convergentes Cuando a medida que n crece, los términos de la sucesión se van acercando a un número, se dice que la sucesión es convergente. n–1

1 Si el término general de una sucesión es an = ( __ , entonces la sucesión será: 2) 1 ; __ 1 ; __ 1 ; ___ 1 ; ___ 1 ; ___ 1 ;… 1; __ 2 4 8 16 32 64

A medida que crece, la sucesión se va acercando a 0. Se dice que la sucesión converge a 0 o que su límite es 0. Sucesiones divergentes Cuando a partir de un n ∈ , los módulos de los términos de la sucesión son mayores que cualquier número positivo k, la sucesión es divergente.

Si el término general de una sucesión es an = 2n, entonces la sucesión será: 2; 4; 6; 8; 10; 12; 14;... La sucesión diverge porque para cualquier número positivo k, existe un n tal que an > k. Para k = 2 300 000, existe n = 1150 001, tal que an = 2 300 002 > k A medida que crece, la sucesión se va acercando a valores cada vez más altos. Se dice que la sucesión diverge a +∞ o que su límite es +∞. Sucesiones oscilantes Cuando una sucesión no es convergente ni divergente, es oscilante.

Si el término general de una sucesión es an = 2 – (–1)n, entonces la sucesión será: 3; 1; 3; 1; 3; 1; 3;... Esta sucesión oscila entre los números 1 y 3. En este caso, la sucesión no tiene límite. Por lo tanto, es oscilante. Sucesiones definidas por recurrencia Una sucesión está definida por recurrencia si para definir el término enésimo se utilizan términos anteriores de la sucesión.

a1 = 1; a2 = a1 + 2; a3 = a2 + 3; …; an = an–1 + n ‰ a2 = 1 + 2; a3 = 1 + 2 + 3; a4 = 1 + 2 + 3 + 4 En general, tenemos que an es la suma de los n primeros números naturales. n . (n – 1)

En este caso, se puede expresar con la fórmula: an = ________ 2 42

Test

de comprensión

1. Respondan y expliquen las respuestas. a. Si una sucesión tiene términos positivos, ¿se puede asegurar que es convergente a un número real? b. ¿Qué condiciones debe cumplir una sucesión para ser divergente? ¿Y para ser oscilante? a. No. No asegura que sea convergente. Para serlo, a medida que n crece infinitamente, sus términos deben acercarse a un número real. b. Para ser divergente, al crecer n infinitamente, el valor absoluto de sus términos deben ser mayores que un número real positivo muy grande. Para ser oscilante, no debe ser convergente ni divergente.

12

ACTIVIDADES Clasificación de sucesiones

17. Clasifiquen las sucesiones en convergentes, divergentes u oscilantes. Si es posible, calculen su límite. a. 5; 4,5; 4, 3,5; 3,3; 3,1; 3,01; 3,001; 3,0001… Sucesión convergente y su límite es 3.

b. 10; 100; 1 000; 10 000; 100 000; 10 000 000;… Sucesión divergente y su límite se aproxima a infinito positivo.

c. –2; 3; –2; 3; –2; 3; –2; 3; –2; 3;… Sucesión oscilante y no existe su límite.

18. Clasifiquen las sucesiones en convergentes, divergentes u oscilantes. Si es posible, calculen su límite. a. an = __32

( )

n

4 ___ 2 ; __ Sucesión convergente y su límite es 0, pues su desarrollo es __ ; 8 ;... 3 9 27

b. bn = n2 – 3n Sucesión divergente y su límite se aproxima a infinito positivo, pues su desarrollo es –2; –2; 0; 4; 10; 18; 28; 40;… –1 ______ c. cn = 2n n+1

5 __ 7 __ 3 __ 1 11 __ Sucesión convergente y su límite es 2, pues su desarrollo es __ 2 ; 1; 4 ; 5 ; 2 ; 7 ;...

d. dn =

{

3 si n es par 5 si n es impar

Sucesión oscilante y su límite no existe, pues su desarrollo es 5; 3; 5; 3; 5; 3;...

19. Indiquen para qué valores de t las sucesiones son convergentes, divergentes u oscilantes. a. t; t2; t3; t4; t5;... La sucesión es convergente para –1 < t < 1, divergente para t ≥ 1 y t < –1. Es oscilante para t = –1.

b. t2; t6; t10; t14;...

La sucesión es convergente para –1 ≤ t ≤ 1. Es divergente para t < –1 y t > 1. No es oscilante para ningún valor de t.

2 3 4 c. _1t ; ( _1t ) ; ( _1t ) ; ( _1t ) ;...

La sucesión es convergente para t ≥ 1 ∧ t < –1 y divergente para –1 < t < 1 ∧ t ≠ 0. Es oscilante para t = –1.

43

12

ACTIVIDADES Clasificación de sucesiones

20. Propongan para cada caso, si es posible, el término general de una sucesión que cumpla las condiciones pedidas. a. Una sucesión de términos negativos que sea divergente. Por ejemplo, an = –3n, pues sus términos son –3; –6; –9;...

b. Una sucesión de términos positivos que sea convergente a 2. 199

5 1 __ ____ Por ejemplo, bn = 2 – __ n , pues sus términos son 1; 1,5; 3 ;… ; 100 ;…

c. Una sucesión oscilante que cumpla que para todo n: –2 ) an ) 2. gn = 2 . (–1)n es oscilante, sus términos son –2, 2, –2,... y está acotada entre –2 y 2.

d. Una sucesión convergente en 3 y que sea monótona decreciente. 5 __ 5 __ –1 8 8 ______ __ __ Por ejemplo en = 3n n sus términos son 2; 2 ; 3 ;… es monótona decreciente, pues 2 ≤ 2 ≤ 3 ≤ ...

e. Una sucesión convergente a 0 que cumpla que para todo n: 0  an ) 1. 1 1 __ 1 __ 1 1 __ ____ Por ejemplo: fn = __ n sus términos son 1; 2 ; 3 ; 4 ;… ; 100 ;…

21. Escriban V (Verdadero) o F (Falso). Expliquen las respuestas donde escribieron F. a. Una sucesión de términos negativos siempre es convergente. F Por ejemplo, an = –n tiene términos negativos y es divergente. 2 b. La sucesión an = __ es convergente a 0. n3

V

c. Una sucesión oscilante tiene todos sus términos positivos. F Por ejemplo, an = (–1)n.

22. Tengan en cuenta las siguientes sucesiones y resuelvan. 1 2 an = _____ n + 1 ; bn = n a. Calculen los siguientes términos.

a1 = b1 =

1 __ 2

1

a2 = b2 =

1 __ 3

4

a10 = b10 =

1 __ 11

100

a100 =

1 ___ 101

b100 = 10 000

a1 000 =

1 _____ 1 001

b1 000 = 1 000 000

a10 000 =

1 ______ 10 001

b10 000 = 100 000 000

b. Clasifiquen las sucesiones en convergentes, divergentes u oscilantes. La sucesión an es convergente a 0 y la sucesión bn es divergente a infinito positivo.

c. Obtengan una fórmula para la sucesión cn = an + bn y para la sucesión dn = an . bn. 2

n 1 _____ 2 cn = _____ n + 1 + n y dn = n + 1

d. Clasifiquen cn y dn en convergentes, divergentes u oscilantes. cn y dn son divergentes.

23. Hallen el término general de una sucesión convergente a 3 que sea producto de una sucesión divergente y una convergente a 0. 1 1 __ Por ejemplo, an = 3n . __ n , donde la sucesión 3n es divergente y n es convergente a cero. 44

12

ACTIVIDADES Clasificación de sucesiones

24. Tengan en cuenta la siguiente sucesión definida por recurrencia y resuelvan.

{

a =3

an = a1 = 2a + 1 para n ≠ 1 n n–1 a. Hallen los 10 primeros términos de esta sucesión. Los 10 primeros términos son 3, 7, 15, 31, 63, 127, 255, 511, 1 023, 2 047.

b. ¿Es una sucesión monótona creciente? ¿Por qué? Es una sucesión monótona creciente, porque cada término es menor o igual que el siguiente.

c. Calculen la suma de los términos calculados. S10 = 4 082

25. Tengan en cuenta los primeros siete términos de esta sucesión y hallen una fórmula definida por recurrencia. 3; 16; 82; 412; 2 062; 10 312; 51 562;… a1 = 3; a2 = 16; an = 5an–1 + 2 para n ≥ 3.

26. Resuelvan. Fibonacci, matemático italiano del siglo XIII, descubrió una sucesión que tiene numerosas aplicaciones en biología, en ciencias de la computación, en matemática y en la teoría de juegos. La sucesión es: 1; 1; 2; 3; 5; 8; 13; 21; 34; 55; 89; 144; 233; 377; 610; 987; 1 597; 2 584; 4 181; 6 765;… a. Hallen una fórmula por recurrencia para la sucesión de Fibonacci. f1 = f2 = 1 y fn = fn–1 + fn–2 para n ≥ 2.

b. Construyan una nueva sucesión donde cada término esté formado por cocientes de términos consecutivos de la sucesión de Fibonacci. b1 = 1 : 1 = 1; b2 = 2 : 1 = 2; b3 = 3 : 2 = 1,5;... 1; 2; 1,5; 1,666...; 1,6; 1,625; 1,6153...; 1,61904...; 1,6176...; 1,6181818...; 1,61797...; 1,61805...

c. La sucesión anterior ¿es convergente o divergente? Si es convergente, indiquen a qué número se acercan sus términos. Es una sucesión convergente y se acerca al número irracional 1,618033989..., también llamado número de oro.

27. Tengan en cuenta que el término general de una sucesión aritmética es an = 2 + 3n y resuelvan. Escriban los 5 primeros términos y hallen una fórmula de la misma sucesión, pero por recurrencia. Los 5 primeros términos son 5; 8; 11; 14 y 17. Una fórmula por recurrencia es a1 = 5 y an = an–1 + 3 para n ≠ 1.

mente ACTIVA Una ciclista tarda 40 segundos en dar la primera vuelta a una pista; por los efectos del cansancio, en cada vuelta tarda 6 segundos más que en la anterior. a. Calculen los seis primeros términos de la sucesión que representa los segundos que tarda por cada vuelta. b. Definan la sucesión por recurrencia. ¿Qué tipo de sucesión es? a. Los seis primeros términos son 40; 46; 52; 58; 64; 70. b. Por recurrencia es a1 = 40 y an+1 = an + 6 si n > 1. Es una sucesión aritmética. 45

INTEGRACIÓN 28. Escriban el término general de las sucesiones. Indiquen si son aritméticas o geométricas y hallen la razón. an = n2 – 4 a. –3; 0; 5; __12; 21;…__ __ __ __ b. 33 ; 2 . 33 ; 3 . 33 ; 4 . 33 ;... an = n . 33 n–1 1 1 ___ an = __ c. 0,2; 0,02; 0,002; 0,0002... 5 . 10 an = –7 + 5n d. –2; 3; 8; 13;… an = 2 . 3n–1 e. 2; 6; 18; 54; 162; 486;...

( )

29. Las siguientes sucesiones no son aritméticas ni geométricas. Hallen los términos pedidos. a. 1; 2; 3; 9; 8; 7; 1; 2; 3; 9; 8; 7;... a14 =

2

; a120 =

7

b. 1; 2; 3; 4; 5; 11; 12; 13;…

33. Resuelvan. a. Los ángulos de un triángulo están en sucesión aritmética de razón 30 grados. Calculen cada ángulo. 30°; 60°; 90°. b. Los ángulos de un cuadrilátero están en sucesión geométrica y el último es 4 veces el segundo. Calculen dichos ángulos. 24°; 48°; 96°; 192°.

34. Lean atentamente y resuelvan. En un cuadrado de lado 1 se unieron los puntos medios de sus lados determinando otro cuadrado en su interior. Se repitió el procedimiento en el segundo cuadrado y en los sucesivos como se ve en la figura. __

a. a1 = 4; a2 =__2 . 32 ; a3 = 2; a4 = 32 .

b21 = 41 ; b50 = 95

__

2 b. Geométrica; q = 3___ . 2

30. Tengan en cuenta la sucesión de término – 16 _______ general an = 4n y respondan. n a. ¿Puede tener un término que valga 20? ¿Por qué? No puede valer 20, porque n = –1 y n debe ser natural. b. ¿Para cuál valor de n uno de los términos vale 2,4? Para n = 10.

31. Resuelvan. a. Una sucesión aritmética tiene término general an = –5 + 4n. Calculen la razón, a395 y el orden del término de valor 2 011. b. Una sucesión geométrica tiene a1 = 4 y a2 = 2. Calculen la razón, el término general y a10. c. Una sucesión geométrica tiene a1 = 3, q = 2 y Sk = 1 533. Calculen el valor de k. d. Una sucesión geométrica tiene q = –0,5 y a2 = __43 . Calculen S15. a. 4; 1 575; 504. 1 1 __ b. __ 2; 4 . 2

n–1

( )

. c. 10. d. –10,0000305

32. Los números 2 y 20 son los extremos de una sucesión aritmética de siete términos. Hallen los números que completan la sucesión. 5; 8; 11; 14 y 17.

__

32 c. an = 4 . ___ n–1

2

d. Monótona decreciente.

a. Calculen los primeros cuatro términos de la sucesión formada por el perímetro de cada cuadrado que se va formando. b. La sucesión ¿es aritmética o geométrica? c. Escriban el término general de la sucesión. d. ¿Es monótona creciente o decreciente?

35. Indiquen si estas sucesiones son monótonas crecientes o decrecientes. a. 1,2; 1,23; 1,234; 1,2345; 1,23456... b. 0; 3; 5; 0; 3; 5; 0;… c. 6,7; 6,07; 6,007; 6,0007;… a. Monót, crec.; b. No es monót. c. Monót. decrec.

36. Tengan en cuenta la siguiente sucesión y resuelvan. 5; 5; 4; 4; 3; 3; 2; 2;... a. ¿Es una sucesión monótona decreciente? b. ¿Está acotada superior o inferiormente? c. Hallen, si es posible, tres cotas superiores y tres cotas inferiores. d. ¿Es convergente, divergente u oscilante? a. Sí. b. Superiormente. c. Cotas sup: 6; 8; 9. d. Divergente.

46

capítulo

CONTENIDOS

8*9*10*11*12 37. Clasifiquen las sucesiones en convergentes, divergentes u oscilantes. Luego, indiquen si son monótonas crecientes o decrecientes y si son acotadas inferior o superiormente. –n a. an = 4_____ n+1 3 –n b. bn = n______ 2n

Convergente a –1, monótona 3 decreciente, acotada entre –1 y __ 2. Divergente, monótona creciente, acotada inferiormente. Oscilante, no es monótona, está acotada entre –3 y 3.

2

39. Resuelvan. 1 a. La sucesión an = __ ¿tiene términos negatin2 vos? ¿Es convergente? No. Sí, es convergente a 0. b. Escriban una sucesión divergente a infinito negativo y una oscilante.

Por ejemplo, –1; –5; –9; –13;… es una sucesión divergente y 2; –2; 2; –2;… es oscilante.

40. Tengan en cuenta que el término general de

una sucesión es an = 4 – __2n y resuelvan. 1 a. Escriban los 4 primeros términos y represen__ si n es par 10 _7_ ___ 11 ,… d. dn = n___ 1 , , 18 , __ ten en un sistema cartesiano. 2, 3, ___ 3 2 5 3 2n si n es impar Convergente a 0, no es monótona, está acotada entre b. ¿Es monótona creciente o decreciente? 1 0 y __ 2. c. ¿Está acotada superior o inferiormente? 38. Observen los gráficos y clasifiquen las suced. Si es posible, hallen el ínfimo y/o supremo siones en convergentes, divergentes u oscilantes. de la sucesión. a. y e. Clasifiquen la sucesión en convergente, 1 divergente u oscilante. c. cn = 3 . (–1)

{

n+1

b. Es monótona creciente. c. Acotada entre 2 y 4. d. El ínfimo es 2 y el supremo, 4. e. Convergente a 4.

0,5

41. Hallen los términos 2, 5 y 8 de las siguien0

1

2

3

4

5

x

–0,5

1 __ 1 Oscilante entre – __ 2 y 2.

–1

2

3

4 x

–2 –4 –6 –8 –10 –12 –14 –16

Divergente hacia infinito negativo.

c.

y 3 2 1 0

1

2

3

4

–1

Convergente a 0.

5

{

b1 = –2 b

n–1 bn = ___ 2n si n * 2 1 1 ; b = – _________ 1 __ b2 = – 2 ; b5 = – ____ 960 8 2 580 480

y 1

{

b. bn =

b. 2 0

tes sucesiones. a = 5 a2 = 17; a5 = 53; a8 = 89. a. an = a1 = a + 12 si n * 2 n n–1

x

42. Tengan en cuenta la actividad anterior y marquen las opciones correctas. a. ¿Cuáles son las características de la sucesión de término general an? Es geométrica. X Es aritmética. X Tiene razón 12. Tiene razón 5. X Es monótona creciente. Es monótona decreciente. X Se la puede definir con a = 12n – 7. n Se la puede definir con an = 5n + 7. b. ¿Cuáles son las características de la sucesión de término general bn? X Es convergente a 0. Es divergente. No tiene supremo. X Tiene supremo. X Está acotada. No está acotada.

47

capítulo

2

AUTOEVALUACIÓN Marquen las opciones correctas 43. Respondan. a. ¿Cuáles son los términos a1 y a20 de la sucesión aritmética que tiene r = –6 y S20 = 250? a1 = –69,5 y a20 = 44,5

a1 = –44,5 y a20 = 69,5

X a = 69,5 y a = –44,5 1 20

b. ¿Cuál es el término a2 y la suma de los primeros 11 términos de la sucesión geométrica que tiene a7 = –22 y q = __21 ? a2 = 704 y S11 = 2 814,625

X a = –704 y S = –2 814,625 2 11

a2 = 704 y S11 = –2 814,625

44. Lean atentamente y respondan. Pablo ahorra dinero todas las semanas. La primera semana guardó $20 y cada semana guarda $5 más que la anterior. ¿Cuánto dinero tendrá ahorrado después de 40 semanas? ¿Cuánto tendrá que guardar en la semana número 30? a. En la semana 40 tendrá ahorrados $4 700 y en la semana 30 deberá guardar $160. X b. En la semana 40 tendrá ahorrados $4 700 y en la semana 30 deberá guardar $165.

c. Ninguna de las opciones anteriores es correcta.

45. ¿Cuáles son las características de la sucesión fn =

(–1)n _____ 1 + n?

a. Es una sucesión no acotada. b. Es una sucesión divergente. X c. Es una sucesión que tiene supremo __1 y ínfimo – __1 . 2 3

d. Es una sucesión convergente a – __21 . X e. Es una sucesión convergente a 0.

46. Tengan en cuenta las sucesiones y respondan. – 5n a. an = 9______ 2+n

¿Cuáles son las características de an? X Es una sucesión monótona decreciente. X Es una sucesión convergente a –5.

Es una sucesión oscilante.

{

b =6

b. bn = b1 = b + 2n si n * 2 n n–1 ¿Cuáles son las características de bn? X Es una sucesión divergente.

Es una sucesión acotada superiormente y su supremo es 6. X Es una sucesión acotada inferiormente y su ínfimo es 6.

48

Contenidos

3

13. El conjunto de los números complejos. 14. Módulo de un complejo. Complejos conjugados. 15. Adición y sustracción. 16. Potencias de la unidad imaginaria. Cuadrado y cubo de un complejo. 17. Multiplicación y división. 18. Operaciones combinadas. 19. Ecuaciones.

El italiano Girolamo Cardano es uno de los personajes más pintorescos de la historia de la matemática: fue médico, jugador (según dicen, algo tramposo) y pasó un buen tiempo en prisión acusado de herejía. Pero sus hallazgos matemáticos son muy importantes y variados. Entre otros, en su Ars Magna propone el problema de disociar 10 en dos sumandos cuyo producto sea 40. Si bien aclara que la cuestión es imposible, lo resuelve: su método le permite encon____ ____ trar dos soluciones que escribe como 5 + 3–15 y 5 – 3–15 . Esto no es más que un simple ejercicio, aunque tiene el mérito enorme de ser la primera referencia escrita a los números complejos. Claro que la historia a partir de allí no es sencilla. Tiempo después otro italiano, Bombelli, logró “dar sentido” a las expresiones de Cardano, aunque él mismo reconoció que su razonamiento era “un tanto salvaje”. Más de un siglo después otro grande, el alemán Leibniz, reconoció el valor del número imaginario aunque todavía sin entenderlo del todo: en sus escritos, lo define como “una especie de anfibio entre el ser y el no ser”.

1. Lean atentamente y respondan. a. ¿Por qué les parece que costó tanto asimilar los números complejos dentro de la matemática? b. ¿Cómo se puede verificar que la solución de Cardano es correcta? a. Abierta. Obviamente es chocante pensar en raíces cuadradas de cantidades negativas porque eso no es posible con los números reales. Pero fueron aceptados porque se comprobó que resultaba de gran utilidad para resolver muchísimos problemas concretos. b. Una____ manera sencilla ____ es pensarlo ____como una diferencia de cuadrados: (5 + 3–15 ) . (5 – 3–15 ) = 52 – ( 3–15 )2 = 25 – (–15) = 40.

capítulo

Números complejos

13

12

14

15

16

17

18

19

20

21

22

El conjunto de los números complejos ¿Para qué sirve?

INFOACTIVA

PÁGINA 5

Los números complejos La radicación de base negativa e índice par no tiene solución en el conjunto de los números reales ____ 4 ____ (3–4 ; 3–25 ; 3–16 ; etc.), ya que no existe ningún número real que elevado a una potencia par dé por resultado un número negativo. ___

Se define entonces un nuevo número, llamado i, cuyo cuadrado es igual a –1. Dicho número es la unidad imaginaria en el conjunto de los números complejos. __ 3a

{

__

i = –1 ⇔ 3–1 = i __ = b ⇔ b2 = a (–i) 2 = 1 . i2 = 1 . (–1) = –1 ⇔ 3–1 = –i

___

__

2

3–4 = ± i . 34 = ± 2i

___

__

i2 = –1

__

i = ±3–1

__

3–3 = ± i . 33 = ± 33 i

Representación gráfica y expresión cartesiana de un complejo

b

z = (a;b)

a

Se define al conjunto de los números complejos ( ) como: = {(x;y) ∈ 2 / x ∈ ∧ y ∈ } A cada número complejo le corresponde un punto del plano.

z = (a;b) ← Expresión cartesiana Componente imaginario. Componente real.

Todos los números de la forma (a;0) son números reales y los de la forma (0;b) son números imaginarios puros. Un número real es un número complejo cuya segunda componente es igual a 0. k = (k;0) El número imaginario de segunda componente igual a 1 es la unidad imaginaria. i = (0;1)

Expresión binómica de un complejo Para multiplicar un número complejo por un escalar, se multiplica cada componente del complejo por el escalar.

z = (a;b) = (a;0) + (0;b) = a . (1;0) + b . (0;1) = a + bi ← Expresión binómica Parte imaginaria [Im(z)] Parte real [Re(z)] z1 = (3;4) = 3 + 4i z2 = (0;3) = 3i

50

z3 = (–1;1) = –1 + i z4 = (–2;0) = –2

Test

de comprensión

1. Respondan y expliquen las respuestas. a. El conjunto de los números complejos ¿está incluido en el conjunto de los números reales? b. Los números complejos ¿pueden tener solamente componente real? a. No; b. Sí. La segunda componente es cero.

13

ACTIVIDADES El conjunto de los números complejos

1. Resuelvan las siguientes raíces e indiquen si pertenecen al conjunto de los números reales o complejos. ____

a. 3–25 =

±5i. Complejo

___ 3

5

±9i. Complejo ±35

d. 3–5 =

____

–2. Real

e. 3–32 = __

__

___

–2. Real

b. 3–8 =

____

c. 3–81 =

i. Complejo

±3.

f. 39 =

Real

2. Resuelvan. a. Representen los números complejos en un par de ejes cartesianos. a = (9;7), b = (15;21), c = (0;28)

b. Escriban la expresión binómica de cada número complejo.

b

5

28 c a

2 1

b

21

–5

14

0

–2

1

3

a

7

c 9

4

15

a=

3 + 2i

b=

c=

–5 – 4i

d. (–1;–2) =

–1 – 2i

–2 + 5i

3. Escriban la expresión binómica de los siguientes números complejos. a. (–3;2) =

–3 + 2i

b. (0;5) =

5i

c. (7;0) =

7

4. Escriban la expresión cartesiana de los siguientes números complejos. a. –3 + i =

(–3;1)

b. –i =

(0;–1)

c. 2 – __2i =

( 2;– __21 )

d. 3 =

(3;0)

5. Hallen los números reales x e y que verifiquen las siguientes igualdades. a. (3 + xi) + (3i + y) = 5 + 2 i x = –1; y = 2

b. (3x;5y) = 21 + i 1 x = 7 y = __ 5

c. 5x + 0,5i – (3 – yi) = __31 ;__23

( )

2; y = 1 x = __ 3

d. 3x + 4 . (x – 1) + 2 – __21 i = 7x – 2;– __21

(

)

Infinitas soluciones. 51

14

13

15

16

17

18

19

20

21

22

23

Módulo de un complejo. Complejos conjugados INFOACTIVA Módulo de un complejo

__› A cada número complejo z = (a;b) le está asociado un vector v , con origen en el punto (0;0) y extremo en el punto (a;b). De este modo se puede hacer corresponder un vector a cada número complejo.

z = (a;b)

b →

v

El módulo de ese vector es el módulo del complejo y se representa con la letra ρ. ______ ρ = | z | = 3a2 + b2 _______

_____

a

___

z = 2 + 3i ‰ | z | = 322 + 32 = 34 + 9 = 313

z = (a;b) |z

|

b

ρ

=

Al ángulo ^ φ se lo llama argumento.

φ a

Complejos conjugados

_

Dado un complejo z, se define como su conjugado z al complejo que tiene su parte real igual y su parte imaginaria opuesta. _

z = a + bi ‰ z = a – bi

z = (a;b)

b

Un complejo y su conjugado son simétricos respecto del eje x. __

z1 = 4 + 2i ‰ z1 = 4 – 2i __

z3 = –11i ‰ z3 = 11i

a

__

z2 = 1 – 5i ‰ z2 = 1 + 5i

–b z = (a;–b)

__

z4 = –3 ‰ z4 = –3

Forma polar o trigonométrica de un complejo

{

a ^ cos ^ φ = __ ρ ‰ a = ρ . cos φ b __ ^ sen φ = ρ ‰ b = ρ . sen ^ φ

z = a + bi ‰ z = ρ . cos ^ φ + i . ρ . sen ^ φ ^ ^ z = ρ . (cos φ + i . sen φ ) ← Forma polar o trigonométrica

z = (a;b)

b ρ=

|z

|

φ a

a. Expresen z = 4 +___ 5i en forma trigonométrica. _______el complejo ________ 2 2 ρ = 34 + 5 = 316 + 25 = 341 5 = 1,25 ‰ ^ φ = __ tg ^ ___ φ = arc tg 1,25 = 51º 20’ 24,7’’ 4 z = 4 + 5i = 341 . (cos 51º 20’ 24,7’’ + i . sen 51º 20’ 24,7’’) b. Expresen el complejo z = 8 . (cos 30º + i . sen 30º) en forma binómica y cartesiana. cos 30º = 0,866 ∧ sen 30º = 0,5 ‰ z = 8 . (0,866 + 0,5i) ‰ z = 6,928 + 4i = (6,928;4)

52

de comprensión

Test

1. Respondan y expliquen las respuestas. a. ¿Qué signo le corresponde a ρsi el argumento es negativo? b. El conjugado de un número complejo ¿es igual a su opuesto? a. El módulo es siempre positivo independientemente del argumento. b. No siempre, solo en los casos que los números complejos tengan componente imaginaria.

14

ACTIVIDADES Módulo de un complejo. Complejos conjugados

6. Hallen el módulo de los siguientes números complejos. ___

a. z1 = (–2;3)

|z| =

313

c. z3 = (–4;–3)

|z| =

5

b. z2 = –6 + 8i

|z| =

10

d. z4 = 3 + i

|z| =

310

___

7. Escriban dos números complejos distintos que tengan el mismo módulo. Existen infinitas soluciones, por ejemplo 2 + 3i y –2 – 3i.

8. Hallen el conjugado de cada uno de los siguientes números complejos. a. z1 = 2 – 3i b. z2 = –1 + 13i

__

z1 = __

z2 =

2 + 3i –1 – 13i

z3 = __

d. z4 = 2i

z4 =

9. Expresen en forma trigonométrica. a. z1 = 2 + 2i

__

c. z3 = –2 – 7i

__

–2 + 7i –2i

__

b. z2 = – 32 + 32 i

__

38 . (cos 45° + i . sen 45°)

2 . (cos 135° + i . sen 135°).

10. Expresen en forma binómica. a. z1 = 2 . (cos 60° + i . sen 60°)

b. z2 = 4 . (cos 315° + i . sen 315°)

__

__

1 + 33 i

__

2 . 32 – 2 . 32 i

11. Observen la representación de los números complejos e indiquen su expresión binómica. a.

b.

y 2__ 33 1

y

4 l=

120°

π 6

–1

x

0 __

a=

–1 + 33

x __

b = 2 . 33 + 2i 53

15

14

16

17

18

19

20

21

22

23

24

Adición y sustracción INFOACTIVA Adición y sustracción gráfica de números complejos

y 6

Para sumar gráficamente dos números complejos (z1 + z2), se pueden seguir estos pasos. 1. Se traza la recta paralela al vector asociado a uno de los números complejos (z1 ) que pase por el otro (z2 ). 2. Se traza un vector con origen en z2 con el mismo módulo y sentido que z1. El extremo de ese vector determina z3 (z1 + z2).

z3

5 4

z1

3

z2

2 1

–3

–2

–1

0

1

2

3

4

5

3

4

5

x

y

Para restar gráficamente dos números complejos (z1 – z2 ), se pueden seguir estos pasos. 1. Se traza la recta paralela al vector asociado a uno de los números complejos (z2 ) que pase por el otro (z1 ). 2. Se traza un vector con origen en z1 con el mismo módulo y sentido opuesto a z2. El extremo de ese vector determina z3 (z1 – z2).

z2

3

z1

2 1

–3

–2

–1

0 –1

1

2

x

z3

–2

Adición y sustracción de números complejos Para sumar o restar dos números complejos como pares ordenados, se suman o restan las componentes reales e imaginarios, respectivamente. (a;b) + (c;d) = (a + c;b + d) (a;b) – (c;d) = (a – c;b – d)

(–4;3) + (–2;–5) = [–4 + (–2);3 + (–5)] (–4;3) + (–2;–5) = (–6;–2)

(–4;3) – (–2;–5) = [(–4 – (–2);3 – (–5)] (–4;3) – (–2;–5) = (–2;8)

Para sumar o restar dos números complejos en forma binómica, se suman o restan las partes reales e imaginarias, respectivamente. (a + bi) + (c + di) = (a + c) + (b + d) i (a + bi) – (c + di) = (a – c) + (b – d) i

(–4 + 3i) + (–2 – 5i) = [–4 + (–2)] + [3 + (–5)] i (–4 + 3i) + (–2 – 5i) = –6 – 2i

(–4 + 3i) – (–2 – 5i) = [–4 – (–2)] + [3 – (–5)] i (–4 + 3i) – (–2 – 5i) = –2 + 8i

Adición y sustracción de complejos conjugados

54

La suma de dos complejos conjugados es igual al duplo de la componente real. (a + bi) + (a – bi) = (a + a) + (b – b)i = 2a

La resta de dos complejos conjugados es igual al duplo de la componente imaginaria. (a + bi) – (a – bi) = (a – a) + [b – (–b)] i = 2bi

(5 + 2i) + (5 – 2i) = (5 + 5) + (2 – 2)i = 10

(5 + 2i) – (5 – 2i) = (5 – 5) + [2 – (–2)] i = 4i

Test

de comprensión

1. Respondan y expliquen las respuestas.

__

a. ¿La diferencia entre dos números complejos z1 y z2 es equivalente a la suma entre z1 y z2? b. ¿Qué resultado se obtiene al sumar un número complejo y su conjugado? a. No siempre, solamente si z2 tiene únicamente parte imaginaria. b. El resultado de la suma siempre da un número que se encuentra sobre el eje real.

15

ACTIVIDADES Adición y sustracción

12. Representen el número complejo que falta, sabiendo que z1 + z2 = z3. a.

b. z3

z3 z2 z2

z1

z1

0 0

13. Resuelvan. Verifiquen su resultado realizando la representación gráfica en una hoja. a. (2;3) + (3;–1) =

=

(5;2)

c. (3;1) – (0;–2) =

b. (–1;3) + (7;2) =

=

(6;5)

d. (7;–2) – (7; 3) =

= =

(3;3) (0;–5)

14. Resuelvan cada una de las siguientes operaciones combinadas. a. 3i + 2 + 3i – 5i =

3 11 i c. 1 – __31 i – __41 + i + 5i = __4 + __ 3

(

2+i

b. (3 + 2i) – (5 – 3i) + (– i) =

–2 + 4i

) (

)

21 i d. 3 – 2i – –2 + __51 i + 3 – 2i = 8 – ___ 5

(

)

15. Resuelvan mentalmente. a. (2 + 3 i) + (2 – 3 i) =

4

d. (7 – 2 i) – (7 + 2 i) =

–4i

b. (–5 – i) + (–5 + i) =

–10

e. (1 + 4 i) – (1 – 4 i) =

8i

c. (7 – 5 i) + (5 i + 7) =

14

f. (19 i – 2) – (–2 – 19 i) =

38i 55

INTEGRACIÓN 16. Resuelvan. _____ a. 3–169 = ____ b. 3_____ –12 = c. 3–289 = ___ d. 3121 =

±3i

3

__

22. Hallen un número que cumpla con cada con-

____

e. 3216 = –6 ___

__

±2 . 33 i f. 3–8 = ±2 . 32 i ___ __ ±17i g. 3–7 = ±37 i ____

±11

h. 3–36 = ±6i

17. Hallen la expresión binómica de los siguientes números complejos. a. (–2;7) = –2 + 7i d. (–1;–1)= –1 – i __ 3 __ __ __ 3 b. (0;4) = 4i e. ( 32 ;32 ) = 32 + 32 i c. __31 ;0 = __31 f. (–5;–2) = –5 – 2i

( )

dición. a. La componente real es el triple de la componente imaginaria. Por ejemplo, (15;5) b. La componente imaginaria es el opuesto de 7 y la componente real es la mitad de dicho número. __27 ;–7 c. Un número imaginario puro. Por ejemplo, (0;5) d. Un número complejo que pertenezca a la bisectriz del segundo cuadrante. Por ejemplo, (–4;4) __ e. Un número complejo cuyo módulo sea 37 .

(

)

__

( 36 ;–1 )

18. Hallen la expresión cartesiana de los siguientes números complejos. a. –3 + 5i = (–3;5) d. 7 + 3i = (7;3) __ __ (0;–5) e. –9 + 35 = ( –9;35 ) b. –5i = (2;0) c. –2 + __41 i = –2;__41 f. 2 =

(

números complejos. a. –2 + 4i d. (3;–2) b. –1 – 2i e. (6;1) c. 6i f. __21 ;0

( )

Solución a cargo del alumno.

20. Calculen el___módulo de cada número. 353

___

337

5 c. 5i ___ d. –3 – 3i 318

21. Observen el gráfico y escriban la expresión binómica de cada número complejo.

_

z

–z

–3 + 5i

3 – 5i

–3 – 5i

2+i

–2 – i

2–i

i

)

19. Representen gráficamente los siguientes

a. 7 + 2i b. 6 – i

23. Completen la tabla. z

–i

–i

–1 – 2i

1 + 2i

–1 + 2i

–3 + 4i

3 – 4i

–3 – 4i

24. Lean atentamente y respondan. a. La suma entre un número complejo y su conjugado es 16 y el módulo de uno de ellos es 10. ¿Cuál es ese número complejo? b. La diferencia entre un número complejo y su conjugado es –10 i y el módulo de uno de ellos ___ es 334 . ¿Cuál es ese número complejo? __ c. El módulo de un número complejo es 3 . 38 . Si la componente real y la imaginaria son iguales, ¿cuál es ese número complejo?

a. z = 8 + 6i o z = 8 – 6i c. z = 6 + 6i o z = –6 – 6i b. z = 3 – 5i o z = –3 – 5i

25. Hallen los valores de x e y para que se 5

–2

0

z3

z1

2

–5 z2

cumplan las siguientes igualdades. 1 __ a. (5x + 2;3y – 1) = (7;0) x = 1; y = 3 7 b. 7 – 2x – 5i = 3x – 5i x = __5 c. 2 + __35 x – (3y – 2) i = 2x – 3y i Absurdo. d. 3x + 2 + (2y + 4) i = __31 x + (3y + 2) i + 10 e. 2 + (2 + 5y) i = 5x + 7 + [5y + 3 . (3 – x)] i

(

d. x = 3; y = 2 e. Absurdo.

z1 = 4 + 5i; z2 = –4 – 10i; z3 = 5i

56

)

capítulo

CONTENIDOS

3

13*14*15 26. Expresen los siguientes números complejos

31. Representen el número complejo que falta,

en forma trigonométrica. sabiendo que z1 + z2 = z3. __ a. z1 = 7 . 33 + 7i z1 = 14 . (cos 30° + i . sen 30°) b. z2 = –2i __ z2 = 2 . (cos 270° + i . sen 270°) 3 3 c. z3 = – __41 + 3__ z3 = __1 . (cos 120° + i . sen 120°) 4 2

z2

27. Expresen los siguientes números complejos en forma binómica. __ a. 9 . (cos 30° + i . sen 30°) = 4,5 . 3__3 + 4,5 i b. 4 . (cos 150° + i . sen 150°) = –2 . 33 + 2i __ c. 32 . (cos 225° + i . sen 225°) = –1 –__ i __ d. 18 . (cos 315° + i . sen 315°) = 9 . 32 – 9 . 32 i

28. Resuelvan las siguientes sumas y restas de números complejos en forma cartesiana. 8 7;__ a. (2;3) + 5;– __31 = 3 (–1;3) b. (–1;0) + (0;3) = (0;8) c. (1;4) – (1;–4) = (6;0) d. (7;–8) – (1,–8) = e. (2;–3) + (5;–1) – (–1;–3) = (8;–1) 3 1 ;– __ – __ f. –1;__23 – __41 ;3 + 1;__43 = 4 4 16 __ ___ g. 5;– __31 + __31 ;0 – (0;2) = ;– 7

(

(

(

)

) ( ) ( ) ) ( )

( )

( (3

)

3

)

29. Resuelvan las siguientes sumas y restas de números complejos en forma binómica. a. (–3 + 2i) + (4 – 7i) = 1 – 5i b. –i + (3 – i) = 3 – 2i c. (–2 + 3i) – (5 + 2i) = –7 + i d. (3 + 2i) – (8 – i) = –5 + 3i e. 2 + 3i – (5 – 4i) + (3 – 2i) = 5i f. (1 – i) + (7 – i) – 1 – 3i = 7 – 5i g. (4 – 8i) – 7 + 2i – (–3 – 4i) = –2i

1 z1 –2

–1

0

1

2

3

–1

32. Resuelvan mentalmente. a. (3 + 2i) + (3 – 2i) = b. (–1 + 3i) + ( – 1 – 3i) = c. (3 – 5i) – (3 + 5i) = __ __ __ __ d. ( 32 + 32 i ) – ( 32 – 32 i ) =

6 –2 –10i

__

2 . 32 i

33. Completen con los números que faltan en cada suma.

6 4

z1

2

z2 –2

0

2

4

6

–2

z3

–4

30. Resuelvan en forma gráfica. a. (–2;3) + (1;–2) = b. (–1;–3) – (–2;–1) = c. (–1 + i) + (5 – 3i) = d. –3 – i + (–2 – i) = e. 1 – 2i – (7 + i) = f. –3 – i – (–4 + i) =

z3

2

z4

–6

Solución a cargo –8

del alumno.

( b. z + ( c. z + ( d. z + ( a. z1 + 2

3

2

)=z ; –6 ) = z ; –6 ) = z ; 0 )=z

4 ; –8 9 8 1

4

4

4

3

57

16

15

17

18

19

20

21

22

23

24

25

Potencias de la unidad imaginaria. Cuadrado y cubo de un complejo INFOACTIVA A través de las propiedades de unidad imaginaria i. i0 = 1 i4 = i3 i1 = i i5 = i4 2 i = –1 i6 = i5 i3 = i2 . i = –1 . i = –i i7 = i6

la potenciación en , se puede hallar la potencia enésima de la . . . .

i i i i

= = = =

–i . i = –i2 = –(–1) = 1 1.i=i i . i = i2 = –1 –1 . i = –i

i8 = i7 . i = –i . i = –i2 = –(–1) = 1 i9 = i8 . i = 1 . i = i i10 = i9 . i = i . i = i2 = –1 i11 = i10 . i = –1 . i = –i

y así sucesivamente, se observa que: i0 = i4 i8 = 1 i1 = i5 = i9 = i i2 = i6 = i10 = –1 i3 = i7 = i11 = –i

i0 = 1 i1 = i i2 = –1 i3 = –i

Los resultados de las potencias de i son 1, i, –1 y –i; se repiten periódicamente.

¨ © ª

in = i4c+r = ( i4 )c . ir ‰ in = ir ∧ n 4 1 r c El resultado de elevar la unidad imaginaria a un número natural n es igual a elevarlo al resto de la división entera entre n y 4.

i75 = i3 = –i

i117 = i1 = i

i138 = i2 = –1

i164 = i0 = 1

75 3

117 1

138 2

164 0

4 18

4 29

4 34

4 41

Cuadrado y cubo de un complejo Para elevar al cuadrado o al cubo un complejo, se desarrolla el cuadrado o el cubo de un binomio.

58

(4 + i)2 = 42 + 2 . 4 . i + i2 = 16 + 8i – 1 = 15 + 8i

(4 + i)3 = 43 + 3 . 42 . i + 3 . 4 . i2 + i3 = 64 + 48i – 12 – i = 52 + 47i

(3 – 6i)2 = 32 + 2 . 3 . (–6i) + (–6i)2 = 9 – 36i – 36 = –27 – 36i

(2 – 5i)3 = 23 + 3 . 22 . (–5i) + 3 . 2 . (–5i)2 + (–5i)3 = 8 – 60i – 150 + 125i = –142 + 65i

Test

de comprensión

1. Respondan y expliquen las respuestas. a. ¿Cuál es la forma práctica de hallar la potencia enésima de i ? b. Las expresiones (a + bi)2 y (a2 + b2 . i2) ¿son equivalentes? a. Dividir la potencia de i por cuatro, y luego elevar i al resto de la división. b. No, se debe desarrollar el cuadrado del binomio.

16

ACTIVIDADES Potencias de la unidad imaginaria. Cuadrado y cubo de un complejo

34. Resuelvan cada potencia de i. a. i45 =

i

b. i17 =

i

c. i32 =

1

d. i30 = –1

e. i5 =

i

f. i19 = –i

35. Resuelvan las operaciones. a. 3i3 + 5i6 – 3 – __51 i3 =

14 –8 – ___ 5 i

22 – 5i ___ 15 5 3 __ __ __ – ___ 28 – 5 i c. __53 i3 + __41 – __73 i20 + 32 + 32 i2 = 7 ___ + 21 i – __ 4 10 d. __53 i21 – 3 – __23 i3 – __41 – i18 = 31 ___ – 2i e. __32 + 5i8 – 3i23 + __21 i10 – 5i9 = 6 19 __ 5 ___ + i 2 4 f. – __43 + __23 i35 – (–1 – i12) + __27 – i27 =

b. – __31 + 5i3 – __54 i2 + 3 – 2i108 =

(

(

)

)

36. Unan con la respuesta correcta. __ ˆ ˆ ˆ ˆ ˆ ˆ

a. 33 i35 . i29 = b. i23 : i13 = c. i30__. i12 : i = d. 33 i18 : i13 = e. i12 . i15 : i17 = f. i13 : (i4 . i18) =

–1__ 33 i 1 i __ 33 –i

37. Desarrollen las siguientes potencias. a. (3 – 5i)2 =

16 – 30i

b. (2 – 5i)2 =

–21 – 20i

c. (–1 + 3i)3 =

26 – 18i

d. (2 – i)3 =

2 – 11i

38. Hallen el valor de a para que se cumplan las siguientes igualdades. a. (a + 2i)2 = 5 + 12i

b. (–3 + ai)2 = –40 – 42i

a=3

a=7

59

17

16

18

19

20

21

22

23

24

25

26

Multiplicación y división INFOACTIVA Multiplicación de complejos Para multiplicar dos números complejos en forma binómica, se aplica la propiedad distributiva de la multiplicación respecto de la suma (o resta). (a + b i) . (c + d i) = ac + ad i + cb i + bd i2 = ac + ad i + cb i – bd = ac – bd + (ad + cb) i

(4 + 5 i) . (–2 + 6 i) = 4 . (–2) + 4 . 6 i + (–2) . 5 i + 5 . 6 i2 = –8 + 24 i – 10 i – 30 = –38 + 14 i El producto de dos complejos en forma trigonométrica es otro complejo cuyo módulo es el producto de los módulos y su argumento es la suma de los argumentos de los complejos dados. φ 1 + i . sen ^ φ 1 ) ∧ z2 = ρ2 . (cos ^ φ 2 + i . sen ^ φ2 ) z1 = ρ1 . (cos ^ ^ ^ ^ Entonces, z . z = ( ρ . ρ ) . [cos ( φ + φ ) + i . sen ( φ + ^ φ )] 1

2

1

2

1

2

1

2

z1 = 6 . (cos 40° + i . sen 40°) ∧ z2 = 2 . (cos 50° + i . sen 50°) z1 . z2 = 6 . 2 . [cos (40° + 50°) + i . sen (40° + 50°)] = 12 . (cos 90° + i . sen 90°) = 12 . (0 + i) = 12 i Producto de complejos conjugados El producto de dos números complejos conjugados es igual a la suma de los cuadrados de la parte real e imaginaria. _

_

_

_

_

z . z = (a + b i) . (a – b i) ‰ z . z = a2 – (b i)2 ‰ z . z = a2 – b2 . i2 ‰ z . z = a2 – b2 . (–1) ‰ z . z = a2 + b2

(5 + 6 i) . (5 – 6 i) = 52 + 62 = 25 + 36 = 61

(–3 – 4 i) . (–3 + 4 i) = (–3)2 + 42 = 9 + 16 = 25

División de complejos Para dividir dos números complejos en forma binómica, se multiplican el divisor y el dividendo por el conjugado de este último y luego se resuelven las operaciones resultantes. + 8 i + 3 i – 4 = _______ 3 + 4 i = 2__________________ . 3 + 2 . 4 i + 3 i + 4 i2 = 6 2 + 11 i = ___ 2 + ___ 2 + i = _____ 2 + i . ______ 11 i _____ ____________ 3 – 4i 3 – 4i 3 + 4i 25 25 25 9 + 16 32 + 42 6i = 5 . 6 i – 2 . 6 i2 = ________ 5 – 2 i . __ 30 i + 12 = ___ 30 i = __ 5i 5 – 2 i = _____ 12 + ___ 1 + __ _____ ___________ 3 6 –6 i –6 i 6 i 36 36 36 62

El cociente de dos complejos en forma trigonométrica es otro complejo cuyo módulo es el cociente de los módulos y su argumento es la diferencia de los argumentos de los complejos dados. z1 = ρ1 . (cos ^ φ 1 + i . sen ^ φ 1 ) ∧ z2 = ρ2 . (cos ^ φ 2 + i . sen ^ φ2 ) z ρ 1 1 __ __ ^ ^ ^ ^ Entonces, = . [cos ( φ – φ ) + i . sen ( φ – φ )] z2

ρ2

1

2

1

2

z1 = 9 . (cos 90° + i . sen 90°) ∧ z2 = 3 . (cos 30° + i . sen 30°)

__

z1 __ 3 9 1 3 __ __ ___ z2 = 3 .[cos (90° – 30°) + i . sen (90° – 30°)] = 3 . (cos 60° + i . sen 60°) = 3 . 2 + 2 i

(

60

__

) = __32 + __32 . 33 i

Test

de comprensión

1. Respondan y expliquen las respuestas. a. El producto de dos números imaginarios puros ¿pertenece al conjunto de los números reales? b. El cociente de dos números complejos ¿da como resultado siempre un número complejo? 3i a. Sí. b. Sí; teniendo en cuenta que los números reales son complejos. Por ej., __ = 3. i

17

ACTIVIDADES Multiplicación y división

39. Resuelvan las siguientes multiplicaciones. a. (–2 + 3i) . (–2 – 3i) =

13

b. (4 – 5i) . (–2 – i) =

13 + 6i

__

__

__

__

c. ( 37 + 35 i ) . ( 37 – 35 i ) =

12

d. 5;__23 . 4;__31 =

39 ___ 23 ( ___ 2;3 )

e. ( 32 ;–1 ) . ( 1;32 ) =

(2 . 32 ;1)

(

) (

)

__

__

__

__

__

__

__

__

(1;2 . 36 )

f. ( 33 ;32 ) . ( 33 ;32 ) =

40. Tengan en cuenta los números complejos y resuelvan las multiplicaciones. z1 = 3 . (cos 135° + i . sen 135°); z2 = 2 . (cos 30° + i . sen 30°); z3 = 7 . (cos 60° + i . sen 60°); z4 = 4 . (cos 315° + i . sen 315°); z5 = 12 . (cos 225° + i . sen 225°) a. z1 . z3 = d. z1 . z5 = 21 . (cos 195° + i . sen 195°)

36 . (cos 360 + i . sen 360°)

b. z4 . z2 =

e. z2 . z5 =

8 . (cos 345° + i . sen 345°)

24 . (cos 255° + i . sen 255°)

c. z3 . z5 =

f. z3 . z4 =

84 . (cos 285° + i . sen 285°)

28 . (cos 15° + i . sen 15°)

41. Completen la tabla. z

_

_

z

z.z

2 + 6i

2 – 6i

40

–1 – i

–1 + i

2

5i

–5i

25

–3 + 4i

–3 – 4i

25

1 – 2i

1 + 2i

5 61

ACTIVIDADES Multiplicación y división de complejos

17

42. Resuelvan las siguientes divisiones. 1 – 3i a. ______ = 2 + 2i 1 – __ 2 –i

1__– 2i e. ______ =

+i 32 __ __ –2 + 32 – ( 1 + 2 . 32 ) i ___________________ 3

__

+ 3i _______ b. –2 = –3 – i

3__2 – i__ f. _________ = 32 + 33 i

3 11 i ___ – ___ 10 10

33 – 2 + (36 + 32 ) i

–i c. 3_____ = 2i

– 2i g. 3______ = 1 __

__

__

3 1 __ – __ 2 – 2 i

+i 2 14 ___ 16 ___ 5 – 5 i

–__ 2i d. 3______ =

5__ – 33__i h. 3________ =

__

33 i 2–i 33 . – __ 3 __

(

)

__

__

35___+ 33 i 1 – 315 i ________ 4

43. Resuelvan las siguientes divisiones de números complejos expresados en forma trigonométrica. a. z1 = 18 . (cos 90° + i . sen 90°) y z2 = 3 . (cos 30° + i . sen 30°) z1 __ z = 6 . (cos 60° + i . sen 60°) z2 =

b. z3 = 20 . (cos 150° + i . sen 150°) y z4 = 20 . (cos 100° + i . sen 100°) z3 z = cos 50° + i . sen 50° __ z4 =

c. z5 = 3 . (cos 60° + i . sen 60°) y z6 = 2 . (cos 30° + i . sen 30°) 3 z5 z = __ __ 2 . (cos 30° + i . sen 30°) z6 =

d. z7 = 2 . (cos 45° + i . sen 45°) y z8 = cos 30° + i . sen 30° z7 z = 2 . (cos 15° + i . sen 15°) __ z8 = 62

17

ACTIVIDADES Multiplicación y división de complejos

44. Lean atentamente y resuelvan. a. Hallen el valor de a para que el producto entre a + 5i y su conjugado sea igual a 29. a = –2 o a = 2

b. Hallen el valor de a para que el producto entre a – 2i y su conjugado sea igual a 13. a=3oa=–3

45. Hallen el valor de ^ φ para que se cumplan las condiciones pedidas en cada caso.

a. Si z1 = 2 . (cos 30° + i . sen 30°); z2 = 4 . (cos ^ φ + i . sen ^ φ ) y z1 . z2 es un número imaginario puro positivo. φ = 60° a. ^

b. Si z3 = 9 . (cos 50° + i . sen 50°); z4 = 4 . (cos ^ φ + i . sen ^ φ ) y z3 . z4 es un número imaginario puro negativo. b. ^ φ = 220°

c. Si z5 = 3 . (cos 150° + i . sen 150°); z6 = 12 . (cos ^ φ + i . sen ^ φ ) y z5 . z6 es un número real positivo. φ = 210° c. ^

d. Si z7 = 6 . (cos 150° + i . sen 150°); z8 = 2 . (cos ^ φ + i . sen ^ φ ) y z7 . z8 es un número real negativo. φ = 30° d. ^

46. Hallen el valor de a para que z =

2 – ai ______ cumpla con la condición pedida en cada caso. 1+i

a. Sea un número real.

b. Sea un número complejo.

a = –2

Cualquier número real menos el –2.

mente ACTIVA El producto de dos números complejos ¿es igual al producto de sus opuestos?

Sí.

63

18

17

19

20

21

22

23

24

25

26

27

Operaciones combinadas INFOACTIVA Las operaciones combinadas entre números complejos se resuelven respetando la jerarquía de cada una de ellas. 1.° Se resuelven las potencias y raíces; 2.° se resuelven las multiplicaciones y divisiones; 3.° se resuelven las sumas y restas. Los paréntesis alteran el orden de resolución de las operaciones. . 4 + 3 . (–2 i) + i . 4 + i . (–2 i) (3 + i) . (4 – 2 i) 3_________________________ _____________ = (5 + i)2 52 + 2 . 5 . i + i2

Se resuelven la multiplicación y el cuadrado del binomio.

– 6i + 4i + 2 _____________ = 12 25 + 10 i – 1 24 – 10 i 14 – 2 i . ________ = ________ 24 + 10 i 24 – 10 i

Se resuelve la división.

14 . 24 + 14 . (–10 i) – 2 i . 24 – 2 i . (–10 i) = ___________________________________ 242 + 102 – 140 i – 48 i – 20 ____________________ = 336 576 + 100 316 – 188 i = __________ 676 316 – ____ 188 i = ____ 676 676 79 – ____ 47 i = ____ 169 169

Se pueden realizar ciertas operaciones combinadas en forma trigonométrica. z .z

1 2 Hallen _____ z 3

z1 z2

° φ = –30

ρ=

z1 = 3 . (cos 45° + i . sen 45°) z2 = 2 . (cos 120° + i . sen 120°) 5 . (cos 300° + i . sen 300°) z3 = __ 2

ρ

2

z1 . z2 _______________________ 6 . cos 165° + i . sen 165° _____ z3 = __ 5 . (cos 300° + i . sen 300°) 2

64

φ = 45° φ = –60°

2,5

12 . (cos 225º + i . sen 225º) = ___ 5

3

ρ=

12 . [cos (–135º) + i . sen (–135º)] = ___ 5

=

z3

Test

de comprensión

1. Respondan y expliquen las respuestas. a. ¿Cómo se resuelve un cálculo combinado que contiene potencias de la unidad imaginaria mayores o iguales que 4? b. ¿Es necesario separar en términos en un cálculo con números complejos? a. Es conveniente primero transformarlas en su equivalente –1; 1, –i o i. b. En todo cálculo combinado hay que separar en términos.

18

ACTIVIDADES Operaciones combinadas

47. Resuelvan los siguientes cálculos combinados.

12

1______ – 17i 29

i – (3 + 2 i) – 14i _______ d. ________________ = –2 105

2–i b. ______ = 2+i

6 – 3i ______ 5

–2 + 3 i7 e. ________________ = (–3 + i2) . (2 – i19)

3 + 3 i2 + i c. i_________ = –1 – i7

3 __ 2 . (1 + i)

i . (i – 2 i ) f. _______________ = 10

3–i a. ________ = 2 i12 – 5 i3

14

(7 – i ) – (3 + 2 i)

33

3

4

(–1 – 2 i) . (i – i)

25

7 ___ + __1 i 20 5

7 ___ 1 – ___ 10 – 10 i

48. Unan cada cálculo con su solución. 2

(1 + 5 i) a. _______ = 2 i3 – 3 i

ˆ –1 + i

12 i17 b. _________________ = (3 + 2 i6) . (3 – 2 i7)

24 ˆ –2 – ___ 5 i

3 i8 . (1 – 3 i7) c. ____________ = 6 i7 – 3 i2

ˆ – __41 + __43 i

i13 – 5 i2 _________ d. 12 = i3 – 3 i2

ˆ 8i

+ 25 i2 e. 1_______ = 6 i3 + 9 i

+ 36 i ________ ˆ 24 13

9 2

(2 – i ) f. _______________ 7 12 = (3 – i ) . (–1 – i )

+ 41 i ˆ 3_______ 10 65

18

ACTIVIDADES Operaciones combinadas

49. Resuelvan los siguientes cálculos combinados.

66

– 2 i3 2 i7 a. 3______ – ______ = 5+i 4 – i18

i32 – 2 i18 ________ f. –7 – __ = 5 – 4 i20 i60

17 ____ ___ + 87 i 26 130

–6

3 + i20 3 i24 ____ b. ______ 19 + 5 – i3 = 3–i

+ 4 i6 i387 _______ g. –5 + ___ = 3 + 3 i33 i386

31 ___ 17 ___ 5 – 5i

5 3 __ – __ 2 + 2i

5 i3 + 2i c. 3______ + ______ = 5 – i15 4 – i18

–2 + i15 + 11 i29 ________ h. _______ + 12 + i500 = 4 + 4 i99 8 i66

53 23 ____ – ____ i 130 130

5 __ 7 – __ 8 – 4 i

– 4 i20 + 2i d. 3______ – 5_______ = i15 3 – i50

3 i___________ . (i2 – 3 i7) 5 – 3 i18 1 – 3 i30 _______ i. _______ + = 40 – 6 2 – 3i 4i –3 i

3 __ __ – 1i 4 2

77 __ 3 – ___ 12 – 4 i

– 4 i12 – 3 i4 e. 5_______ – 8______ = –3 – i20 3 – i19

2 – 3 i7 –5 + 3 i21 –2 + i ______ ________ = j. __________ + – 32 13 12 i i . (1 – 3i) i – 3i

7 __ + 1 i – __ 4 2

24 ___ 13 – ___ 5 + 5 i

18

ACTIVIDADES Operaciones combinadas

50. Tengan en cuenta los números complejos y resuelvan las operaciones combinadas. z1 = 3 . (cos 50° + i . sen 50°); z2 = 5 . (cos 200° + i . sen 200°); z3 = 4 . (cos 30° + i . sen 30°); z4 = 9 . (cos 230° + i . sen 230°) z4 d. _____ z .z =

z1 . z2 a. _____ = z 3

15 ___ . (cos 220° + i . sen 220°) 4

z4 . z2 b. ______ = z

3

z3 . z4 e. ______ z .z =

1

3

15 . (cos 20° + i . sen 20°)

1

3 . (cos 180° + i . sen 180°)

z1 . z4 f. ______ z .z =

z2 c. _____ z .z = 1

1

3 __ . (cos 150° + i . sen 150°) 4

3

3

5 ___ 12 . (cos 120° + i . sen 120°)

2

27 ___ . (cos 50° + i . sen 50°) 20

51. Resuelvan. i a. (3 – i32) . (4 – i20) – ________ = 5 i12 – 3 i2

i53 . (7 – 3 i24) i12 . (1 – 3 i14) d. ____________ + ___________ = 50 i50 + 5i 2 . (i + 5 i)

1i 6 – __ 8

3 11 i ___ – ___ 13 13

21

– 3 i3 b. 5______ 2 – i20

(

–1

)

3 – i12 : ______ = 2 – 3 i4

24 i3 . (3 – i38) i___________ . (3 – 2 i5) e. __________ – = 39 38 i53 – 2 i –i

5 3 ___ – ___ 68 + 68 i

18 __ 11 ___ 5 – 5i

5 + i30 3 i2 c. _______ : i25 + ______ = 5 – i18 –2 – i20

2 – 3 i12 2 f. __________ : ______ + 1 – 3 i20 = 18 i . (i3 – 1) i15 – 3

4 1 __ – __ 2 + 3i

1 –1 – __ 2i

67

19

18

20

21

22

23

24

25

26

27

28

Ecuaciones INFOACTIVA (FXDFLRQHVFRQFRH¿FLHQWHVUHDOHV\VROXFLyQFRPSOHMD Existen ecuaciones con coeficientes reales cuya solución es la raíz de índice par de un número negativo. Dichas ecuaciones no tienen solución en el conjunto de los números reales, pero sí en el de los complejos. Las soluciones complejas de una ecuación cuadrática con coeficientes reales son complejos conjugados. ___ 2 2 x + 9 = 0 ‰ x = –9 ‰ x = i . 3–9 ‰ x = ±3i

x2 – 3x + 8 = 0

_____________

______

____

___

___

___

3 ± 3(–3)2 – 4 . 1 . 8 3 ± 39 – 32 3 ± 3–23 3 ± 323 i 23 i 3 + 3____ x1;2 = _________________ = __________ = ________ = ________ ‰ x1 = __ 2 2.1 2 2 2 2

23 i 3 – 3____ ∨ x2 = __ 2 2

Para verificar una solución compleja, se procede de igual manera que para las soluciones reales.

(

___

23 i 3 + 3____ __ 2 2

2

) –3.( ___

___

23 i 3 + 3____ __ 2 2 ___

)+8=

∨ ___

(

___

23 i 3 – 3____ __ 2 2

2

) –3.( ___

23 i + 3 23 i 2 – __ 23 i + 8 = 9–3.3 9 + 2 . __ 3.3 __ ____ ____ ____ 4 2 2 2 2 2 ___ ___ 9 – __ 9 + __ 3 . 323 i – ___ 23 – __ 3 . 323 i + 8 = __ 4 2 4 2 2



9+8= 9 – ___ 23 – __ __ 4 4 2

9+8= 9 – ___ 23 – __ ∨ __ 4 4 2

9 – 23 – 18 + 8 = ___________ 4

– 23 – 18 + 8 = ∨ 9___________ 4

32 + 8 = 0 – ___ 4

32 + 8 = 0 ∨ – ___ 4

(

)



___

23 i 3 – 3____ __ 2 2 ___

)+8= ___

23 i + – 3 23 i 2 – __ 23 i + 8 = 9+3.3 9 – 2 . __ 3.3 __ ____ ____ ____ 4 2 2 2 2 2 ___ ___ 9 + __ 9 – __ 3 . 323 i – ___ 23 – __ 3 . 323 i + 8 = __ 4 2 4 2 2

(

)

Para hallar una ecuación cuadrática, dadas sus soluciones complejas, se aplica el mismo procedimiento que con soluciones reales: (x – x1 ) . (x – x2 ) = 0.

x1 = 5 – 2 i ∧ x2 = 5 + 2 i [x – (5 – 2 i)] . [x – (5 + 2 i)] = 0 2 x – x . (5 + 2 i) – (5 – 2 i) . x + (5 – 2 i) . (5 + 2 i) = 0 x2 – 5x – 2 i x – 5x + 2 i x + 25 + 10 i – 10 i + 4 = 0 x2 – 10x + 29 = 0

(FXDFLRQHVFRQQ~PHURVFRPSOHMRV Las ecuaciones con números complejos deben resolverse aplicando las operaciones y propiedades de los mismos. 3+i ‰ –8 + 4i . ____ 3z + 8 – z i = 4i ‰ z . (3 – i) = –8 + 4i ‰ z = ______ 3–i 3+i – 8i + 12i – 4 ‰ z = ________ –28 + 4i ‰ z = – ___ 2i 14 + __ _______________ ‰ z = –24 10 9+1 5 5

También pueden plantearse ecuaciones con números complejos y solución real.

Hallen x e y reales que verifiquen: –3x + 5xi = y + 2 – 2yi

= y + 2 ‰ y = –3x – 2 {–3x 5x = –2y ‰ 5x = –2 . (– 3x – 2) ‰ 5x = 6x + 4 ‰ x = –4 ∧ y = 10 68

Test

de comprensión

1. Respondan y expliquen las respuestas. a. ¿Cómo se verifica la solución de una ecuación cuando da como resultado un número complejo? b. Si z = 1 – 3i es solución de una ecuación de segundo grado, ¿cuál es la otra solución? a. Se reemplaza el resultado en la variable. b. Es el conjugado, en este caso 1 + 3i.

19

ACTIVIDADES Ecuaciones

52. Marquen las soluciones de las siguientes ecuaciones. a. z2 + 25 = 0 X 5i

X –5 i

25 i

25 i

b. z2 – 3 = –13 ___

___

X 310 i

___

313 i

___

X – 310 i

– 313 i

c. z2 + 3z + 3 = 0 __

__

3 3 3___ X – __ 2 + 2 i

3 __ + 3i 2 3

__

__

3 3 3___ X – __ 2 – 2i

3 __ 2 – 33 i

d. z2 + 26 = 2z 5i

X 1 – 5i

X 1 + 5i

–5 i

e. z2 – 2z = – 5 X 1 – 2i

X 1 + 2i

–1 – 2 i

–1 + 2 i

53. Resuelvan las ecuaciones. Luego, verifiquen las soluciones. a. z2 – 2 . (z – 5) = 4z – 3

d. 5 . (z2 – 1) – (2 + z) = 4 . (z2 – 3) – 3z

3 ± 2i

–1 ± 2i

b. 3 . (z2 + 20) = 12z

e. 3 . [z . (z – 10) + (2 + z)] = –81 + 3z

2 ± 4i

5 ± 2i

c. z2 + 20 = 2 . (z – 3)

f. 5z2 – 3z + 10 = –3z . (z + 1) + 4 . (z2 + 1)

1 ± 5i

± __i . 36

__

2

69

19

ACTIVIDADES Ecuaciones

54. Unan con flechas las ecuaciones que tienen el mismo conjunto solución. a. z2 – 2z = –5 b. 3z . (z – 4) = – 15 c. 4 . (z2 – 4) + 2z = 2 . (10 + z) d. z . (z – 2) – 3 = – 13 e. z2 = – 4 f. z2 – 3 . (z + 10) = –(43 – z)

g. z . (4 – z) = 13 h. 2z2 – 3z + 1 = z . (z – 1) – 9 i. z . (z – 3) = (9 – 3z) j. 2z2 – 5z + 2 . (z2 – 3) = 3z – 26 k. z2 – 3 – 20z = – 20z – 7 l. z2 – 6 . (2z – 3) + 2 . (z2 – 1) = 1

55. Resuelvan las ecuaciones. Luego, verifiquen las soluciones. – 2i –i a. z______ = 1____ 3 + 2i 1+i

– 3i – 3i d. 5______ = 2______ z–i i

z=2–i

32 z = – ___ + ___ i 13 13

z–i b. _____ =1–i 2+i

3–i – 2i e. ______ = z______ 2 + 2i i

z=3

5 z = 1 + __ 2i

–3 c. z_____ = 2z – i 1–i

– 3i 2i f. 1______ = ______ 5i z + 3i

9

4 __ 3 z = – __ 5 – 5i

z = –1 – 6i

56. Hallen una ecuación cuyas raíces sean los números complejos dados. a. z1 = 2 – 3 i y z2 = 2 + 3 i

d. z1 = 1 – __21 i y z2 = 1 + __21 i

z2 – 4z + 13 = 0

5 z2 – 2z + __ =0 4

b. z1 = –7 i y z2 = 7 i

e. z1 = 2 – i y x2 = 2 + i

2

z2 – 4z + 5 = 0

z + 49 = 0

__

70

__

__

__

c. z1 = 1 – 36 i y z2 = 1 + 36 i

f. z1 = 2 – 35 i y z2 = 2 + 35 i

z2 – 2z + 7 = 0

z2 – 4z + 9 = 0.

19

ACTIVIDADES Ecuaciones

57. Hallen los valores de x e y que verifiquen las igualdades. a. (x + i) . (2 – y i) = 6 – y i

– 2i c. x______ –1=i 1 – yi

x=y=2

x = 4; y = 3

– xi b. 2______ = y – 4i 1+i

–3 + 2i + 2y 1 d. ___________ = _____ 4 + xi 2+i

x = 6; y = –2

x = 7; y = 3

58. Lean atentamente y resuelvan. __

x+i a. Hallen el valor de x e y para que ______ sea equivalente a 32 . (cos 225° + i . sen 225°). 2 – yi

x = –5; y = 3

b. Hallen los valores de x para los cuales (1 + x i)2 es un número imaginario puro. x = –1 y x = 1

__

+i c. Hallen los valores de x para los cuales z = x____ tiene como módulo 3 5 . 1+i

x = 7; x = –7

59. Planteen la ecuación y resuelvan. Sean z1 = x – 2 i; z2 = y + i; z3 = 17 – 7 i, hallen el valor de x e y sabiendo que z1 . z2 = z3. (x – 2i) . (y + i) = 17 – 7i 3 x = 3 ∧ y = 5 o x = –10 ∧ y = – __. 2

mente ACTIVA El número complejo 3 + 2i es una solución de una ecuación cuadrática cuyo coeficiente principal es 1. Hallen los valores del coeficiente del término lineal y del término independiente. b = –6 y c = 13

71

INTEGRACIÓN 60. Calculen cada potencia de i. 1

16

a. i = b. i23 = c. i63 = d. i14 = e. i34 =

–i –i –1 –1

39

f. i = g. i60 = h. i106 = i. i170 = j. i81 =

65. Resuelvan las siguientes multiplicaciones.

–i 1 –1 –1 i

61. Resuelvan las siguientes operaciones. a. 23 i5 – 12 i7 – 4 i = 31 i b. 12 i19 – 3 i4 – 3 + 2 i12 = –4 – 12 i c. 43 i34 + 12 – 35 i20 + 15 i15 = – 66 – 15 i d. (2 – 3 i5) – (23 – 14 i7) + 3 = –18 – 17 i e. 56 i5 – (18 + 23 i15) = –18 + 79 i f. –(3 – 4 i9) – (3 – 12 i6) + (2 – 3i) = –16 + i g. –(2 – i7) – (3 – i57) – (13 – 2 i8) = –16 h. 34 – i8 – (35 + 2 i6) + i69 = i

62. Resuelvan teniendo en cuenta las propiedades de la potenciación. a. 22 i45 : 2 i23 = –11 b. 2 i7 . 3 i29 = 6 c. 4 i34 . 5 i45 : 2 i28 = –10 i 1 __ d. i56 : (4 i32 . 2 i25) = – 8 i 63. Desarrollen las siguientes potencias. 2

2

a. (2 – 3i) = –5 – 12i f. (–7 + 2i) = 45 – 28i. b. (3 + 3i)2 = 18 i g. (1 – 2i)3 = –11 + 2 i c. (1 – 4i)2 = –15 –8 i h. (2 + 3i)3 = –46 + 9 i d. (8 – 2i)2 = 60 – 32 i i. (–1 + 2i)3 = 11 – 2 i e. (2 + 3i)2 = –5 + 12i j. (–2 – 3i)3 = 46 – 9 i

64. Hallen el valor de a para que se cumplan las siguientes igualdades. a=4 a. (1 + 2i)2 = –3 + ai 2 a = –21 b. (2 – 5i) = a + 20i 2 c. (–3 + 7i) = –40 – 7ai a = 6 a = –4 d. (–a – 4i)2 = –32i 2 e. (7 + 2ai) = 13 + 84i a = 3 f. (2a – 5i)2 = –9 – 20ai a = –2 o a = 2 g. (–4 + ai)2 = 7 + 24i a = –3

a. (2 – 4 i) . (3 – 4 i) = b. (–2 + 3 i) . (4 + 12 i) = c. (–5 + 3 i) . (– 2 – i) = d. (7 – 2 i) . (– 1 – i) = e. (2 – 3 i) . (2 + 3 i) = f. (5 – 7 i) . (5 + 7 i) = g. (1 + 4 i) . (1 – 4 i) = h. (17 – 3 i) . (17 + 3 i) =

–10 – 20i – 44 – 12i 13 – i – 9 – 5i 13 74 17 298

66. Tengan en cuenta los números complejos y resuelvan las operaciones. z1 = 3 . (cos 40° + i . sen 40°) z2 = 2 . (cos 10° + i . sen 10°) z3 = 12 . (cos 35° + i . sen 35°) z4 = 6 . (cos 120° + i . sen 120°) z5 = 2 . (cos 180° + i . sen 180°) a. z2 . z3 = b. z1 . z5 = c. z2 . z4 = d. z2 : z3 =

e. z5 : z2 = f. z4 : z3 = g. z2 . z1 = h. z4 : z2 =

a. 24 . (cos 45° + i . sen 45°) b. 6 . (cos 220°+ i . sen 220°) c. 12 . (cos 130°+ i . sen 130°) 1 d. __ . (cos 335° + i . sen 335°) 6 e. cos 170° + i . sen 170° 1 f. __ . (cos 85° + i . sen 85°) 2 g. 6 . (cos 50° + i . sen 50°) h. 3 . (cos 110° + i . sen 110°)

67. Resuelvan las operaciones teniendo en cuenta que z1 = 1 – 3 i; z2 = 5 – i; z3 = 3 i; z4 = 1 – i. __

a. z1 . z2 = 2 – 16i b. z3 . z4 = 3 + 3i __

c. z1 . z2 = 8 – 14i __

__

d. z3 . z2 = 3 – 15i

z1 ___ 1 8 ___ e. __ z2 = 13 + 13 i z4 1 __ 1 __ f. __ z3 = – 3 – 3 i z1 7 4 ___ ___ g. __ z2 = 13 – 13 i z3 15 3 ___ ___ h. __ z2 = – 26 + 26 i

68. Escriban V (Verdadero) o F (Falso) según corresponda. a. La suma de dos números complejos conjugados siempre es otro número complejo. V b. La diferencia entre dos números complejos F Siempre es imaginario.

conjugados siempre es un número real.

c. El producto de dos números complejos conjugados siempre es un número real.

72

V

capítulo

CONTENIDOS

3

16*17*18*19 69. Determinen el valor de φ para para que

73. Resuelvan las ecuaciones. Luego, verifiquen

cumpla las condiciones pedidas en cada caso.

las soluciones. __ 1 ± 33 i a. z . (z – 2) = 4 5± i b. z2 – 5 . (z – 6) = 5z + 4 __ 1 ± 33 i c. z . (3z – 4) + 10 = 2 . ( z – 1) __ d. 7 . (z2 – 1) + z = –26 + 3z . (2z + 1) 1 ± __3 . 32 i__ { –33 i;0;33 i } e. 3z3 – 6z + 15z = 0

a. z1 : z2 es un número imaginario puro positivo; z1 = 12 . (cos 130° + i . sen 130°); z2 = 4 . (cos ^ φ + i . sen ^ φ ). l = 40°

b. z1 : z2 es un número imaginario puro negativo; z1 = 5 . (cos 330° + i . sen 330°); z2 = 2 . (cos ^ φ + i . sen ^ φ ). l = 60°

c. z1 : z2 es un número real positivo; z1 = 3 . (cos 50° + i . sen 50°); z2 = 12 . (cos ^ φ + i . sen ^ φ ). l = 50°

d. z1 . z2 es un número real negativo; z1 = 6 . (cos 225° + i . sen 225°); z2 = cos ^ φ + i . sen ^ φ. l = 45°

70. Hallen los números complejos pedidos en cada caso. a. Por ejemplo, z1 = 12 . (cos 90° + i . sen 90°) y z2 = 4 . (cos 45° + i . sen 45°). a. Dos números complejos cuyo cociente sea z = 3 . (cos 45° + i . sen 45°). b. Dos números complejos cuyo cociente sea z = –3 . (cos 45° + i . sen 45°). b. No es posible porque el cociente entre dos números positivos es positivo.

71. Resuelvan los siguientes cálculos combinados. 13

4 – 2i 3i 3–i 29 b. ______ + _______ – _______ = –2 + ___ 8 i i50 – i105 5 + 3 i20 i23 – i12 2

(3 – 2 i) 3 i27 . (3 – 5 i) 39 ____ 122 c. ____________ – ________ = ___ 17 – 17 i i20 – 3 3 i20 – 5 i 5 i40 5 2 i3 ________ __ d. ____________ –i 30 12 4 + 4 i12 – 3 i2 = 7 i . (i – 3 i ) 25

las soluciones. 4i + 3i a. z______ = _____ 2–i 2+i – 3i +i b. 1______ = 2_____ 2+i z+i –i –i c. 3_____ = z____ i 1+i

3 16 __ ___ 5 – 5i 9 3 – ___ + ___ i 10 10

2 – 3i

10 ___ – 2 i ___ 1+i + 62 i d. ______ = 5______ z – 2i 1 – i 29 29

75. Hallen una ecuación cuyas raíces sean los números complejos dados. a. z1 = 5 – 7 i y z2 = 5 + 7 i b. z1 = 1 – 3 i y z2 = 1 + 3 i c. z1 = __35 – 7 i y z2 = __35 + 7 i __ __ d. z1 = 3 – 3__ 7 i y z2 = 3 + 3__ 7i 1 1 __ __ e. z1 = 2 – 33 i y z2 = 2 + 33 i

z2 – 10z + 74 z2 – 2z + 10 466 10 z + ____ z2 – ___ 3 9 2 z – 6z + 16 13 z2 – z + ___ 4

76. Hallen los valores de x e y que verifiquen la

–4 i 19 ___ –1 a. _______ – ___ + __41 = – ___ – 1 i 20 15 i12 – 3 i3 3 i3 12

74. Resuelvan las ecuaciones. Luego, verifiquen

igualdad. 4x + i a. x – i = ______ y + 2i

x=2y=3

y + 3i 1 b. _______ = ______ –4 + x i 2 + 2i

x=8y=1

3

i . (i – 2 i) 24 127 ___ 12 i3 e. ___________ – _______ = – ____ + i 4 i3 – 2 i 65 65 i20 . (i4 – 8 i) i3 – 3 i24 3 i3 44 ___ 27 f. 2________ + ________ . 2 i26 = ___ – i i32 . i15 2 i12 – 3 i3 13 13 36 20 15 3 i . (2 – 5 i ) i_________ . (i – i) 31 ___ 23 1 g. ____________ : 3 i18 – 4 i3 + _______ = – ___ + i i13 – 2 i2 i15 – 3 i2 20 20 3 12 i – 3i 1 1 i. h. _______ : (i7 + i10) – ______ = –2 + __ i19 + i3 2 i60 – 2 i4

72. Calculen. +3i a. z . z2 para z = 2 . 1______ + i18. –11 – 2 i 2+i _

13

–2i b. z . z para z = 3_______ . 2 – i5

13 ___ 5

77. Planteen la ecuación y resuelvan. Sean z1 = a – 3 i; z2 = b – 6 i; z3 = 2 – 39 i, hallen el valor de a y b sabiendo que z1 . z2 = z3. 5 a = 4 y b = 5 o a = __ 2 y b = 8.

78. Determinen el valor de x para que el número 3 – xi complejo z = ______ cumpla con la condición 1 + 2i pedida en cada caso. a. El vector correspondiente al número complejo esté incluido en la bisectriz del primer y tercer cuadrante. b. El vector correspondiente al número complejo esté incluido en la bisectriz del segundo y cuarto cuadrante. a. x = 9; b. x = –1

73

capítulo

3

AUTOEVALUACIÓN Marquen las opciones correctas

79. ¿Cuál es el resultado de las siguientes raíces? 4

____

__

a. 3–48 3

3i

–6 i

X –6

_____

b. 3–216

4 X 2.3 3i

–3 i

6i

80. ¿Cuál es el resultado de las siguientes operaciones? a. (3 + i) – (–5 + 3i) + 4 – i = X 12 – 3 i

12 + 3 i

10 – 3 i

12 + 13 i

b. (–3 + 2 i) . (–5 + 3 i) = –9 – 9 i

–9 – 19 i

X 9 – 19 i

9 + 19 i

13 1 ___ ___ 10 + 10 i

13 1 ___ ___ 10 – 10 i

– 3i c. 5______ = 2 + 4i 14 – ___ 10 i

13 1 ___ X – ___ 10 – 10 i

81. ¿Cuál es el resultado de las siguientes potencias? a. (–3 – 5 i)2 = 16 + 16 i

–16 – 30 i

X –16 + 30 i

30 + 16 i

b. (–1 + 2 i)3 = X 11 – 2 i

–11 – 2 i

–11 + 2 i

11 + 2 i

82. ¿Cuál es el resultado de las siguientes operaciones combinadas? 6

5

i . (3 + 2 i ) a. ____________ 20 6 = 5 i . (1 – 3 i )

3 1 ___ X – ___ 20 – 10 i

3 1 ___ ___ 20 – 10 i i35 i26 b. ___________ + ______ : i63 = i7 – 3 i i24 . (i5 – 3 i2) 3 1 ___ – ___ 20 – 10 i

1 1 ___ ___ 20 + 10 i

3 3 3 1 ___ ___ ___ X ___ 20 – 10 i 20 – 10 i

1 1 ___ – ___ 20 – 10 i

1 1 ___ – ___ 20 – 10 i

83. ¿Cuáles son las soluciones de las siguientes ecuaciones? a. z2 + 26z + 313 = 0 13 + 12 i

X –13 + 12 i

13 – 12 i

X –13 – 12 i

b. z2 – 6z + 21 = 0 ___

–3 + 312 i

74

___

–3 – 312 i

___

X 3 + 312 i

___

X 3 – 312 i

__

4 X –2 . 3 3i

6

Contenidos

4

20. Circunferencia. 21. Elipse. 22. Parábola. 23. Hipérbola.

Hacia el año 430 a. C. se produjo en Atenas una terrible epidemia y la población se vio rápidamente diezmada. Tras la muerte del gobernador Pericles, varios hombres decidieron consultar el legendario oráculo de Delfos, y la respuesta fue, según cuenta la leyenda, que la peste cesaría si se duplicaba un altar de forma cúbica dedicado al dios Apolo. Los hombres duplicaron entonces cada uno de los lados, pero no sirvió, ya que el volumen del cubo aumentó ocho veces. Poco tiempo después, un geómetra llamado Hipócrates de Quíos demostró que la duplicación es posible x si se pueden encontrar curvas que cumplan las relaciones __ax = __yx = ___ 2a . Algunas décadas más tarde, un sabio llamado Menecmo descubrió otro hecho notable: las curvas mencionadas se pueden obtener como secciones de un cono. En los siglos siguientes, Apolonio escribió su famoso tratado sobre las cónicas, en el cual, entre otras cosas, les da su nombre definitivo: elipse, hipérbola y parábola. Pero pasaron muchos siglos hasta que se pudo resolver el extraño problema planteado por los dioses: la duplicación del cubo, que empleando únicamente regla y compás es imposible.

1. Lean atentamente y respondan. a. ¿Se puede duplicar un cubo? ¿O es imposible? b. ¿Qué aplicaciones prácticas de las cónicas conocen? a. Sí se puede duplicar el volumen de un cubo. La imposibilidad se refiere al hecho de hacerlo usando únicamente regla y compás. b. Abierta. Por ejemplo, la parábola se usa para la construcción de antenas satelitales, reflectores, etc. Las elipses sirven para entender el movimiento planetario, con un foco en el Sol.

capítulo

Cónicas

20

19

21

22

23

24

25

26

27

28

29

Circunferencia ¿Para qué sirve?

INFOACTIVA

PÁGINA 6

Se llama circunferencia al conjunto de puntos de un plano que equidistan de otro punto fijo. Ese punto fijo es el centro de la circunferencia y la distancia del centro a cualquier punto de la misma se denomina radio. Para determinar una circunferencia se necesita conocer su centro y su radio. R

o: centro

o

R: radio C(o;R)

Si se ubica la circunferencia en un sistema de ejes cartesianos, se obtiene la ecuación canónica de la misma. ˆ El centro de la circunferencia es el punto (0;0). y R

x2 + y2 = R2 o = (0;0) x

ˆ El centro de la circunferencia está desplazado al punto (a;b). Si se aplica el teorema de Pitágoras, se obtiene: (x – a)2 + (y – b)2 = R2

Ecuación canónica de la circunferencia.

Si se desarrollan los cuadrados de los binomios de la expresión (x – a)2 + (y – b)2 = R2, se obtiene la ecuación general de la circunferencia.

y y1 R b

y1 – b

o x1 – a

2

2

2

(x – a) + (y – b) = R x2 – 2ax + a2 + y2 – 2by + b2 – R2 = 0 x2 + y2 – 2ax – 2by + a2 + b2 – R2 = 0 x2 + y2 – Ax – By + C = 0

0

a

x1

x

Ecuación general de la circunferencia

Hallen la ecuación canónica de las siguientes circunferencias: a. Centro (4;–2) y R = 3 b. x2 + y2 + 4x – 12y + 4 = 0 x2 + 4x + y2 – 12y = –4 (x – a)2 + (y – b)2 = R2 (x2 + 4x + 4) – 4 + (y2 – 12y + 36) – 36 = –4 (x – 4)2 + (y + 2)2 = 9 (x + 2)2 + (y – 6)2 = 36 76

de comprensión

Test

1. Respondan y expliquen las respuestas. a. ¿Cuál es el centro de la circunferencia (x + 1)2 + y2 = R2? b. Dos circunferencias que tienen el mismo radio ¿son concéntricas? a. El centro es el punto (–1;0). b. No, deben tener el mismo centro.

20

ACTIVIDADES Circunferencia

1. Indiquen el centro y el radio de cada circunferencia. a. (x –2)2 + (y + 3)2 = 9 centro:

(

)

2 ; –3

b. (x + 1)2 + y2 = 25 radio: 3

centro:

(

)

–1 ; 0

radio:

5

2. Escriban la ecuación canónica de cada una de las siguientes circunferencias. a.

b. y

1 –6 –5

3 2 1 –3 –2 –1 0

y

p

p

4

c.

y

–4 –3

–2 –1 0 –1

4

x

3

–2

o

o 1

2 3 4 5 x

p

–3

2 o

–4

1

–5

–6

(x – 1)2 + y2 = 16

–2

0

–1

(x + 3)2 + (y + 3)2 = 9

1

2 x

x2 + (y – 2)2 = 4

3. Escriban la ecuación canónica de cada circunferencia y grafíquenlas. __

b. centro: __27 ;–3 y R: 4

(

a. centro: (–3;1) y R: 36 Ecuación canónica:

2

2

(x + 3) + (y –1) = 6 y

Ecuación canónica: 1 0

5 4 3 2 1 0 –7 –6 –5 –4 –3 –2 –1 0

)

1

x

–1–1 –2 –3 –4 –5 –6 –7

( x – __27 )

2

+ (y + 3)2 = 16

y 1

2 3 4

5 6 7 8 9

x

4. Hallen la ecuación general de cada circunferencia. a. (x – 1)2 + (y + 4)2 = 25 x2 + y2 – 2x + 8y – 8 = 0

Ecuación general:

b. (x + 2)2 + (y – 5)2 = 16 x2 + y2 + 4x – 10y + 13 = 0

Ecuación general: 77

21

20

22

23

24

25

26

27

28

29

30

Elipse INFOACTIVA Se llama elipse al conjunto de puntos de un plano, tales que la suma de sus distancias a dos puntos fijos llamados focos es constante. p1 ___

___

___

___

p1f1 + p1f2 = p2f1 + p2f2

f2

f1

p2 b1

Elementos principales de una elipse ˆ El centro: o. ˆ Los vértices: a1; a2; b1 y b2. ˆ Los focos: f1 y f2. ___ ˆ La distancia focal: f1f2 = 2C. ____ ˆ El diámetro mayor: a____ a = 2A. 1 2 ˆ___ El diámetro menor: b b = 2B. ___ ___ ___1 2 b2f2 + b2f1 = 2A ‰2b2f1 = 2A ‰b2f1 = A 

A a2

C o

f2

f1

a1

B

A2 = B2 + C2

b2

C C __ Al cociente CA o __ A se lo denomina excentricidad. Como en la elipse C < A, entonces 0 ) A < 1. En una circunferencia, C = 0, ya que los dos focos coinciden en el centro de la misma, por lo tanto la excentricidad de la circunferencia es cero. Cuanto más alejados están los focos del centro en una elipse, mayor es su excentricidad y se la ve más “achatada”.

Ecuación de la elipse Para determinar una elipse se necesitan conocer las coordenadas del centro y las medidas de los diámetros. Ubicando la elipse en un sistema de ejes coordenados cartesianos, se obtiene la ecuación de la misma. y ˆ El centro de la elipse es el punto (0;0). y

f1 A

2

y x2 __ + __ =1 A2 B2

f2

B 0

f1

2

y x2 __ + __ =1 B2 A2

x

A

0 f2

ˆ El centro de la elipse está desplazado al punto (a;b). y

(y – b)2 (x – a)2 _______ + _______ =1 A2 B2

b

B

f2 0

a 78

f1 A

x

B

x

de comprensión

Test

1. Respondan y expliquen las respuestas. 2 2

y

x a. En la ecuación de la forma __ + __ = 1 ¿qué curva se forma cuando A = B? A2 B2 b. La distancia focal ¿es siempre menor que el diámetro mayor?

a. Una circunferencia. b. Sí.

21

ACTIVIDADES Elipse

5. ¿Cuál es el gráfico que corresponde a la elipse a.

y2 (x – 2)2 _______ ___ 100 + 36 = 1?

X c.

b.

y

y

10

10

d. y

y 6

6

–6 0 2

–4

8

0

x

6

x

0 2

–8

12

x

–2 0

–12

–6

8 x –6

–10

–10

6. Tengan en cuenta la elipse de la actividad anterior y calculen lo pedido.

Distancia focal =

16

. Diámetro mayor =

20

12

. Diámetro menor =

.

7. Completen con la letra de la ecuación correspondiente. (y – 5)2

y2 __ x2 ___ B: 25 + 4 =1

(x – 3)2

A: _______ + _______ =1 4 16 a.

b.

(x – 5)2

c.

y

y

(y – 3)2

y2

2

x ___ D: 25 + __ =1 4

C: _______ + _______ =1 4 16

d.

y

y

9

5

2

5 5

3 –2

1 0 –1

1

5

9

0

–5

0

2

x

5 x

–2

x

0

–1

1

3

5 x

–5

C

B

8. Calculen 2la excentricidad de las siguientes elipses. y x2 ___ a. ____ 100 + 64 = 1

c = 6, entonces e = 0,6

A

D

2

y x2 __ b. ___ 25 + 9 = 1

c = 4, entonces e = 0,8

79

21

ACTIVIDADES Elipse

9. Escriban las ecuaciones de las elipses. a.

y 3,5 3

c.

y 5

2

4

1

2

–2 1

2

–3 –3,5

x

3

– 3)2 (x + 2)2 (y _______ _______ + =1 25 4

Ecuación: b.

Ecuación:

y2 x2 _____ + __ =1 12,25 4

d.

y 1

–1

2

3 4

x

2

–1

1 –8 –7 –6 –5 –4 –3 –2 –1

1

–2 –1

3

5 6 7

y

x

3

–2

2

–3

1

–4

–4 –3 –2 –1

–5

+ 3)2 (x – 4)2 (y _______ + _______ =1 9 4

Ecuación:

Ecuación:

–1

x

(y – 1)2 (x + 2)2 _______ + _______ =1 1 4

10. Completen con los elementos pedidos. Luego, grafiquen cada elipse en una hoja. (y – 3)2 (x + 2)2 _______ a. _______ =1 25 + 4

centro =

(y + 2)2 (x – 1)2 c. _______ + _______ =1 9 36

(–2;3)

centro =

(1;–2) __

___

a1 = a2 =

(3;3) (–7;3)

; b1 = (–2;5)

; f1 = (–2 + 321 ;3)

(–2;1)

(–2 – 321 ;3)

; b2 =

___

; f2 =

(y+ 1)2 (x – 1)2 _______ b. ______ =1 25 + 4

a1 = (1;4) a2 =

(1;–8)

; b1 = (4;–2) ; b2 =

(–2;–2)

__

; f2 =

(1;–2 – 3 . 33 )

(y – 2)2 (x – 3)2 d. _______ + _______ =1 9 49

centro = (1;–1)

centro = (3;2) ___

___

80

; f1 = (1;–2 + 3 . 33 )

a1 = (1;4)

; b1 = (3;–1)

; f1 = (1;–1 + 321 )

a1 = (10;2)

; b1 = (3;5)

; f1 = (3 + 2 . 310 ;2)

a2 = (1;–6)

; b2 = (–1;1)

; f2 = (1;–1 – 321 )

a2 = (–4;2)

; b2 = (3;–1)

; f2 = (3 – 2 . 310 ;2)

___

___

21

ACTIVIDADES Elipse

11. Grafiquen cada una___de las elipses y escriban las ecuaciones correspondientes. a. a1 = (7;0); f1 = (333 ;0)

c. a1 = (3;7); a2 = (3;–3); f1 = (3;6)

y

y 5 4 3 2 1

–7 –6 –5 –4 –3 –2 –1

Ecuación:

1 2 3 4 5 6 7 –1 –2 –3 –4 –5

7 6 5 4 3 2 1

x

–1 –2 –3

y2 x2 ___ + ___ = 1 49 16

Ecuación:

1 2 3 4 5 6 7

(y – 2)2 (x – 3)2 _______ + _______ =1 25 9 __

b. a1 = (4;1); a2 = (–6;1); f2 = (–4;1)

d. b1 = (–2;7); b2 = (–2;–1); f1 = (–2 + 2 . 35 ;3)

y

y 7 6 5 4 3 2 1

5 4 3 2 1 –8 –7 –6 –5 –4 –3 –2 –1

Ecuación:

x

1 2 3 4 5 6 7 –1 –2 –3 –4

– 1)2 (x + 1)2 (y _______ + _______ = 1 25 16

x

–8 –7 –6 –5 –4 –3 –2 –1

Ecuación:

–11 2 3 4 5 –2 –3

x

– 3)2 (x + 2)2 (y _______ + _______ =1 36 16

mente ACTIVA Hallen la ecuación de la elipse sabiendo que el centro es (0;0), a1 = (6;0) y pasa por __ el punto (4;35 ). y2 x2 + __ ___ =1 9 36 81

22

21

23

24

25

26

27

28

29

30

30

Parábola INFOACTIVA Se llama parábola al conjunto de puntos de un plano que equidistan de un punto fijo llamado foco y de una recta llamada directriz.

{

____

___

P1f = P1d1 ____ ___ P2f = P2d2

P2

P1

f

D d1

d2

Elementos principales de una parábola ˆ ˆ ˆ ˆ ˆ

D

El foco: f. La recta directriz: D. El vértice: v. El eje: E. La distancia del foco a la directriz: P.

E

v

f

P

Ecuación de la parábola Para determinar una parábola se necesita conocer el valor de P y las coordenadas del vértice. Si se ubica la parábola en un sistema de ejes coordenados cartesianos, se obtiene la ecuación de la misma. ˆ El vértice de la parábola es el punto (0;0). D

y y

y2 = 2Px

x2 = 2py

f

f

x

x D

a. y2 = 16x D

b. y2 = –16x

y

y

c. x2 = 16y

d. x2 = –16y y

y

D

4

D 4

–3 x

–4

4

x

–4

x

4

x D

ˆ El vértice de la parábola está desplazado al punto (a;b). y

(y – b)2 = 2P . (x – a)

D P

b

a 82

f

x

–4

–4

Test

de comprensión

1. Respondan y expliquen las respuestas. a. El vértice de una parábola ¿está a la misma distancia del foco y de la directriz? b. La recta directriz ¿es eje de simetría de una parábola? a. Sí. b. No; el eje de simetría es perpendicular a la recta directriz y pasa por el vértice de la misma.

22

ACTIVIDADES Parábola

12. Tengan en cuenta las parábolas y completen con la letra de la ecuación correspondiente. a.

A: x2 = 6y; B: (y – 2)2 = 4 . (x + 3). c.

y

y

6

0

–6

–3

3

6

x

3

6

x

0

–6

–3

b.

3

6

x

A

–6

d.

y

y

6 2 0

–6

–3

0

–6

–3 –2

2 3

6

x

–2

B

13. Escriban la ecuación de cada parábola y de cada recta directriz. a.

b.

y

y

5 –3 0

–1

2

4

f

1

f

0

4

11

x

x –3

Ecuación de la parábola: (y – 1)2 = 8 . (x – 2)

Ecuación de la parábola: (x – 4)2 = 16 . (y + 3)

Ecuación de la directriz: x = 0

Ecuación de la directriz: y = –7

14. Escriban la ecuación de cada parábola. a. f = (0;3) y D: y = 1

b. f = (5;2) y D: x = –3 (y – 2)2 = 16 . (x – 1)

x2 = 4 . (y – 2)

Ecuación:

Ecuación: 83

23

22

24

25

26

27

28

29

30

31

32

Hipérbola INFOACTIVA Se llama hipérbola al conjunto de puntos de un plano, tales que el valor absoluto de la diferencia de sus distancias a dos puntos fijos llamados focos es constante. ___

___

___

p1

___

| p1f1 – p1f2 | = | p2f1 – p2f2 |

f2

f1 p2

Elementos principales de una hipérbola

R1

ˆ El centro: o. ˆ Los vértices: a1 y a2. ˆ Los focos: f1 y f2. ___ ˆ La distancia focal: f1f2 = 2C. ____ ˆ El eje real: a1a2 = ____ 2A. ˆ El eje imaginario: b1b2 = 2B. ˆ Las asíntotas R1 y R2, (rectas que, prolongadas indefinidamente, se acercan continuamente a una curva sin llegar a encontrarla nunca).

b1 a2 o

f2

B a1 A

b2

f1

C R2

En la hipérbola se cumple que: C2 = A2 + B2.

C Como en la hipérbola C > A, la excentricidad es mayor que uno __ A > 1 . En una parábola C = A, por lo tanto la excentricidad de la parábola es uno.

(

)

Ecuación de la hipérbola Para determinar la ecuación de una hipérbola se necesitan conocer las coordenadas del centro y los valores de A y B. Si se ubica la hipérbola en un sistema de ejes cartesianos, se obtiene la ecuación de la misma. ˆ El centro de la hipérbola es el punto (0;0). y

y2 x2 __ – ___ =1 A2 B2

y

y2 x2 __ – ___ =1 A2 B2

B A

A y = __ x B

B y = __ x A

x

A B x

B A

y = – __ x

A y = – __ x B

ˆ El centro de la hipérbola está desplazado al punto (a;b). (y – b)2 (x – a)2 _______ – _______ =1 A2 B2

y b

B A

a 84

x

Test

de comprensión

1. Respondan y expliquen las respuestas. a. El eje real ¿siempre es paralelo al eje x? b. ¿Cuántos ejes de simetría tiene una hipérbola? a. No siempre; puede ser también paralelo al eje y. b. Tiene dos ejes de simetría.

23

ACTIVIDADES Hipérbola

15. Marquen las respuestas correctas. 2

y2

x __ ¿Qué puntos pertenecen a la hipérbola __ 9 – 4 = 1?

X b. (–3;0)

a. (0;–4)

___

2 X d. 310 ;– __ 3

(

c. (–1;2)

)

16. Hallen la excentricidad de las siguientes hipérbolas. (y + 1)2 (x – 4)2 _______ a. _______ 25 = 1 64 –

y2 (x + 3)2 _______ b. ____ 100 – 36 = 1

e = 1,18

e = 1,166

17. Escriban las ecuaciones de cada una de las siguientes hipérbolas. a.

c.

y

y 4

2 –6

3

–3

x

–6

Ecuación:

–2

– 1)2 (x + 2)2 (y _______ – _______ =1 9 16 Ecuación:

y2 x2 __ __ – 4 =1 9

b.

y

d.

y

–4 –1

Ecuación:

2 y2 __ __ – x1 = 1 4

x

3

–2

–2

1

1

x

x

(y + 2)2 (x + 1)2 _______ – _______ =1 25 9 Ecuación: 85

23

ACTIVIDADES Hipérbola

18. Grafiquen las siguientes hipérbolas y completen. a. a1 = (6;3) y b1 = (2;5)

b. a1 = (–1;1) y b1 = (–4;6) y 6

y 5

3

3

1 –7

1 –5 –4 –3 –2

2

–1

–1

–4

x

x

6

–4

(

Centro = A=

4

)

2 ; 3

Centro= __

2

;B=

Ecuación:

2 . 35 ;C=

A=

– 3)2 (x – 2)2 (y _______ _______ – =1 4 16

( –4 ; 1 )

5

___

3

;B=

;C=

334

(y – 1)2 (x + 4)2 _______ _______ – =1 25 9

Ecuación:

19. Resuelvan. a. Completen la siguiente tabla. Centro

A

B

f1

f2

__

(0;0)

2

2 . 33

(4;0)

3

37

(2;1)

6

4

y2 x2 – ___ __ =1 4 12

(–4;0)

(4;0)

__

Ecuación de la hipérbola

(4;4)

y2 (x – 4)2 __ – _______ =1 7 9 2 (y – 1)2 (x – 2) _______ _______ – =1 36 16

(4;–4)

___

___

( 2 + 2 . 313 ;1 )

(2 – 2 . 313 ;1 )

b. Grafiquen las hipérbolas del punto a.

–2

y

y

3

3

1

1

0

2

x

0

y 5

4

1

x –4

–3

86

0

2

8 x

23

ACTIVIDADES Hipérbola

20. Tengan en cuenta la ecuación 4x2 – 9y2 = 144 y resuelvan. Sí; representa una hipérbola. a. ¿Representa una hipérbola? b. En caso de serlo, hallen la ecuación de la hipérbola y la distancia focal.

Ecuación:

y2 x2 ___ ___ – 16 = 1 36

___

Distancia focal:

4 . 313

21. Hallen la ecuación de la hipérbola que cumpla con las condiciones pedidas en cada caso. Luego, grafiquen. a. Un vértice es (8;0) y un foco es (10;0). b. Eje real: 8; eje imaginario: 6; un foco es (–6;–3).

Ecuación:

y2 x2 ____ ___ 100 – 36 = 1

Ecuación:

y

(y + 3)2 (x + 1)2 _______ _______ =1 9 16 –

y

6 –5

2 –8

–2 0

2

8

x

–1 0

3

x

-3

–6 -6

mente ACTIVA Hallen las ecuaciones de las asíntotas de la hipérbola cuyos vértices son a1 = (3;–1) y b1 = (–1;1). 3 1 1 1 __ __ __ y = __ 2 x – 2; y = – 2 x – 2

87

INTEGRACIÓN 22. Completen la tabla y grafiquen.

26. Indiquen el centro, los vértices y los focos. Luego, grafiquen las elipses. a. C: (–3;0),

Centro

Radio

Ecuación de la circunferencia

(0;0)

5

x2 + y2 = 25

(2;–3)

4

(x – 2)2 + (y + 3)2 = 16

__

(–1;4)

37

(x + 1)2 + (y – 4)2 = 7

(–3;0)

3

(x + 3)2 + y2 = 9

___

(5;–2)

2

2

(x –5) + (y + 2) = 15

315

a1 = (–3;4), a2 = (–3;–4), b1 = (–1;0), b __2 = (–5;0), __ f1 = (–3;2 . 33 ), f2 = (–3;–2 . 33 ). b. C: (–3;4), (y – 4)2 (x + 3)2 a1 = (4;4), a2 = (–10;4), _______ _______ b. 49 + 36 = 1 b = (–3;10), b2 = (–3;–2), ___ ___ 1 f1 = (–3 + 313 ;4), f2 = (–3 – 313 ;4). y2 (x + 3)2 _______ a. ___ + =1 4 16

27. Escriban las ecuaciones de las siguientes elipses. 6 a.

23. Escriban la ecuación canónica de cada circunferencia.

C1: x2 + (y + 2)2 = 9 C2: (x – 2)2 + (y – 3)2 = 25

y

1 –7

8

–1

x

–6

b.

1 0

–1

y2 (x + 4)2 _______ ___ + =1 9 36

3

–3

–4

1 2 3

4

6 7 x

–2 1 –4

24. Hallen la ecuación canónica de las circunfe-

–3

–1 0 1 –1 –2

2

7

x

(y – 1)2 (x – 2)2 _______ _______ =1 9 25 +

rencias y determinen el centro y el radio. Luego, 28. Escriban las ecuaciones de cada parábola y grafiquen. a. 2 2 de cada directriz. 2 2 a. x + y – 2x – 6y – 2 = 0 (x – 1) + (y – 3) =__12 y C: (1;3); R = 2 . 3 . 3 a. b. x2 + y2 + 8x + 4y + 4 = 0 2 2 c. x2 + y2 – 10y + 16 = 0 b. (x + 4) + (y + 2) = 16 5 (y –2)2 = –8 . (x – 1) C: (–4;–2), R = 4. c. x2 + (y – 5)2 = 9; C: (0;5), R = 3.

D: x = 5

25. Marquen las opciones correctas.

f

v

¿Cuáles de las siguientes ecuaciones corresponden a elipses? 2

0

y2

2

b.

(x – 4)2 = –6 . (y – 2) D: y = 5

2

y

x X c. – ____ – ____ = –1 0,49 0,16

v

2 1 –1 0 1 –1

88

x

–3

x __ a. – ___ 25 + 4 = 1

X b. 9x2 – 1 = –25y2

2

–1

4

f

9

x

capítulo

CONTENIDOS

4

20*21*22*23 29. Escriban las ecuaciones de cada parábola y

33. Marquen las opciones correctas.

luego grafíquenlas. a. f = (0;2) y D: y = 4 a. x2 = –42 . (y – 3). b. (y – 2) = 8 . (x – 3). b. v = (3;2) y D: x = 1 c. (x + 4)2 = –6 . (y – 1). c. v = (–4;1) y f = (–4;–2)

¿Cuáles de las siguientes ecuaciones corresponden a una cónica? Indiquen de qué cónica se trata. a. x2 + y2 – 4x = x2 – y2 X b. x . (x –3) + 6 . (y + 1) = –y2

30. Marquen las opciones correctas. ¿Cuáles de los siguientes puntos pertenecen a la parábola (x – 1)2 = 16 . (y + 3)? (–2;5) X (5;–2) __ (0;4 . 33 ) X (1;–3)

31. Escriban las ecuaciones de cada hipérbola. a.

y2

2

– 1)2 (x – 2)2 (y _______ – _______ =1 4 16

x __ X c. ____ 100 – 1 = – 4

X d. (y + 1)2 – 4 . (x – 1) = 0 X e. 36y2 = 4 . (x2 – 1)

34. Completen los gráficos con la letra de la ecuación correspondiente. A: y2 = –8x D: y2 = 8x (y – 1)2 (x – 2)2 B: _______ – _______ =1 9 4

3

a. 4 3 2 1

1 2

–2

6

x

–2

–2 –1 0 1 –1 –2

y

(y + 3)2 (x – 1)2 _______ – _______ =1 1 9

2 3 4

5 6

x

F y

1 –1

(y – 1)2 (x – 2)2 E: _______ – _______ =1 9 4

(y – 1)2 (x – 2)2 C: (x –2)2 + (y – 1)2 = 9 F: _______ + _______ =1 9 4

y

b.

b. Circunferencia. c. Elipse. d. Parábola. e. Hipérbola.

2

x

3 2 1

b.

–3

–3 –2 –1 0 –1 –2 –3

–6

1

2 3

x

A

32. Grafiquen las siguientes hipérbolas, escriban

c.

las ecuaciones de las mismas e indiquen los valores del centro, de A, B y C. (y + 1)2 – 3)2 _______ _______ a. a1 = (8;–1) y b1 = (3;2) a. (x – = 1; 9 25 b. a1 = (–2;4) y b1 = (0;0) o: (3;–1); A = 5; B = 3; ___ . c. a1 = (4;3) y b1 = (1;4) C =y2334(x – 2)2 b. ___ –__ _______ = 1; 4 16

o: (–2;0), A = 4; B2 = 2; C = 2 . 35 . ___ (y – 3) (x – 1)2 _______ c. _______ – = 1; o: (1;3); A = 3; B = 1; C = 310 . 1 9

y 3 1 0

–2

2

5

x

–2

B

89

capítulo

4

AUTOEVALUACIÓN Marquen las opciones correctas 35. ¿Cuál es la expresión canónica de la ecuación x2 + y2 + 4x – 10y + 20 = 0? a. (x + 2)2 + (y – 5)2 = 20 X b. (x + 2)2 + (y – 5)2 = 9

c. (x – 5)2 + (y + 2)2 = 9 d. Ninguna de las anteriores.

36. ¿Cuál es la ecuación de la elipse cuyo semidiámetro mayor coincide con el radio de la circunferencia x2 + y2 – 6x – 4y – 12 = 0, el centro coincide con el centro de la misma y uno de los focos es (7;2)? (y – 2)2 (x – 3)2 _______ X a. _______ =1 25 + 9

(y – 2)2 (x – 3)2 _______ c. _______ =1 25 + 4

(y – 2)2 (x – 3)2 _______ b. _______ =1 25 – 9

d. Ninguna de las anteriores. y2 x2 ____ ___ + 225 81 = 1?

37. ¿Cuál es el valor de la excentricidad de la elipse a. 1,25

X c. 0,8

b. 0,6

d. Ninguna de las anteriores.

38. ¿Cuál es la ecuación de la siguiente parábola? 3 2 1

f

–4 –3 –2 –1 0 1 –1 –2 –3

2 3 4 5 6

7 8 9

x

v

a. (x – 3)2 = –12 . (y + 2)

X c. (x – 3)2 = 12 . (y + 2)

b. (x + 3)2 = 12 . (y – 2)

d. Ninguna de las anteriores.

39. ¿Cuál gráfico corresponde a la hipérbola

(y – 4)2 (x – 3)2 _______ – _______ = 1? 4 16

X b.

a.

c.

d.

y

y y

8

y 7 6 5 4 3 2 1

4

–1

1

3

5

x

–2 –1

6 5 4 3 2 1 7

x

–1

3 2 8

x

–1

4

6 x

40. En la hipérbola de vértices a1 = (7;2) y b1 = (1;10), ¿cuál es el valor de C? __

a. 2 . 37

90

X b. 10

c. 6

d. Ninguna de las anteriores.

Contenidos

5

24. Funciones. 25. Función inversa. 26. Interpretación y análisis de gráficos. 27. Función lineal. 28. Función cuadrática. 29. Ecuaciones cuadráticas. 30. Sistemas de ecuaciones lineales. 31. Sistemas de ecuaciones mixtos.

Las ecuaciones lineales y cuadráticas se conocen desde la Antigüedad: los babilonios ya sabían cómo resolverlas y los griegos las emplearon en diversos problemas geométricos. Pero fue recién en los tiempos modernos cuando se empezó a pensar en tales ecuaciones a partir del contexto más general de la teoría de funciones. En esta construcción tuvo un rol fundamental un ilustre personaje que vivió en Italia en el siglo XVII: se trata de Galileo, quien se basó en los principios de Euclides como forma de entender el mundo. En sus estudios sobre el movimiento de los cuerpos aparece “lo cuadrático” como modelo de la variación de la posición respecto del tiempo. Para Galileo, la parábola ya no es tan solo un objeto geométrico sino, fundamentalmente, un punto que se mueve; a partir de allí, el estudio matemático de la naturaleza estuvo siempre basado en las funciones y sus propiedades.

1. Lean atentamente y respondan. a. ¿Por qué les parece que las funciones matemáticas son importantes para entender el mundo? b. ¿Por qué Galileo considera la parábola como un punto en movimiento? a. Para estudiar los distintos fenómenos se emplean modelos que se basan en relaciones matemáticas. b. Porque no le interesaba tanto la parábola como objeto matemático estático sino más bien como parte de sus estudios acerca del movimiento de los cuerpos.

capítulo

Funciones

24

23

25

26

27

28

29

30

31

32

33

Funciones INFOACTIVA Una función es una relación entre dos variables en la cual a cada valor de la primera (independiente) le corresponde un único valor de la segunda (dependiente). El conjunto dominio de la función ( Df ) está formado por los valores que puede tomar la variable independiente. El conjunto codominio ( Cf ) está formado por todos los valores que puede tomar la variable dependiente. El conjunto imagen ( Im ) es un subconjunto del codominio formado por los valores que toma la función. La imagen de x a través de la función f se denota con la expresión y = f(x). f: A → B es función de A en B ⇔ ∀ x ∈ A: ∃! y ∈ B / y = f(x)

(∀ : “para todo”; ∃! : “existe un único”.)

La representación gráfica de una función es el conjunto de todos los puntos (x;y), para los cuales (x;y) es un par ordenado de f.

Clasificación de funciones Una función es inyectiva si y solo si a elementos distintos del dominio les corresponden imágenes distintas en el codominio. ∀ x ∈ A: x1 ≠ x2 ⇒ f(x1) ≠ f(x2) ⇔ ∀ x ∈ A: f(x1) = f(x2) ⇒ x1 = x2 f es inyectiva si no existe una recta paralela al eje x que pase por dos puntos de f. f no es inyectiva si existe una recta paralela al eje x que pase por dos puntos de f. Una función es sobreyectiva si y solo si a todo elemento del codominio le corresponde una preimagen en el dominio. ∀ y ∈ B: ∃ x ∈ A / y = f(x) Una función es biyectiva si y solo si es inyectiva y sobreyectiva.

Las siguientes funciones están definidas de [–3;5] → [–2;3] y 3 0

–6

–3

1 2 3 4 5

x

–2

y 5 4 3 2 0

–6

1 2 3 4 5

–3 –2

92

x

f no es inyectiva porque existe una recta A, paralela al eje x, que pasa por dos puntos de la función f. f es sobreyectiva porque a todo elemento del codominio [–2;3] le corresponde una preimagen en el dominio [–3;5]. f no es biyectiva porque no es inyectiva.

f es inyectiva porque no existe una recta A, paralela al eje x, que pase por dos puntos de la función f. f es sobreyectiva porque a todo elemento del codominio [–2;4] le corresponde una preimagen en el dominio [–3;5]. f es biyectiva porque es inyectiva y sobreyectiva.

Test

de comprensión

1. Respondan y expliquen las respuestas. a. En una función ¿el codominio está incluido en la imagen o es al revés? b. ¿Es cierto que si el conjunto codominio es igual al conjunto imagen, la función es sobreyectiva? a. Es al revés.; b. Sí, porque significa que cada elemento del conjunto codominio tiene una preimagen.

24

ACTIVIDADES Funciones

1. Marquen las opciones correctas. ¿Cuáles gráficos corresponden a una función de a. X b. y

→ ? c.

d. X

y

y

y

x

x

x

x

2. Escriban el dominio y la imagen de las funciones representadas en cada gráfico. a.

c.

e.

y

y

y 2 1 2

x

–2

0

–1

1

2

x

–1 0

–2

; Im =

Df =

Df =

b.

d.

–2

x

; Im = [2;+')

; Im = {2}

Df = f.

y

y

y

5 3

–2

0

2

0 1

2

3

4 x

x –2

0 –2

–4

1

3 x

–3 –4

–2

Df = [–2;1]

1

–1

; Im = [–2;3]

Df =[–4;–2) F [0;3] ; Im = [–2;0] F (2;5]

Df =

+

; Im = (–';0) 93

24

ACTIVIDADES Funciones

3. Hallen el dominio de cada función. 3 a. f(x) = _____ 1 –x __ 2

Df =

5–x b. f(x) = _______ x2 – 25

Df =

e. f(x) = 3x + 4

Df =

f. f(x) = ln( 5x –2 )

Df =

1 – __ 2

{ }





– { 5;–5 }

c. f(x) = x2 – 25

Df =

g. f(x) = –e5x

Df =

h. f(x) = ln( 3x )x

Df =

( __52;+')



 x__– 2 d. f(x) = _______ 2 3x – 9

Df =

+

(–';–3) F (3;+')

4. Completen con las imágenes pedidas en cada caso. 1 a. f(x) = ______ x2 + 5 1 __

f(–2) = 9

f __51 =

( )

25 ____ 126

1 ___ f(–5) = 30

__1 b. g(x) = _______ 2

–2 3x __ __ 2 – 1 g( –32 ) = – 3___ 2

g(–2) = No está definida.

g(0) =

94

1 – __ 2

______

c. h(x) = log(3x)

e. n(x) =3x2 + 4

h(–3) = ’

n(–4) = 2 . 35

h __31 =

0

10 h ___ 3 =

1

( )

__

n(0) =

2

__

( )

n(1) =

35

d. m(x) = e–3x

f. p(x) = 2– x + 1

m – __31 = e

p(–1) = 4

( )

m __31 =

( )

__1 e

p(1) =

1

m(0) =

1

p(2) =

1 __ 2

24

ACTIVIDADES Funciones → .

5. Marquen las opciones correctas. Tengan en cuenta que son funciones definidas de a. ¿Cuáles funciones son inyectivas? X y

y

y

x

x

y

x

x

b. ¿Cuáles funciones son sobreyectivas? X

X y

y

y

x

x

y

x

x

c. ¿Cuáles funciones son biyectivas? X y

y

x

y

x

y

x

x

mente ACTIVA Definan un dominio y un codominio para que las funciones sean biyectivas. a. f(x) = x2 c. h(x) = __x1 1 2 b. g(x) = x + 1 d. i(x) = _____ x+1 Solución a cargo del alumno.

95

25

24

26

27

28

29

30

31

32

33

34

Función inversa ¿Para qué sirve?

INFOACTIVA

PÁGINA 7

Dada f: A → B / y = f(x) se puede obtener una función inversa f –1: B → A / x = f –1(y) que solo existe en el caso de que la función f sea biyectiva. Para obtener la fórmula de la función inversa, se debe realizar un cambio de variables.

f:



/ f(x) = 2x – 5 y 2

0

–4

ˆ f es inyectiva porque no existe una recta A paralela al eje x que pase por dos puntos de la función f. ˆ f es sobreyectiva porque a todo elemento del codominio le corresponde una preimagen en el dominio. ˆ f es biyectiva porque es inyectiva y sobreyectiva.

4 x

y+5

5 1 y + __ = x ⇒ x = __ y = 2x – 5 ⇒ _____ 2 2 2

Realizando el cambio de variable se obtiene la función inversa. →

f –1: –5

5 1 x + __ / f –1(x) = __ 2 2

Las representaciones gráficas de dos funciones inversas son simétricas respecto de la función y = x.

f:



/ f(x) = x3 – 2 y f –1:



_____

3 / f –1(x) = 3 x+2

y

f(x) = x3 – 2

5

f(x) = x _____

3 f –1(x) = 3 x+2

0 –2

5

–5

96

x

de comprensión

Test

1. Respondan y expliquen las respuestas. a. Si f(x) = 3x + 2, ¿cuánto vale f –1(2)? ¿Y f –1(5)? b. ¿Es cierto que f(x) = – __31 x y g(x) = 3x son funciones inversas?

4 a. f –1(2) = – __ y f –1(5) = 1; b. No, porque la inversa de f(x) sería f –1(x) = –3x. 3

25

ACTIVIDADES Función inversa

6. Marquen las opciones correctas. ¿En cuáles de los siguientes gráficos están representadas funciones inversas? a. X

–2

b.

c.

d.

y

y

y

y

2

2

4

2

1

1 0

–1

1

x

2

1 0

–1

–1

–1

–2

–2

1

x

0

–1

1

2

3

x

–1 0

–2

4 x –1

–3

c. Una de las curvas no es función.

7. Determinen un dominio para cada función de manera que se pueda obtener la función inversa. Luego, hallen la inversa de cada función e indiquen su dominio. a. f(x) = 3x – 6

f –1(x) =

1x+2 __ 3

;

b. g(x) = 4x + 2

g –1(x) =

x_____ –2 4

;

c. h(x) = 2x2 + 3

Df =

d. k(x) = x3

Df =

k –1(x) =

–1

x–3

2 3 _____

e. l(x) = 3x + 1

Dg =



l –1(x) =

–1

+

Dh = –1

[3;+')

; Dk = –1

_____



Dh =

;

3 __

3x

Dg =

x2 – 1

Dl =

m –1(x) =

(_______ x – 1 )2

2

[–1;+∞)

; Dl =

[0;+∞)

Dm =

[0;+∞)

; Dm =

[1;+∞)

–1

___

f. m(x) = 32x + 1

_____

h –1(x) =

Dk =

–1

97

26

25

27

28

29

30

31

32

33

34

35

Interpretación y análisis de gráficos INFOACTIVA Analizar el gráfico de una función consiste en describir el comportamiento de la misma. Se debe tener en cuenta: ˆ La continuidad y el dominio. ˆ Las variables que relacionan la función. ˆ Intersecciones con los ejes, máximos y mínimos. ˆ Intervalos de crecimiento, de decrecimiento y constantes. Distancia (en m) y

t El siguiente gráfico representa a qué distancia (en metros) se encuentra una persona de su casa en función del tiempo transcurrido.

500 400 300 200 100 0

1

3

13

15 x

5 7 9 11 13 Tiempo (en meses)

15 x

5 7 9 11 Tiempo (en min)

a. ¿Cuáles son las variables independiente y dependiente? La variable independiente es el tiempo y la dependiente es la distancia. b. ¿Cuál es el dominio de la función? Es una función continua definida para 0 ≤ x ≤15. c. ¿Dónde se encuentra la persona en x = 0? La persona se encuentra en su casa. d. ¿En qué momento alcanza la máxima distancia a su casa? En el minuto 4. e. En algún momento ¿la distancia se mantiene constante? Sí, entre el minuto 6 y el 10. f. ¿Cuáles son los intervalos de crecimiento y de decrecimiento de la función? La función crece en (0;4) y decrece en (4;6) y (10;15). Ganancia (en $) y

t El siguiente gráfico representa la ganancia mensual (en $) de una empresa.

50.000 40.000 30.000 20.000 10.000 0

1

3

a. ¿Cuáles son las variables independiente y dependiente? La variable independiente es el tiempo y la dependiente es la ganancia. b. ¿Cuál es el dominio de la función? Es una función discontinua definida para 1 ≤ x ≤ 12 ∧ x ∈ ∧ x : tiempo en meses. c. ¿En qué mes se obtuvo la mayor ganancia? ¿Y la menor? En el mes 7 ($50 000). En el mes 1 ($10 000). d. ¿En qué meses obtuvo la misma ganancia? En los meses 6, 9 y 10 ($40 000) y en los meses 5 y 8 ($45 000). 98

Test

de comprensión

1. Respondan y expliquen las respuestas. a. Según la convención, ¿es cierto que la variable independiente siempre se coloca en el eje x y la dependiente, en el eje y? b. En un intervalo de crecimiento, ¿se representan valores correspondientes al eje x o al eje y? a. Sí, es correcto. b. En cada intervalo se representan los valores de x en los cuales la función crece.

26

ACTIVIDADES Interpretación y análisis de gráficos

8. En el siguiente gráfico se representa la altura de un avión desde que sale del aeropuerto de Salta hasta que llega al aeropuerto de Buenos Aires. a. Completen las coordenadas del gráfico. Altura (m)

y

10 000

8 750 6 250

0

1 __ 6

1 __ 3

1 __ 2

5 __ 6

1

7 __ 6

3 __ 2

13 ___ 6

x

Tiempo (h)

b. ¿Cuál es la variable independiente? ¿Y la dependiente? Variable independiente: tiempo; variable dependiente: altura.

c. ¿Cuánto tiempo duró el viaje? 2 horas y 10 minutos.

d. ¿Cuál fue la altura máxima? ¿Cuánto tiempo tardó en llegar a esa altura? 10 000 m; 30 minutos

e. ¿Cuántas veces voló a una altura constante? ¿Cuánto tiempo voló a esa altura? Voló dos veces a una altura constante. En total, 40 min.

f. ¿Cuántas veces estuvo a 4 km de altura? 2 veces. g. ¿A qué altura estaba luego de 1 hora y 10 minutos de viaje? ¿Y luego de 2 horas? A 10 000 m de altura. A 2 500 m de altura.

h. ¿En qué momentos ascendió? ¿Y cuándo descendió? Escriban los intervalos de crecimiento y de decrecimiento. Ascendió durante la primera media hora y desde 1 h hasta 1 h 10 min. Descendió desde los 50 min hasta 1 h 5 7 3 ___ 13 1 __ __ __ y desde 1 h 30 min hasta 2 h 10 min. Crece en 0;__ 2 y 1; 6 . Decrece en 6 ;1 y 2 ; 6 .

( ) ( )

( ) (

)

i. ¿Cuál fue la distancia total del viaje? No se puede saber cuál fue la distancia total del viaje porque lo que se representa en el gráfico es la altura. 99

27

26

28

29

30

31

32

33

34

35

36

Función lineal INFOACTIVA Una función lineal es aquella de la forma f(x) = ax + b, siendo a y b números reales. Los coeficientes principal e independiente de la función reciben los nombres de pendiente y ordenada al origen, respectivamente. La representación gráfica de una función lineal es una recta. La pendiente de una recta es el cociente entre la variación de la variable dependiente (Δy) y la variación de la variable independiente (Δx) de cualquier punto de la misma. y2 – y1

Δy

___ a = ______ x – x = Δx 2

1

La ordenada al origen es el valor donde la recta interseca al eje y. y

f(0) = b y2

Δy

y1

Δx

b 0 x1

x2

x

Ecuaciones de la recta La ecuación de una recta se puede expresar de distintas formas. Ecuación explícita Ecuación implícita Ecuación segmentaria y x __ __ y = ax + b cx + dy + e = 0 m + n = 1 ↑ ↑ Abscisa al origen. Ordenada al origen. Dos rectas son paralelas si y solo si sus pendientes son iguales. M: y = a1x + b1 ∧ P: y = a2x + b2 ∧ M // P ‹ a1 = a2 Dos rectas son perpendiculares si y solo si sus pendientes son inversas y opuestas. 1 M: y = a1x + b1 ∧ N: y = a2x + b2 ∧ S Œ N ‹ a1 = – __ a 2

Las siguientes fórmulas permiten encontrar la ecuación de una recta. ˆ Dada su pendiente (a) y un punto que pertenece a la misma ( x1;y1 ). ˆ Dados dos puntos que pertenecen a la misma ( x1;y1 ) y ( x2;y2 ).

100

y – y1 = a . (x – x1 ) y – y1 x – x1 ______ ______ y2 – y1 = x2 – x1

Test

de comprensión

1. Respondan y expliquen las respuestas. a. ¿Es cierto que en y = x + __21 , la ordenada al origen es __21 y la pendiente es 0? b. ¿Es cierto que las rectas y = – __31 x e y = 3x + __31 son perpendiculares? 1 a. La ordenada al origen sí es __ 2 , pero la pendiente es 1. b. Sí.

27

ACTIVIDADES Función lineal

9. Marquen las opciones correctas. ¿Cuáles de las siguientes funciones tienen pendientes negativas? a. X f(x) = –3x + 2

f(x) = __31 x – 5

b.

c. X f(x) = – __32 x – __41

d.

f(x) = 7x – 1

10. Completen con la función correspondiente a cada gráfico. f(x) = 2x + 2; g(x) = 2x – 2; h(x) = –2x + 2; i(x) = –2x – 2 a. b. c. y y

1 –2

–2

0

–1

1

0

–1

y

1

1

2

d.

y

1

x

2

2

0

–1

–1

–1

–2

–2

2

1

1

x

x

2

–1

–1

h(x) = –2x + 2

g(x) = 2x – 2

0

1

2

x

2

x

–1

i(x) = –2x – 2

f(x) = 2x + 2

11. Escriban la fórmula correspondiente a cada una de las funciones. a.

b.

y

c.

y

4

–2

–1

2

3

6 x

0

–1

f(x) = –0,5x + 3

1

2

x

1

–1

1

–2 0 –2

y

0

2

2

d.

y

1

2

–2

x

0

–1

–1

–3

–1

–2

–4

–2

f(x) = 4x – 2

f(x) = –2x – 4

1

f(x) = x

12. Grafiquen las siguientes funciones lineales. b. f(x) = __45 x – 4

a. f(x) = – __21 x + 5 5

y

y

1 2 3 4 5 6

-1

2

4

6

8

10

x

x

-4 101

27

ACTIVIDADES Función lineal

13. Escriban la ecuación explícita y segmentaria de cada una de las rectas. a.

b.

c.

y

y

y

y

1

1

1

2 0

–1

d.

1

2

3

0

x

1

2

1

4 x

3

–1

0

2

3

x

d. G: y = __23 x – __32 ; H: y – __23 x = __23

G

//

H

–1

–1

1

–1 0

–1

1

2

x

–2

–2

2x–2 y = __ 3

1x+1 y = – __ 4

y=2

1 y = __ 2x–1

y x + ___ __ =1 3 –2

y __ x __ 1 + 4 =1

y __ 2 =1

y x + __ __ =1 2 –1

–2

–1

14. Hallen la ecuación segmentaria de las rectas dadas. a. y = – __31 x – 3 b. y = –4x + __51 c. y = __32 x – __34 d. y = – __25 x + 2 e. y = –4

x + ___ –9 x ____ 0,05

y ___ =1 –3 y + ___ =1 0,2

y x + __ ___ =1 0,8 2 y ___ = 1 –4

y x ___ __ =1 2 + __ 4 –3

15. Relacionen las tres ecuaciones que definen a la misma recta. a. y = –3x + 3 b. y = –3x – 3 c. y = 3x – 3 d. y = 3x + 3

y

e. __x1 + ___ =1 y–3 f. __x1 + __3 = 1 y x __ g. __ –1 + 3y = 1 x ___ h. __ –1 + –3 = 1

i. y – 3x – 3 = 0 j. y + 3x – 3 = 0 k. y + 3x + 3 = 0 l. y – 3x + 3 = 0

a con f y j.; b con h y k.; c con e y l.; d con g e i.

16. Hallen la ecuación de la recta, teniendo en cuenta las condiciones pedidas. a. La ecuación segmentaria de una recta cuya pendiente es – __31 y pasa por el punto (–2;–1). y x ___ 1 = ___ –5 + – 5 __ 3

b. La ecuación segmentaria de una recta que pasa por los puntos (2;2) y (–2;–8). y x + ___ __ =1 6 –3 __ 5

17. Completen con // o ⊥ según corresponda, cuando sea posible.

102

a. A: y = – __21 x + 2; B: y = 2x – 5

A

ΠB

b. C: y = 7x + __31 ; D: y = __71 x – 2

C

D

e. I: y + 4x = __21 ; J: y = – __41 x – 2

I

c. E: y = –5x – 4; F: y = –5x + __41

E

F

f. K: y + __43 x = –5; L: __34 x = y – 2

K

//

J Œ

L

27

ACTIVIDADES Función lineal

18. Hallen la ecuación de la recta pedida según las condiciones dadas. a. Una recta que pasa por el punto (4;3) y es paralela a y = –2x + 2. y = –2x + 11

b. Una recta que pasa por el punto (2;–1) y es perpendicular a y = __51 x – 3. y = –5x + 9

19. Hallen gráfica y analíticamente la ecuación de la recta solicitada. a. Una recta paralela a A y que pase por el punto (4;–2).

c. Una recta perpendicular a C que pase por el origen de las coordenadas.

y

A

y

4 3 2 1

4 3 2 1

C

–5 –4 –3 –2 –1 0 1 –1 –2 –3

2 3 4 5

–5 –4 –3 –2 –1 0 1 –1 –2 –3

x

2 3 4 5

x

y = – __41 x – 1

y = – __21 x

b. Una recta paralela a B que pase por el punto (1;1).

d. Una recta perpendicular a D que pase por el punto (0;2).

y

y

4 3 2 B 1 –5 –4 –3 –2 –1 0 1 –1 –2 –3

y=

2x – 1

4 3 2 1

D

2 3 4 5

x

–5 –4 –3 –2 –1

–1 –2 –3

y=

0 1

2 3 4 5

x

1 __ 2x+2

mente ACTIVA a. ¿Cuántas rectas perpendiculares a otra se pueden trazar? Infinitas. b. ¿Cuántas rectas paralelas a otra y que pasen por un punto determinado se pueden trazar? Una. c. ¿Cuántas rectas que pasen por dos puntos determinados se pueden trazar? Una.

103

28

27

29

30

31

32

33

34

35

36

37

Función cuadrática ¿Para qué sirve?

INFOACTIVA

PÁGINA 8

A la función polinómica de segundo grado f(x) = ax2 + bx + c, siendo a, b y c números reales y a ≠ 0, se la denomina función cuadrática. Los términos de la función reciben los siguientes nombres: f(x) = ax2 + bx + c Término cuadrático

Término independiente Término lineal La representación gráfica de una función cuadrática es una parábola.

Gráfica de la parábola Para realizar el gráfico de una parábola, f(x) = ax2 + bx + c, se deben calcular los elementos de la misma y luego representarla. ˆ Raíces de la parábola. Son los puntos de intersección de la parábola con el eje x. Es decir, cuando f(x) = 0. ________

–b ± 3b2 – 4ac x1;2 = ______________ 2a

y

(Δ = b2 – 4ac) ← Discriminante ˆ Vértice de la parábola. Las coordenadas del vértice son: v = ( xv;f(xv ) )

Ordenada al origen Raíz x1

x1 + x2 xv = ______ 2

–b xv = ___ 2a

Vértice

7 6 5

yv = f(xv)

Punto simétrico a (0;4)

3 2 1

–2 –1 –1 –2

Raíz x2 1 2 3

4 5 6 7 8

x

Eje de simetría

ˆ Eje de simetría. Es la recta que tiene por ecuación x = xv.

ˆ Ordenada al origen. Es el punto de intersección de la parábola con el eje y. Es decir, cuando f(0) = c.

Ecuación polinómica, canónica y factorizada La función cuadrática puede ser expresada de distintas formas.

Se desarrolla el cuadrado de un binomio.

Polinómica f(x) = ax2 + bx + c

Se aplica la propiedad distributiva.

Se buscan las raíces.

Canónica f(x) = a . (x – xv)2 + yv 104

Se busca el vértice.

Factorizada f(x) = a . (x – x1 ) . (x – x2 )

de comprensión

Test

1. Respondan y expliquen las respuestas. a. En una función cuadrática, ¿qué valores puede tomar el discriminante para que tenga dos raíces reales? ¿Y para que tenga solo una? ¿Y para que no tenga ninguna? b. Si una función cuadrática tiene raíces complejas, ¿corta al eje x en algún punto? a. Para que tenga 2 raíces reales, Δ > 0; para que tenga una sola, Δ = 0 y para que no tenga ninguna, Δ < 0. b. No.

28

ACTIVIDADES Función cuadrática

20. Completen con el ítem de la función correspondiente en cada caso. b. y = (x + 1)2

a. y = x2 + 1 y

–2

d. y = (x – 1)2

c. y = x2 – 1

y

y

y

3

3

3

3

2

2

2

2

1

1

1

1

0

–1

1

x

2

–2

0

–1

–1

1

x

2

–2

0

–1

–1

1

2

x

–2

0

–1

–1

b.

1

2

x

–1

d.

a.

c.

21. Marquen las opciones correctas. a. ¿Cuál es la parábola que corresponde a la función y = x2 – 2x + 2? X y

–2

y

y

3

3

3

1

2

2

2

1

1

1

0

–1

y

2

x

1

–1 –1 –2

0

1

–1

2

x

–1 0 –1

1

2

x

–1

0

1

2

x

–1

b. ¿Cuáles de las siguientes funciones tienen raíces reales? X y = –4x2 + 2x

y = –3x2 – 2

y = x2 – 2x + 6

y = x2 + 5

22. Tengan en cuenta el gráfico de la función y escriban V (Verdadero) o F (Falso) según corresponda. y

–1

3

a. Δ < 0 F

2

b. Tiene raíces reales. V

1

c. Alcanza su valor mínimo en el vértice. F

0 –1

1

2

3

4 x

d. Crece en el intervalo (–∞;2). V e. Tiene ordenada al origen negativa. V

–2 105

28

ACTIVIDADES Función cuadrática

23. Completen la siguiente tabla. Luego, grafiquen las funciones en una hoja. a

b

c

Vértice

Eje de simetría

Raíces reales

Ordenada al origen

Conjunto imagen

y = __21 x2 – 8 y = x2 + 5x + 4

1 __ 2

0

–8

(0;–8)

x=0

4y–4

(0;–8)

[–8;+')

1

5

4

(–2,5;–2,25)

x = –2,25

–4 y –1

(0;4)

[–2,25;+ ')

y = –3x2 + 6x

–3

6

0

(1;3)

x=1

0y2

(0;0)

(– ';3]

y = –x2 + 2x – 3

–1

2

–3

(1;–2)

x=1

No tiene.

(0;–3)

(– ';–2]

Función

24. Hallen la fórmula correspondiente a cada gráfico. a.

c.

e.

y

y

y

4 1

1 –3

–2

0

–1

1

x

3 0

–1

–1

–1

–2

–2

–3

–3

–4

–4

1

2

3

x

2 1 –2

0

–1

1

x

2

–1

y = x2 + 2x – 3

y = 4x2 – 16x + 12

b.

y = –4x2 + 4

d.

f.

y

y

y

5

2 1

4 1

–1

1 –1 –2 –3

2

3

3 –1

1

–3

106

3

2 1

–2

–4

y = –2x2 + 6x – 4

2

–1

–1

1 –1

3 1 2 __ y = – __ 2x +x+ 2

y = 3x2 – 6x + 5

2

x

28

ACTIVIDADES Función cuadrática

25. Hallen la forma polinómica, canónica y factorizada de las siguientes funciones cuadráticas. a. El coeficiente principal es __41 y tiene raíces en x = –2 y x = 6. 1 x2 – x – 3; y = __ 1 . (x – 2)2 – 4; y = __ 1 . (x + 2) . (x – 6) y = __ 4 4 4

b. El coeficiente principal es –2 y tiene raíces en x = – __21 y x = __41 . 2 1 1 1 __ __ y = –2x2 – __ 2 x + 4 ; y = –2 . x + 8

(

)

9 1 1 __ + ___ ; y = –2 . x + __ 2 . x– 4 32

(

)(

)

c. El coeficiente principal es –1 y el vértice es (–2;1). y = –x2 – 4x – 3; y = – (x + 2)2 + 1; y = – (x + 3) . (x + 1)

d. El coeficiente principal es __52 y el vértice es (3;–1).___

(

)(

___

13 __ 10 10 2 2 ___ 12 2 2 3___ 3___ ___ __ 2 y = __ 5 x – 5 x + 5 ; 5 . (x – 3) – 1; y = 5 . x – 3 – 2 . x – 3 + 2

)

26. Grafiquen las funciones de la actividad anterior en un mismo par de ejes coordenados. Luego, escriban el máximo o el mínimo y el intervalo de crecimiento y de decrecimiento de cada función. a. Máx =

; Mín =

y 6 5 4 3 2 1 –6 –5 –4 –3 –2 –1

c.



–1 –2 –3 –4 –5 –6

Crece:

(2;+')

b. Máx =

d.

Crece: 01

2 3 4 5 6 b.

; Decrece:

(2;–4) (–';2)

9 ( __81 ;___ 32 ) ; Mín = ( –';__81 ) ; Decrece: ( __81 ;+' )

x

c. Máx =

(–2;1)

a.

Crece:

(–';–2)

d. Máx = Crece:

; Mín = ; Decrece: ; Mín =

(3;+')

; Decrece:

(–2;+') (3;–2) (–';3)

mente ACTIVA En la función y = __21 x2 + bx + c, la distancia entre las raíces es 6 y xv = –1. Hallen los valores de b y de c. b = 1; c = –4

107

INTEGRACIÓN 27. Clasifiquen las siguientes funciones.

30. Calculen el dominio de cada función.

y

a. f: [–3;3] → [–3;3]

3 2 1

Sobreyectiva –3 –2 –1

–1 –2 –3

0 1

2 3

x

a. f(x) = –3x + __31



1 (–3;+') _____ e. f(x) = ______

b.  f(x) = x2 – 4



f. f(x) = 3x + 1

3x + 3 _____

3

2x +5 x ______ ______ c.  f(x) = x + 3 – {–3} g. f(x) = 4x – 1 _____

d. f(x) =

1 – __ 4

{ }

1 x + __21 – __1 ;+' h. f(x) = ___ 3 __ 2 3x

3

[

)

– {0}

31. Hallen la función inversa de cada función. b. f: [–3;3] → [–3;3]

y 3 2 1

Inyectiva

–3 –2 –1 0 1 –1 –2 –3

2 3

x

28. Indiquen el dominio y el codominio para que cada gráfico represente una función. a. y 4 3 2 1 –4 –3 –2 –1 0 1 –1 –2 –3

2 3 4 x

_____

a. f(x) = __51 x + 8

d. f(x) = 3x + 3

b. f(x) = 5x2 – 2

e. f(x) = x3 – 1

+2 ______ c. f(x) = 3x 2x + 1_______

f. f(x) = 33 x + 2 (x – 2)3

__

_____ 3 1 2 x–2 __ ______ 2 a. 5x – 40; b. __ 5 x + 5 ; c. 3 – 2x ; d. x – 3; e. 3x + 1

3

32. El siguiente gráfico representa la recaudación por entradas vendidas según el tipo de ubicación, para un festival durante un fin de semana. Precios por ubicación: ˆ Súper Pullman (filas 1 a 16): $400. ˆ Pullman: $250. ˆ Campo: $200. ˆ Cabecera: $100. Recaudación (en miles) y 300

Df =

[–4;4]

; Co =

[–2;4]

200

b.

y

150

4 3 2 1 –4 –3 –2 –1 –1 –2 –3

Df =

[–1;2]

100

1

; Co =

2 3 4 x

Valor de la entrada

0 100

[–2;3]

29. En la actividad anterior, ¿cuántos dominios diferentes se pueden hallar? ¿Y codominios? Un único dominio e infinitos codominios.

108

250

200 250 300

400

x

a. ¿Cuál es la variable dependiente? ¿Y la independiente? Recaudación y valor de entrada. b. ¿Cuál es el dominio? Dom = {100;200;250;400} c. ¿Qué ubicación tuvo mayor recaudación? d. ¿Cuál fue la recaudación del fin de semana? e. ¿Cuántas entradas de cada ubicación se venc. Campo. d. $900 000. dieron? e. Súper Pullman: 500; Pullman: 1 000; Campo: 1 500; Cabecera: 1 500.

capítulo

CONTENIDOS

5

24*25*26*27*28 33. Hallen la ecuación explícita de la recta que

37. Escriban fórmulas que puedan representar a

tenga la pendiente y pase por el punto dado en cada caso. y = 2x + 3 a. Pendiente: 2; a = (1;5) y = –2 b. Pendiente: 0; a = (3;–2) 1 __ y = –5x c. Pendiente: –5; a = – 5 ;1 3 3 __ y = __ d. Pendiente: 2 ; a = (2;–4) 2x–7 y = –4x + 5 e. Pendiente: –4; a = __21 ;3

las funciones. a.

(

4 3 2 1

)

( )

d.

1 – __ 3

4 2 ; d. y = – __ 1 c. y = __ x – __ 2x–3 3 3

36. Hallen lo pedido para cada una de las siguientes funciones. Luego, grafíquenlas. Dominio, imagen, raíces, vértice, eje de simetría, crecimiento, decrecimiento y máximo o mínimo. a. – __21 x2 + 3x – __25 b. x2 + 2x + 3 c. 2x2 + 3x – 4 d. –(x + 1)2 + 2 e. (3x – 12) . (x – 5) f. – __21 x2 + 4x Por ejemplo, a. Df = ; Im = (–∞;2]; x1 = 1; x2 = 5; v(máx) = f(3) = 2; eje: x = 3; C: (–∞;3); D: (3;+∞).

x

y f(x)

4 3 2 1

8

con las condiciones pedidas en cada caso. Luego, grafiquen. a. La recta que pasa por el punto (–2;1) y es paralela a la recta y = – __31 x + 3. y = – __31 x + __31 b. La recta que pasa por el punto (4;–3) y es 23 paralela a la recta y = __52 x. y = __52 x – ___ 5 c. La recta que pasa por el punto (–1;–2) y es perpendicular a la recta y = – __43 x + 5. d. La recta que es perpendicular a la recta y = 2x – __21 y pasa por el punto (–2;–2).

2 3

b.

+ 1 = 1; e. 1 + –4 = 1 __

35. Hallen la ecuación de la recta que cumpla

Pueden ser: f(x) = x2 + 1 g(x) = x2 h(x) = x2 – 3

f(x)

–3 –2 –1 0 1 –1 h(x) –2 –3

34. Hallen la ecuación segmentaria de la recta que pase por los puntos dados en cada caso. y x ___ __ a. a = (2;4); b = (1;–1) a. __6 + __1 = 1; – 6 y 5 b. a = (–1;1); b = (3;4) x + __ b. ___ 7 =1 7 __ c. a = (2;–3); b = (–4;1) – 3 __ 4 y x + ___ c. ___ =1 d. a = (5;2); b = (–3;–1) 5 5 __ – 2 – __ 3 e. a = (1;0); b = (2;4) ___ y x x __ __x ___

g(x) y

–3 –2 –1 0 1 –1 –2

38. Escriban corresponda.

g(x)

2 3 4 5

h(x)

Pueden ser: f(x) = (x + 1)2 g(x) = x2 h(x) = (x – 3)2

x

V (Verdadero) o F (Falso) según

a. Si el coeficiente principal de una función cuadrática es positivo, el vértice de la función es un mínimo. V b. Si una función tiene raíces complejas, el gráfico corta al eje x en un solo punto.

F

c. El intervalo de crecimiento representa todos los puntos donde la función es positiva.

F

d. Si las raíces de una función cuadrática son –2 y 6, el eje de simetría es x = 4. F e. Para calcular el eje de simetría de una función cuadrática, se suman las raíces y se divide por dos.

V

109

29

28

30

31

32

33

34

35

36

37

38

Ecuaciones cuadráticas INFOACTIVA Una ecuación de segundo grado es aquella cuya forma general es: ax2 + bx + c = 0 ∧ a D ∧ b D ∧ c D ∧ a ≠ 0

Ecuaciones incompletas ˆ Si b = 0, la ecuación de segundo grado es incompleta de la forma ax2 + c = 0. __ 2 Se debe tener en cuenta que 3x = | x | cuando está igualado a un número real positivo. 2

9 =0 2x2 – ___ 50

x +5=0 – __ 3

9 x2 = ____ 100

1 x2 + 1 = 0 __ 9

x2 = 15

____ __ 9 3x2 = ____ 100 3 | x | = ___ 10 3 ; x = – ___ 3 x1 = ___ 10 2 10

__

3

x2 = –9

___

___

x  ; x D ⇒ x = 3–9

3x2 = 315

___

|

___

___

x1 = 315 ; x2 = –315

__

x = ± i . 39

x | = 315

x1 = 3 i; x2 = –3 i

ˆ Si c = 0, la ecuación de segundo grado es incompleta de la forma ax2 + bx = 0. Se debe tener en cuenta que: m . n = 0 ‰ m = 0 ∨ n = 0 2

2x2 – 4x = 0

x + __ 1x = 0 – __ 5 2

2x = 0 –5x2 – __ 3

2x . (x – 2) = 0

1 x + __ 1 =0 x . ( – __ 2) 5

2 =0 –x . ( 5x + __ 3)

=0 {2x x–2=0

{

x1 = 0; x2 = 2

5 x1 = 0; x2 = __ 2

x=0 1 x + __ 1=0 – __ 2 5

{

–x = 0 2=0 5x + __ 3

2 x1 = 0; x2 = – ___ 15

Ecuaciones completas Si la ecuación es completa, o sea que ninguno de sus coeficientes es nulo, los valores de x que la verifican se hallan aplicando la siguiente fórmula. ________

–b ± b2 – 4ac

3 x1;2 = ______________ 2a

ˆ Δ > 0 ⇒ x1 D

∧ x2 D

2

–3x + 7x – 4 = 0

________________

__

–(7) ± 3(7)2 – 4 . (–3) . (–4) –7 ± 31 = ______ –7 ± 1 ⇒ x = __ 4; x = 1 x1;2 = _______________________ = _______ 1 –6 3 2 –6 2 . (–3)

ˆ Δ = 0 ⇒ x1 D

∧ x2 D

–2x2 + 8x – 8 = 0

________________

x1;2

–(8) ± 3(8)2 – 4 . (–2) . (–8) = _______________________ 2 . (–2)

ˆ Δ < 0 ⇒ x1 D

__

± 30 = __ 8=2 ⇒x =x =2 _______ = –8 2 1 4 –4

∧ x2 D

5x2 + 6x + 5 = 0

______________ 2

____

–(6) ± 3(6) – 4 . (5) . (5) –6 ± 3–64 = –6 ± 8 i ⇒ x = – __ 3 – __ 3 + __ 4 i; x = – __ 4i _______ x1; x2 = _____________________ = _________ 1 10 5 5 2 5 5 10 2 . (5) 110

Test

de comprensión

1. Respondan y expliquen las respuestas. a. ¿Es correcto decir que 0x2 es igual a x2? b. ¿Es cierto que si el discriminante es menor a cero, la ecuación no tiene solución? a. No, porque 0x2 = 0 y x2 = 1x2. b. No tiene solución en el conjunto de los números reales, pero sí tiene solución en el conjunto de los números complejos.

29

ACTIVIDADES Ecuaciones cuadráticas

39. Resuelvan las siguientes ecuaciones. a. x2 – 5 = 15

d. x2 + 6 = –x2 + 4x

x2 = 20

2x2 – 4x + 6 = 0

___ |

x | = 320 __

__

__

x1 = 2 . 35 ; x2 = –2 . 35

__

x1 = 1 + 32 i; x2 = 1 – 32 i

e. x2 + x = 4 + __31 x

b. 4x2 – x = 2x

2x–4=0 x2 + __ 3

4x2 – 3x = 0 x . (4x – 3) = 0

___

___

37 37 1 + 3____ 1 – 3____ x1 = – __ ; x2 = – __ 3 3 3 3

3 x1 = 0; x2 = __ 4

f. (x – 3)2 = 16

c. (2x – 1) . (x + 3) = –3 2x2 + 6x – x – 3 = –3 2x2 + 5x = 0 x . (2x + 5) = 0

x2 – 6x + 9 = 16

5 x1 = 0; x2 = – __ 2

x1 = 7; x2 = –1

x2 – 6x – 7 = 0

40. Planteen la ecuación y resuelvan sabiendo que la solución es negativa. a. El cuadrado del siguiente de un número, aumentado en el triple de dicho número, es igual a 1. ¿Cuál es ese número? (x + 1)2 + 3x = 1 x2 + 2x + 1 + 3x = 1 x . (x + 5) = 0

x = –5

b. El anterior del cuadrado de un número desconocido aumentado en su cuádruple da como resultado el doble del mismo número. ¿Cuál es ese número? x2 – 1 + 4x = 2x __

x2 + 2x – 1 = 0

x = –1 – 32

41. Simplifiquen y resuelvan la ecuación. 3 1 __1 __ 2 – __ 5 . (–10x + 3x) = 5 . – 5 x – 1 . (–x + 1) – 5x

(

)

3 3 __ 2x2 – __ 5 x = (–x – 5) . (–x + 1) – 5 x

2x2 = x2 – x + 5x – 5 x2 – 4x + 5 = 0

x1 = 2 + i, x2 = 2 – i 111

30

29

31

32

33

34

35

36

37

38

39

Sistemas de ecuaciones lineales INFOACTIVA Clasificación de los sistemas Un sistema de ecuaciones lineales es aquel que posee al menos dos ecuaciones lineales. Los sistemas de ecuaciones lineales se clasifican de la siguiente manera. ˆ Sistema Compatible Determinado (SCD): tiene una única solución. Las rectas se cortan en un punto. ˆ Sistema Compatible Indeterminado (SCI): tiene infinitas soluciones. Las rectas son coincidentes. ˆ Sistema Incompatible (SI): no tiene solución. Las rectas son paralelas.

Métodos para la resolución analítica de un sistema ˆ Sustitución: se despeja una de las variables en una de las ecuaciones y luego se la reemplaza en la otra. ˆIgualación: se despeja de ambas ecuaciones la misma incógnita y luego se igualan las ecuaciones. ˆ Sumas y restas: se multiplica una de las ecuaciones por un número de manera que, mediante una suma o una resta de ecuaciones, se cancele uno de los términos con variable.

{

4 x – 5y = __ 1 __ 3 2 2 – __ x + 2y = 7 3

+

4 x + 4y = 14 ⇒ – __ 3

4 x – 5y = __ 1 __ 3 2

4 x – 5. __ 3

29 1 – ___ 4 x = __ __ 3 2 10 4 x = –72 __ 3

4 x + 4y = 14 – __ 3 29 ⇒ y = – ___ 29 0x – y = ___ 2 2

.2

29 = __ 1 (– ___ 2 ) 2

x = –54

{

S: (–54;–

145 ____ 2

)}

ˆDeterminantes: para resolver un sistema de ecuaciones lineales, se pueden usar determinantes. Un determinante se resuelve de la siguiente manera:

a b =a.d–c.b c d

Para resolver un sistema se utiliza la regla de Cramer:

{

e b f d

ax + by = e cx + dy = f

a e c f .d – f .b = e__________ a .d – c .b

x= a b c d

Por ejemplo,

{

1 x + 3y = 2 __ 2 4 y = –3 –2x + __ 3

2 3 4 –3 __ 3

x=

= 1 __ 2

3

4 –2 __ 3

112

4 – (–3) . 3 2 . __ 3 ____________ 4 – (–2) . 3 1 . __ __ 2 3

a .f – c .e = ___________ a .d – c .b

y= a b c d

35 ___ 3 = __ 7 ___ = 20 ___ 4 3

1 2 __ 2 –2 –3

y=

= 1 3 __ 2 4 –2 __ 3

1 . (–3) – (–2) . 2 __ 2 ______________ 1 . __ 4 – (–2) . 3 __ 2 3

3 7 ;__ S: ( __ 4 8)

{

5 __ 3 2 = __ = ___ 20 8 ___ 3

}

Test

de comprensión

1. Respondan y expliquen las respuestas.

{

y = x2 + 1

a. ¿Cuántas soluciones tiene el siguiente sistema? y = x2 – 1

b. ¿Es correcto decir que un sistema compatible siempre tiene una única solución? a. No tiene solución. b. No, porque puede tener infinitas soluciones. En ese caso, sería compatible indeterminado.

30

ACTIVIDADES Sistemas de ecuaciones lineales

42. Marquen las opciones correctas. a. ¿Cuáles de los sistemas tienen como solución al punto (–2;3)? X

{x2x=+1 y– =y –1

–y=3 {–3x x−y=5−y

{–y−4 == 7x –+ 2x2y

{xx –+ y1 == –5−y

b. ¿Cuáles de los sistemas son compatibles determinados?

{xx =– yy =– 57

+ 3y = 1 {–2x 6y = 2 + 4x

X

{2xx −=24=+−2yy

{2x−3y– +8 2x= 3y= 10

43. Determinen los valores de k para que el sistema cumpla la condición en cada caso.

{

–2x + ky = 2 4x − 5y = k

5 __ a. SCD k ≠ 2

5 __ c. SI k = 2

b. SCI No existe.

44. Resuelvan los sistemas por el método de sustitución.

{ x – 2y = 6

a. 2x + 5y = 3 S: {(4;–1)}

{

–4x – 2y = __1

b. –4x – y = 72 S:

{ ( – ___278 ;___132 ) }

{

– __3 x = –9 + 3y

c. 6x2 – 6y = –18 S: {(0;3)}

d.

{

5 2 __ __ 6x – 4y = 3 1 __ 1 – 2 y = – __65 x

No tiene solución.

113

30

ACTIVIDADES Sistemas de ecuaciones lineales

45. Resuelvan los sistemas por el método de igualación. a.

{

3 1 __ __ 2 x – 2 y = –6 . (x – 3y) 3__________ = –18 2

Infinitas soluciones. S:

{

4x – 3y _______ = 15

b. –2x–3+ y = –20 S:

105 { ( ____ 2 ;85 ) }

c.

{ ( x;__3x + 4 ) }

{

S:

d.

{

S:

5x + __72 = –4y + 2 –y + 1 –6x + 1 = ______ 2 109 40 ____ ; { ( ____ 371 371 ) }

7x + __51 y = –3x + __41 2 1 __ __ 5 y + 5x = – 4 5 1 ;– __ { ( ___ 20 4 ) }

46. Resuelvan los sistemas por el método de sumas y restas. a.

{

1 __ 2x – 1 __ 5y =

4 __ 5 y = –3 2 – __21 x

S: {(2;5)}

{

3 __ x + __1 y = y – 8

4 4 b. 2y –8+x=0

S:

114

40 ___ ; 56 { ( – ___ 9 9 )}

{

4x = 5y + 1

c. 4 . (x – 3) = 5 . (y – 1) No tiene solución.

d.

{

S:

3x – __21 y = –1 4x –2 ______ –2=y–1 5 17 ___ ;– 25 { ( – ___ 26 13 ) }

30

ACTIVIDADES Sistemas de ecuaciones lineales

47. Planteen los sistemas y resuelvan. a. Un quiosquero ahorra monedas de $0,50 y $0,10 para poder dar vuelto. Si hay 850 monedas de esos valores por un total de $225, ¿cuántas monedas tiene de cada una? x + y = 850 { 0,50x + 0,10y = 225

S: {(350;500)}

Tiene 350 monedas de $0,50 y 500 monedas de $0,10.

b. Las edades de Camila y Florencia suman 105 años, pero Florencia tiene 13 años menos que Camila. ¿Cuántos años tiene cada una?

{ xx ++ 13y ==105y

S: {(46;59)}

Florencia tiene 46 y Camila 59.

48. Resuelvan aplicando el método de determinantes.

{

{ –12y = –12 – 2x

– __1 y + 5 = –2x

a. x 4– 4y = 2x – 5 S:

{

c. x – 2 = 6y

25 ___ 20 { ( – ___ 11 ; 11 ) }

No tiene solución.

{

8 – 10y _______ =x

Infinitas soluciones. S:

–2 y = 5 + x_____

d. 2x = y + 44

b. –x +2 4 = 5y +4 { ( x;–x______ 5 )}

S:

34 ___ 40 { ( ___ 7 ; 7 )}

mente ACTIVA Determinen el valor de a y b para que el siguiente sistema cumpla con las condiciones pedidas en cada caso.

{xax– =ayy= b a. SCD a ≠ ( 1, b D

b. SCI a = (1, b = 0

c. SI a = (1, b ≠ 0

115

31

30

32

33

34

35

36

37

38

39

40

Sistemas de ecuaciones mixtos INFOACTIVA Los sistemas de ecuaciones formados por una ecuación de primer grado y otra de segundo, o por dos ecuaciones de segundo grado, se denominan sistemas mixtos.

Intersección entre parábolas y entre parábolas y rectas Para reconocer cuántas soluciones tiene un sistema mixto, se analiza el discriminante (Δ) de la ecuación cuadrática que surge de resolver el sistema por el método de igualación o sustitución. Δ>0 Dos puntos de intersección.

Δ=0 Un punto de intersección.

y

Sistema formado por una recta y una parábola.

y

x

{

y = mx + d y = ax2 + bx + c

La recta es tangente a la parábola.

y = ax2 + bx + c y = dx2 + ex + f

x

La recta es exterior a la parábola.

y

y

{

y

x

La recta es secante a la parábola.

Sistema formado por dos parábolas.

Δ – __ 2

Dos soluciones:

1 k = – __ 2

Una solución:

4 k > __ 5

Ninguna solución:

1 k < – __ 2

52. Completen con el ítem correspondiente en cada caso. a.

{

y = __41 x2 + __43 x – 1 y = – __21 x + 2

{

y = (x + 2)2 – 1

b. y = –x2 + 3 c.

c.

{

y = __41 x2 – __43 x – 1 y = __21 x + 2

{

y = (x – 2)2 – 1

d. y = x2 + 3 a.

2 3 4 x

1

d.

y = (x – 2)2 – 1

1

2 3 4 x

y

–2 –1 –1 –2

4 3 2 1 1

–2 –1 –1 –2

2 3 4

h.

x

–4 –3 –2 –1 –1 –2

2 3 4 x

2 3 x

1

2 x

y

4 3 2 1 1

1

b.

y

4 3 2 1 2 3 4 x

y = (x + 2)2 – 1

e.

y

1

{

h. y = x2 + 3

y

f.

4 3 2 1

{

y = __41 x2 – __43 x – 1 y = – __21 x + 2

4 3 2 1

–4 –3 –2 –1 –1 –2

y

118

{

g.

f. y = –x2 + 3

y 4 3 2 1

–2 –1 –1 –2

{

y = __41 x2 + __43 x – 1 y = __21 x + 2

g.

y 4 3 2 1 –2 –1 –1 –2

e.

–4 –3 –2 –1 –1 –2

4 3 2 1 1

2 x

–4 –3 –2 –1 –1 –2

ACTIVIDADES Sistemas de ecuaciones mixtos

31

53. Resuelvan en forma analítica los siguientes sistemas. Luego, verifiquen los resultados realizando los gráficos correspondientes en una hoja. a.

{

y = 3x2 – 3x – 3 y = 3x – 6

d.

S: {(1;–3)}

b.

{

e.

No tiene solución.

{

)

(

)}

{

y = –2x2 – 6 y = 4x2 + 2x + 2

No tiene solución.

2

y = – __32 x – 1 + 2 c. y = – __31 x + 1 15 __ ;– 1 S: (0;1); ___ 4 4

{

y = 2x2 + 4x – 4 y = –(x – 3)2 – 1

No tiene solución.

y = – __51 x + __51 y = __51 x2 + __53 x + __57

(

{

f.

{

y = 3x2 + 1 y = (x – 1)2

S: {(0;1);(–1;4)}

mente ACTIVA Encuentren la ecuación de una recta que tenga el vértice y la ordenada al origen de y = x2 – 2x + 5. Luego, realicen el gráfico del sistema. y = –x + 5

119

INTEGRACIÓN 54. Escriban V (Verdadero) o F (Falso).

58. Hallen los valores de b para que la siguiente

a. Si el discriminante de una ecuación cuadrática V

es negativo, las soluciones son complejas.

b. Si el discriminante de una ecuación cuadrática es cero, la solución es única.

V

ecuación tenga las soluciones pedidas en cada caso. x2 + bx = – 25 b < –10 ∨ b > 10 a. Dos soluciones. b = 10 ∨ b = –10 b. Una solución. c. Ninguna solución. –10 < b < 10

c. En una ecuación cuadrática que no tiene término lineal, una de sus soluciones es cero.

F

55. Marquen las opciones correctas.

2

X x = 8x

X (x – 5) . (–x + 1) = 3 2

2

3 − 4x = 4 . (–x + 3)

b. ¿Cuáles son las soluciones de la ecuación x2 + x – 6? x 1 = 2 y x2 = 3 X x = 2 y x = –3 1 2

x1 = −2 y x2 = 3 x1 = −2 y x2 = −3

56. Relacionen cada ecuación con su solución. a. x2 – 25 = 0

e. x1 = 5 y x2 = 5

b. (x – 5)2 = 0

f. x1 = –5i y x2 = 5i

c. (x + 5)2 = 0

g. x1 = 5 y x2 = –5

d. –x2 – 20 = 5

h. x1 = –5 y x2 = –5

a. y g.; b. y e.; c. y h.; d. y f.

57. Resuelvan las siguientes ecuaciones. ___ a. __21 . (x – 2)2 – 5 = 0 x = 2 ± 310 ___ x = –2 ± 310 b. (x + 2)2 = 10 ___

1 x = 2 ± 310 i c. __2 . (x – 2)2 = –5 ___ x = –2 ± 310 i d. (x + 2)2 = –10 e. 7x . (–x + 2) = –3 + 2 . (x2 + x) f. 7x . (x – 2) = –3 + 2 . (–x2 – x) g. 7x . (x + 2) = 3 + 2 . (x2 – x) h. 7x . (–x__ – 2) = 3 + 2 . (–x2___+ x)

± 379 2 ± 37 1 ; g. –8 1 ________ e. _______ ; f. 1 y __ ; h. –3 y – __ 5. 5 3 3

120

analítica y gráfica. Luego, clasifíquenlos.

{ 2x – y = 4

a. x – 3y = 6

a. ¿Cuáles de las siguientes expresiones son ecuaciones cuadráticas? X 2x2 – x2 + 5 = 3x

59. Resuelvan los siguientes sistemas de forma

b. c.

{

{

3x = y – __41 1 __ 3y + 1 = x

( __56;– __58 )

SCD

No tiene solución. SI

8y + 16 = 7x Infinitas soluciones. y + 2 = __87 x SCI 7 S: x;__ 8x – 2

{(

)}

60. Planteen y resuelvan los sistemas. a. Eduardo y Cintia comparten una cuenta en el banco de $14 300. Si Eduardo tiene el 25% de lo que tiene Cintia aumentado en $3 000, ¿cuánto dinero del depósito le corresponde a cada uno? Cintia $9 040 y Eduardo $5 260. b. ¿Cuál es la amplitud de los ángulos de un triángulo acutángulo e isósceles, si la diferencia entre el ángulo mayor y el menor es 30°? c. La suma de un número y el doble de otro es igual al triple de cuatro, y la diferencia entre el quíntuplo del primero y el doble del segundo es igual a la mitad de 96. ¿Cuáles son esos x + 2y = 12, 5x – 2y = 96 : 2 números? x = 10, y = 1. d. Unos amigos fueron al cine con sus hijos. La entrada para los adultos costaba $45 y la de los menores, $30, siempre cada uno. Compraron una gaseosa por adulto a $18 cada una y un paquete de pochoclos por niño a $20 cada uno. Si en las entradas gastaron $495 y en las consumiciones $270, ¿cuántos adultos y cuántos niños fueron al cine? b. 70°, 70°, 40° o 80°, 50°, 50°. d. 45x + 30y = 495, 18x + 20y = 270; adultos: 5, niños: 9.

capítulo

CONTENIDOS

29*-30*-31 62. Escriban V (Verdadero) o F (Falso).

61. Marquen las opciones correctas. Luego, resuelvan los sistemas analíticamente. ¿Cuál gráfico representa a cada sistema? a.

{

5

a. Un sistema es compatible determinado si

y = __41 x – 1 y = x2 – x – __43

tiene infinitas soluciones. F b. Un sistema formado por ecuaciones de rec-

X

tas paralelas es incompatible. V y

S:

4 3 2 1

{(

15 3 __ 1;– __ ; 1 ;– ___ 4 4 16

)(

)}

c. Un sistema formado por dos ecuaciones de rectas secantes tiene infinitas soluciones.

F

d. En un sistema mixto formado por una ecua-

–4 –3 –2 –1 0 1 –1 –2

2 3 4

x

ción lineal y otra cuadrática, si el discriminante de la ecuación cuadrática es cero, el sistema tiene solución única.

y

e. En un sistema mixto formado por dos ecua-

1 –4 –3 –2 –1 0 1 –1 –2 –3 –4

{

F

2 3 4

x

ciones cuadráticas, si el discriminante de una de las ecuaciones es positivo, el sistema tiene dos soluciones. F

63. Hallen el conjunto solución de los sistemas, y = –x2 – 4x – 2

de forma analítica y gráfica.

b. y = –(–x + 1)2 – 1

{ y = x + 2x + 3 2

X y

S: { (0;–2) }

2 1

a. y = –2x + 3 b.

–4 –3 –2 –1 0 1 –1 –2 –3 –4

2 3 4 x

c.

{ e. {

y = 5x2 – __25 . (x + 1) No tiene solución. y = __54 x – 4 2 – 2x + 1 y = x__________ 3

y = – __32 x + __31

3 2 1

f.

{

S:

{ ( 0;__31 ) }

y = x2 + 3

d. y = 5x2 + 3

y

01 –4 –3 –2 –1 –1 –2 –3 –4

{ {

S: {(0;3);(–4;11)}

S: {(0;3)}

y = –x2 + 4x – 6 y = x2 + 2

No tiene solución.

y = x2 + x – 6 y = – __51 x . (x – 29)

S: {(5;24);(–1;–6)}

2 3 4 x

121

capítulo

5

AUTOEVALUACIÓN Marquen las opciones correctas 64. ¿Cuáles son los gráficos que pueden corresponder a una función biyectiva? X b.

a. y

c. y

x

d. y

x

y

x

x

65. ¿Cuáles son las funciones que tienen la ordenada al origen indicada? a. – __45 y = – __45 x + __45

5 X y = – __ 4 x – 1,25

5 5 __ X y = __ 4x – 4

b. __32

y = – __31 . (x + 2)2 + __32 X y = __31 . (x + 1)2 + __31

y = – __31 x2 – __32

y = –1,26x + 1,25

X y = – __1 . (x + 2)2 + 2 3

66. Lean atentamente y respondan. Un día de examen se dispuso utilizar un cierto número de aulas. Si se ubicaban 21 alumnos por aula, faltaban 13 asientos; pero si se ubicaban 24 alumnos por aula, quedaban libres 8 asientos. Se pide la cantidad de alumnos presentes y aulas utilizadas ese día. a. ¿Cuál es el sistema que representa la situación? 21x – 13 = y { 24x +y=8

21x + 13 = –y { 24x −y=8

21x – 13 = y { 24x + y = −8

X

21x + 13 = y { 24x −y=8

b. ¿Cuál es la solución del problema? x = 15; y = 328

X x = 7; y = 160

x = 9; y = 173

x = 5; y = 92

67. Respondan. a. ¿Cuál es el sistema que se corresponde con el siguiente gráfico? y 2 1 0 1 –4 –3 –2 –1 –1 –2 –3 –4

{

y = –x2 – 2 y = −x2 + 4x – 4

2 3 4 x

{

y = –x2 – 2 y = −x2 – 4x – 4

X

{

y = –x2 + 2 y = −x2 + 4x – 4

{

y = –x2 + 2 y = −x2 – 4x – 4

b. ¿Cuál es la solución del sistema? X

122

( __23;– __41 )

( __41 ;– __23 )

( __32;– __41 )

( __41 ;– __32 )

6

Contenidos 32. Función polinómica. 33. Análisis de la función polinómica. 34. Función racional. 35. Representación gráfica de funciones racionales. 36. Función homográfica.

La ciencia moderna, en especial a partir de Newton, se caracterizó por el afán de explicar objetivamente la realidad por medio de modelos matemáticos, en los cuales las funciones tuvieron un rol fundamental. Por tal motivo, el teorema que publicó en 1712 el matemático inglés Brook Taylor cobraría, con el correr de los tiempos, una importancia extraordinaria. Gran parte de la matemática se expresa mediante funciones reales; entre ellas, los polinomios constituyen los ejemplos más simples, pues su construcción solo requiere de las operaciones de suma y multiplicación. Lo que Taylor demostró es que una gran cantidad de dichas funciones, y sin duda todas las que aparecen en las aplicaciones concretas, pueden aproximarse mediante polinomios de una manera fácil de calcular. Los polinomios de Taylor permiten obtener valores aproximados de funciones de un modo notable, dando origen a algoritmos que todavía emplean las calculadoras y computadoras actuales.

1. Lean atentamente y respondan.

__

Según el teorema de Taylor, para valores de x cercanos a 1, 3x se puede (x – 1)2 (x – 1)3 – 1) ______ _______ _______ aproximar mediante el polinomio f(x) = 1 + (x . ¿Cuál es – + 2 ___ ___ 8 16 el valor aproximado de 31,1 ? ¿Y de 31,2 ? Comparen los resultados usando la calculadora. Los valores aproximados respectivos son: 1,0488125 y 1,0955, mientras que los valores “exactos” (aunque obviamente, no del todo) son 1,048808848 y 1,095445115.

capítulo

Funciones polinómicas y racionales

32

31

33

34

35

36

37

38

39

40

41

Función polinómica ¿Para qué sirve?

INFOACTIVA

PÁGINA 9

Una función de la forma f(x) = an xn + an–1 xn–1 + ... + a2 x2 + a1 x + a0, siendo n un número natural y an, an–1, ..., a2, a1, a0, números reales, es una función polinómica. ˆ ˆ ˆ ˆ

Si an ≠ 0, entonces la función es de grado n. El dominio de las funciones polinómicas es el conjunto de los números reales ( ). Las funciones polinómicas son continuas. El orden de multiplicidad de una raíz es el número de veces que esa raíz se repite como tal.

f(x) = 2x4 . (x + 2)3 . (x – 1) = 2x . x . x . x . (x + 2) . (x + 2) . (x + 2) . (x – 1) x1 = 0 → x1 tiene orden de multiplicidad 4. x2 = –2 → x2 tiene orden de multiplicidad 3. x3 = 1 → x3 tiene orden de multiplicidad 1. El conjunto de positividad (C+) de una función polinómica está formado por todos los valores del dominio para los cuales la función es positiva. C+: x ∈ Df ∧ f(x) > 0 El conjunto de negatividad (C–) de una función polinómica está formado por todos los valores del dominio para los cuales la función es negativa. C–: x ∈ Df ∧ f(x) < 0

Teorema de Bolzano Si una función f(x) es continua en un intervalo de su dominio, y tiene distinto signo en los extremos del mismo, entonces la función tiene por lo menos una raíz real en ese intervalo. y f(b)

a f(c) f(a)

x1

b

x2

c

x

f(a) < 0 ∧ f(b) > 0 ⇒ f(x1) = 0 ∧ x1 D (a;b) f(b) > 0 ∧ f(c) < 0 ⇒ f(x2) = 0 ∧ x2 D (b;c)

Usando el teorema de Bolzano, determinen si la siguiente función tiene al menos una raíz en el intervalo (–2;1). 1x + 3 f(x) = 3x5 – 2x3 + __ 2 1 . (–2) + 3 f(–2) = 3 . (–2)5 – 2 . (–2)3 + __ 2 f(–2) = 3 . (–32) – 2 . (–8) – 1 + 3 f(–2) = –96 + 16 – 1 + 3 f(–2) = –78

1.1+3 f(1) = 3 . 15 – 2 . 13 + __ 2 1 __ f(1) = 3 – 2 + 2 + 3 9 f(1) = __ 2

f(–2) < 0 ∧ f(1) > 0 ⇒ f(x1) = 0 ∧ x1 ∈ (–2;1) 124

Test

de comprensión

1. Respondan y expliquen las respuestas. a. ¿Cuál es el grado de la siguiente función? f(x) = 3x5 – 2x + 1 b. ¿Cuáles son las raíces de la siguiente función? ¿Cuál es el orden de multiplicidad de cada una? f(x) = (x + 3)4 . (x – 2) a. La función es de grado cinco. b. –3 es de orden cuatro y 2 es de orden uno.

32

ACTIVIDADES Función polinómica

1. Escriban en cada caso una función que cumpla con el orden de multiplicidad de las raíces indicadas. a. 0 es una raíz de orden 2, y –2 es de orden 1.

c. –1, –2 y –3 son de orden 1.

f(x) = x2 . (x + 2)

f(x) = (x + 1) . (x + 2) . (x + 3)

b. 3 es de orden 3, y –3 es de orden 2.

d. 2 es de orden 3, y 0 es de orden 4.

f(x) = (x – 3)3 . (x + 3)2

f(x) = x4 . (x – 2)3

2. En la actividad anterior, ¿cuántas funciones posibles hay en cada caso? Infinitas. Porque puede variar el coeficiente principal.

3. Para cada uno de los siguientes gráficos, indiquen los intervalos de positividad y negatividad. a.

c.

y

y

2

2 1

–5 –4 –3 –2 –1 0

1

2

3 4

5

x

–5 –4 –3 –2 –1–1 0 1

2

3 4

5

x

2

3 4

5

x

–2

–2

–3 –4

C+ = (–3;2) F (2;+')

C+ = (2;+')

C– = (–';–3)

C– = (–';–1) F (–1;2) y

b.

y

d.

2

2 1 –5 –4 –3 –2 –1 0 1 –1

2

3 4

5

–5 –4 –3 –2 –1

x

0 1

–2

–2 –3

–4

C+ = (–';–3) F (–3;2) F (2;+')

C+ = (1;2) F (4;+')

C– = ’

C– = (–';1) F (2;4)

4. ¿Cuáles son las funciones que tienen al menos una raíz en el intervalo (–1;3)? X a. f(x) = x3 – 3x – 2

b. f(x) = x2 – 2x – 8

X c. f(x) = x3 – 6x2 + 9x – 4 125

33

32

34

35

36

37

38

39

40

41

42

Análisis de la función polinómica ¿Para qué sirve?

INFOACTIVA

PÁGINA 10

Para realizar el gráfico aproximado de una función polinómica, se pueden seguir estos pasos: 1. Se expresa su fórmula en la forma factorizada: f(x) = an . (x – x1) . (x – x2) ... (x – xn–1) . (x – xn). Para ello, es importante recordar el teorema de Gauss y el teorema del resto. Teorema de Gauss: Si P(x), de grado n, con coeficientes enteros y término independiente no nulo, p admite una raíz racional __ q (fracción irreducible), entonces p es divisor del término independiente y q lo es del coeficiente principal. Teorema del resto: El resto de P(x) : (x − a) es P(a). 2. Se determinan las raíces, que indican las intersecciones con el eje x, y su orden de multiplicidad: a. Si el orden de multiplicidad es par, la gráfica de la función toca el eje x, pero no lo atraviesa. b. Si el orden de multiplicidad es impar, la gráfica de la función atraviesa el eje x. 3. Se hallan los intervalos de positividad (C+) y negatividad (C–), para lo cual se buscan los valores del dominio entre dos raíces consecutivas para determinar si la función es positiva o negativa en ese intervalo.

Grafiquen la función polinómica f(x) = 2x3 + x2 – 7x – 6. 1. Se busca la forma factorizada: 3 ; ±3; ±2; ± __ 1 ; ±1. a. Por teorema de Gauss, las posibles raíces racionales son: ±6; ± __ 2 2 b. Se aplica el teorema del resto para buscar una de las raíces. ← 1 no es raíz de f(x). f(1) = 2 . 13 + 12 – 7 . 1 – 6 = –10 3 2 ← –1 es raíz de f(x). f(–1) = 2 . (–1) + (–1) – 7 . (–1) – 6 = 0 c. Se aplica la regla de Ruffini: 2

1

–7

–6

–2

1

6

–1

–6

0

Cálculos auxiliares (–1) . 2 = –2 (–1) . (–1) = 1

+ –1 2

(–1) . (–6) = 6

f(x) = (x + 1) . (2x2 – x –6) _______

–b ± 3b2 – 4ac para factorizar 2x2 – x –6. d. Se usa la fórmula _____________ 2a _______________

–(–1) ± 3(–1)2 – 4 . 2 . (–6) _____ x2;3 = _______________________ = 1 4± 7 2.2 3 x2 = 2 ∧ x3 = – __ 2 3 f(x) = 2 . (x + 1) . (x – 2) . x + __ 2

(

y 15

)

10 5

2. Todas las raíces tienen multiplicidad 1. Entonces, la gráfica atraviesa el eje x en todas sus raíces. –3

3 ;–1 F (2;+') y C– = –';– __ 3. C+ = ( – __ ( 32 ) F (–1;2) ) 2

–2 –3 –1 2

0 –5 –10 –15

126

1

2

3

x

Test

de comprensión

1. Respondan y expliquen las respuestas. a. ¿Es cierto que el gráfico de la función f(x) = x . (x – 2)2 . (x + 3)2 no atraviesa al eje x? b. Si en una función polinómica, C+ = (–∞;2) y C– = (2;+∞), ¿en x = 2 hay una raíz? a. No es cierto, la atraviesa en x = 0. b. Sí.

33

ACTIVIDADES Análisis de la función polinómica

5. Realicen los gráficos teniendo en cuenta las características indicadas. a. 3 es una raíz de multiplicidad 1, la ordenada al origen es (0;–3).

c. C+ = (–';2) F (2;+').

b. 1 y –1 son raíces de multiplicidad 1, la ordenada al origen es (0;–2).

d. C+ = (–';0) F (3;+'), 3 es una raíz de multiplicidad 3, y 2 es una raíz de multiplicidad 4.

Solución a cargo del alumno.

6. Observen los gráficos y completen las fórmulas. Luego, calculen la ordenada al origen. a.

b. y

y

6

4

4 2 2

–3

–2

–1

0

1

2

–4

x

–3

–2

–1

0

1

2

3

x

–2 –2

Las soluciones no son únicas.

f(x) = x

1

.

(

x+1

Ordenada al origen:

) 1 .( (0;0)

x+3

)1

f(x) = __81 .

(

x+4

Ordenada al origen:

)1 .(

x–1

)2

( 0;__21 ) 127

33

ACTIVIDADES Análisis de la función polinómica

7. Expresen las siguientes funciones en forma factorizada. a. f(x) = x3 – 3x – 2

d. f(x) = x3 – 2x2 – 5x + 6

f(x) = (x – 2) . (x + 1)2

f(x) = (x + 2) . (x – 1) . (x – 3)

b. f(x) = x2 – x – 6

e. f(x) = x3 – 4x

f(x) = (x + 2) . (x – 3)

f(x) = x . (x – 2) . (x + 2)

c. f(x) = x4 – 2x2 + 1

f. f(x) = x4 + 4x + 3 __

f(x) = (x – 1)2 . (x + 1)2

__

f(x) = (x + 1)2 . (x – 1 + 2 . 32 i) . (x – 1 – 2 . 32 i)

8. Observen el gráfico y marquen las opciones correctas. y 3 2 1

–7

–6

–5

–4

–3

–2

–1

0

1

2

3

4

5

6

7

x

–1 –2

a. ¿Cuál es el conjunto de positividad de la función? X (2;+')

(–2;2)

(–';–2) F (2;+')

b. ¿Cuál es la ordenada al origen? (–2;0)

(0;2)

X (0;–2)

c. ¿Cuál es la multiplicidad de las raíces? X –2 es par y 2 es impar.

–2 es par y 2 es par.

2 es impar y 2 es impar.

d. ¿Cuál es la fórmula factorizada de la función? f(x) = (x – 2) . (x + 2)2 128

f(x) = __41 . (x – 2)2 . (x + 2)

X f(x) = __1 . (x – 2) . (x + 2)2 4

33

ACTIVIDADES Análisis de la función polinómica

9. Realicen el gráfico de las siguientes funciones polinómicas. a. f(x) = x3 – 4x

–3

–2

d. f(x) = x7 – 6x6 + 12x5 – 8x4 y 3

y

2

2

1

1

–1

0

1

2

3

x

–4

–3

–2

–1

0 –1

–2

–2

–3

–3

1

2

3

x

e. f(x) = –x2 + 4x – 4

–2

y

y

2

2

1

1

–1

0

1

x

–3

–2

–1

0

–1

–1

–2

–2

c. f(x) = x3 – x2

–3

–2

–1

b. f(x) = x2 + 6x + 9

–5

–3

1

2

3

x

f. f(x) = x4 – 2x2 + 1 y

y

2

2

1

1

–1

0

1

2

3

x

–3

–2

–1

0

–1

–1

–2

–2

–3

–3

1

2

4

x

129

INTEGRACIÓN 10. Escriban la fórmula de una función polinómica

13. Tengan en cuenta los datos y resuelvan.

que cumpla con las condiciones pedidas en cada caso. a. La función es de grado 3. x1 = –2; raíz de orden 1. x2 = –1; raíz de orden 2 f(0) = 4 b. La función es de grado 4. x1 = –5; raíz de orden 2. x2 = –1; raíz de orden 2. f(0) = 3 c. La función es de grado 3. x1 = 3; raíz de orden 2. x2 = 2; raíz de orden 1. f(–1) = 2 d. La función es de grado 3. x1 = 2; x2 = –1; x3 = 3 f(–3) = 4

f(x) = x4 – ax3 + bx2 x1 = 5 y x2 = –3 son raíces de la función. a. Hallen los valores de a y de b. a = 2; b = –15 b. Indiquen todas las raíces del polinomio y su 5 y –3 de grado 1 y grado de multiplicidad. 0 de grado 2. c. Escriban la función polinómica en forma factorizada. f(x) = x2 . (x – 5) . (x + 3). d. Realicen el gráfico aproximado. e. Indiquen: dominio, ordenada al origen, conjunto de positividad y conjunto de negatividad. Df = ; Ordenada al origen: (0;0); C+ = (-–';–3) F (5;+'); C– = (–3;0) F (0;5)

14. Hallen las raíces, la ordenada al origen y los conjuntos de positividad y negatividad de las funciones representadas. y a. 4

2

Solución a cargo de los alumnos.

11. Usen el teorema de Bolzano para verificar si las funciones tienen una raíz en los intervalos indicados. No. a. f(x) = x3 + 2x2 en (1;2) 3 __ 2 b. f(x) = x + 3x + 2 en – 2 ;2 Sí. Sí. c. f(x) = x3 – 4x en (1;3) 3 No. d. f(x) = x – 4x en (–5;–3) e. f(x) = x4 – 3x + 2 en (–1;1) Sí. f. f(x) = x4 – 3x + 2 en (–1;0) No.

(

–2

–1

0

1

2

x

–2

)

–4

a. Raíces: –1 y 2; ordenada: –2; C+: (2;+'); C–: (– ';–1) F (–1;2)

b.

y 3

12. Para cada una de las siguientes funciones indiquen: raíces, ordenada al origen, conjunto de positividad y conjunto de negatividad. a. f(x) = x2 . (x + 2) b. f(x) = (x – 3)2 . (x + 3)2 c. f(x) = (x + 1) . (x + 2) . (x + 3) d. f(x) = x4 . (x – 2)3 e. f(x) = 2x . (x – 2)2 . (x + 3) f. f(x) = 3x . (x + 2) . (x – 1) . (x – 3) a. –2 y 0; (0;0);(–2;0) F (0;+'); (–';–2) b. 3 y –3, (0;81), (–';3) F (–3;3) F (3;+'); no tiene; c. –1, –2 y –3; (0;6); (–3;–2) F (–1;+'); (–';–3) F (–2;–1); d. 0 y 2; (0;0); (2;+');(–';0) F (0;2) e. 0; 2 y –3; (0;0); (–';–3) F (0;2) F(2;+'); (–3;0); f. 0, –2, 1 y 3; (0;0); (–';–2) F(0;1) F(3;+'); (–2;0) F (1;3)

130

2

1

–2

–1

0

1

–1

b. Raíces: –1 y 1; ordenada: 1; C+: (–';–1) F (–1;1) F (1;+')

2

x

capítulo

CONTENIDOS

6

32*33 15. Observen el gráfico y marquen las opciones

17. Realicen, en cada caso, un gráfico que cumpla

correctas.

con las características indicadas. a. Tiene a x = 2 como raíz doble y a x = –1 como raíz de orden 1. b. Tiene a x = –3 como raíz de orden 1 y a x = –1 como una raíz de orden 4. La ordenada al origen es (0;–4). c. Tiene tres raíces de orden 1: x = –2; x = 1; x = 3 y el intervalo de negatividad es (–';–2) F (1;3). d. Tiene dos raíces x = 0 y x = 3 y el intervalo de positividad es (3;+')

y 4

2

–5

–4

–3

–2

–1

0

1

2

x

–2

–4

–6

Solución a cargo de los alumnos.

a. ¿Qué tipo de orden de multiplicidad tiene la raíz x = –4? X Par.

Impar. b. ¿En cuáles intervalos la función es positiva? (3;+∞) X (–∞;–4) X (–4;–1) X (2;+∞) c. ¿En cuáles intervalos la función es negativa? (–4;–1) (–∞;–4) X (–1;2) (–3;0) d. ¿Cuáles intervalos verifican el teorema de Bolzano? (–3;–2) X (–2;1) (–5;–3) e. ¿Cuál es el grado de la función?

(2;+∞)

X 4 2 3 f. ¿Cuál es la ordenada al origen?

5

(–2;0)

(0;–4)

(–4;0)

18. Escriban V (Verdadero) o F (Falso) teniendo en cuenta la siguiente función. Expliquen los casos donde escribieron F. f(x) = (x – 2) . (x + 1)2 a. El gráfico de la función atraviesa al eje x en el punto (1;0). F

La imagen de 1 es -4.

b. En x = 2 hay una raíz de orden 1. V c. En x = 1 hay una raíz de orden 2. V d. La función f(x) es de grado 2.

F

Es de grado 3.

e. La ordenada al origen de f(x) es (0;1). F Es (0;–2)

f. C+: (–1;2).

F

(2;+')

g. C+: (–';–1) F (2;+'). F

(2;+')

X (0;–2)

19. Factoricen las siguientes funciones y realicen 16. Respondan. 2

Dada la función f(x) = (x – a) . (x – b) (con a < b), su conjunto de negatividad es C–: (b;+'). a. ¿Cuál es el conjunto de positividad? b. ¿El valor de la ordenada al origen es positivo o negativo? c. ¿Las dos raíces pueden ser positivas?

el gráfico de cada una. a. f(x) = x3 – 8x2 + 16x f(x) = x . (x – 4)2 b. f(x) = x3 + 3x2 f(x) = x2 . (x + 3) c. f(x) = x3 – x f(x) = x . (x – 1) . (x + 1) d. f(x) = x4 – 2x3 + x2 f(x) = x2 . (x – 1)2 e. f(x) = x3 – 7x – 6 f(x) = (x + 2) . (x + 1) . (x – 3) f. f(x) = x3 + 3x2 – 4x – 12 f(x) = (x + 3) . (x + 2) . (x – 2)

a. C+: (–';a) F (a;b). b. Es positivo. c. Sí.

131

34

33

35

36

37

38

39

40

41

42

43

Función racional INFOACTIVA P(x)

Una función de la forma f(x) = ____ , siendo P(x) y Q(x) polinomios ∧ Q(x) no es nulo, es una función Q(x) racional. 3

2

x + 3x – 2 es una función racional. f(x) = 2___________ x2 – 4x + 9

Para calcular el dominio de una función racional, se iguala el denominador a 0 (ya que ese es el valor que no puede tomar) y se hallan los valores de x. El dominio es el conjunto de los reales menos los valores de x encontrados. Para calcular el conjunto imagen de una función racional, se iguala la función a y y se despeja x. Los valores que no puede tomar y son los que no pertenecen a la imagen. Una asíntota es una recta. Entre las funciones racionales se pueden estudiar diferentes casos particulares: ˆ Una función de la forma f(x) = __kx , siendo k un número real, es una función de proporcionalidad inversa. Dominio: El denominador de la función debe ser distinto de cero: x ≠ 0. Df = – {0} La recta de ecuación x = 0 es asíntota vertical (A. V.) de la función. Imagen:

y = __kx ⇒ x = __ky ⇒ y ≠ 0 Im = – {0} La recta de ecuación y = 0 es asíntota horizontal (A. H.) de la función.

k ˆ Una función de la forma f(x) = ____ , siendo k un número real y Q(x) un polinomio que no es nulo. Q(x)

Dominio:

El denominador de la función debe ser distinto de cero: Q(x) ≠ 0.

5 f(x) = _____ x–3

x – 3 = 0 ⇒ x = 3 → 3 es el valor que no puede tomar x. Df = – {3} La recta de ecuación x = 3 es asíntota vertical (A. V.) de la función. Imagen: 5 f(x) = _____ x–3 5 ⇒ x – 3 = __ 5 ⇒ x = __ 5 + 3 → 0 es el valor que no puede tomar y. y = _____ y y x–3

Im = – {0} La recta de ecuación y = 0 es asíntota horizontal (A. H.) de la función.

132

Test

de comprensión

1. Respondan y expliquen las respuestas. 3

x 2 a. La función f(x) __ x ¿es racional? ¿Es equivalente a x ? 3 b. ¿Es cierto que el codominio de la función f(x) = _____ x + 2 puede ser ? ¿Y la imagen?

a. No, porque es igual a x2, aunque su dominio es

34

– {0}. b. Sí, puede ser. La imagen es

– {0}.

ACTIVIDADES Función racional

20. Marquen las opciones correctas. a. ¿Cuáles funciones son racionales? –5 X f(x) = _____ x–1

x3 X f(x) = __ 4

–1 f(x) = x_____ –5

4

x

x f(x) = __ x3

4 c. f(x) = _____ x–3

3 X d. f(x) = __ x

b. ¿Cuáles funciones son de proporcionalidad inversa? 3 X f(x) = – __ x

x b. f(x) = ___ –2

21. Unan cada función con su dominio. 2 a. f1(x) = _____ x+2

Df =

– {2}

2 e. f5(x) = ______ –x – 2

2 b. f2(x) = _____ x–2

Df =

– {–2}

2 f. f6(x) = _______ 1 x+1 __ 2

2 c. f3(x) = ______ –x + 4

Df =

– {4}

2 g. f7(x) = ________ 1 __ –2 x – 2

2 d. f4(x) = _______ 1 x+2 __ 2

Df =

– {–4}

2 h. f8(x) = _______ 1 x–1 __ 2

22. Escriban el dominio, la imagen y las asíntotas de las siguientes funciones. 5 a. f(x) = ______ 3x + 4

Df =

4 – – __ 3

A. V.:

4 x = – __ 3

{ }

3 c. f(x) = ______ 4x + 3

= ; Im ; A. H.:

– {0} y=0

1 b. f(x) = ______ 2x – 1

Df =

1 – __ 2

A. V.:

1 x = __ 2

{ }

Df =

3 – – __ 4

A. V.:

3 x = – __ 4

{ }

= ; Im A. H.:

– {0} y=0

4 d. f(x) = – __ x2

= ; Im ; A. H.:

– {0} y=0

Df = A. V.:

– {0} x=0

= (–∞;0) ; Im A. H.:

y=0 133

35

34

36

37

38

39

40

41

42

43

44

Representación gráfica de funciones racionales INFOACTIVA Para representar una función racional, se deben seguir estos pasos: 1. Se determinan los conjuntos dominio e imagen. 2. Se hallan las ecuaciones de las asíntotas. 3. Se hallan distintos puntos de la función, entre ellos los puntos de intersección con los ejes. En el caso de la función de proporcionalidad inversa, las asíntotas siempre son x = 0 ∧ y = 0; por lo tanto no hay intersección con los ejes. 3. Representen la función f(x) = __ x 1. Df = – {0} ∧ Im = – {0} 2. Asíntota vertical: x = 0; asíntota horizontal: y = 0 3. Se hallan distintos puntos de la función.

x

f(x)

–3

–1

–2

__ –3 2

–1

–3

1

3

2

3 __ 2

3

1

y 4 3 2 3 2

f(x)= 3 x

1 y =0 –4

–3

–2

–1

0

1

2

3

4

x

–1 –3 2

–2 –3 –4 x =0

En el caso de esta función, la asíntota horizontal siempre es y = 0; por lo tanto no hay intersección con el eje x. y

5 . Representen la función f(x) = ______ 2x – 3

4

3→D = 3 1. 2x – 3 = 0 ⇒ x = __ – __ f 2 2 5 5 5 ______ __ __ y = 2x – 3 ⇒ 2x – 3 = y ⇒ x = y + 3

{ } ( ) . __12

5 + __ 3→C = ⇒ x = ___ f 2y 2

3

2 5 3

– {0}

1

3; 2. Asíntota vertical: x = __ 2 asíntota horizontal: y = 0. 3. Se hallan distintos puntos de la función y se representa gráficamente.

y =0 –3

–2

–1

0

1

2

–5 7

5 f(x)= 2x – 3

–1

–5 3

x

–2

–1

0

1

2

3

f(x)

– 5__ 7

–1

– 5__ 3

–5

5

5 __ 3

–2

–3

–4

134

x=3 2

3

4

x

de comprensión

Test

1. Respondan y expliquen las respuestas. a. En la función y = __kx ¿las asíntotas coinciden con los ejes cartesianos para cualquier valor de k? b. ¿Es cierto que ninguna función racional tiene intersección con el eje y? a. Sí, porque es una función de proporcionalidad inversa. b. No, depende de cuál es el valor de la asíntota vertical.

35

ACTIVIDADES Representación gráfica de funciones racionales

23. Completen la siguiente tabla. Luego, grafiquen cada función. Función

Dominio

Imagen

A. V.

A. H.

f(–1)

f(1)

f(2)

f(x) = 4__x

– {0}

– {0}

x=0

y=0

–4

4

2

3 g(x) = ______ 2x + 1 –2 h(x) = _____

1 – – __ 2

– {0}

1 x = – __ 2

y=0

–3

1

3 __ 5

1 – – __ 2

– {0}

1 x = – __ 2

y=0

4

4 – __ 3

4 – __ 5

4 p(x) = _____ 3–x

– {3}

– {0}

x=3

y=0

1

2

4

{ } { }

1 x + __ 2

y

p(x)=

4 3–x

5

2 h(x)= x+ 1 3

4

3

2

4 f(x)= x

1

–9

–8

–7

–6

–5

–4

–3

–2

–1

3 g(x)= 2x + 1

0

1

2

3

4

5

6

7

8

9 x

–1

–2

–3

–4

–5

135

35

ACTIVIDADES Representación gráfica de funciones racionales

24. Grafiquen las siguientes funciones. 3 a. f(x) = ______ 2x – 3

3 d. f(x) = ______ 1 __ 2x +

4

y 3

–8 –7 –6 –5 –4 –3 –2

y

2

4

1

2

–1

0 1

2

3

4

5

6

7

8 x

–8 –7 –6 –5 –4 –3 –2

–1

–1

–2

–2

–4

0 1

2

0

1

0 1

2

3

4

5

6

7

8 x

–3

5 b. f(x) = ___ 2x

3 e. f(x) = – ___ 4x y 3 y 2

20

1

10

–8 –7 –6 –5 –4 –3 –2

–1

0 1

2

3

4

5

6

7

8 x

–4

–3

–2

–1

–10

–1

–20

–2

2

3

4 x

–3

–2 c. f(x) = _____ x+3

–2 f. f(x) = _______ 1 x–2 __ 2 y 3 y

–8 –7 –6 –5 –4 –3 –2

4

2

2

1

–1

0 1

2

3

4

5

6

7

8 x

–8 –7 –6 –5 –4 –3 –2

–1

–2

–1

–4

–2

–3

136

3

4

5

6

7

8 x

35

ACTIVIDADES Representación gráfica de funciones racionales

25. Completen con el ítem de la función correspondiente. k a. f(x) = ______ con k < 0; a < 0, b = 0 ax + b

k c. f(x) = ______ con k < 0; a > 0; b < 0 ax + b

k b. f(x) = ______ con k > 0; a > 0; b < 0 ax + b

k d. f(x) = ______ con k < 0; a > 0; b = 0 ax + b

c.

d.

y

–7

–6

–5

–4

–3

–2

y

2

2

1

1

–1

0

1

2

3

4

5

6

7

x

–7

–6

–5

–4

–3

–2

–1

0

–1

–1

–2

–2

–3

–3

a.

–6

–5

–4

–3

–2

2

3

4

5

6

7

x

1

2

3

4

5

6

7

x

b.

y

–7

1

y

2

2

1

1

–1

0

1

2

3

4

5

6

7

x

–7

–6

–5

–4

–3

–2

–1

0

–1

–1

–2

–2

–3

–3

26. Escriban en cada caso una ecuación de la función de la forma f(x) =

k ______ . ax + b

a. Tiene asíntota vertical x = 2 y la ordenada al origen es (0;3). –6 Por ejemplo, f(x) = _____ x–2.

b. Tiene asíntota vertical x = –1 y la ordenada al origen es (0;2). 2 Por ejemplo, f(x) = _____ x+1.

mente ACTIVA Matías necesita representar la relación entre la base y la altura de los rectángulos que tienen 36 cm2 de área y decide expresar la base en función de la altura. a. Escriban la función con la que debe trabajar Matías. b. ¿Cuál es el dominio de la función? ¿Y la imagen? c. ¿De qué tipo de función se trata? d. Represéntenla gráficamente. 36 a. b = ___ . b. h

+

;

+

. c. Función de proporcionalidad inversa. d. Solución gráfica. 137

36

35

37

38

39

40

41

42

43

44

45

Función homográfica ¿Para qué sirve?

INFOACTIVA

PÁGINA 11

+ b ______ Una función de la forma f(x) = ax , siendo a, b, c y d números reales ∧ c ≠ 0, es una función cx + d homográfica.

Dominio de una función homográfica d El denominador de la función debe ser distinto de cero: cx + d ≠ 0 ⇒ x ≠ – __ c d __ Por lo tanto: Df = – – c d La recta de ecuación x = – __ c es asíntota vertical (A. V.) de la función.

{ }

Imagen de una función homográfica Im = – { __ac } La recta de ecuación y = __ac es asíntota horizontal (A. H.) de la función.

Representación gráfica +b ______ Para representar una función homográfica de la forma f(x) = ax , se debe: cx + d 1. Determinar los conjuntos dominio e imagen. 2. Encontrar las ecuaciones de las asíntotas. b 3. Determinar el punto de intersección con el eje y: x = 0 ⇒ f(0) = __ d ax +b b ______ 4. Encontrar el punto de intersección con el eje x: cx + d = 0 ⇒ x = – __ a

3x – 1 ⇒ a = 3; b = –1; c = 1; d = –2 f(x) = ______ x–2 –2 ⇒ D = – – ___ – {2} ∧ Im = f 1 2. A. V.: x = 2 ∧ A. H.: y = 3

1. Df =

{

3 ⇒I = – __ m 1

}

{ }

– {3}

y 8

–1 ⇒ f(0) = __ 1 → 0;__ 3. f(0) = ___ ( 12 ) es punto –2 2 de intersección con el eje y.

7

3x – 1 = 0 ⇒ x = ___ 1 → __ ⇒ x = __ 4. ______ – –1 ( 13;0 ) es punto 3 3 x–2 de intersección con el eje x.

11 2

Para realizar una gráfica más aproximada, se pueden calcular otros puntos de la función.

4

6

5

y =3 3

138

x

f(x)

–1 0

4 __ 3 1 __ 2

1 __ 3

0

1

–2

–1

3

8

–2

4

11 ___ 2

–3

2 3x – 1 f(x)= x – 2

4 3

1 1 2

–4

–3

–2

0 1

–1

3

1

2

3

x =2

4

x

Test

de comprensión

1. Respondan y expliquen las respuestas. 3x + 2 a. La función f(x) = ________ ¿es una función homográfica? 1 __ –

2

+ 2x

–2 – 3x b. ¿Cuál es la asíntota horizontal de f(x) = _______ –2x + 3 ? 3 a. Sí. b. No, la asíntota horizontal es y = __ 2.

36

ACTIVIDADES Función homográfica

27. Indiquen el dominio de cada función. 2x – 1 a. f(x) = _______ 1 x+3 __ 2 3 – – __ 2

1 x + __ 4 _____ c. f(x) = 1 x – __ 2 1 – __ 2

3x + 2 b. f(x) = ______ 1 __

3 d. f(x) = _______ 3x + 4

{ }

3x –

1 – ___ 12

{ }

{ }

1 __ x+2

4

4 – – __ 3

{ }

28. ¿Cuál es el conjunto imagen de cada función? +2 ______ a. f(x) = 3x x

X

– {3}

– {2}

2x – 1 b. f(x) = ______ 4x + 3

– {1}

– __43

{ }

X

– __21

– {3}

X

– __21

– {3}

X

– {2}

{ }

x–3 c. f(x) = ______ 2x – 1

– {–3}

{ }

2x d. f(x) = _____ x+5

29. Unan con flechas cada ecuación con las asíntotas correspondientes. x+1 a. f(x) = ______ 2x – 3

ˆ A. V.: x = __23 y A. H.: y = __23

+3 ______ b. f(x) = 2x x+2

ˆ A. V.: x = –2 y A. H.: y = 1

–2 ______ c. f(x) = 3x 2x – 3

ˆ A. V.: x = __23 y A. H.: y = __21

x d. f(x) = _____ x+2

ˆ A. V.: x = –2 y A. H.: y = 2

30. Escriban en cada caso una función homográfica que cumpla con las condiciones. a. A. V.: x = 2; A. H.: y = 1 +3 Por ejemplo, f(x) = x_____ x–2

b. A. V.: x = – __41 ; A. H.: y = __21 –3 ______ Por ejemplo, f(x) = 2x 4x + 1 139

36

ACTIVIDADES Función homográfica

31. Tengan en cuenta la siguiente función y calculen las preimágenes pedidas. 5x + 2 h(x) = ______ 1 __ 3x –

2

a. h(a) = 0

a=

–2

d. h(d) = 1

d=

5

b. h(b) = –4

b=

c. h(c) = 4

c=

–5 4

0

4 7

e. h(e) = __29

e=

1 f. h(f) = __ 4

f=

1 2

–1 2

32. Escriban V (Verdadero) o F (Falso) según corresponda. a. x = – __21 es A. H. F

4

b. f(0) = –6 V 2

c. Df =

3 4

–8

–6

–4

–2

–1 2

0

–2

2

4

6

8

– – __21

{ }

V

d. y = __43 es A. V. F e. Im =

– – __43

{ }

F

f. Raíz: (0;–6) F –4

g. C+ = –';– __21 ∪ (4;+') V –6

h. C–

( ) 1 = ( – __2 ;4 ) V

mente ACTIVA Tengan en cuenta los datos de la función de la actividad 32 y hallen su fórmula. – 12 _______ f(x) = 3x 4x + 2 140

36

ACTIVIDADES Función homográfica

33. Grafiquen las siguientes funciones. +5 ______ d. f(x) = 2x x+2

3x – 1 a. f(x) = ______ 2x – 3

–8 –7 –6 –5 –4 –3 –2

y

y

4

4

2

2

–1

0 1

2

3

4

5

6

7

8 x

–8 –7 –6 –5 –4 –3 –2

–1

–2

–2

–4

–4

–6

–6

–8

–8

1 __ x+3

2 b. f(x) = _______ 2x – 3

0 1

2

3

4

5

6

7

8 x

0 1

2

3

4

5

6

7

8 x

0 1

2

3

4

5

6

7

8 x

+3 ______ e. f(x) = –x 2x – 1 y

y

4

3

2

2

1 –8 –7 –6 –5 –4 –3 –2

–1

0 1

2

3

4

5

6

7

8 x

–2 –8 –7 –6 –5 –4 –3 –2

–1

–4

–1

–6

–2

–8

–3

–4 c. f(x) = x_____ x–3

+3 _______ f. f(x) = –2x –x + 1 y

y

4

3

2

2

1 –8 –7 –6 –5 –4 –3 –2

–1

0 1

2

3

4

5

6

7

8 x

–2 –8 –7 –6 –5 –4 –3 –2

–1

–4

–1

–6

–2

–8

–3

141

INTEGRACIÓN 34. Hallen el dominio de las siguientes funciones

38. Marquen las opciones correctas.

racionales.

¿Cuál es el conjunto imagen de cada función? 3x – 2 a. f(x) = _______ 1 __

2

– 3x a. f(x) = x______ x3 – 1 4

x +1 b. f(x) = ______ x2 – 4 3 –x c. f(x) = x_____ x+1 x d. f(x) = _____ x2 + 1 + x2 e. f(x) = x_____ x+3 4 f. f(x) = _____ x3 + 1

– {1}

2

– {2;–2} – {–1}

{ }

– {8}

X

– {–1}

– {4}

1 2x + __

2 c. f(x) = ______ x–3

X

– {3}

– {2}

– – __41

– – __31

– {3}

{ }

x–2 d. f(x) = ______ 3x + 1

2 a. f(x) = _____ x–5 –3 b. f(x) = ___ x

X Solución gráfica a cargo del alumno.

36. Escriban una función racional que cumpla con las condiciones indicadas en cada caso. a. Que tenga a x = 2 como raíz y a x = –1 –2 como asíntota vertical. x_____ x+1 b. Que tenga a x = 3 como asíntota vertical y a (0;–1) como la ordenada al origen. c. Que tenga – {2} como dominio y – {2} como conjunto imagen. d. Que tenga a y = 3 como asíntota horizontal 3x –9 +3 +1 ______ ______ y f(3) = 0. b. x_____ ; c. 2x x –2 ; d. x + 1 . x–3

– __31

{ }

{ }

39. Hallen el dominio y la asíntota vertical de cada función. x+1 a. f(x) = ______ 3x – 4

4 4 – __ ; x = __ 3 3 – {–1}; x = –1

{ }

–x b. f(x) = 4_____ x+1 –2 ______ c. f(x) = 3x 2x – 4 x–6 d. f(x) = ______ 3x + 1

– {2}; x = 2 1 ; x = – __ 1 – – __ 3 3

{ }

40. Tengan en cuenta las funciones dadas y escriban V (Verdadero) o F (Falso) según corresponda. –1 ______ a. f(x) = 2x x–3

Las soluciones no son únicas.

37. Hallen las asíntotas de cada función.

Df =

F

– {–3}

Im =

– __21

{ }

F F

2 a. f(x) = _____ x+5

x = –5; y = 0

A. V.: x = 3

V

A. H.: y = __21

b. f(x) = __7x

x = 0; y = 0

Raíz = __21 ;0

V

f(0) = 2

x–3 c. f(x) = ______ 2x + 1

1 1 __ x = – __ 2; y = 2

4x – 1 d. f(x) = _______ 1 x+2 __ 2 –2 + x e. f(x) = ______ 3x + 6 ______ f. f(x) = 3x–6– 2

142

– __32

– {6}

x+1 b. f(x) = _______ 1 x–2 __ 4

35. Grafiquen las siguientes funciones.

–5 c. f(x) = _____ x+2 1 d. f(x) = ___ 2x

X

– {4}

– {–3} – {–1}

x+2

( )

F

+2 _______ b. f(x) = –3x –5 + 2x

x = –4; y = 8

Df =

– __25

{ }

2 x = –2; y = ___ –3

A. V.: x = – __25

2; y = 0 x = __ 3

Raíz = 2__3 ;0

(

)

V F V

Im =

– __53

{ }

A. H.: y = – __23 f(0) = 1 F

F V

capítulo

CONTENIDOS

34*-35*-36 41. Calculen el valor de a y b en cada caso.

43. Marquen las opciones correctas. ¿Cuáles son las funciones que cumplen con las condiciones pedidas en cada caso? a. Una asíntota es x = 3.

– 3 __ 3 1 ______ ; es raíz; A. H.: y = __ a. f(x) = ax 3. bx + 1 2 +3 ______ b. f(x) = ax ; Df = bx – 1

– __41 ; Im =

– __41 .

{ }

{ }

a. a = 2; +a ______ c. f(x) = 2x ; 2 es raíz; A. H.: y = 4. b = 6; bx + 1 b. a = 1; –3 ______ d. f(x) = ax ; f(0) = 1; Im = – {3}. b = 4; x–b 1 c. a = –4; b = __ 2 ; d. a = 3; b = 3.

2

–2 f(x) = x______ x–3 –2 X f(x) = x_____ x–3 –2 f(x) = x_____ x+3

42. Observen los gráficos y completen con el

b. Las asíntotas son x = –2 y y = –2.

número que corresponde a cada ítem.

–2 ______ f(x) = 3x x+2

+3 ______ 3 a. f(x) = 2x x+2

–2 _______ f(x) = –2x x–3

–3 ______ 1 b. f(x) = 2x x–2

–2 _______ X f(x) = –2x x+2

+3 ______ 2 c. f(x) = 2x x–2

d. x = –2 es asíntota.

c. El dominio es

3

e. El punto __23 ;0 es raíz.

( )

f. El dominio es

6

1

– {–2}.

1

– {–1}.

2 –2 f(x) = x______ x–1 2 –2 X f(x) = x______ x+1 x2 –2 f(x) = ______ 3x – 3

3

y

d. Una asíntota es y = –2.

2

2

–7

–6

–5

–4

–3

–2

–1

0

1

2

3

4

5

6

7

x

–2 f(x) = x______ x–3

–2

–2 ______ f(x) = 2x x–3

–4

–2 ______ X f(x) = 2x –x – 3

–6

e. La ordenada al origen es (0;–2). –2 ______ X f(x) = 2x x+1

2

y

2

–2 f(x) = x______ x–1

4

2

+2 f(x) = x______ x+1

2

–7

–6

–5

–4

–3

–2

–1

0

1

2

3

4

5

6

7

x

–2

f. Es de proporcionalidad inversa. 2

–2 f(x) = x______ x–3

–4

–2 ______ f(x) = 2x x–3

3

–2 X f(x) = ___ x

y 4

44. Grafiquen las siguientes funciones.

2

–7

–6

–5

–4

–3

–2

–1

0 –2

–4

1

2

3

4

5

6

7

x

x+1 a. f(x) = ______ 2x – 4

Solución a cargo del alumno.

–x b. f(x) = 3_____ x+1 –1 ______ c. f(x) = 4x x+4 –3 ______ d. f(x) = 3x x+1

143

capítulo

6

AUTOEVALUACIÓN Marquen las opciones correctas

45. ¿Cuál es el grado de la función (x – 2)2 . (x – 3)2? a. 2

b. 3

X c. 4

46. Tengan en cuenta la función polinómica f(x) = x3 . (x – 2)2 . (x + 1) y respondan. a. ¿Cuál es el orden de multiplicidad de x = 0? 2

1

X 3

No tiene.

3

X No tiene.

3

No tiene.

b. ¿Cuál es el orden de multiplicidad de x = 1? 2

1

c. ¿Cuál es el orden de multiplicidad de x = 2? X 2

1

47. ¿Cuál es el intervalo de negatividad de la función f(x) = (x – 2) . (x + 3)? a. [–3;2]

X c. (–3;2)

b. (–3;+')

48. ¿En qué intervalos se puede asegurar que la función f(x) = x2 + x – 6 tiene una raíz? a. (–2;–1)

b. (–2;0)

X c. (–2;3)

49. ¿Cuáles son las raíces de la función f(x) = x3 – 3x + 2? X b. 1 y –2

a. –1 y 2

50. Tengan en cuenta la función f(x) =

c. 1 y 2

2 _____ x – 1 y respondan.

a. ¿Cuál es la ordenada al origen? (0;–1) (1;0) b. ¿Cuál es la preimagen de 4? –7

2 __ 3

X (0;–2)

(2;0)

3 X __

3 __ 4

2

51. Tengan en cuenta la función f(x) =

2x –3 ______ y respondan. 3x – 6

a. ¿Cuál es el dominio de la función? X

– {2}

– __21

{ }

– {–2}

b. ¿Cuál es la asíntota horizonal de la función? x=2

2 X y = __ 3

y = __21

c. ¿En cuál punto de la gráfica la función corta al eje x?

( 0;__21 ) 144

(2;0)

X

( __23;0 )

Contenidos 37. 38. 39. 40. 41.

7

Función exponencial. Logaritmos. Función logarítmica. Ecuaciones exponenciales. Ecuaciones logarítmicas.

Cuenta la historia que Pitágoras, el gran filósofo y sabio, caminaba por la calle cuando escuchó los sonidos que provenían de una herrería. Observando los golpes que daba el herrero con su martillo descubrió que las leyes que rigen a los sonidos se basan en proporciones; más tarde probó golpeando recipientes llenos de líquidos y haciendo vibrar cuerdas de distintas longitudes, para concluir que la regla es siempre la misma. Si una cuerda vibra con cierta frecuencia, al partirla y tomar la mitad vibra el doble de rápido y la nota que se produce es la misma, una octava más aguda. Si, en cambio, se toman sus dos terceras partes, la frecuencia se multiplica por __23 y la nota es lo que se llama una quinta respecto de la original. Esto le bastó para construir la escala musical con 12 notas (contando los sostenidos) que se relacionan a través de una proporción constante. Lo que Pitágoras no podía saber, en ese entonces, es que veinte siglos más tarde su descubrimiento iba a perfeccionarse con ayuda de los logaritmos. A modo de ejemplo, los logaritmos sirven para averiguar cuál es una nota x, si se conoce su frecuencia y la frecuencia de do.

1. Lean atentamente y respondan. a. Para construir la escala musical, ¿se habrá dividido en 12 partes iguales? b. Si la frecuencia de la es 440 y la de la siguiente la es 880, ¿cómo calcularían las frecuencias de las notas intermedias? a. No, porque la relación es proporcional constante, lo que indica que la frecuencia de cada nota se obtiene multiplicando la anterior por una constante. b. Para calcular la frecuencia (F) entre la primera la y la siguiente, se multiplica por 2. Como hay 12 notas, cada una de las cuales se obtiene multiplicando la frecuencia de la anterior por una __ 12 constante (c), se obtiene FLA2 = 2FLA1 = FLA1 . c12 ⇒ c = 32 .

capítulo

Funciones exponenciales y logarítmicas

37

36

38

39

40

41

42

43

44

45

46

Función exponencial ¿Para qué sirve?

INFOACTIVA

PÁGINA 12

Se denomina función exponencial a toda función de la forma f(x) = k . ax – b + c ∧ a > 0 ∧ a ≠ 1. ˆ Funciones de la forma f(x) = ax 1. 0 < a < 1 2. a > 1 1 f(x) = __ 3

x

( )

–2

y

y

2

2

1

1 0

–1 –1

x 1 2 y = 0 → A. H.

ˆ Funciones de la forma f(x) = k . ax ∧ k D

–2

f(x) = 3x

0

–1 –1

x 1 2 y = 0 → A. H.

– {0}

y f2(x) 5

Modifica el valor de la ordenada.

f1(x)

4

f(x) = k . ax

k

f1(x) = 3x

3 f3(x)

1 x

2

x

5

f2(x) = 2 . 3 f3(x) = 5 . 3

2 1 –2

2

x

f1(x)

4 3

f(x) = ax–b

b

Corrimiento

f1(x) = 3x

0

No tiene.

f2(x) = 3x+1

–1

1 hacia la izquierda.

1

1 hacia la derecha.

f3(x) = 3

1

y

Indica el corrimiento sobre el eje x.

x–1

0

–1

ˆ Funciones de la forma f(x) = k . ax – b

y = 0 ← A. H.

f3(x)

f2(x) 2 1 –2

0

–1

ˆ Funciones de la forma f(x) = ax + c ∧ c D

y = 0 ← A. H. 1

2

x

3

y 4

Indica el corrimiento sobre el eje y.

3

f(x) = ax + c x

c

Corrimiento

A. H.

2

0

No tiene.

y=0

f2(x)

f2(x) = 3 + 1

1

Hacia arriba, 1.

y=1

f1(x)

f3(x) = 3x – 1

–1

Hacia abajo, 1.

y = –1

–2

f1(x) = 3 x

f3(x)

146

y=1

1 0

–1 –1

1

2

x

y=0 y = –1

Test

de comprensión

1. Respondan y expliquen las respuestas. a. En la definición de función exponencial k . ax–b, ¿por qué a no puede ser 1? b. ¿Cómo tiene que ser la función exponencial g(x) para que resulte simétrica a f(x) = k . ax con k > 0 con respecto al eje x? a. Porque si a = 1 la función f(x) = k . ax–b = k . 1x–b = k es c. ¿Cuándo una función exponencial es decreciente? un número real k, por lo tanto f(x) = k, que es una función constante.; b. g(x) = –k . ax; c. Cuando a < 1.

37

ACTIVIDADES Función exponencial

1. Completen las tablas y grafiquen f1(x), f2(x) y f3(x) en un mismo eje y f4(x), f5(x) y f6(x) en otro. a. f1(x) = 2x x

–2

–1

0

1

2

f1(x)

1 __ 4

1 __ 2

1

2

4

b. f2(x) = 3 . 2

y

x

x

–2

–1

0

1

2

f2(x)

3 __ 4

3 __ 2

3

6

12

f2(x)

–2

–1

0

1

2

f3(x)

3 – __ 4

3 – __ 2

–3

–6

–12

d. f4(x) = __21

f1(x)

–5 –4 –3 –2 –1 0 1 2 –1 –2 –3 –4 f3(x) –5

c. f3(x) = –3 . 2x x

5 4 3 2 1

3

4 5

x

3

4 5

x

x

( )

x

–2

–1

0

1

2

y

f4(x)

4

2

1

1 __ 2

1 __ 4

5 4 3 2 1

e. f5(x) = 3 . __21

f3(x)

x

( )

x

–2

–1

0

1

2

f5(x)

12

6

3

3 __ 2

3 __ 4

–1

0

1

2

–3

3 – __ 2

3 – __ 4

f. f6(x) = –3 . __21

–5 –4 –3 –2 –1 0 1 2 –1 –2 f (x) –3 5 –4 –5

x

( )

x f6(x)

–2 –12

–6

f4(x)

2. Hallen los valores de a y k para que la función g(x) = k . ax cumpla con la condición pedida en cada caso. x

a. Que sea simétrica a f(x) = __31 con respecto al eje x.

( ) b. Que sea simétrica a f(x) = ( __51 ) c. Que su imagen sea (–';0).

x

con respecto al eje y.

1 k = –1; a = __ 3

k = 1; a = 5

k = –1; existen infinitas soluciones para a. Por ejemplo, a = 20.

d. Que pase por los puntos (1;3) y (2;6).

3 k = __ 2; a = 2 147

37

ACTIVIDADES Función exponencial

3. Grafiquen las siguientes funciones en los mismos ejes de coordenadas. Luego, resuelvan. f(x) = 5x; g(x) = 5x + 1; h(x) = 5x – 2; i(x) = 5x – 5. a. Completen con las ordenadas al origen de cada función. y 9 8 7

h(0) = 25

5

g(0) =

i(0) =

1 _____ 3 125

b. Completen con las imágenes o preimágenes según corresponda.

6 5 4 3 2 1

f(–1) = g(–2) = 0 1

–5 –4 –3 –2 –1

1 ___

1

f(0) =

2 3

4 5 6 7

x

1 __ 5 1 __ 5

1 __

h(1) = 5 i(2) =

( 3 ) = 125 1 g( –3 ) = ___ 25

1 ____ 125

( ) = 25 1 i( 2 ) = ____ 125 h 4

f

4. Tengan en cuenta las funciones exponenciales dadas y hallen g(x) y h(x). b. f(x) = __31

x

( )

a. f(x) = 4x y g(x)

–3

–2

x+2 g(x) = y = 4

–1

y f(x)

6

h(x)

g(x)

f(x)

5

5

4

4

3

3

2

2

1

1 0

1

2

; h(x) =

3

x

y = 4x–1

–3

–2

1 __ g(x) = y = 3

h(x)

6

–1

0

1

x+1

( )

2

3

x–2

; h(x) =

( __31 )

5. Tengan en cuenta la relación entre las fórmulas de g(x) y h(x) con f(x) y completen. f(x) = k . ax

1 f(–2) = ___ 18

f(–1) = __61

a. g(x) = k . ax+2 1 __

g(–2) = 2 1 ___

g(–4) = 18 g(0) =

148

9 __ 2

f(0) = __21

f(1) = __23

f(2) = __29

b. h(x) = k . ax–3

( ) = __23 g( –3 ) = __61 1 g( –4 ) = ___ 18 g –1

h(1) = h(4) = h(5) =

1 ___ 18 3 __ 2 9 __ 2

( 5 ) = __29 h( 3 ) = __21 h( 2 ) = __61 h

x

37

ACTIVIDADES Función exponencial

6. Grafiquen las siguientes funciones de la forma f(x) = k . ax + c y completen la tabla. a. f(x) = 4x + 1

b. g(x) = 4x – 1

c. h(x) = 2 . 4x + 1

d. i(x) = –4x – 1

y 6 5 4 3 2 1 –5 –4 –3 –2 –1 –1 –2 –3 –4

Funciones

1 2

3

k

a

c

Imagen

1

4

1

(1;+')

y=1

1

4

–1

(–1;+')

y = –1

h(x) = 2 . 4 + 1

2

4

1

(1;+')

y=1

i(x) = –4x – 1

–1

4

–1

(–';–1)

y = –1

x

f(x) = 4 + 1 x

g(x) = 4 – 1 x

x

4 5 6 7

Asíntota

7. Unan con flechas la función con la asíntota horizontal correspondiente. a. f(x) = b. f(x) = c. f(x) = d. f(x) = e. f(x) =

ˆ ˆ ˆ ˆ ˆ

3x – 3 5x+3 – 2 x+2

( __31 )

3 + 2 . 4x–2 1 __ x–3 5 .5 + 2

y y y y y

= = = = =

0 –2 –3 2 3

8. Completen con los datos pedidos. Luego, grafiquen cada función en sus carpetas. a. f(x) = 25 . __51

x–2

Df:

; Im: (–1;+')

( )

f(0) = 624 C+:

(–';4)

b. g(x) = –8 . 2x–1 + __21

–1

; A. H.: y = –1 ; C–:

(4;+')

1 __ ; Im: ( –'; 2 )

Df: 7 __ f(0) = – 2

C+:

1 __ ; A. H.: y = 2

(–3;+')

; C–: (–';–3)

mente ACTIVA ¿Cuáles son las raíces de las siguientes funciones? a. f(x) = 3x – 1 b. g(x) = 9 . 3x + __31

c. h(x) = –4 . 2x+3 + __81

a. (0;0); b. No tiene. c. (–8;0)

149

38

37

39

40

41

42

43

44

45

46

47

Logaritmos INFOACTIVA La logaritmación es una operación entre dos números reales a y b, llamados base y argumento, respectivamente, que se define como: loga b = c ‹ ac = b ∧ a > 0 ∧ a ≠ 1 ∧ b > 0

log2 16 = 4 ⇔ 24 = 16 Existen dos logaritmos cuya notación es especial: ˆ el decimal (base 10), que se simboliza log10 b = log b; ˆ el natural o neperiano (base e ≅ 2,71), que se simboliza loge b = ln b Aproximadamente.

Propiedades de los logaritmos 1. loga 1 = 0 ⇔ a0 = 1

log3 1 = 0 ⇔ 30 = 1 2. loga a = 1 ⇔ a1 = a 1 1 = 1 ⇔ __ log__1 __ ( 12 ) = __12 2 2

3. loga (xy) = loga x + loga y ∧ x > 0 ∧ y > 0

log5 (5 . 25) = log5 5 + log5 25 = 1 + 2 = 3 4. loga __yx = loga x – loga y ∧ x > 0 ∧ y > 0 81 = log 81 – log 27 = 4 – 3 = 1 log3 ___ 3 3 27

5. loga bn = n . loga b

log6 2164 = 4 . log6 216 = 4 . 3 = 12 Para calcular logaritmos en los cuales el argumento no es potencia de la base, se debe recurrir a un cambio de base, utilizando logaritmos con bases convenientes o logaritmos decimales o neperianos, los cuales pueden resolverse con la calculadora científica. log b

log b

log 8

log 8

ln b c 6. loga b = ______ = _____ = ____ logc a log a ln a 2 ln 8 log16 8 = _______ = ______ = _____ log 16 log 16 ln 16 2

150

de comprensión

Test

1. Respondan y expliquen las respuestas. a. ¿Es cierto que log2 64 = log2 32 + log2 32? log2 81 b. ¿Es correcta la siguiente igualdad? log3 81 = _______ log2 3 a. No. Por ejemplo, log2 64 = log2 32 + log2 2. b. Sí.

38

ACTIVIDADES Logaritmos

9. Calculen aplicando la definición. a. log3 27 =

3

8 f. log__3 ___ = 2 27

b. log7 49 =

2

g. log3 –9 = No existe.

1 c. log2 ___ 16 =

–4

d. log5 0,04 = e. log10 0,0001 =

–3

h. loge 1 = –2 –4

0

i. ln e =

1

j. ln __e1 =

–1

10. Indiquen V (Verdadero) o F (Falso) según corresponda. a. log3 7 + log5 9 = log8 (7 + 9) b. log3 242 = 2 . log3 24

F

V

c. ln (3 . 27) = 3 . ln 27 F

d. ln (10 . 11) = ln 10 + ln 11 V log 5 V e. log8 53 = 3 . _____ log 8 log 41

f. log941 = ______ log 9

V

11. Resuelvan aplicando propiedades. __

a. log3 ( 27 . 33 ) = __

b. ln ( e3 . 3e ) =

__

( 3)

2 . 3 __21 =

__ 1 ____ . 35 125___ ________ e. log5 3 = 325 __

3 33 . 27 f. log3 _______ = 81

___ 1 __ = g. log7 _________ 5 349 . 37 __ 1 __ 33 . 9 ______ h. log3 81 =

0,001 ___ i. log _____

( 310 )

7 __ 2 5 __ 2

8__ = c. log2 ___ 32

d. log8

7 __ 2

3

=

2 __ 9 19

– ___ 6 2 – __ 3 9

– ___ 10 11 – ___ 2 21 – ___ 2

151

38

ACTIVIDADES Logaritmos

12. Calculen los siguientes logaritmos sin usar calculadora. a. log4 8 + log4 512 = 6

d. log6 1 080 – log6 5 = 3

b. log18 486 + log18 12 = 3

e. log 0,0002 – log 2 = –4

c. log15 75 + log15 45 = 3

f. log2 224 – log2 7 = 5

13. Calculen los logaritmos aplicando propiedades. Tengan en cuenta las siguientes igualdades. log 2 = 0,301030; ln 4 = 1,3862. a. log 4 =

0,60206

b. log 256 =

2,40824

c. log 4 096 = 3,61236 d. log __41 =

–0,60206

e. ln 16 =

2,7724

1 f. ln ___ 16 =

–2,7724

g. ln 64 =

4,1586

____

h. ln 3256 =

2,7724

14. Expresen los siguientes logaritmos en función de a y b sabiendo que log7 2 = a y log7 3 = b. __

a. log7 6 = a+b

b. log7 12 = 2a + b

d. log7 36 = 1 __ 2 . (a + b)

3

__

e. log7 39 = 3 __ 2 b

__

c. log7 72 = 3a +2 b

152

__

f. log7 ( 32 . 33 ) = 1 a + __ 1 __ 2 b 2

38

ACTIVIDADES Logaritmos

15. Expresen los siguientes cálculos como un solo logaritmo. a. log 3 + 2 . log 4 – __23 . log 100 =

c. 2 . log 3 + __21 . log 9 – 5 . log 1 =

6 log ____ 125

log 27

b. log __32 – log __59 + log 27 =

21 d. log 25 + 2 . log 21 + log ___ 25 =

1

log 21

16. Completen la siguiente demostración de una de las propiedades de los logaritmos. loga (x . y) = loga x + loga y Supongamos que:

{

loga x = P ‰ a loga y = Q ‰ a

loga (x . y) = loga ( a . a p

( ‰(

q

p

=

q

=

) ‰ loga a p +

x y q

(Por definición de logaritmo). (Por definición de logaritmo).

(Por propiedad de la potenciación).

) . log a (Por propiedad de los logaritmos). p q + ) . 1 (Por propiedad de los logaritmos). p q Luego, log (x . y) = ( + )

 ‰

p

+

q

a

a

Como loga x = P ∧ loga y = Q ‰ loga (x . y) = loga x + loga y

17. Expresen como un solo logaritmo. a. log5 x + log25 x3 = 5 __ 2 log5 x

b. log2 x3 – 3 log2 x2 + log2 x = 2 log2 ( __x1 )

+3 c. log (x + 3) – log x_____ 2 + log x =

log 2x

–2 d. log5 x_____ x + 3 + log5 (x + 3) + log5 x =

log5 ((x – 2) . x)

153

39

38

40

41

42

43

44

45

46

47

48

Función logarítmica INFOACTIVA Se define función logarítmica de base a, a la función inversa de la función exponencial de base a. f(x) = y = loga x ⇔ ay = x ∧ x > 0 ∧ a > 0 ∧ a ≠ 1

t f(x) = y = log3 x ⇔ 3y = x Df = (0;+∞) ∧ Cf = A. V.: x = 0 0 Intersección con el eje x: f(x) = 0, entonces log3 x = 0 ⇒ 3 = x ⇒ x = 1 y

x

y = log3 x

3

1 __ 9 1 __ 3

–2

2

–1

1

1

0

3

1

9

2

–3

–2

g(x) – 3x

f(x) = log3 (x)

0

–1

1

2

3

x

–1 –2

t f(x) = log3 x + 1 Df = (0;+∞) ∧ Cf = A. V.: x = 0 1 Intersección con el eje x, f(x) = 0, entonces log3 x + 1 = 0 ⇒ 3–1 = x ⇒ x = – __ 3 x

y = log3 x + 1

1 __ 9 1 __ 3

–1

1

1

y 2

3

2

9

3

f(x) = log3 (x) + 1

1

0 –1

0

1

2

3

x

–1 –2

t f(x) = y = ln (x – 1) ⇔ ey = x – 1 A. V.: x = 1 x – 1 > 0 ⇒ x > 1 ⇒ Df = (1;+∞) ∧ Cf = Intersección con el eje x, f(x) = 0, entonces ln (x – 1) = 0 ⇒ e0 = x – 1 ⇒ x = 2 y

x

x=1

y = ln (x – 1)

f(x) = log3 (x – 1)

2

2

0

e+1

1

e2 + 1

2

3

e +1

3

1 0

–1 –1 –2

154

1

2

3

4

5

x

Test

de comprensión

1. Respondan y expliquen las respuestas. a. ¿Es cierto que la función y = loga x puede tomar valores negativos? b. Si se tiene el gráfico de una función de la forma y = ax, ¿es posible saber cuál es el gráfico de la función y = loga x? a. Sí, para valores 0 < x < 1, y < 0; b. Sí, porque las funciones y = ax e y = loga x son simétricas con respecto a la recta y = x.

39

ACTIVIDADES Función logarítmica

18. Completen la tabla, hallen el dominio y grafiquen cada una de las siguientes funciones logarítmicas. a. y = log4 x

b. y = log x

x

1 ___ 16

y = log4 x

–2

1 __ 4

1

4

16

x

1 ____ 100

1 ___ 10

1

10

100

0

1

2

y = log x

–2

–1

0

1

2

1

2

–1

y

y

2

2

1

1

–1

1

2

3

4

5 x

–1

–1

–1

–2

–2

Df: (0;+')

3

4

5 x

Df: (0;+')

19. Grafiquen sobre un mismo eje de coordenadas y completen. a. f(x) = log2 x

b. g(x) = log__1 x

c. h(x) = log3 x

2

d. i(x) = log__1 x 3

y f(x) = log2 x

4 3

h(x) = log3 x

2 1 –1

–1 –2 –3 –4

2

4

6

8

10

12

14

16

18

20

22

x

24

i(x) = log__1 x 3

g(x) = log__1 x 2

Df: (0;+') ; Cf:

+ – ; Raíz de f: x = 1 ; f(0) = ∃ ; A. V.: x = 0 ; C f : (1;+∞) ; C f : (0;1)

Dg: (0;+') ; Cg:

+ ; Raíz de g: x = 1 ; g(0) = ∃ ; A. V.: x = 0 ; C g: (0;1)

Dh: (0;+') ; Ch:

+ – ; Raíz de h: x = 1 ; h(0) = ∃ ; A. V.: x = 0 ; C h: (1;+∞) ; C h: (0;1)

Di: (0;+') ; Ci:

; Raíz de i:

x = 1 ; i(0) =

∃ ; A. V.: x = 0 ; C +: i

– ; C g: (1;+∞)

(0;1) ; C –: i

(1;+∞) 155

39

ACTIVIDADES Función logarítmica

20. Completen escribiendo la función correspondiente. y

a. f(x) = log3 (x + 1)

2

b. g(x) = log3 (x – 1)

1

c. h(x) = log3 (x + 3) –2

d. i(x) = log3 (x – 3)

0

–1

1

2

3

4

5

6

7

x

8

–1 –2

h(x)

f(x)

g(x)

i(x)

21. Tengan en cuenta la actividad anterior y completen. Df: (–1;+'); Cf:

+ – ; Raíz de f: x = 0 ; f(0) = 0 ; A. V.: x = –1 ; C f : (0;+∞); C f : (–1;0)

Dg: (1;+') ; Cg:

; Raíz de g: x = 2 ; g(0) =

∃ ; A. V.:x = 1

Dh: (–3;+'); Ch:

; Raíz de h: x = –2 ; h(0) =

1 ; A. V.:x = –3 ; C +: (–2;+∞) ; C – : (–3;–2) h h

Di: (3;+') ; Ci:

; Raíz de i:

x = 4 ; i(0) =

+ – ; C g: (2;+∞) ; C g: (1;2)

∃ ; A. V.: x = 3 ; C +i :

(4;+∞); C –i :

(3;4)

22. Completen la tabla. Función

Dominio

A. V.

log7 x

(0;+')

x=0

log8 (x + 1)

(–1;+')

x = –1

log (x – 3)

(3;+')

x=3

ln (x + 5)

(–5;+')

x = –5

log__1 (x – m)

(m;+')

x=m

5

23. Tengan en cuenta la función y resuelvan. f(x) = log4 (x + 2) a. Grafiquen. b. Hallen su función inversa y grafíquenla.

y

f–1(x) = 4x – 2

3 2

f–1(x) = 4x – 2

f(x) = log4 (x + 2) 1

–3

c. Completen con el dominio y el codominio de cada función. Df: (2;+') ; Cf: Df –1:

156

;

; Cf –1: (–2;+') .

–2

0

–1 –1 –2 –3

1

2

3

4

x

39

ACTIVIDADES Función logarítmica

24. Tengan en cuenta la función y resuelvan. f-1(x) y

f(x) = ln x – 3 a. Grafiquen. b. Hallen su función inversa y grafíquenla.

6 4

f–1(x) = ex+3 2

–6

c. Completen con el dominio y el codominio de cada función. Df: (0;+'); Cf:

0

–2

2

4

6

8

10 x

–2 –4

;

–6

; Cf –1: (0;+') .

Df –1:

–4

f(x)

25. Tengan en cuenta las funciones dadas y escriban V (Verdadero) o F (Falso) según corresponda. f(x) = log2 x; g(x) = log2 x + 1; h(x) = log2 (x + 1). V

a. f(x) y g(x) tienen el mismo dominio.

b. f(x) y h(x) tienen el mismo dominio. F c. f(x) y g(x) tienen la misma imagen.

V

d. f(x) y h(x) tienen la misma imagen. V e. La asíntota vertical de g(x) es x = 1. F f. La asíntota vertical de h(x) es x = 1. .

F

26. Completen la tabla tomando como referencia f(x) = log x y m > 0. Función

Dominio

log x + 1

(0;+')

Codominio

Desplazamiento

A. V.

1 unidad hacia arriba

x=0

log x – 2

(0;+')

2 unidades hacia abajo

x=0

log (x + 1)

(–1;+')

1 unidad hacia la izquierda

x = –1

log (x – 3)

(3;+')

3 unidades hacia la derecha

x=3

log (x + m)

(–m;+')

m unidad hacia la izquierda

x = –m

log x + m

(0;+')

m unidad hacia arriba

x=0

mente ACTIVA Una función logarítmica f(x) tiene dominio (–9;+∞) y codominio . a. La función inversa ¿es exponencial? a. Sí, siempre es exponencial. b. ¿Cuál es el dominio y el codominio de esa función inversa? b. Dominio: ; codominio: (–9;+∞). 157

INTEGRACIÓN 27. Grafiquen las siguientes funciones.

30. Indiquen la asíntota que corresponde en cada caso. a. f(x) = 6x + 2 b. f(x) = 9x–1 – 3 x c. f(x) = __41 – __23 d. f(x) = 8x e. f(x) = 12x + 3 f. f(x) = 4 – 5x

x

a. f(x) = 4

b. g(x) = 2 . 4x c. h(x) = –4x

( )

x d. p(x) = __41 x e. q(x) = – __41

( ) ( )

y=2 y = –3 3 y = – __ 2 y=0

y=3 y=4

f. r(x) = 5 . 4x g. s(x) = 5 . __41

31. Marquen las opciones correctas.

x

( ) h. t(x) = –5 . (__41 )

x

x

a. ¿Cuál es la imagen de f(x) = –2 . __31 ?

( )

Solución gráfica.

X (–';0)

(–2;+')

28. Grafiquen la función y = 7x–1 y respondan. a. ¿Cuál b. ¿Cuál c. ¿Cuál d. ¿Cuál e. ¿Cuál f. ¿Cuál

es es es es es es

el la el el la la

dominio? imagen? (0;+') conjunto de positividad? conjunto de negatividad? No tiene. raíz de la función? No tiene. ordenada al origen? 0;__1

(

7

)

29. Grafiquen la función f(x) = 7x + 1 y escriban V (Verdadero) o F (Falso) según corresponda. a. El dominio de la función son todos los números reales. V b. La imagen de la función son todos los números reales. F c. La función tiene asíntota horizontal en x = 0. F d. La función corta al eje de ordenadas en y = –7. F e. La función tiene asíntota vertical en

x

b. ¿Cuál es la asíntota de f(x) = __23 + 1?

( )

y = __23

X y = 1

No tiene. x

c. ¿Cuál es la ordenada al origen de f(x) = __23 + 1?

( )

X (0;2)

(2;0)

(0;1)

32. Dadas las siguientes funciones exponenciales, indiquen dominio, imagen, ordenada al origen, asíntota horizontal, conjunto de positividad y conjunto de negatividad. Luego, grafiquen sin realizar tabla de valores. x a. Df: ; Im: (1;+'); f(0) = 2; a. f(x) = __31 + 1 A. H.: y = 1; C+: . b. g(x) = 4x+1 – 2 b. D : ; I : (–2;+'); g(0) = 2;

( )

g

m

1 1 __ – A. H.: y = –2; C+: – __ 2 ;+' . C : –';– 2 .

(

)

(

)

33. Observen los gráficos y completen con la letra de la función que corresponde. a. f(x) = 8x + 1 b. g(x) = __81

c. h(x) = 8x+1

x

d. p(x) = __81

( )

–x

( )

y g(x)

x = 1. F

6

f. La función es creciente. V g. La intersección de la función con el eje y es

5 4

(0;2). V

p(x)

3 2 f(x)

1

h(x) –3

158

–2

–1

0

1

2

3

x

capítulo

CONTENIDOS

7

37*38*39 34. Observen los gráficos y escriban una fórmula

37. Hallen el valor de las siguientes expresiones

que se corresponda con cada uno. a. y = 2x + 1

sabiendo que ln a = 3 y ln b = 5. a. ln a2 + ln b2 = 16

y

a b. ln ( __ = –6 b) 3

4

3

2 1 0 1

–3 –2 –1

b. y = 2x+3

2

3

x

y

3 ab 3__ 3a ___ __ e. ln b = – 27 __ 3 1 __ b 3__ f. ln ___ = 6 3a

a. ¿Cuál es el dominio f(x) = log6 (x – 1)?

6 4

X (1;+')

(–';–1) b. ¿Cuál es el dominio f(x) = log6 x + 1?

2 0 2

x

f. log 100 = g. log3 81 = 8 h. log__2 ___ = 3 27 i. log__1 25 = 5 16 j. log__7 ___ 49 =

2

c. ¿Cuál es la asíntota de f(x) = log6 x + 1? X x = 0

3

x

y

1 ___ 16 1 __ 4

4

1

0

16

–4

No tiene.

d. ¿Cuál es la ordenada al origen de f(x) = ln (x + e)?

–1 –2

(0;e)

(0;e – 1)

X (0;e + 1)

e. ¿Cuál es la raíz de f(x) = __21 . log6 x – 1?

36. Completen las tablas, hallen el dominio y Df: (0;+').

x = –1

4

4

grafiquen. a. f(x) = –2 . log4 x

X (0;+')

(–';0)

35. Calculen los siguientes logaritmos. 3 a. log2 8 = b. log7 49 = 2 1 c. log__1 ___ = 4 2 16 2 2 d. ln e = e. log 1 000 = 3

8

38. Marquen las opciones correctas.

8

–10 –8 –6 –4 –2

___

c. ln 3ab = __3 1 ___ d. ln ____ = – __8 3

3

(–6;0)

X (36;0)

No tiene.

39. Observen el gráfico de la función de la forma y = loga (x + b) y respondan. a. ¿Cuál es el valor de a? a = 3 b. ¿Cuál es el valor de b? b = 1 c. ¿Cuál es el dominio de la función? d. El codominio de la función ¿puede ser ?

2

c. (–1;+'). d. Sí. y

b. g(x) = log5 x + 1 x

y

1 ___ 25 1 __ 5

–1 0

Df: (0;+').

2 1 0

–1

1

1

–1

25

3

–2

125

4

1

2

3

4

5

6

7

8

x

159

40

39

41

42

43

44

45

46

47

48

49

Ecuaciones exponenciales INFOACTIVA Una ecuación exponencial es aquella en la que la incógnita aparece en el exponente. Para resolver una ecuación exponencial, hay que tener en cuenta: 1. ax ⇒ a > 0 ∧ a ≠ 1 2. ax = ax ⇒ x1 = x2 3. Las propiedades de las potencias. 1

2

En la página 20 pueden repasar las propiedades de la potenciación.

Resuelvan las siguientes ecuaciones exponenciales. a. 32x+1 = 81 3 32x+1 = 34 ⇒ 2x + 1 = 4 ⇒ x = __ 2

b.

2x+2

____

__

32x+3 = 38

x+3 _____

3 __

3⇒x=0 x+3 = __ 22x+2 = 22 ⇒ _____ 2x+2 2

c. 4x–2 + 4x + 4x+1 = 324 1 +1+4 4x + 4x + 4x . 4 = 324 ⇒ 4x . ___ ___ 16 42 81 = 324 ⇒ 4x = 43 ⇒ x = 3 4x . ___ 16

(

) = 324

d. Sn = 1 + 3 + 9 + 27 + … + 3n–1 = 3 280 qn – 1

Se utiliza la fórmula de la suma de n términos de una sucesión: Sn = a1 . _____ q–1 x–1

3 – 1 = 3 280 1 . _______ 3–1

3x–1 – 1 = 6 560 ⇒ 3x–1 = 6 561 ⇒ 3x–1 = 38 ⇒ x – 1 = 8 ⇒ x = 9 e. 32x+1 – 2 . 3x – 1 = 0 3 . 32x – 2 . 3x – 1 = 0 Se usa una variable t = 3x ⇒ 32x = (3x)2 = t2 3t2 – 2t – 1 = 0

{

t1 = 1 ⇒ 3x = 1 ⇒ x1 = 0 1 ⇒ 3x = – __ 1 ⇒ x no es solución. t2 = – __ 2 3 3

En algunos casos, para resolver las ecuaciones exponenciales es necesario despejar la incógnita. Para esto, se aplica en cada miembro el logaritmo cuya base es la base de la potencia. ax = b loga ax = loga b ⇒ x . loga a = loga b ⇒ x = loga b

Hallen el valor de x. 10x–2 = 8 log 10x–2 = log 8 ⇒ (x – 2) . log 10 = log 8 ⇒ x = log 8 + 2 ⇒ x = 2,903

160

de comprensión

Test

1. Respondan y expliquen las respuestas. a. ¿Cuál es la solución de la siguiente ecuación? 8x = 0 b. ¿Es cierto que la solución de 2 x = 8 es x = 4? a. No tiene solución. b. No. La solución es x = 3 porque 23 = 8.

40

ACTIVIDADES Ecuaciones exponenciales

40. Unan las ecuaciones con su solución. a. 3x = 243 b. 5x = 625 c. 2x + 1 = 257 d. 2x + 1 = 8 192 e. 22x = 2x + 1

x x x x x

= = = = =

8 1 5 12 4

41. Completen para que se cumpla la igualdad. a. 3

–2

6

–3

5

b. c. 7

d. 10

= 729

–2

g. 3

= 0,01

5

. 2 = 486

2

1 = ____ 125

e.

3 3

1 = ___ 49

f. 3

. 2 = 18

h. 312 : 3

: 6 = __29

i. 3

3

7

= 243

. 4 = 108

42. Escriban como una sola potencia. a3x a. ___ ax =

a2x

3x + 1 .a = b. a______ a2x

ax + 2

c. ( a2x )3 . a3x =

a9x

21x d. [ ( a3x )2 . ax ]3 = a

43. Resuelvan las siguientes ecuaciones. a. 2 x + 5 = 256

d. 25x + 3 = 5x + 2

x=3

x = –4

__

b. 5x + 3 : 25 = 35 1 x = – __ 2

e.

4x + 1

____

332x –

1 x = __ 4

9 __ 2

= __91

____

c. 7x – 3 . 98 = 14 x=2

f. 332x – 2 = 93x + 2 x = –1

161

40

ACTIVIDADES Ecuaciones exponenciales

44. Hallen las raíces de las siguientes funciones. a. f(x) = 3x + 1 + 3x + 3 – 10

c. h(x) = 9 x + 5 – 3x – 2

x = –1

x = –12

2x + 3

3 d. i(x) = ____ – 81 33x + 1

b. g(x) = 7x + 1 – 6 . 7x – 1 x=0

x = –2

45. Marquen las opciones correctas. ¿Cuáles son las soluciones de las siguientes ecuaciones? a. 3 . 2x + 2 – 5 . 2x – 1 = 76 X 3

–3

8

32x + 1 b. ____ – 17 . 3x = 30 3x – 2

X 1

____ x+1

c. 34

–1 1 __ 2x + 3

+4

= 1 056

3 x+2

d. __23

( )

1–x

+ __32

30

( )

–4

X 4

X –2

2

x

11 – 2 . __23 = ___ 27

( )

3 __ 2

46. Resuelvan las siguientes ecuaciones. a. __41

x+1

( )

+ __41

x+2

( )

= 1 280

x = –6

b. 5 . 3x + 1 + 2 . 3x + 1 = 7 x = –1

c. 4x + 1 + 4x + 2 – 320 = 0 x=2

162

d. 2x + 1 + 2x + 3 + 2x – 1 = 168 x=4

x e. __31 – 2 . __31

( )

x+2

( )

= 21

x = –3

f. 7x + 1 + __71

–x–2

( )

x = –2

53 – 3 . 7x = ___ 49

40

ACTIVIDADES Ecuaciones exponenciales

47. Resuelvan las siguientes ecuaciones. a. 2 x + 2 + 2 . 4x = 160 x=3

h. 32x – 5 . 3x – 2 = –8 x = 1; x = log3 2

b. 42x – 3 . 22x = 4 x=1

i. 42x – 5 . 4x+2 + 1 024 = 0 x = 2; x = 3

c. 2x – 72 = –4x x=3

j. 7x – 71 – x = 6 x=1

d. 72x + 4 . 7x + 3 = 0 No tiene solución.

x

15 e. 5x – ___ 5 = 2

( )

x=1

k. 92x – 82 . 32 . (x – 1) + 1 = 0 x = 1; x= –1

l. 16x + 31 . 22 . (x – 2) = __81 x = –2

x

27 f. 3x – ___ 3 = 26

( )

No tienen solución.

g. 22x – 3 . 2x+1 + 8 = 0 x = 1; x = 2

m. 252x + 7 . 52x + 10 = 0 No tiene solución.

1 n. 43x – 23x – 2 = – ___ 64

x = –1

163

41

40

42

43

44

45

46

47

48

49

50

Ecuaciones logarítmicas INFOACTIVA Las ecuaciones logarítmicas son aquellas en las que la incógnita aparece afectada por un logaritmo. Para resolver ecuaciones logarítmicas, hay que tener en cuenta: En la página 150 1. loga x ⇒ a > 0 ∧ a ≠ 1 pueden repasar las propiedades de 2. loga x1 = loga x2 ⇒ x1 = x2 los logaritmos. 3. Las propiedades de los logaritmos. 4. Se deben verificar las soluciones para asegurar que no se obtengan logaritmos nulos o negativos.

Resuelvan las siguientes ecuaciones logarítmicas. a. log 3 + log (6 + x2) = 2 . log (4 + x) log [3 . (6 + x2)] = log (4 + x)2 3 . (6 + x2) = (4 + x)2 18 + 3x2 = 16 + 8x + x2 2 2x – 8x + 2 = 0

x1 = 2 + 33 ∧ x2 = 2 – 33

x 3 . log x = 5 + log ___ 10 3 . log x = 5 + log x – log 10 3 . log x – log x = 5 – 1 2 . log x = 4 log x = 4 : 2 log x = 2

x = 100

__

__

b.

c. ln x + ln (x – 2) = 2 . ln (x – 1) ln [x . (x – 2)] = ln (x – 1)2 x . (x – 2) = (x – 1)2 x2 – 2x = x2 – 2x + 1 0 = 1 ← absurdo. No tiene solución. d.

log (9 + x2) __________ =2 log (4x + 3)

log (9 + x2) = 2 . log (4x + 3) log (9 + x2) = log (4x + 3)2 9 + x2 = (4x + 3)2 9 + x2 = 16x2 + 24x + 9 17x2 + 24x = 0 x . (17x + 24) = 0

164

x1 = 0 ← Es solución. 24 ← No es solución porque hace negativo al argumento. x2 = – ___ 17

Test

de comprensión

1. Respondan y expliquen las respuestas. a. ¿Es cierto que logx x = 1 para todo x D ? b. ¿Cuál es el valor de x en la ecuación log3 –x = –2? 1 a. No. Solo para x > 0 y x ≠ 1.; b. x = – __ 9

41

ACTIVIDADES Ecuaciones logarítmicas

48. Unan cada ecuación logarítmica con su solución. a. log3 (x + 2) = 5 b. log2 (x – 1) = 3 c. log__1 (x + 3) = –2 2 d. log2 (–x – 1) = 1 e. log x = 3

x x x x x

= = = = =

–3 1 241 9 1 000

49. Resuelvan aplicando la definición de logaritmo. Luego, verifiquen la solución. a. log2 x = 3 x=8

b. log5 x = –1 1 x = __ 5

c. log3 (x + 2) = 2 x=7

d. 3 . log5 x = 3 x=5

e. ln x = 2 x = e2

f. ln (–x) = –1 1 x = – __ e

50. Resuelvan las siguientes ecuaciones. a. log2 (x + 3) – log2 (x – 5) = 3 43 x = ___ 7

b. log2 x + log2 __8x = 1 x=4

c. log3 x2 + log3 x – 3 = 0 x=3

d. log3 (x + 2) + log3 (x + 1) = log3 (x2 – 1) No tiene solución.

e. log5 x + log5 (2x – 1) – log5 (2x + 2) = 0 x=2

f. log (x – 1) – log (x – 3) = log 2 x=5

165

ACTIVIDADES Ecuaciones logarítmicas

41

51. Hallen las raíces de las siguientes funciones logarítmicas. a. f(x) = log3 4x + log3 __9x

d. i(x) = log2 (3x + 1) + log2 (x – 3) – 3

b. g(x) = log x5 – 2 log x2 + 3

e. j(x) = log7 (2x + 1) – log7 (x – 5)

3 x = __ 2

11 x = __ 3

1 x = _____ 1 000

No tiene solución.

c. h(x) = log2 (x – 1) + log2 (3x + 1) – 2 5 x = __ 3

f. k(x) = 5 . log5 x – log5 x3 – 2 x=5

52. Marquen las opciones correctas. ¿Cuáles son las soluciones de las siguientes ecuaciones? 3 a. log5 (x – 2) – log5 (x + 3) = log5 ___ 13 7 X __

2 __ 7

3______ __ b. 3log3 x4 + 4 . log3 3x = 4

2

X 3

2 c. log2 (x – 1) – [log2 x + log2 (x + 1)] = log2 (2x – 3) – log2 x

4

2

–2 __ d. ln x – ln 3x + __51 . ln x–2 = __21 X e5

X 2

3

1 __

__

3

__ 2

__ 6

e2

e2

5

– __51

e. log3 3x – log3 3x = log3 35 X __1 5

53. Resuelvan las siguientes ecuaciones aplicando previamente cambio de base. a. 5 . log4 x – 7 . log2 x = –27 x = 64

b. log6 x – log216 x = log6 4 x=8

21 c. log3 x + log9 x = log27 x + ___ 6

x = 27

166

11 d. log5 x2 + log125 x – log25 x = __ 3

x = 25

76 e. log2 x3 – log8 x2 + log32 x = ___ 15

x=4

245 11 ____ f. __37 . log4 x + __29 . log64 x – __ 6 . log16 x = 4

x = 242

41

ACTIVIDADES Ecuaciones logarítmicas

54. Resuelvan las ecuaciones. a. log2 (9 – 2x) + log2 x = 2 1 S: __ 2 ;4

g. log125 x – log5 x = –2 S: 125

(x – 4)

b. log2 (x + 3) + log2 ______ + log2 (x – 1) = 2 (x + 3) S: 5

h. log2 x + log4 x – log16 x = 5 S: 16

c. log3 (x2 – 6x – 7) – log3 (x – 7) = log2 4 S: 8

i. log2 x + log4 x2 – log16 x4 = 5 S: 32

d. log 7 x2 – 4 = 0

j. log3 x2 + 9 log27 x2 = 4 __

__

S: –33 ;33

S: –49;49

e. log (x + 1)2 – log (x + 2) = log 2x __

k. log2 (x – 1)2 – log2 (x2 – 4x + 3) = log2 4 11 S: __ 3

S: –1 + 32

f. log6 x + log6 (1 – x) = 1 No tiene solución.

l. log3 __x1 = 6 – 2 . log3 x S: 729

mente ACTIVA ¿Es correcta la resolución de la siguiente ecuación? (ln x)2

ln x (ln x)2 = ln x ⇒ ______ = ____ ⇒ ln x = 1 ⇒ x = e ln x ln x

No es correcta porque al dividir por ln x no se tiene en cuenta que no puede ser 0.

167

INTEGRACIÓN 55. Completen las siguientes igualdades. 7

a. 2

¿Cuántas soluciones tienen las siguientes ecuaciones? a. 5x = 4x

= 128 0

b. 345

=1

5

c. 3

1 __ 2

e.

g. 5

Una

i. 4

X Infinitas.

exponenciales. a. 52x + 25x = 250 b. 2x+2 + 2x+1 + 2x = __87

. 3 = 375 11

= 36

29 c. 5x+1 – 5x + 5x+2 = ____ 625 d. 10x + 10x+1 – 110 = 0 e. 72x – 56 . 7x + 343 = 0 f. 22x – 17 . 2x–3 = – __41

. 2 = 32 = 256

¿Cuáles son las soluciones de cada ecuación? a. 23x+2 = 32 X x = 1

x = 10

x=0

X x = – __1 3

x=3

x = –4

X x = 2

____

353x+2 = 5

x = __31 c. 3x+1 + 3x = 36 x=4

X x = –2

x = __21

x=2

x . 31 – 1 = ___ 1 e. 3_________ 27 22

x = __31

1 x = ___ 27

X x = –3

f. 7x+1 + 7x – 8 = 0 x=8

3 x = __ 2 x = –3

x = –4 x=1 x = 1; x = 2 x = –3; x = 1

60. Tengan en cuenta la siguiente función e indiquen el valor de x que corresponde en cada caso. f(x) = 4x . 2 a. f(x) = 512 x=4 b. f(x) = 32 x=2 1 ___ c. f(x) = 32 x = –3 d. f(x) = –1 No es posible. 61. Hallen las raíces de las siguientes funciones.

d. 5x+2 – 1 = 0

X x = 0

57. Respondan. ¿Es correcta la siguiente resolución? e2x = ex ex e2x ___ __ ex = ex ex = 1 x=0 Sí, porque ex no puede ser 0.

168

Ninguna.

?

59. Resuelvan las siguientes ecuaciones

= 16 807

56. Marquen las opciones correctas.

b.

Infinitas.

= 16 2

j. 82 . 4

10 ___ 3 –x

Ninguna. –x

( )

–4

h. 613 : 6 2

b. 3x = __31

= 100 000

f. 73 . 7 3

X Una

= 243 5

d. 10

58. Marquen las opciones correctas.

No tiene.

x=1 a. f(x) = 4x + 4x+2 + 4x–1 – 69 x+3 x x+1 b. g(x) = 5 + 5 – 5 – 3 025 x = 2 c. h(x) = 9x – 90 . 3x + 729 x = 2; x = 4 No tiene. d. i(x) = 2x+2 + 2x+3 + 2x x x x =0 e. j(x) = e + 3 . e – 4 1 2x x x = __ f. k(x) = 3 + 4 . 9 – 15 2 x = 1; x = 2 g. l(x) = 32x – 4 . 3x+1 + 27

62. Respondan. ¿Para cuáles valores de x se cumple cada igualdad? a. 12___ . 32x = 8___ 748 x=3 x x+6 x–1 2x b. 33 = 33 x=3

capítulo

CONTENIDOS

7

40*41 63. Observen el gráfico de la función y = 2x + 2 + 3

66. Resuelvan las siguientes ecuaciones logarít-

y respondan sin resolver la ecuación. Luego, verifiquen la respuesta.

micas. a. log2 (x + 2) + log2 (5x + 2) = 8 x = 6 b. log6 (x + 10) + log6 (x – 2) = 2 x = 6 c. log4 x – log4 (x – 1) = 1 x = __34 d. log2 x + log8 x – 3 . log64 x = __35 x = 4 e. 5 . log2 x2 – 26 . log2 x + 5 = 0 x = 32 f. log3 x2 + 3 . log3 x = 10 x = 9 g. (log3 x)2 + 3 . log3 x + 2 = 0 x = __1 ; x = __1

y

7 6 5

9

3

4 3

67. Hallen las raíces de las siguientes funciones.

2

a. f(x) = log5 x + log5 33 x – log5 x2 – __34 x = 25 b. g(x) = –3 . log4 x + 2 . log4 x5 – log4 x8 + 3

1 –3 –2 –1

1 ___

__

0 1

2

c. h(x) = (log4 x)2 – 2 . log4 x – 3 x = __41 ; x = 64 d. i(x) = log125 x – log5 x2 + log25 x3 + __31

x

1 31 x = __ e. j(x) = log4 x – __81 + log4 x + ___ 4 4

¿Cuál es la solución de 2x + 2 + 3 = 4?

(

x = –2

)

(

)

64. Marquen las opciones correctas.

f. k(x) = log__1 (x + 13) – log__1 (x + 31) – 1

¿Cuáles son las soluciones de las siguientes ecuaciones? a. log5 (x + 2) = 2

g. l(x) = log (3 – x2) – log x – log 2

X x = 23

x=8

3

x = 27

b. log5 (2x + 5) = log5 (x + 3) X x = –2

x = __21

x=2

X x=5

x=6

d. 2 . log3 (x + 2) = log3 729 X x = 25

x=5

x=3

e. ln (x + 3) = 2 x=e

x=1

X x = e2 – 3

f. 4 . log3 x – log3 9 + 2 = 0 x=0

X x=1

1 b. x = 64; d. x = ___ 25 ; f. x = –4; g. x = 1

68. Respondan.

¿Cuáles son los valores de x para los cuales las siguientes funciones tienen la misma imagen? f(x) = log 2 + log (11 – x2) g(x)= 2 . log (5 – x) 1 x = 3; x = __ 3

c. log2 (x – 1) + log2 (3x + 1) = 6 x = –5

3

No tiene.

65. Lean atentamente y respondan. El logaritmo de base cinco del logaritmo de base tres de un número es igual a uno. ¿Cuál es ese número?

69. Hallen las soluciones de las siguientes ecuaciones. a. 4 . log3 x = 20 b. 3x + 2 = 2 c. 3x = 2x + 1 d. e2x = 20 e. 3x + 2 = 21 – 2x

x = 243 x = –1,37 x = 1,71 x = 1,5 x = 5,23

70. Respondan. ¿Para cuáles valores de x se cumple cada igualdad? a. 3 + log3 (x + 2) = 5 x = 7 b. 3 + log3 (x + 2)2 = 5 x = 1; x = –5

x = 243

169

capítulo

7

AUTOEVALUACIÓN Marquen las opciones correctas

71. ¿Cuál es el gráfico que corresponde a f(x) = 4x+1 – 2?

–6

y

y

X a.

–4

y

b.

c.

4

4

4

2

2

2

0

–2

2

4

x

–6

–4

0

–2

2

4

x

–6

–4

0

–2

–2

–2

–2

–4

–4

–4

2

4

x

72. Tengan en cuenta la función f(x) = log2 (x + 1) – 3 y respondan. a. ¿Cuál es su dominio?

X (–1;+')

(1;+')

(3;+')

b. ¿Cuál es su imagen?

(3; +')

c. ¿Cuál es la asíntota vertical?

x=3

d. ¿Cuál es su raíz?

(3;0)

X (7;0)

(1;0)

e. ¿Cuál es la ordenada al origen?

(0;3)

X (0;–3)

(0;0)

X

(1;3) X x = –1

x=1

73. ¿Cuál es el gráfico de la función de la actividad anterior? a.

X b.

y

c.

4

y

y

2

2

2 –4 –4

–2

–2

0

2

4

6

x

–4

–2

0

–2

–2

–2

–4

–4

–4

–6

–6

0

2

4

6

x

74. ¿Cuáles son las soluciones de las siguientes ecuaciones? a. 3x + 1 – 3x + 2 – 3x = –63 X x = 2

x = –3

x = –63

b. 22x – 3 . 2x + 1 + 5 = 0 X x = 1; x = 5

x = 1; x = –5

x = –1; x = 5

c. log5 (x + 98) + log5 (32 – x) = 4 x = –98; x = 27 d. log6 x3 – log36 x = 12

170

X x = –93; x = 27

x = –27; x = 93

14 x4 + __54 . log216 x5 = ___ 3

x=6

X x = 36

2

4

6

x

Contenidos

8

42. Sistema de medición de ángulos. 43. Razones trigonométricas. 44. Valores exactos y aproximados. 45. Ecuaciones trigonométricas. 46. Triángulos rectángulos. 47. Teoremas del seno y del coseno. 48. Triángulos oblicuángulos.

Unos doscientos años antes de nuestra era, la ciudad de Alejandría fue testigo de un sorprendente hecho: con ayuda de apenas una vara, Eratóstenes logró calcular en forma aproximada la circunferencia de la Tierra. Por supuesto, hace falta algo más que una vara: se dice que Eratóstenes necesitó además la ayuda de un regimiento de soldados y, lo más importante de todo, algo de trigonometría. ¿Cómo es esto? Eratóstenes sabía que en Asuán, durante el solsticio de verano, los objetos verticales no proyectaban sombra y se le ocurrió que, midiendo las sombras en Alejandría y conociendo las distancias entre ambas ciudades, entonces podría hacer el cálculo. Dicho y hecho: mandó al regimiento a contar los pasos hasta Asuán (una pequeña caminata de casi 1 000 km), después midió la sombra proyectada por cualquier arbolito alejandrino y... ¡ listo !

1. Lean atentamente y respondan. a. ¿Por qué en Asuán los objetos verticales no proyectaban sombra? ¿Por qué en Alejandría sí? b. ¿Para qué otros cálculos podrá usarse la trigonometría? a. El texto se refiere a un momento determinado en el que el sol está justo encima de Asuán. Por lo tanto, no proyecta la sombra de objetos verticales allí, pero sí los proyecta en Alejandría, ya que el sol no está encima de esa ciudad. b. Abierta. Cualquier problema de resolución de triángulos requiere de la trigonometría. Por ejemplo, el cálculo de distancias entre planetas o estrellas, o algo más cotidiano como el GPS.

capítulo

Trigonometría

42

41

43

44

45

46

47

48

49

50

51

Sistema de medición de ángulos INFOACTIVA Para medir ángulos se utilizan distintos sistemas de medición. ˆ Sistema sexagesimal: la unidad de medida en este sistema es el grado sexagesimal (1°), que se obtiene al dividir el ángulo recto en 90 partes iguales. 1 recto 1° = ______ 90 ⇒ 1 recto = 90°

ˆ Sistema circular: la unidad de medida en este sistema es el radián. Se llama radián al ángulo central que abarca un arco de circunferencia cuya longitud es igual al radio de la misma. a ___

r = ob

R

___

_ o

b

ab = ob ^ _ = 1 radián ab ^ ___ _ = ___ ob

Equivalencia entre los sistemas Los ángulos centrales son proporcionales a los arcos que intersecan, por lo tanto: ___

^ 360° ___ 360° 1 radián 2/___ob _ ___ ___ ___ ___ ⇒ ______ = ___________ ⇒ _______ = ______ = 360° ⇒ 2/ = 360° ab long C(o;ob) ob 2π ob ob

En la siguiente tabla figuran algunas equivalencias entre los dos sistemas de medición de ángulos.

172

Sistema sexagesimal

Sistema circular

90°

/ __ 2

180°

/

270°

3/ __ 2

360°

2/

Test

de comprensión

1. Respondan y expliquen las respuestas. / a. ¿Es cierto que 1° equivale a ____ 180 radianes? b. Si en la calculadora aparece como resultado 125,2°, ¿es equivalente a 125° 2’?

a. SÍ. b. No, equivale a 125° 12’.

42

ACTIVIDADES Sistema de medición de ángulos

1. Marquen las opciones correctas. / a. ¿Cuál ángulo es equivalente a __ 3?

90°

120°

X 60°

30°

b. ¿Cuál ángulo es equivalente a 45°? / __ 2

/ X __  4

/ __ 3

/

2 X __ 5 /

X 10°

c. ¿Cuáles ángulos son agudos? / X __ 

100°

8

2. Calculen en grados sexagesimales. a. 4/ =

720°

c. 2 radianes = 114° 35’ 30’’

e. __45 / =

/ b. __ 6 =

30°

d. 1,5 / =

f. 2,75 / =

270°

225° 495°

3. Expresen los siguientes ángulos en radianes, en función de /. a. 120° = b. 225° =

2 / __ 3 5 __ / 4

c. 315° = d. 100° =

7 / __ 4 5 __ / 9

e. 135° = f. 270° =

3 / __ 4 3 __ 2 /

4. Escriban V (Verdadero) o F (Falso) según corresponda. Expliquen los casos donde escribieron F. / a. 90° = __ 2 V

f. 82,5° = 82° 50’ F

/ F b. 180° = __ 2

c. 4 /= 360°

d. 180 /= 180°

/= 180°

F

720°

F

e. 21° 4’’ = 75 840’’

g. 137,5° = 137° 30’

V

/ F h. 120° = /+ __ 2

2 / __ 3

180° /= 32 400°

i. __41 /+ __61 /= 75°

F

j. __47 /– /= 135°

75 604’’

82° 30’

V

V

173

43

42

44

45

46

47

48

49

50

51

52

Razones trigonométricas ¿Para qué sirve?

INFOACTIVA

PÁGINA 13

Se llaman razones trigonométricas a las que relacionan las longitudes de los lados de un triángulo rectángulo con los ángulos agudos del mismo. Razones directas

Razones recíprocas

cateto opuesto sen ^ _ = ______________ hipotenusa



hipotenusa cosec ^ _ = ______________ cateto opuesto

cateto adyacente cos ^ _ = _______________ hipotenusa



hipotenusa sec ^ _ = _______________ cateto adyacente

cateto opuesto tg ^ _ = _______________ cateto adyacente



cateto adyacente cotg ^ _ = _______________ cateto opuesto

Hipotenusa

Cateto opuesto

_ Cateto adyacente

Según el cuadrante en donde se ubica el ángulo, las razones trigononométricas pueden ser positivas o negativas. y 90° = __/ rad 2 x ^ ^ ^ ^ II I ˆ Si 0° < ` < 90° (cuadrante I): sen ` > 0; cos ` > 0; tg ` > 0 y ^ ^ ^ ^ ˆ Si 90° < ` < 180° (cuadrante II): sen ` > 0; cos ` < 0; tg ` < 0 180° = / rad 0° = 2/ rad _ x ^ ^ ^ ^ ˆ Si 180° < ` < 270° (cuadrante III): sen ` < 0; cos ` < 0; tg ` > 0 ^ ^ ^ ^ ˆ Si 270° < ` < 360° (cuadrante IV): sen ` < 0; cos ` > 0; tg ` < 0 III IV 3 270° = __ 2 / rad

Identidades trigonométricas Las identidades trigonométricas son igualdades en las cuales aparecen razones trigonométricas y resultan verdaderas para cualquier valor de los ángulos agudos de un triángulo rectángulo. Para demostrar o resolver una identidad trigonométrica se desarrollan uno o ambos miembros de la misma, tratando de obtener expresiones equivalentes. Para ello se utilizan las relaciones que se establecen entre las razones trigonométricas de un mismo ángulo.

a. 1 + tg2 ^ _= sec2 ^ _

^ = cosec2 _ ^ b. 1 + cotg2 _

^ + 1) . (sen _ ^ – 1) = –cos2 _ ^ c. (sen _

sen2 ^ _ = ______ 1 _ + ______ cos2 ^ ______ 2^ _ _ cos2 ^ cos _ cos2 ^

cos2 ^ _ = ______ sen2 ^ _ + ______ 1 ______ 2^ _ sen _ sen2 ^ _ sen2 ^

^ – 1 = –cos2 _ ^ sen2 _

^ + sen2 _ ^=1 cos2 _

^ + cos2 _ ^=1 sen2 _

^ + cos2 _ ^=1 sen2 _

d. cosec ^ _. tg ^ _= sec ^ _

2 e. (sen ^ _+ cos ^ _) = 2. tg ^ _. cos2 ^ _+ 1

sen ^ _ = _____ 1 . _____ 1 _____ ^ sen _ cos ^ _ cos ^ _

sen ^ _ . cos ^ sen2 ^ _ + 2 . sen ^ _. cos ^ _+ cos2 ^ _= 2 . _____ _. cos ^ _+ 1 ^ cos _

sen2 ^ _+ cos2 ^ _+ 2 . sen ^ _. cos ^ _= 2 . sen ^ _. cos ^ _+ 1 1 Los casos a., b., c. y e. se verifican por la relación pitagórica. El caso d. se verifica por llegar a la misma expresión en ambos miembros.

1 = _____ 1 _____ cos ^ _ cos ^ _

174

Test

de comprensión

1. Respondan y expliquen las respuestas. a. ¿Es cierto que el producto entre una razón trigonométrica y su recíproca siempre es igual a 1? ¿Y el cociente? b. En un triángulo rectángulo, si sen _ = cateto opuesto, ¿cuál es el valor de la hipotenusa? a. Sí. El cociente es igual al cuadrado de la razón trigonométrica. b. La hipotenusa vale 1.

43

ACTIVIDADES Razones trigonométricas

5. Tengan en cuenta el siguiente triángulo y completen. c _

` a

b

___

__

ab __ a. sen ^ _ = ___

bc ___ d. cosec ^ _ = ___

bc

ab

___

___

^ bc ___ k. sec ` = ___

bc

__

ab __ c. tg ^ _ = ___ ac

__

^ ab __ h. cos ` = ___

bc __ e. sec ^ _ = ___ ac

bc

^ bc __ j. cosec ` = ___ ac

bc

__

__

ac __ b. cos ^ _ = ___

__

__

^ ac __ g. sen ` = ___

ab

___

__

^ ac ___ i. tg ` = ___

ac ___ f. cotg ^ _ = ___ ab

^ ab __ l. cotg ` = ___ ac

ab

6. Resuelvan usando la calculadora. a. sen 46° =

0,7193

f. sen ^ _ = –0,76 ‰ ^ _= 310° 32’ 9’’ o 229° 27’ 51’’

b. cos 132° 10’ =

–0,6713

^ ^ g. cos `= 0,85 ‰ `=

31° 47’ 18’’ o 328° 12’ 42’’

^ ^ h. tg b= –1,532 ‰ b=

123° 8’ o 303° 8’

c. tg 222° 25’ 36’’ = 0,9139 d. sen 305° 12’ =

–0,8171

^= 0,14 ‰t ^= i. sen t

8° 2’ 52’’ o 171° 57’ 8’’

e. tg 125° =

–1,4281

j. cos ^ ¡= –0,9 ‰^ ¡=

154° 9’ 29’’ o 205° 50’ 31’’

7. Expresen teniendo en cuenta el siguiente cuadrado. d

c _

___

a. ao en función de ^ _. ___

___ ___ ___ ___ ob ao = cos ^ _ . ab ∨ ao = sen ^ _ . ab ∨ ao = ____ ___

tg ^ _

__ b. ac en función de ^ _.

__ ^ c. bc en función de `.

__

__

___

sen ^ _

cos ^ _

__ ab bc ac = ______ ∨ ac = ______

o

`

__ ^ __ __ oc bc = cos `. ac ∨ bc = ______ __

^ cos `

a

b 175

43

ACTIVIDADES Razones trigonométricas

8. Dadas las siguientes gráficas, escriban las razones trigonométricas pedidas. a.

c. 3

–1

_

t

1 –1

sen ^ _ = cos ^ _ = tg ^ _ =

___

__

3______ . 310 10 ___ 10 3___ 10

^= sen t

2 – 3___ 2

^= cos t

2 – 3___ 2

__

^= tan t

3

b.

1

d. 3

¡

`

4

–2 –2

^ sen ` = ^ cos ` = ^ tg ` =

__

___ 3 ___ . 13 13 3 ___ 13 –2 . 3___ 13 3 – __ 2

5 – 3___ 5

sen ^ ¡ =

2 __ __ 5 . 35 1 – __ 2

cos ^ ¡ = tg ^ ¡ =

9. Tengan en cuenta el dato y calculen las razones pedidas. Recuerden que los ángulos son del primer cuadrante. a. cos ^ _ = __52

__

3 ^ = 3___ c. tg t 2

^ _ = 66° 25’ 19’’ sen ^ _ = _ = tg ^

0,9165

^ = 40° 53’ 36’’ sen t ^= t ^= cos t

2,2913

0,6547

0,7559

__

^ 2 b. sen ` = 3___ 2

^ ` = ^ tg ` = 176

d. cosec ^ ¡ = 2,366

^ cos ` =

45° 1

__

2 3___ 2

^ ¡ =

25° 8’’

cos ^ ¡ =

sen ^ ¡ =

0,9063

0,4227

43

ACTIVIDADES Razones trigonométricas

10. Verifiquen las siguientes identidades. a. cotg ^ _ . sen ^ _= cos ^ _ cos ^ _ ______ . sen ^ _ = cos ^ _ sen ^ _

cos ^ _ = cos ^ _

1 b. sec ^ _. tg ^ _. cosec ^ _= ______ 2 ^

cos _ sen ^ _ ______ 1 . ______ 1 1 ______ ______ . = cos ^ _ cos ^ _ sen ^ _ cos2 ^ _ 1 1 ______ ______ = cos2 ^ _ cos2 ^ _

–1 c. (sec ^ _– cos ^ _) . (sen ^ _) = tg ^ _

^ cos ^ _ ______ sen ^ _ sen ^ _ sen ^ _ sen ^ _ 1 _ = ______ _ 1 – cos2 ^ sen2 ^ ____________ – ______ = sen _ ⇒ ____________ = ______ ⇒ ____________ ⇒ ______ = ______ cos ^ _ . sen ^ _ sen ^ _ cos ^ _ _ _ cos ^ _ cos ^ _ cos ^ _ . sen ^ _ cos ^ cos ^ _ . sen ^ _ cos ^

2 d. (sen ^ _+ cos ^ _) – 1 = 2 . sen ^ _. cos ^ _

sen2 ^ _ + 2 . sen ^ _ . cos ^ _ + cos2 ^ _ – 1 = 2 . sen ^ _ . cos ^ _ sen2 ^ _ + cos2 ^ _ – 1 = 0 1–1=0⇒0=0 –1 e. cotg ^ _+ tg ^ _= tg ^ _. (sen2 ^ _)

^ cos ^ cos2 ^ _ ______ _ + sen2 ^ _ sen ^ _ ______ 1 1 1 1 ______ + sen _ = ______ . = ____________ ⇒ ______________ = ____________ ⇒ ____________ sen ^ _ cos ^ _ cos ^ _ sen2 ^ _ sen ^ _ . cos ^ _ cos ^ _ . sen ^ _ sen ^ _ . cos ^ _ cos ^ _ . sen ^ _

f. (sen ^ _+ cos ^ _) . tg ^ _– sec ^ _= sen ^ _– cos ^ _ 2 ^ sen2 ^ _ _ ______ _ – 1 sen2 ^ 1 = sen ^ ______ _________ + sen ^ _ – ______ _ – cos ^ _ ⇒ ______ – 1 = –cos ^ _ ⇒ sen = –cos ^ _ ^ ^ ^ ^ cos _ cos _ cos _ cos _ cos ^ _

–cos2 ^ _ = –cos ^ _ ⇒ –cos ^ _ = –cos ^ _ ⇒ _______ cos ^ _

c

mente ACTIVA

_

Demuestren que si en un triángulo rectángulo el sen ^ _ = cos ^ _, el triángulo es isósceles. ___

__ ___ __ ab ac __ = ___ __ sen ^ _ = cos ^ _ ⇒ ___ ⇒ ab = ac ⇒ El triángulo bc bc abc es isósceles.

a

b

177

44

43

45

46

47

48

49

50

51

52

53

Valores exactos y aproximados INFOACTIVA La tabla muestra los valores exactos de las funciones trigonométricas de ángulos particulares.

_ sen ^ _ cos ^



/ __ 6 = 30°

0

1 __ 2

1

/ __ 4 = 45° __

__

3 3___ 2

1

1 __ 2

0

2

__

__

3 3___ 2

2 3___ 2

__

3 3___ 3

0

/ __ 2 = 90°

2 3___

__

_ tg ^

/ __ 3 = 60°

1

33

No existe.

Relaciones entre las funciones trigonométricas de un mismo ángulo del cuadrante I Las funciones trigonométricas de ángulos de cualquier cuadrante se pueden relacionar con funciones trigonométricas de ángulos del cuadrante I.

ˆ Ángulos suplementarios:

/ ^ / ^ __ ^ sen ^ _ = cos ( __ 2 – _) ∧ cos _ = sen ( 2 – _) sen ^ _ = sen (/ – ^ _) ∧ cos ^ _ = –cos (/ – ^ _)

ˆ Ángulos que difieren de /:

sen ^ _ = –sen (/ + ^ _) ∧ cos ^ _ = –cos (/ + ^ _)

ˆ Ángulos que suman 2/:

sen ^ _ = –sen (2/ – ^ _) ∧ cos ^ _ = cos (2/ – ^ _)

ˆ Ángulos que difieren de 2/:

sen ^ _ = sen (2/ + ^ _) ∧ cos ^ _ = cos (2/ + ^ _)

ˆ Ángulos opuestos:

sen ^ _ = –sen (–^ _) ∧ cos ^ _ = cos (–^ _)

ˆ Ángulos complementarios:

Escriban cada expresión trigonométrica como una expresión de un ángulo del primer cuadrante. Luego, resuelvan. 1 a. sen 150º = sen (180º – 30º) = sen 30º = __ 2 __

2 b. cos 225º = cos (180º + 45º) = –cos 45º = – 3___ 2 __

c. tg 300º =

3 – 3__ __ sen (360º – 60º) ________ 2 = –33 –sen 60º _______________ ____ = = cos 60º 1 cos (360º – 60º) __ 2 __

33 2__ = 2____ 1__ = ___ 11 π = ___________ 1 1 = ___ _____ d. sec ___ π π = __ __ 6 3

(

cos 2π –

6

)

cos

6

3__ 3

2

5 π = __________ 1 1 = __ 1 _____ e. cosec __ π 1 =2 π = __ __ 6

(

sen π –

178

6

)

sen

6

__ 2

33

Test

de comprensión

1. Respondan y expliquen las respuestas. a. Si sen ^ _ = –sen (–^ _), ¿es correcto pensar que cos ^ _ = –cos (–^ _)? b. ¿Es cierto que sen 330° = sen 30°? a. No, porque ambas funciones no se identifican con el mismo cateto. b. No, porque también se debe considerar el cuadrante al que pertenece el ángulo, para evaluar el signo sen 330° = –sen 30°.

44

ACTIVIDADES Valores exactos y aproximados

11. Marquen las opciones correctas. a. ¿Cuál es el seno de 210°? 1 __ 2

X – __1 2

__

__

3 3___ 2

3 – 3___ 2

/ b. ¿Cuál es la tangente de __ 4?

0

–1

X 1

sen ( / – ^ _)

X −sen ( –^ _)

No existe.

c. ¿Cuál es el seno de ^ _? / ^ sen __ 2 – _

(

)

X sen ( 2/ + ^ _)

12. Unan con una flecha las relaciones que pertenecen al mismo ángulo. / a. sen ( / – __ 8)

/ ˆ –cos __ 8

/ b. cos ( /+ __ 8)

/ ˆ cos ( – __ 8)

/ c. tg ( – __ 8)

/ ˆ sen __ 8

/ __ / d. sen ( __ 2 – 8)

/ ˆ –sen __ 8

/ e. cos __ 8

/ ˆ –tg __ 8

/ f. sen ( 2/– __ 8)

/ ˆ cos __ 8

13. Resuelvan buscando la equivalencia con un ángulo del primer cuadrante. __

3 3___ 2

a. sen __31 / =

( )

e. –cos __23 / =

( )

__

b. cos – __45 / =

(

)

2 – 3___ 2

__

c. tg __35 / =

( )

– 33

1 11 – __ d. –sen – __ 2 6 / =

(

)

0

__

– 3 f. –tg __34 / = 3

( )

__

2 3___ g. sen __43 / = 2

( )

__

2 3___ h. –cos __47 / = – 2

( )

179

45

44

46

47

48

49

50

51

52

53

54

Ecuaciones trigonométricas INFOACTIVA Las ecuaciones trigonométricas son aquellas en las que la incógnita está afectada por una función trigonométrica. Se pueden resolver utilizando distintos procedimientos. Los siguientes son ejemplos de la resolución de ecuaciones trigonométricas.

Hallen los valores de ^ _que hacen verdaderas las ecuaciones, sabiendo que ^ _D [0;2/) a. 2 . sen ^ _– 1 = 0 1⇒^ _= __ _= 30º ∨ ^ _= 150º sen ^ 2 1 _) + 1 = __ b. cos (π – ^ 2 1⇒ ^ –cos ^ _= – __ _= 60º ∨ ^ _= 300º 2 3 _= __ c. cos2 ^ 4 __

3 cos ^ _= ± 3___ 2 __

3⇒ ^ cos ^ _= + 3__ _= 30º ∨ ^ _= 300º 2 __

3⇒ ^ _= – 3__ _= 150º ∨ ^ _= 210º cos ^ 2

_– 2 . sen ^ _= –1 d. sen2 ^ _+ 1 = 0 sen2 – 2 . sen ^ _– 1) = 0 (sen ^ 2

_= 1 ⇒ ^ _= 90º sen ^ _– 3 . sen ^ _= 1 e. 4. sen2 ^ _– 3 . sen ^ _– 1 = 0 4. sen2 ^ _) 4 . t2 – 3t – 1 = 0 (t = sen ^ 1 _= 1 ∨ sen ^ _= – __ sen ^ 4

^ _= 90º ∨ ^ _= 194º 28’ 39,04’’ ∨ ^ _= 345º 31’ 20,96’’

180

Test

de comprensión

1. Respondan y expliquen las respuestas. a. ¿Cuántas soluciones tiene la ecuación sen2 ^ _ = __21 en el intervalo [0;/)? b. ¿Es cierto que el cos ^ _ nunca puede ser 1? 3 / ^ a. Tiene 2 soluciones: ^ _1 = __  y _2 = __ /. b. No. Por ejemplo, cos 0° = 1. 4 4

45

ACTIVIDADES Ecuaciones trigonométricas

14. Marquen las opciones correctas.

__

2 / 3___ a. ¿Cuál es la solución de cos ( x + __ 6) = 2 para x D [0;2/)? / X ___ 12

19 X ___ 12 /

/ – ___ 12

19 – ___ 12 /

b. ¿Cuál es la solución de 2 . sen2 x = 1 para x D [0;/)? 5 __ 4/

7 __ 4/

/ X __ 

3 X __ 4/

4

15. Relacionen cada ecuación con su solución, sabiendo que ^ _ D [0;2/). 4 / __ ˆ __ 3; 3 /

a. sen ^ _ = __23 b. cos ^ _ = – __1 __

11 ˆ __67 /; __ 6 /

2

c. tg ^ _ = 33

ˆ No existe.

d. 2 . sen ^ _ = –1

5 / __ ˆ __ 3; 3 /

e. 4 . cos ^ _ = 2

ˆ __32 /; __34 /

__

7 / __ ˆ __ 6; 6 /

f. 3 . tg ^ _ = 33

16. Encuentren los valores de ^ _ D [0;2/) que verifiquen las ecuaciones. a. 2 . cos ^ _ – 1 = 0 5 / __ S: __ ; / 3 3

(

)

3 / __ S: __ ; / 4 4

__

b. 4 . sen ^ _+ 2 . 33 = 0 4 5 /; __ / S: __ 3 3

__

c. tg ^ _+ 33 = 0 5 2 /; __ / S: __ 3 3

/ 1 __ d. sen ( _+ __ 4) = 2 7 S: ___ 12 /

__

2 e. cos ^ _– __23 / + 3___ 2 = 0

f. sen2 ^ _ – __43 = 0

4 5 / __ S: __ ; 2 /; __ /; __ / 3 3 3 3

g. cos2 ^ _– 1 = 0 S: /; 0

h. cos2 ^ _ – 2 . cos ^ _+ 1 = 0 S: 0

181

INTEGRACIÓN 17. Completen la tabla.

20. Escriban V (Verdadero) o F (Falso) según

Sistema sexagesimal

Sistema circular

135°

3 __ / 4 3 __ / 2 / __ 4 5 __ 4 /

270°

45° 225°

177° 53' 14''

3

–135°

5 __ 4 / 7 __ / 6 5 ___ 18 /

210° 50°

corresponda. a. Si el seno de un ángulo es __21 , la secante es 2.

F

b. Si la cosecante de un ángulo es 3,2, el seno es 0,3125. V c. La cotangente de un ángulo es igual a la inversa de la tangente del ángulo complementario.

F

d. La cosecante de un ángulo es igual a la inversa del coseno del mismo ángulo. F

18. Marquen las opciones correctas. __ 3 ^ a. Si sen ^ _ = 3___ 2 y 90° < _ < 180°, ¿cuál es el ^ coseno de _? __

3 3___ 2

1 __ 2

X − __1 2

3 − 3___ 2

__

__

__

33

1

−1

c. Si cos ^ _ = __21 y ^ _ < 90°, ¿cuál es el seno de ^ _? __

__

3 − 3___ 2

3 X 3___ 2

− __21

1 __ 2

d. Si cos ^ _ = __21 y ^ _ < 90°, ¿cuál es la tangente ^ de _? __

__

–33

X 33

0

No existe.

19. Hallen los ángulos de las siguientes expresiones. a. cosec ^ _= 1,753 ^ b. sec ` = 3,094 ^ c. cotg b = 0,251 ^ = 10,75 d. sec t e. cosec ^ ¡ = 2,09 f. cotg ^ a = 5

182

entre la hipotenusa y el cateto adyacente del

__

3 ^ b. Si sen ^ _ = 3___ 2 y 90° < _ < 180°, ¿cuál es la tangente de ^ _?

X –33

e. La secante de un ángulo es igual a la razón

34° 46’ 54’’ 71° 8’ 35’’ 75° 54’ 35’’ 84° 39’ 45’’

ángulo.

V

f. Si el seno de un ángulo es 0,5, el ángulo mide 60°.

F

g. El coseno es positivo en ángulos que pertenecen a los cuadrantes I y II.

21. Resuelvan las siguientes identidades. cotg ^ _ ______ a. ______ = 1^ sec ^ _ sen _ ^ ^ ^ ^ b. cosec `– sen `= cotg `. cos ` ^. tg t ^ + cos t ^ = cos–1 t ^ c. sen t d. 1 – sec2 ^ ¡ = –tg2 ^ ¡ 2 ^ e. (1 – sen a) . cosec2 ^ a = 1 ^ ^ f. (cosec m – cotg m). sen ^ m = 1 – cos ^ m sen2 ^ / g. _________ = 1 – cos ^ / 1 + cos ^ / ^ ^ h. 1 – cosec2 ` = cotg2 ` ^ ^ ^ ^ i. (cotg b+ 1). sen b = sen b + cos b 2 ^ ^ Ÿ . cosec2 Ÿ _______________ ^– cos2 t ^= cos j. cotg2 t 2 ^

sec Ÿ

Solución a cargo del alumno. 28° 35’ 8’’ 11° 18’ 36’’

F

capítulo

CONTENIDOS

8

42*43*44*45 _. 22. Expresen cada relación en función del sen ^ _________

a. cos ^ _= ^ b. tg _ = c. cotg ^ _ = d. cosec ^ _= e. sec ^ _=

_ sen ^ _________ a. ±31 – sen2 ^ _ b. _____________ ±31 – sen2 ^ _ _________ 2 ^ ± 1 – sen _ 1 3 ____________ ______ c. d. sen ^ _ sen ^ _

corresponda.

23. Tengan en cuenta los datos y calculen el valor de las funciones pedidas. __ 2 a. sen ^ _= – 3___ y 180° < _< 270° 2 __ 2 ^ – 3___ 1 cos ^ _= 2 ; tg _= __ __ ^ –32 –32 cosec ^ _= _= sec ; __

__ ^ –33 ; tg `= ^ 2 __ – __ 3 2 33 ; sec `=

3 – 3___ 2

c. tg ^ ¡ = 1 y 0° < ^ ¡< 90° __

2 3___

sen ^ ¡=

2

cotg ^ ¡=

1

a. Si el seno de un ángulo es negativo, el ángulo pertenece al segundo o tercer cuadrante.

F

Al 3° o al 4°.

b. Si el coseno de un ángulo es positivo y el seno del mismo ángulo es negativo, el ángulo

^ ^ b. cos ` = __21 y 270° < `< 360° ^ cosec `=

^ _= 15° 4’

27. Escriban V (Verdadero) o F (Falso) según

1 _________ e. ____________

±31 – sen2 ^ _

^ sen `=

26. Tengan en cuenta los datos y hallen el valor _. de ^ ^ ˆ _ pertenece al primer cuadrante. ˆ cos (55° + ^ _) = 0,342

pertenece al cuarto cuadrante.

V

c. Si un ángulo pertenece al segundo cuadrante, la tangente de dicho ángulo es positiva.

F

d. Si la tangente de un ángulo es positiva, __

^ ; cos ¡= ^ ; cosec ¡=

2 3___

entonces dicho ángulo pertenece al tercer

2

__

32

cuadrante.

F

O al 1.°.

e. Si el coseno de un ángulo es positivo y la

24. Expresen como funciones del primer cuadrante. 16 a. sen ___ 9 / =

2 / –sen __ 9

cuarto cuadrante.

/ __  3 / – tg __  4

f. La tangente y la secante de un ángulo no

–cos

b. cos __34 /= c. tg __43 /=

25. Hallen todos los valores ^ _ entre [0;2/]. a. sen ^ _ = 0 ^ b. sen  _ = 1 __ 3 ^ c. sen _ = – 3___

S: /; 2/; 0

d. cos ^ _ = –1 e. cos ^ _ = __1

S: /

2

2__

2 f. cos ^ _ = 3___ 2 g. tg ^ _ = –1 __ h. tg ^ _ = – 33 __

3 i. tg ^ _ = 3___ 3

tangente es negativa, el ángulo pertenece al

/ S: __ 2 4 5 __ S: 3 /; __ / 3 5 / __ ; / S: __ 3 3 7 / __ S: __ ; / 4 4

V

están definidas para valores que anulan el seno. F

El coseno.

g. Si un ángulo pertenece al segundo cuadrante, el seno y la tangente son negativos.

F

28. Resuelvan las ecuaciones considerando los _ D [0;2/]. valores de ^ 5 / __ a. ^ _ = __ ; / 4 4 /  = 0 a. cos ^ _ + __

(

4

)

3 _ = __ / b. ^

3 7 /; __ / S: __ 4 4 5 2 __ __ S: 3 /; 3 /

4 b. sen 2^ _ + 1 = 0 / __ 2 / __ ^ c. _ = 3; 3 /  c. 2 . sen 2^ _ – __ = 1 2 5 13 _ = ___ /; ___ / d. ^ / 24 24 d. 2 . cos 2^ _ + __ 4 = –1 7 __ 11 __ ^ e. _= /; 6 /; 6 /; e. 2 . sen2 ^ _ + sen ^ _ = 0

7 / __ S: __ ; / 6 6

f. cos2 ^ _ – cos ^ _ = 0

( (

) )

0; 2/

3 / __ f. ^ _= __ 2; 2 /; 0; 2/ 183

46

45

47

48

49

50

51

52

53

54

55

Triángulos rectángulos INFOACTIVA Resolver un triángulo rectángulo consiste en averiguar la longitud de sus tres lados y la amplitud de sus ángulos agudos. Un triángulo rectángulo queda determinado con dos de sus lados o con un lado y uno de sus ángulos agudos.

Resolución de triángulos rectángulos ˆ Dados dos catetos o un cateto y la hipotenusa.

Datos:

Dos catetos

__ bc = 12,1 cm __

Cateto e hipotenusa

{ ab = 8,6 cm

c

__ ^ Hallen b , ^ c y ac. __ Para calcular ac se puede________ aplicar de Pitágoras: ___________________ __ __ __ __ el teorema __ __ __

2 bc2 = ab + ac2 ⇒ ac = 3bc2 – ab2 ⇒ ac = 3__________ (12,1 cm)2 – (8,6 cm)2 ⇒ _____________________ __ __ __ 2 2 ac = 3146,41 cm – 73,96 cm ⇒ ac = 372,45 cm2 ⇒ ac ≅ 8,51 cm

a

b

^ Para calcular __ c se debe recurrir a una función trigonométrica que vincule los datos con el ángulo: 8,6 cm ab __ ⇒ sen ^ c = ________ ⇒^ c = arc sen 0,71 ⇒ ^ c ≅ 45º 17' 43,7'' sen ^ c = ___ 12,1 cm bc

^ Para calcular __ b se razona de forma análoga: ^ __ ^ ^ 8,6 cm ab ⇒ cos ^ cos b = ___ b = ________ ⇒ b = arc cos 0,71 ⇒ b ≅ 44º 42' 16,3'' 12,1 cm bc

ˆ Dados un ángulo agudo y uno de sus lados. Ángulo agudo y cateto c

{

^ b = 35º Datos: __ ab = 12 cm __

Ángulo agudo e hipotenusa

__

Hallen ^ c , ac y bc. Para calcular ^ c se aplica la propiedad de los ángulos agudos: ^ ^ ^ c + b = 90º ⇒ ^ c = 90º – b ⇒ ^ c = 90º – 35º ⇒ ^ c = 55º a

__

b

Para calcular ac se debe recurrir a una función trigonométrica que vincule los datos con el lado: __ __ __ __ ^ __ ^ ___ ac __ tg b = ⇒ ac = ab . tg b ⇒ ac = 12 cm . tg 35º ⇒ ac ≅ 8,4 cm ab

__

Para calcular en__forma análoga:__ __ bc __ __se razona ^ __ ab ⇒ bc = _____ ab ⇒ bc = _______ 12 cm ⇒ bc ≅ 14,65 cm cos b = ___ cos 35º bc

184

^ cos b

Test

de comprensión

1. Respondan y expliquen las respuestas. a. Si se tienen como datos un cateto y un ángulo agudo adyacente, para averiguar el resto de los elementos del triángulo, ¿se deben emplear las relaciones trigonométricas? b. Si se tienen como datos dos catetos, ¿qué relación trigonométrica se puede aplicar para averiguar un ángulo? a. Sí. b. La tangente.

46

ACTIVIDADES Triángulos rectángulos

29. Resuelvan los siguientes triángulos rectángulos. c

a.

a

c. ___

__

ab = 10,5 cm __ bc = 7,2 cm

cb = 9,2 cm ^ b = 32°

a

b

__

ac = 12,73 cm ; ^ a = 34° 26’ 20’’ ; ^ c = 55° 33’ 40’’ b.

c

c

b

___

d.

a

__

ab = 5,5 cm __ bc = 12,8 cm

b

__

^ ac = 11,56 cm ; b = 64° 33’ 8’’ ; ^ c = 25° 26’ 52’’

30. Calculen el perímetro de cada triángulo. ___ c

a. cd = 6 cm

a

d

; ac = 5,75 cm

c

___ a

__

ab = 10,85 cm ; ^ a = 58°

bc = 7,3 cm ^ a = 61°

b

^ b =

29°

___

3,54 cm

6,38 cm ; ab =

c

b. ad = 7 cm ^ c = 40°

b

___

__

; ac =

a

d

b

__

Perímetro = 12 . 33 cm

Perímetro =

54,93 cm 185

46

ACTIVIDADES Triángulos rectángulos

31. Lean atentamente y resuelvan. a. Una escalera de 5,5 cm se apoya en una pared formando un ángulo con el piso de 71°. ¿A qué altura de la pared llega la escalera? 5,20 m escalera pared piso

71°

b. Se debe colocar una rampa de 2,8 m de largo para facilitar el acceso a un edificio cuya entrada se encuentra a 1,2 m de altura. ¿Qué ángulo forma la rampa con el suelo?

25° 22’ 37’’ rampa

suelo

c. La sombra de un árbol que mide 3,5 m a cierta hora del día es de 6,2 m. ¿Qué ángulo forman los rayos del sol con el árbol? 60° 33’ 16’’

rayo de Sol árbol

sombra

d. Una escalera mide 225 cm de longitud. Cuando está totalmente abierta, la distancia entre el pie de la escalera con su soporte es de 98 cm. ¿A qué altura se encuentra el extremo superior de la escalera? 2,20 m escalera

soporte

e. Un niño remonta un barrilete desde una altura de 1,32 m y está usando 25 m de hilo. Si el ángulo que forma el hilo con el suelo es de 35°, ¿a qué altura se encuentra el barrilete? 14,34 m + 1,32 m = 15,66 m altura desde donde sostiene el niño 35°

hilo

suelo 186

46

ACTIVIDADES Triángulos rectángulos

32. Lean atentamente, realicen un gráfico que represente la situación y resuelvan. a. Se quiere apoyar una escalera de 4,8 m de longitud contra una pared. Si para que no haya peligro de que se caiga, debe formar un ángulo de 35° con la pared, ¿a qué distancia se debe ubicar la base de la escalera? 2,75 m

b. Un poste de electricidad de 5 m de altura tiene que sujetarse con dos tensores desde su extremo superior hasta el piso. Si los tensores deben formar ángulos de 50° con el suelo, ¿cuántos metros se necesitan? 13,05 m

c. La sombra de una persona de 1,70 m de alto, generada por un foco de luz, es de 3,40 m. ¿Qué altura tiene el foco si se sabe que este se encuentra a 4 m de esa persona? 3,70 m

d. Un teodolito determina que desde un punto ubicado en el suelo hasta la cima de un monte hay 40°. Si ubica el teodolito a 300 m del punto anterior más cerca de la base del monte, dicho ángulo es de 65°. ¿Cuánto mide el monte? 413,54 m

mente ACTIVA ¿A qué altura se encuentra sentado un observador que está en la terraza de la casa? Altura de la casa: 16,09 m.

Casa

x ∧ tg 65° = __ x tg 50° = _____ y y+6 115°

50° 6m

187

47

46

48

49

50

51

52

53

54

55

Teoremas del seno y del coseno INFOACTIVA Los teoremas del seno y del coseno son dos de los teoremas más usados en trigonometría.

Teorema del seno En todo triángulo los lados son proporcionales a los senos de los ángulos opuestos. ___

c

__

__

ac bc ab ______ = ______ = ______ ^ sen ^ c sen ^ a sen b a

b

Este teorema se puede demostrar de la siguiente manera: ^ ___ H H __ __ sen ^ a = ___ ac ∧ sen b = bc __ __ ^ H = ac . sen ^ a ∧ H = bc . sen b

c

H

__ ^ ac . sen ^ a = bc . sen b __

a

__

__

ac bc ______ = ______ ^ sen ^ a sen b

b

__

__

sen b

sen c

ac ab Análogamente, se puede determinar que ______ = ______ . ^ ^ __

__

__

sen b

sen c

ac bc ab Por lo tanto, ______ = ______ = ______ . ^ ^ ^ sen a

Teorema del coseno El cuadrado de un lado de un triángulo es igual a la suma de los cuadrados de los otros dos lados menos el doble del producto de dichos lados por el coseno del ángulo que determinan. ___

__

c

___

__ __ 2 2 bc = ab + ac2 – 2 . ab . ac . cos ^ a __ ___ __ ___ __ ^ 2 2 2 ac = bc + ab – 2 . bc . ab . cos b ___ __ __ __ __ 2 2 ab = bc + ac2 – 2 . bc . ac . cos ^ c

a

b

El teorema de Pitágoras es un caso particular del teorema del coseno. __

___

__

___ __

c

bc = ab + ac – 2 . ab . ac . cos ^ a __

2 2

2

___

2

2

__

___ __

bc = ab + ac2 – 2 . ab . ac . cos 90° __

2

___

2

__

bc = ab + ac2

a 188

b

56

Test

de comprensión

1. Respondan y expliquen las respuestas. a. En los triángulos rectángulos, ¿se verifica el teorema del seno? b. ¿Es cierto que si se conocen los tres lados de un triángulo, se puede utilizar el teorema del coseno para averiguar cualquiera de sus ángulos interiores? a. Sí, es válido para todos los triángulos. b. Sí.

47

ACTIVIDADES Teoremas del seno y del coseno

33. Observen los triángulos y completen teniendo en cuenta el teorema del seno o del coseno. c

a.

b.

c

20°

17 cm 9,75 cm

45 cm

33,4 cm 120°

40° a

sen 20°

22 cm

17,77 cm

sen 40°

sen 120°

_____________ = _____________ = _____________

17,77

25°

a

b

33,4

45

2

9,75 =

2

17

b

2

+ 22

– 2 . 17

. 22

. cos 25°

34. Tengan en cuenta los datos y verifiquen si las medidas de los triángulos son correctas usando el teorema del seno o del coseno. ^ a. __ a = 46°; ^ c = 52°; __ bc = 7 m; ac = 11 m No.

^ b. ___ a = 20°; ^ c = 125°; __ ab = 15 m; ac = 12 m No.

^ c. ___ b = 49°; ^ c = 108°; __ ab = 30,43 m; bc = 12,5 m Sí.

___

^ d. __ a = 59°; ab = 11 m; __ bc = 16 m; ac = 9 m No.

___

^ e. __ c = 131°; ab = 24,57 m; __ bc = 13 m; ac = 14 m Sí.

___ ^ f. __ b = 27°; ab = 4,7 m; __ bc = 4,7 m; ac = 2,5 m

No.

189

48

47

49

50

51

52

53

54

55

56

57

Triángulos oblicuángulos INFOACTIVA Un triángulo es oblicuángulo cuando ninguno de sus ángulos interiores es recto, y resolverlo es hallar el valor de sus tres ángulos y sus tres lados. Para ello hay que aplicar los teoremas del seno, del coseno y la propiedad de la suma de sus ángulos interiores, que es igual a 180°. Se pueden presentar distintos casos. Dos lados y el ángulo comprendido

Un lado y dos ángulos

Los tres lados

Dos lados y el ángulo opuesto a uno de ellos

Para resolver un triángulo oblicuángulo dados sus tres lados, se pueden seguir estos pasos.

^ ^ 1. aplica__ el teorema del __ Se __ __ coseno para calcular a y b . __ 2 2 bc = ac2 + ab – 2 . ac . ab . cos ^ a __

__

c

__

bc2 – ac2 –__ ab2 cos ^ a = ____________ __

9 cm

–2 . ac . ab

(9 cm)2 – (6,2 cm)2 – (4,8 cm)2 cos ^ a = _________________________ ⇒^ a ≅ 109° 8’ 41,35’’ – 2 . 6,2 cm . 4,8 cm

6,2 cm

__ __ __ __ ^ ac = ab2 + bc2 – 2 . ab . bc . cos b __2

__

a

__

2 ^ __ ac2 – ab bc2 __ –__ cos b = ____________

–2 . ab . bc

^ (6,2 cm)2 – (4,8 cm)2 – (9 cm)2 ^ cos b = _________________________ ⇒ b ≅ 40° 36’ 3,82’’ –2 . 4,8 cm . 9 cm 2. Se aplica la suma de los ángulos interiores de un triángulo para calcular ^ c. ^ ^ a + b +^ c = 180° ^ ^ c = 180° – ^ a–b ^ c = 180° – 109° 8’ 41,35’’ – 40° 36’ 3,82’’ ⇒ ^ c ≅ 30° 15’ 14,83’’ 190

4,8 cm

b

Test

de comprensión

1. Respondan y expliquen las respuestas. a. Si se quiere resolver un triángulo oblicuángulo teniendo como datos dos ángulos y un lado, ¿se puede aplicar el teorema del seno para obtener cualquiera de los otros dos lados? b. ¿Es cierto que para aplicar el teorema del coseno alcanza con tener dos lados y un ángulo? a. Sí, pero dependiendo de qué lado se busque, se puede necesitar calcular el otro ángulo teniendo en cuenta la suma de los ángulos interiores de un triángulo. b. No, el ángulo debe ser el formado por esos dos lados. De lo contrario se debe utilizar el teorema del seno para conocer los otros ángulos.

48

ACTIVIDADES Triángulos oblicuángulos

35. Calculen lo pedido en cada caso. a.

d.

c

c

57°

22° 58 cm

100 cm

63°

45°

a

a

b

___

__ ^ ab = 54,59 cm ; ac = 56,37 cm ; b =

__

60°

c

b.

b

___ ^ bc = 76,82 cm ; ab = 40,70 cm ; b =

e.

113°

c

40° 11 cm

9 cm

40 cm

60 cm

76° a

b

a

___

^ ab = 7,09 cm ; ^ a = 54° 38’ 14’’ ; b = 85° 21’ 46’’

___ ^ b = 40° 18’ 20’’ ; ^ c = 63° 41’ 40’’ ; ab = 55,43 cm

c

c.

c

f. 15 cm

b

16 cm 27 cm

a

29 cm

b

^ ^ a = 21° 24’ 12’ ; b = 20° 23’’ ; ^ c = 138° 35’ 25’’

a

40 cm

38 cm

b

^ ^ a = 73° 47’ 7’’ ; b = 40° 24’ 8’’ ; ^ c = 65° 48’ 45’’ 191

48

ACTIVIDADES Triángulos oblicuángulos

36. Observen las figuras y calculen lo pedido en cada caso. ___ __

a. Calculen los lados del paralelogramo si ac = 80 cm y bd = 72 cm. ___

d

c

__

___

__

ad = bc = 62,30 cm; ab = cd = 43,71 cm.

70°

a

b

b. Calculen el perímetro del siguiente triángulo. c

Perímetro = 212,5 cm

27°

65°

a

b

90 cm

c. Calculen la diagonal del trapecio isósceles. d

Diagonal: 21 cm.

c 15 cm 63°

a

b

23 cm

d. Calculen la longitud de cada lado del siguiente pentágono regular. e

___

ab = 35,23 cm a

d

57 cm

b

c

37. Observen el gráfico y respondan. a. ¿Cuál es la distancia de cada barco hasta el faro? Barco A: 3 402,75 m; barco B: 3 248,99 m.

barco A 71° faro

1560 m 82° barco B

b. ¿Cuál es la distancia entre los dos árboles? árbol

15 m

Poste 63°

17 m 192

árbol

Distancia: 16,81 m.

48

ACTIVIDADES Triángulos oblicuángulos

38. Planteen y resuelvan las siguientes situaciones. a. Un árbol que se encuentra a 1 780 m de una montaña (A) y a 2 013 m de otra montaña (B), forma un ángulo entre ambas de 38°. Calculen la distancia entre ambas montañas. 1 254,38 m

b. Una persona ubicada en una plaza observa dos casas, una de ellas a 150 m y la otra a 120 m, formando un ángulo de 100°, aproximadamente. ¿A qué distancia se encuentran ambas casas? 207,73 m

c. La distancia entre Santa Rosa y Rosario es de 526 km; entre Rosario y Lobos, de 288 km y entre Lobos y Santa Rosa, 495 km. Si se toma cada ciudad como el punto de un triángulo, ¿qué ángulo se forma con Lobos como vértice? 79° 38’ 9,9’’

d. Dos carreteras rectas se cruzan formando un ángulo de 57°. Un edificio (A) está situado a 576 m de ese cruce y otro edificio (B), a 403 m del mismo cruce. Calculen la distancia entre ambos edificios. 491,26 m o 864,31 m (porque se puede pensar que está del otro lado de la ruta, con ángulo de 123°).

mente ACTIVA

c

Dado el triángulo __ abc oblicuángulo, __ calculen la altura ch, ac y ^ a.

4 cm

__

ch = 2,68 cm __

ac = 6,59 cm ^ a = 23° 56’ 38’’

a

h 9 cm

42° b

193

INTEGRACIÓN 39. Tengan en cuenta la siguiente figura y los datos dados en cada caso y resuelvan. p

m

n ___

___

a. mp = 12,3 cm; mn = 15,7 cm ___ n = 38° 4’ 36’’; Calculen ^ n, ^ p y np. ^

41. Lean atentamente y resuelvan. a. Un teodolito determina que desde un punto en el suelo hasta la cima de una montaña hay un ángulo de elevación de 30°. Si luego el teodolito se ubica 200 m más cerca de la montaña, el ángulo de elevación es de 45°. ¿Cuál es la altura de la montaña? 273,21 m. montaña

___ ^ p = 51° 55’ 24’’; np = 19,94 cm

___

___

b. mn = 8,7 cm; mp = 13,4 cm ___ ___ Calculen n^, ^ p y np. np = 15,98 cm; ^ ^ = 57° 22’’ p = ___32° 59’ 38’’; n___

c. np = 10,5 cm; mp = 4,9 cm ___ n = 27° 49’ 5’’; Calculen ^ n, ^ p y mn. ^ ___ ^ p = 62° 10’ 55’’; ___ nm = 9,29 ___ cm ^

d. p = 45° 50’; mp = 314 cm ___ ___ ^ Calculen mn, np___y n . ___

30°

45°

200 m

b. Calculen la altura de un edificio, sabiendo que proyecta una sombra de 230 m cuando los rayos del sol forman un ángulo de 13° 15’ con el horizonte. 54,16 m sol

mn = 3,85 cm, np = 5,37 cm, ^ n = 44° 10’ __

___

e. ^ n = 39° 20’’; np = 2 . 33 cm ___ ___ ^ Calculen mp, mn y p. ___ ___

edificio

mp = 2,18 cm, nm =___ 2,69 cm, ^ p = 50° 59’ 40’’ ___

f. ^ p = 62°; mn = 3 15 cm ___ ___ Calculen ^ n , mp y np.

___ ___ ^ n = 28°, np = 4,39 cm, mp = 2,06 cm

40. Indiquen el valor de los ángulos solicitados en cada caso. a.

b.

42. Hallen el perímetro y el área de los triángulos abc y abd. a d

c

7 cm

Perímetro trián. abc = 45,89 cm, área trián. abc = 88,96 cm2. Perímetro trián. abd = 28,88 cm, área trián. abd = 35,22 cm2.

12 cm

4 cm

_

a

13° 15’

^ _= 34° 51’

b

51° b

c `

^ `= 48° 21’ 59’’

8 cm

c

43. Escriban V (Verdadero) o F (Falso). a. En todo triángulo los lados son proporcionales

a

b

9 cm c

c. 5 cm

b. El teorema de Pitágoras es un caso particular 4 cm a

a

^ a= 63° 12’ 42’’

con tener los tres ángulos como datos.

c 15 cm

del teorema del seno. F c. Para aplicar el teorema del coseno alcanza

b

3,5 cm

d.

a los cosenos de los ángulos opuestos. F

^ ¡ = 29° 55’ 35’’

F

d. Cuando se aplica el teorema del coseno alcanza con tener tres lados como datos. V

a

194

¡

13 cm

b

capítulo

CONTENIDOS

8

46*47*48 44. Verifiquen los datos de los siguientes triángulos. Redondeen a los centésimos. a.

a. Se quiere calcular la distancia entre dos árboles que están separados por un lago. Se sabe que un poste está a 13 m de un árbol y a 16 m del otro y que el ángulo comprendido es de 65°. ¿Cuál es la distancia entre los árboles? b. Un mojón marca el origen de dos bicisendas que se abren con un ángulo de 53° 20’. Dos ciclistas parten del mojón por caminos diferentes y recorren 103 m y 135 m, respectivamente. ¿Qué distancia los separa?

c

No se verifica. 10,62 m 4,2 m 110° a

b

3,5 m

b.

c

Se verifica. 14,81 m

48. Lean atentamente y resuelvan.

12,1 m

a. 15,79 m. b. 110,58 m.

49. Calculen el área y el perímetro de las 42°

55°

a

siguientes figuras. a. d 9,85 cm

b

17,95 m

c

45. Resuelvan los triángulos aplicando el teorema del seno cuando sea posible. c a.

70° a

^ c = 58° ___ bc = 9,29 cm __ ac = 3,53 cm

b d

b. 100° 22°

a

50°

b

8 cm

b.

a 20 cm

c

c

Área: 102,20 cm2 Perímetro: 46,38 cm

^ b = 44° 52’ 32’’ ^ c = 38° 7’ 28’’ __ ac = 11,25 cm a

Área: 109,3 cm2 Perímetro: 43,84 cm

9 cm

18°

7 cm 97° 8 cm

b

46. Calculen lo pedido en cada caso.

b

c.

e

a. El área de un triángulo equilátero cuyo lado es de 12 cm. b. El perímetro de un rectángulo cuya diagonal mide 24 cm y el ángulo formado por esta y uno de sus lados es de 36°.

f

a

c 10 cm

b

a. 62,35 cm2. b. 67,04 cm.

47. Tengan en cuenta los datos y calculen los

d.

lados y ángulos restantes. __ a. ^r = 29°;__^t = 47°; st =__ 12,9 cm ^t = 43°, rs = 16,4 cm; st = 5 cm __ __ __

c

d 12 cm 38°

^ s = 104°, rs = 19,46 cm; rt = 25,82 cm

__ b. ^r = 12°;__^ s = 125°; rt__= 19,7 cm

17 cm

Área: 183 cm2 Perímetro: 73,54 cm

25° 45’ 32”

a

e.

Área: 259,8 cm2 Perímetro: 60 cm

d

b

d

c

c. rs = 33 cm; st = 27 cm; rt = 39 cm

^r = 43° 2’ 57’’,__^t = 56° 32’ __ 42’’; ^ s = 80° 24’ 21’’ __

d. rs = 1,5 cm; st = 1,6 cm; rt = 2,9 cm

^t = 20° 23’’; ^ s = 138° 35’ 25’’; ^r = 21° 24’ 12’’

__ __ e. ^ s = __ 100°; rs = 6 cm; st = 9 cm

rt = 11,65 cm, ^t = 30° 28’ 38’’; ^r = 49° 31’ 22’’

Área: 98,05 cm2 Perímetro: 42,52 cm

8 cm 130° a

b

195

capítulo

8

AUTOEVALUACIÓN Marquen las opciones correctas 50. ¿Cuáles son las expresiones equivalentes a las dadas? a. / X 180°

360°

270°

90°

b. cos 150° X –cos 30°

cos 30°

X cos 210°

cos 60°

51. ¿Cuáles son las igualdades correctas? X a. sen ^ _ = –sen (2/ – ^ _)

X c. sen ^ _= sen (2/+ ^ _)

X b. cos ^ _= cos (2/ – ^ _)

X d. cos ^ _= cos (2/+ ^ _ 

52. ¿En qué intervalo tiene exactamente tres soluciones la siguiente ecuación? 2 . sen2 ^ _+ 2 . sen ^ _= 0 3 X b. 0;__ 2/

[

a. [0;/]

]

X d. [–/;/]

c. [0;2/]

53. ¿Cuáles son las soluciones de la siguiente ecuación en el intervalo [0;/]? 2 . cos 3^ _ = 1. / X b. __ 9

5 X a. __ 9 /

c. – __95 /

d. __92 /

54. Lean atentamente y resuelvan. Una persona observa la terraza de un edificio con un ángulo de elevación de 55° y la de otro edificio con un ángulo de elevación de 72°. Se encuentra a 80 m del primer edificio y a 60 m del segundo. ¿Qué altura tiene cada edificio, sabiendo que la persona observa desde una altura de 1,70 m?

edificio I

55°

80 m

72°

edificio II

a. 114,25 m y 184,66 m

X c. 115,95 m y 186,36 m

b. 114,25 m y 186,36 m

d. 115,95 m y 184,66 m

60 m

55. ¿Qué fórmula conviene utilizar para resolver el siguiente triángulo oblicuángulo? c 20° 10 cm

a

196

8 cm

b

___

__

ab ac a. ______ = ______ ^ ^ sen c

sen b

___

__

ab bc X b. ______ = ______ sen ^ c

sen ^ a

___ __ ___ __ __ ^ 2 2 c. ac 2 = ab + bc – 2 . ab . bc . cos b ___

__

__

__ __ 2 2 d. ab = ac 2 + bc – 2 . ac . bc . cos ^ c

Contenidos

9

Estadística. Intervalos de clase. Parámetros de posición. Parámetros de dispersión. Combinatoria. Permutaciones, variaciones y combinaciones. 55. Probabilidad. 56. Sucesos y probabilidad condicional. 49. 50. 51. 52. 53. 54.

Los censos tienen un origen muy antiguo: tanto en Egipto, unos 3 000 años antes de Cristo, como en el libro Números en la Biblia, se encuentran referencias a diversos conteos de población. Luego, los griegos y los romanos perfeccionaron estas prácticas, tanto con fines militares como tributarios. Esta es la idea inicial de la estadística, palabra que refiere sin duda a la noción de Estado; sin embargo, su estudio como rama de la matemática es mucho más reciente. El verdadero origen de los métodos estadísticos hay que rastrearlo, más que en los aburridos conteos de habitantes, en recursos a veces muy ingeniosos para hacer inferencias. Por ejemplo, ¿cómo hacer para calcular la altura de una muralla contando unas pocas filas de ladrillos? Pero para los historiadores la estadística no nació contando personas ni ladrillos, sino en un trabajo escrito en el siglo IX por un filósofo llamado Al-Kindi, en donde el análisis de frecuencias se emplea con fines más bien detectivescos: el desciframiento de mensajes secretos.

1. Lean atentamente y respondan. a. ¿Por qué son importantes los métodos estadísticos? b. Los métodos estadísticos ¿son siempre confiables? ¿Por qué? a. Por ejemplo, la estadística se usa en muchísimos ámbitos, desde las ciencias experimentales hasta las sociales. b. Los métodos estadísticos permiten inferir a partir de muestras de poblaciones. Si bien los cálculos matemáticos son precisos y exactos, esto no quiere decir, por ejemplo, que las predicciones hechas por la estadística sean certeras. La propia estadística proporciona medidas para estimar el error cometido al efectuar cierta inferencia.

capítulo

Estadística y probabilidad

49

48

50

51

52

53

54

55

56

Estadística ¿Para qué sirve?

INFOACTIVA

PÁGINA 14

Un gráfico estadístico permite hacer una rápida lectura de los datos recolectados de una muestra. La utilización de los distintos gráficos dependerá del tipo de variable utilizada y de la información que se quiera brindar. Si la variable es cualitativa, los gráficos más convenientes son el circular o el de barras.

Gráfico circular Los gráficos circulares sirven para mostrar y comparar los porcentajes, ya que muestran la cantidad de datos que pertenecen a una misma categoría como una parte proporcional de un círculo. En cada sector circular se representa una variable y se la puede identificar con su porcentaje correspondiente. El ángulo central correspondiente a cada sector circular se calcula de la siguiente manera: absoluta . 360° _______________________ Ángulo central = Frecuencia Total de observaciones

La siguiente tabla muestra el resultado de una encuesta realizada a 120 personas acerca de su destino elegido para las vacaciones. Destino elegido

x: destino elegido

13,3%

f

Porcentaje

Noroeste argentino

16

13,3%

Noroeste argentino

Nordeste argentino

35

29,2%

Nordeste argentino

Centro-oeste argentino

18

15%

Sur argentino

27

22,5%

Extranjero

24

20%

20%

Centro-oeste argentino

29,2%

22,5%

Sur argentino Extranjero 15% Destino elegido

Los gráficos de barras muestran la frecuencia correspondiente a cada categoría de la variable, y permiten compararlas entre sí. Se puede utilizar tanto para las variables cualitativas como para las cuantitativas discretas.

Frecuencia absoluta

Gráfico de barras

40 35 30 25 20 15 10

35 27

24

18

16

5 0 Noroeste argentino

Nordeste Centro-oeste Sur argentino argentino argentino

Extranjero

Esquema tallo-hoja El esquema tallo-hoja permite resumir datos numéricos, de dos o más dígitos, cuando la muestra es grande. Generalmente es utilizado con el fin de ordenarlos y poder estudiar la característica de la distribución. Para construir el esquema, se toma cada dato y se lo divide en dos partes: el tallo, que contiene todos los dígitos menos el último y las hojas, que es el último dígito.

198

59 ¨ Tallo © 60 ª 61

1124588 34588 122

¨ « © « ª

Si se tienen los pesos (en kg) de 20 boxeadores de la categoría ligero, el esquema es el siguiente. 59,8 59,4 59,1 61,2 59,8 59,5 60,4 61,1 59,2 60,8 60,3 59,1 61,2 60,8 60,5

Hojas

Test

de comprensión

1. Respondan y expliquen las respuestas. a. Si en un sector de un gráfico circular se debe representar una variable de frecuencia absoluta 16 y el total de las observaciones es 36, ¿es cierto que su ángulo central debe ser de 16°? b. ¿Qué tipo de gráficos conviene utilizar para comparar las frecuencias absolutas de las variables? a. No, el ángulo debe ser de 160°. b. Los gráficos de barras.

49

ACTIVIDADES Estadística

1. La siguiente tabla muestra la distribución de alumnos de una institución por niveles de enseñanza. Determinen en cada caso el ángulo central y construyan un gráfico circular. Nivel de estudios Frecuencia absoluta Porcentaje Inicial

440 alumnos

20%

Primario

550 alumnos

25%

Secundario

1 210 alumnos

55%

^ _ =  90° Nivel primario ^ `= 72° Nivel inicial

Nivel inicial: 72°.

^a= 198° Nivel secundario

Nivel primario: 90°. Nivel secundario: 198°.

2. Observen el gráfico de barras que muestra los deportes que practican los alumnos de 5.° año y

10

5 3

un o

s Ni

ng

Te

ni

le y Vo

ll nd Ha

c. Completen el gráfico.

ba

l

0



Handball: 14 alumnos, voley: 11 alumnos, tenis: 3 alumnos.

14 11

Fú tb o

b. ¿Cuántos alumnos eligieron handball, voley y tenis, si representan el 20,59%, 16,18% y 4,41%, respectivamente?

15

ue t

Básquet: 22,06%, fútbol: 29,41%, ninguno: 7,35%.

20

sq

resuelvan. a. ¿Qué porcentaje representa básquet, fútbol y ninguno, si en total hay 68 alumnos?

3. El siguiente esquema tallo-hoja muestra las edades de cierta cantidad de personas. a. ¿Cuántas personas fueron encuestadas? 17 personas.

Tallo

Hojas

b. ¿Qué porcentaje de personas supera los 17 años?

1 0, 0, 2, 4, 7, 9

70,59%

2 1, 4, 4, 5

c. ¿Qué porcentaje está entre 18 y 30 años?

3 4, 4 4 1, 5

29,41%

d. Si se realiza un gráfico circular, ¿qué ángulo central debe tener el sector circular que representa a las personas entre 30 y 50 años?

5 1, 9 7 8

84° 42' 21,18'' 199

50

49

51

52

53

54

55

56

Intervalos de clase INFOACTIVA Cuando se cuenta con un gran número de datos cuantitativos, es conveniente organizarlos de manera tal que resulte más rápida su lectura. Para ello se puede recurrir a una distribución de frecuencias agrupadas en intervalos de clase con las siguientes características: ˆ Todas las clases deben tener la misma amplitud. ˆ Cada dato debe pertenecer exclusivamente a una clase. ˆ En lo posible no deben quedar clases vacías. Para poder armar los intervalos de clase, se deben identificar el menor y el mayor dato de la distribución, y establecer el número de intervalos que se desea. Dato mayor – Dato menor Amplitud de cada intervalo = _______________________ Número de intervalos

Los datos correspondientes al recuento de plaquetas de 100 pacientes son: 160; 161; 165;…; ml y se los quiere distribuir en 6 intervalos de clase. 400 ____ mm3 400 – 160 Amplitud de cada intervalo = __________ 6

Amplitud de cada intervalo = 40 Habrá 6 intervalos de amplitud 40 Límite inferior [ a Dato incluido

Límite superior ; b ) Dato no incluido

Se debe elegir un representante de cada intervalo. En general, el punto medio del intervalo llamado marca de clase (xn). límite inferior + límite superior xn = ___________________________ 2 f

xn

F

[160;200)

13

180

13

[200;240)

25

220

38

[240;280)

22

260

60

[280;320)

14

300

74

[320;360)

10

340

84

[360;400)

16

380

100

Un histograma es un gráfico de barras que se utiliza para representar intervalos de clase. Un histograma cuenta con: ˆ Una escala horizontal, en la cual se indican el límite inferior y el superior de cada intervalo de clase. ˆ Una escala vertical, en la cual se indican las frecuencias de las distintas clases.

25

Frecuencia absoluta

Recuento de plaquetas (mil/mm3)

22

16 14 13 10

0 160

200

240

280

320

Plaquetas (mil/mm3)

200

360

400

Test

de comprensión

1. Respondan y expliquen las respuestas. a. Se tienen los datos de las estaturas de 50 personas, donde el más bajo mide 168 cm y el más alto, 198 cm. ¿Es conveniente distribuirlos en 15 intervalos? ¿Cuál sería el rango de cada intervalo? b. ¿Es posible representar en un histograma la relación entre razas de perros y su frecuencia?

a. Puede ser, pero dependerá de lo que se quiera analizar. La amplitud sería de 2 cm. b. No, el histograma se utiliza para datos cuantitativos, no sirve para datos cualitativos.

50

ACTIVIDADES Intervalos de clase

4. Lean atentamente y resuelvan. En una empresa se analizaron los ingresos (en pesos) de una semana y se obtuvieron los siguientes datos. 3 145 15 879 6 914 4 572 11 374 12 764 9 061 8 245 10 563 8 164 6 395 8 758 10 755 7 415 9 361 11 606 3 517 7 645 9 537 8 020 12 848 8 438 6 347 5 151 5 253 5 656 21 333 9 280 7 538 7 414 11 707 9 144 7 424 25 639 10 274 4 683 5 045 5 768 5 089 6 904 9 182 12 193 12 472 8 494 6 032 16 012 9 282 3 331 5 889 5 345 a. Realicen una tabla de intervalos de clase que esté comprendida en [0;30 000). Solución gráfica. Solución gráfica. b. Representen los datos en un histograma. c. ¿En qué intervalo se encuentran las ganancias menos frecuentes? ¿Cuál es su porcentaje? En [20 000;25 000) y [25 000;30 000). 2% cada una.

5. Resuelvan. a. Completen la tabla que muestra el peso de los atletas de una competición.

c. Realicen un histograma.

Peso (en kg) Cantidad de atletas Marca de clase [46;52)

30

49

[52;58)

90

55

[58;64)

130

61

[64;70)

100

67

[70;76)

40

73

[76;82)

50

79

[82;88)

10

85

b. ¿Qué porcentaje de atletas pesa entre 58 y 76 kg? 60%. Solución gráfica.

6. Lean atentamente y resuelvan. a. Si las marcas de clase de un histograma son 10 000, 11 000, 12 000, 13 000, 14 000 y 15 000, respectivamente, ¿cuáles son los intervalos de clase? [9 500;10 500), [10 500;11 500), [11 500;12 500), [12 500;13 500), [13 500;14 500), [14 500;15 500).

b. Inventen una situación en la que se puedan utilizar los intervalos del ítem anterior y realicen en una hoja un histograma que la represente. Por ejemplo: los salarios de los empleados de la sección comercial de una empresa son los siguientes: 9 800; 10 200; 11 500; 11 000; 11 300; 13 200; 14 400; 15 400; 14 400; 15 100; 15 050; 13 400. 201

51

50

52

53

54

55

56

Parámetros de posición INFOACTIVA Se llama media aritmética (o promedio) al cociente entre la suma de los productos resultantes de multiplicar cada valor de la variable por su correspondiente frecuencia absoluta, y el total de observaciones.

Y x1 . f1 + x2 . f2 + ... + xn . fn Y xn . fn x = _______________________ = _______ n n _

La tabla muestra las notas obtenidas por 50 alumnos en un examen de ingreso.



 x: notas

f

x.f

F

1

2

2

2

2

3

6

5

3

1

3

6

4

7

28

13

5

7

35

20

6

10

60

30

7

6

42

36

8

7

56

43

9

5

45

48

10

2

20

50

Y x . f = 297

n = 50 El promedio de las notas del

Yx

_

.f

n n 297 ____ examen es: x = ______ n = 50 = 5,94

La mediana es el valor de la variable que ocupa la posición central o el promedio de los dos datos centrales. Si se divide la población en dos partes ( __n2 = 25 ), la mediana es el valor de la variable que contiene dicha frecuencia acumulada F. (me = 6) La moda es el valor de la variable de mayor frecuencia absoluta. (mo = 6)

Parámetros de posición para datos agrupados en intervalos Se aplican las fórmulas utilizando la marca de clase: x

f

xn

xn . f

F

1

12

55

660

12

2

8

65

520

20

3

15

75

1125

35

4

27

85

2295

62

5

18

95

1710

80

Y xn . fn = 6 310

ˆ Cálculo de la media aritmética. _

Yx

.f

_

n n 6 310 ______ x = ______ n ⇒ x = 80 ⇒ 78,875

n __ –B _____ ˆ Cálculo de la mediana: me = A + 2 C . D n n: población. Se debe localizar a __ 2 en la frecuencia acumulada y ver a qué clase pertenece. Dicha clase es la clase principal. A: límite inferior de la clase principal. B: frecuencia acumulada de la clase que antecede a la principal. C: frecuencia absoluta de la clase principal. D: amplitud de la clase principal.

40 – 35 . 10 = 81,85 me = 80 + _______ 27

ˆ Intervalo modal: intervalo de clase con mayor frecuencia.

Intervalo modal: [80;90) 202

n = 80

Gráfico de frecuencias acumuladas

80 70 62

60 50 40

35

30 n 20 12 2

10 0

50

60

70

80 90 100 me = 81,85

de comprensión

Test

1. Respondan y expliquen las respuestas. a. ¿Es cierto que se obtiene la misma mediana tomando los siguientes datos sueltos que agrupándolos en intervalos de amplitud 2? 1, 2, 2, 3, 3, 3, 4, 5, 5, 6, 7, 8, 9, 9, 9 b. ¿Qué diferencia hay entre la moda y la clase modal?

a. Empezando por [0;2), sí, da 5. Si se toman otros intervalos u otra amplitud, la mediana puede variar. b. La moda es el valor de mayor frecuencia absoluta y la clase modal es el intervalo que contiene a la moda.

51

ACTIVIDADES Parámetros de posición

7. Lean atentamente y resuelvan. Un profesor de economía da clases en dos divisiones del último año de la escuela secundaria y quiere analizar el rendimiento de sus alumnos. Notas de 5.° año A: 5, 4, 5, 5, 8, 6, 5, 7, 7, 7, 3, 8, 10, 10, 10, 2, 1, 5, 7, 8, 5, 6, 9, 4, 6, 4. Notas de 5.° año B: 2, 2, 2, 5, 6, 7, 8, 8, 8, 8, 4, 3, 8, 10, 10, 10, 8, 8, 8, 6, 6, 10, 10, 4, 6, 7. ¿Cuál curso tuvo menor rendimiento promedio? Promedio de 5.° año A: 6. Promedio de 5.° año B: 6,69. Tuvo menor rendimiento 5.° año A.

8. Resuelvan. Los resultados al lanzar un dado 300 veces están representados en la siguiente tabla. Resultados

1

2

3

4

5

6

f

55

56

60

f4

40

f4 + 9

Calculen la frecuencia de 4 y de 6 teniendo en cuenta que la media aritmética es 3,336. f4 = 40; f6 = 49

9. Calculen la mediana y la moda teniendo en cuenta los datos disponibles en cada caso. a. 12 - 15 - 21 - 11 - 7 - 10 - 14 - 8 - 15 me = 12 y m0 = 15

b. 23,5 - 12,8 - 21,2 - 10,8 - 14,7 - 18,6 - 21,5 - 12,1 - 23,5 - 13,4 - 16,2 - 23,5 me = 17,4 y m0 = 23,5

10. Lean atentamente y resuelvan. Las alturas de 42 árboles de una determinada especie figuran en la siguiente tabla: Altura (cm)

[5;15)

[15;25)

f

9

7

[25;35)

[35;45)

[45;55)

15

8

3

_

a. ¿Cuál es la altura media de los árboles? x = 27,38 cm

b. Calculen la moda y el intervalo modal. mo = 30 cm y el intervalo modal: [25;35). c. ¿Qué altura marca la mediana de la muestra? me = 28,33 cm _

d. Ordenen de menor a mayor la media aritmética, la moda y la mediana. x < me < mo 203

51

ACTIVIDADES Parámetros de posición

11. Tengan en cuenta el siguiente gráfico y resuelvan. El siguiente gráfico de frecuencias absolutas muestra el peso en kg de 65 personas adultas. 16

a. Construyan en una hoja una tabla donde indiquen: intervalos, marcas de clase, frecuencia absoluta y frecuencia acumulada. Solución a cargo del alumno. b. Calculen la media, mediana y moda.

14 12 10

_

8

x = 79,76 kg, me = 79,06 kg, mo = 75

6 4

c. Construyan en una hoja un gráfico de frecuencias acumuladas. Solución a cargo del alumno.

2 0 50

60 70 80 90 100 110 120

d. ¿Cuántas personas superan en peso el valor de la moda? ¿Qué porcentaje representa? 31 personas y representa el 47,69% de la población tomada.

e. ¿Qué porcentaje deberían bajar las personas que pesan 115 kilos, aproximadamente, para acercarse al valor promedio? Deberían bajar aproximadamente el 30% de su peso actual.

12. Tengan en cuenta la tabla donde se representa la cantidad de palabras por frase de una página de una novela contemporánea y resuelvan. Cantidad de palabras

[1;4)

[4;7)

[7;10)

[10;13)

Cantidad de frases

4

37

59

27

a. ¿Cuál es la media de la cantidad de palabras por frase? _

x = 8,0748

b. ¿Cuál es la clase modal? ¿Qué número representa la moda? La clase modal es el intervalo [7;10) y la moda es 8,5.

c. ¿Qué porcentaje de la muestra representa el intervalo modal? 46,45% de la muestra.

13. Marquen las opciones correctas. Si los siguientes datos expresan la ganancia mensual de 30 comerciantes (en miles de pesos) de un importante shopping, ¿cuál es la ganancia mensual promedio? 72 175 150 

204

80 150 150 129,3

80 95 150

80 95 175 X 129,03

80 95 255

72 95 255

72 95

129,33

115 95

175 150

115 150

150 150

150 150

Ninguna de las anteriores.

51

ACTIVIDADES Parámetros de posición Tallo

14. Marquen las opciones correctas. El diagrama de tallo y hojas muestra las edades de 20 personas que están en una sesión de reiki. ¿Qué edad representa la mediana? X 33,5

34,5

Hojas

2

0344459

3

113466799

4

0015

35,5

Ninguna de las anteriores.

15. Tengan en cuenta la tabla que representa la venta de calzados deportivos para mujer en una casa de deportes y resuelvan. Talle del calzado

34

35

36

37

38

39

40

Cantidad vendida

400

550

1 670

1 500

685

135

60

a. Calculen la media aritmética, la moda y la mediana. _

x = 36,433; mo = 36; me = 36.

b. Si se quiere saber cuál fue el calzado más vendido, ¿qué parámetro es más representativo? La moda, porque en este caso interesa el valor de mayor frecuencia. La mediana casualmente es 36, pero no es representativa de lo que se está buscando porque tiene que ver con la distribución de la muestra.

16. Tengan en cuenta la fórmula y resuelvan. _

30,5 . 5 + 31,5 . 7 + 32,5 . 8 + 33,5 . 12 + 34,5 . 25 + 35,5 . 31 + 36,5 . 22 + 37,5 . 12 + 38,5 . 8 + 39,5 . 4 x = _____________________________________________________________________________________ 134

a. ¿Cuál es la media aritmética? _

x = 35,20

b. Realicen en una hoja una tabla indicando los intervalos, las marcas de clases, las frecuencias absolutas y las frecuencias acumuladas. Solución a cargo del alumno. c. Calculen la moda y la mediana. mo = 35,5 y me = 35,32

d. Realicen en una hoja un histograma y el gráfico de frecuencias acumuladas en la misma representación. Solución a cargo del alumno.

mente ACTIVA Se registró la estatura de los alumnos de un colegio y se obtuvieron los siguientes datos. ˆLa mediana es 169,89 cm y está en el intervalo [169,5;174,5). ˆ58 alumnos están en el intervalo [169,5;174,5) y 33 están en el anterior. n – 33 __ ¿A cuántos alumnos se encuestó? 2 ______ 169,89 = 169,5 +

58

.5

n = 75 205

52

51

53

54

55

56

Parámetros de dispersión INFOACTIVA Desviación estándar La desviación estándar mide la dispersión de los datos con respecto al promedio.

Dos alumnos A y B rindieron la misma cantidad de evaluaciones y obtuvieron distintas calificaciones. Si se busca el promedio de las calificaciones, ambos tienen el mismo promedio 7, pero B tiene un rendimiento más estable que A.

_

Notas de A

(xi – x)2

(xi – x)2

4

(4 – 7)2 = 9

8

(8 – 7)2 = 1

10

(10 – 7)2 = 9

7

(7 – 7)2 = 0

4

(4 – 7)2 = 9

7

(7 – 7)2 = 0

10

(10 – 7)2 = 9

6

(6 – 7)2 = 1 __________

_



Se define como varianza: m = ___

Y ( xi – x )2 . fi ___________ n

_

Notas de B

La desviación estándar es: m =

3

_

Y ( xi – x )2 . fi ___________ n

__

36 mA = 3___ 4 =3

2 ≅ 0,71 mB = 3__ 4

El valor de este parámetro es mayor cuando los datos están muy disgregados o dispersos y es menor cuando están más concentrados. Como mA > mB, entonces A es más disperso que B.

Coeficiente de variación El coeficiente de variación expresa la desviación estándar como un porcentaje de la media aritmética. ___________ _ 2

Y ( xi – x ) . fi ___________ m __ _____________ _n Coeficiente de variación: Cv = _x  . 100 ⇒ Cv = . 100 x

3

Para los alumnos del ejemplo anterior, el coeficiente de variación es: 0,71 3 . 100 ⇒ Cv ≅ 43% Cv = _____ CvA = __ . 100 ⇒ CvB ≅ 10% A B 7 7

Cuando el coeficiente de variación es inferior al 30%, la distribución es bastante homogénea. Se utiliza para comparar la homogeneidad de dos series de datos, aun cuando estén expresadas en distintas unidades. A medida que el coeficiente de variación disminuye, se observa una mayor homogeneidad en los datos, o lo que es lo mismo, los datos están más concentrados alrededor del promedio. _

_

216 ⇒ x ≅ 6,35 x = ____ 34 _____

99,78 m = ______ ⇒ m ≅ 1,71 34

3

1,71 Cv = _____ . 100 ⇒ Cv ≅ 27% 6,35

La distribución es bastante homogénea.

206

_

x: nota

f

x.f

(xi – x)2 . fi

3

2

6

22,45

4

3

12

16,57

5

6

30

10,94

6

8

48

0,98

7

4

28

1,69

8

7

56

19,06

9

4

36

28,09

Test

de comprensión

1. Respondan y expliquen las respuestas. a. Si una muestra A tiene mayor media que otra B, ¿la muestra A tiene los datos más dispersos? b. Si dos muestras tienen la misma desviación estándar, ¿sus datos están dispersos de igual forma? a. No, se debe calcular la desviación estándar. b. No, porque se debe tener en cuenta también el valor de la media aritmética.

52

ACTIVIDADES Parámetros de dispersión

17. Tengan en cuenta los datos de cada muestra y resuelvan. A: 8, 8, 9, 9, 9, 9, 10, 9, 10 B: 1, 3, 9, 6, 9, 11, 14, 13, 15 C: 1, 15, 11, 8, 8, 11, 9, 9, 9 a. Realicen en una hoja una tabla de frecuencias absolutas y acumuladas para cada caso. b. Calculen la media aritmética, la mediana y la moda de cada muestra. ¿Qué tienen en común? _

Los tres conjuntos tienen x = me = mo = 9.

c. ¿Qué muestra se desvía menos con respecto a la media? mA = 0,67; mB = 4,5946; mC = 3,4960. El conjunto que se desvía menos con respecto a la media es A.

18. Tengan en cuenta la tabla del sueldo mensual de los empleados de una empresa y resuelvan. Sueldo (en $)

5 000

5 600

6 000

6 700

7 000

8 200

10 000

Empleados

5

4

6

12

10

5

3

¿Cuánto se desvían los sueldos con respecto a la media? El sueldo promedio es de $6 773,3 y los sueldos se desvían de la media $1 218,45.

19. Lean atentamente y resuelvan. En un colegio, los alumnos deben elegir con qué profesor cursar biología. Registraron las calificaciones de los profesores y los organizaron en la siguiente tabla. _

x

m

Profesor Pistilo

6,5

2,3

Profesora Corola

6,5

0,6

a. ¿Qué profesor debe elegir un alumno que aspira a tener la nota máxima? Con el profesor Pistilo, ya que las notas de los alumnos son más variadas y, si bien puede haber más aplazos, suele haber notas más altas también.

b. ¿Y un alumno que se conforma con aprobar con 6? Elegiría a la profesora Corola, pues sus notas están cercanas al promedio del curso, 6,5. Seguramente no hay notas tan altas, pero tampoco muchos aplazos. 207

52

ACTIVIDADES Parámetros de dispersión

20. Marquen las opciones correctas. Expliquen las respuestas. Se realizó un estudio de las estaturas (en cm) de los jugadores de tres equipos de un colegio. Balvanera

Pachanga

Los Estón

x

173,2

176

170,1

m

3,7

6,2

4,1

_

a. ¿Qué equipo tendrá al jugador más alto? X Pachanga

Balvanera

Los Estón

Pachanga tiene el promedio más alto y la desviación mayor, así que es casi seguro que tendrá el jugador más alto.

b. ¿Cuál es el equipo con la distribución más homogénea? X Balvanera

Pachanga

Los Estón

Balvanera tiene la distribución más homogénea porque tiene el coeficiente de variación más bajo. Cv Balvanera = 2,13%, Cv Pachanga = 3,52% y Cv Los Estón = 2,41%.

21. Observen la tabla donde se registran los resultados de arrojar dos dados y sumarlos. Luego, resuelvan. Números

2

3

4

5

6

7

8

9

10

11

12

Veces

3

8

9

11

20

19

16

13

11

6

4

a. Calculen la media aritmética, la varianza y la desviación estándar. _

x = 7,025, m2 = 5,9244, m = 2,4340

b. Obtengan el coeficiente de variación y decidan si la muestra es homogénea. Cv = 34,65%. No es homogénea porque supera el 30%.

22. Marquen las opciones correctas. Los resultados de un examen de inglés están agrupados en la siguiente tabla. Intervalo Frecuencia

[80;89) 4

[89;98)

[98;107)

10

21

[107;116) [116;125) [125;134) [134;143) 23

9

5

3

a. ¿Cuál es la media aritmética y la desviación estándar? _

x = 108 y m= 17,19

_

x = 108 y m= 19,19

X Ninguna de las anteriores. _ x = 108,5 y m = 12,5

b. ¿Cuál es el coeficiente de variación? Cv = 15,91% 208

X Cv = 11,52%

Ninguna de las anteriores.

52

ACTIVIDADES Parámetros de dispersión

23. Lean atentamente y resuelvan. El siguiente gráfico representa la distribución de diferentes muestras. Todas estas curvas tienen la misma media aritmética (18). La desviación estándar determina el grado de apuntamiento de la curva. Cuanto mayor sea el valor de esta, más se dispersan los datos en torno a la media, y la curva es más plana. Un valor pequeño de este parámetro indica, por tanto, una gran probabilidad de obtener datos cercanos al valor medio de la distribución.

18

Completen con el color de la curva correspondiente a cada desviación estándar. a. m = 3,01 b. m= 3,0112 c. m= 3,012 d. m= 3,001 

Naranja

Amarilla

Verde

Azul

24. Resuelvan. Un entrenador de básquet realizó un histograma de marcas de clase con las estaturas (en cm) de 96 varones pensando en formar equipos homogéneos. f

30 25

a. Calculen la media aritmética y la desviación estándar.

27

_

x = 170,83 cm; m = 7,42 cm.

20

22

15

16

b. La muestra ¿es homogénea?

13

10

La muestra es homogénea, ya que el Cv = 4,34 % < 30%

8

5

6 4

0 157,5 162,5 167,5 172,5 177,5 182,5

187,5

Altura (en cm)

mente ACTIVA Las siguientes curvas responden al modelo de distribución de Gauss llamado “campana de Gauss”. La fórmula genérica de esta distribución es 1 x–x __ _____ 1___ f(x) = _______ . e– 2 . ( m ) m . 32/ Determinen las fórmulas de cada curva. _

_

x = 0; m2 = 1 _ x = –2; m2 = 0,5

0,6 0,5 0,4

2

1 . e– __21 x ; ___ Verde: f(x) = ____

0,3 0,2 0,1

2

32/

0

–5 –4

–3

–2

–1

0

1

2

3

4

2

+2 ___ 1__ . e– __21 . ( x_____ 30,5 ) violeta: f(x) = ___ 3/

209

INTEGRACIÓN 25. Observen la tabla donde se registran las

27. Marquen las opciones correctas.

camisas vendidas en un negocio y resuelvan.

¿Cuáles son los parámetros de posición asociados a los siguientes datos? 1 - 1 - 1 - 3 - 4 - 5.

Talle de la camisa

Cantidad vendida

37

90

39

38

42

105

S

110

L

63

XL

44

a. Realicen una tabla con los porcentajes de cada talle y la frecuencia acumulada. b. ¿Qué porcentaje de camisas se vendió de los talles mayores a 42? Aprox. el 48,22%. c. ¿Cuántas camisas se vendieron entre el talle 39 y el L inclusive? ¿Qué porcentaje representa de toda la venta? 316 camisas; aprox. el 70,22%. d. Realicen un gráfico de barras con la frecuencia absoluta y un gráfico circular con la frecuencia porcentual.

26. Observen la tabla donde se registran la cantidad de hermanos (x) que tiene cada alumno de un colegio y resuelvan. x

f

0

70

1

140

2

97

3

45

4

350

5

12

6

15

7

5

8

3

a. Calculen la media _aritmética y la moda. 3x b. Hallen A, si A = ___ mo + 3,5 . mo. c. Determinen el porcentaje de la moda y la frecuencia acumulada posterior al valor que representa. _

a. x = 2,82; mo = 4. b. A ≅ 16,115. c. 47,48%; 714.

210

_

X a. m = 1; m = 2 y x = 2,5 o e _

b. mo = 1, me = no existe y x = 2,5 c. Ninguna de las anteriores.

28. Resuelvan. El promedio de las edades de tres primos es 17 años. ¿Puede alguno de ellos tener 60 años? ¿Por qué? No, porque alguno de los otros primos tendría edad negativa.

29. Tengan en cuenta los datos y resuelvan. 23 - 25 - 27 - 25 - 31 - 25 - 23 - 29 - 31 - 31 - 31 a. Calculen la media aritmética. b. ¿Qué dato habría que agregar para que la media aritmética sea exactamente 27? c. Calculen la mediana de la muestra original y de_ la modificada.

a. x = 27,36; b. 23; c. La original 26 y la otra, 27.

30. Tengan en cuenta el registro que realizó la profesora de literatura de una prueba de velocidad de lectura de palabras por minuto y resuelvan. 72, 54, 70, 80, 40, 105, 102, 71, 96, 81, 58, 57, 80, 81, 73, 99, 57, 74, 87, 48, 90, 47, 109, 90, 69, 79, 75, 52, 72, 81, 91, 56, 67, 66, 79, 90, 106, 100, 87, 104, 75, 101, 53, 98, 99. a. Si se quieren agrupar los datos en 7 intervalos, ¿qué amplitud debería tener cada uno? 10 b. Realicen una tabla de frecuencias absolutas y acumuladas, indicando las marcas de clases. c. Calculen la media aritmética, la moda, la mediana, la varianza, la desviación estándar y la clase modal. d. La muestra ¿es homogénea? ¿Por qué? e. Realicen un histograma de frecuencias absolutas y un gráfico de frecuencias acumuladas. _

c. x = 79; me = 74,1304; mo = 75; clase modal: [70;80); m2 = 339,56; m = 18,43. d. Sí (Cv = 23,33% < 30%).

capítulo

CONTENIDOS

9

49*50*51*52 2 - 4 - 6 - 8 - 10 a. Calculen la media aritmética, la varianza y la desviación estándar. b. Sumen 30 a cada uno de los datos iniciales y vuelvan a calcular los parámetros anteriores. c. Escriban una conclusión sobre el efecto que se produce en la media y en la desviación estándar cuando se le suma un valor constante _ a. x = 6; m2 = 9,6; m = 3,1. _ a los datos iniciales. 2

35. Observen el histograma que representa las edades de un grupo de personas y resuelvan. 14 12 Frecuencias

31. Tengan en cuenta los datos y resuelvan.

10 8 6 4 2 0 22

31

40

b. x = 36; m = 9,6; m = 3,1. c. A x se le suma la constante y m no varía.

32. Observen las muestras y resuelvan. _ _ B: x = 49; m = 8,3. A: x = 13,6; m = 2,3. Calculen el coeficiente de variación de cada muestra y expliquen cuál muestra tiene los datos más agrupados alrededor de la media. Cv(A) = 16,91% y Cv(B) = 16,94%. Muestra A.

49

58

67

76

Marcas de clase

_

a. Realicen una tabla con los intervalos de clase, la frecuencia absoluta y acumulada. b. Calculen la media aritmética, la mediana y la _ moda. x = 46,975; me = 46,75; mo = 49. c. La muestra ¿es homogénea? Cv= 29,21%. Es homogénea porque es menor a 30%.

33. Lean atentamente y resuelvan.

36. Respondan.

Se pesaron las bolsas de batatas que empacó una envasadora y se realizó el siguiente registro. Peso (kg)

f

[50;110)

7

¿En cuál de las siguientes muestras los datos están más dispersos respecto de la media? A: 1; 5; 8; 11; 15; 19. B: 125 000, 132 000, 137 000, 136 000, 140 000, 141 000. Cv(A) = 47,78% y Cv(B) = 2,49%.

[110;170)

55

Los datos están más dispersos en la muestra A.

[170;230)

90

37. Lean atentamente y respondan.

[230;290)

75

El gráfico muestra tres distribuciones que tienen media aritmética 0.

[290;350)

38

a. ¿Qué peso representa cada intervalo de clase? 80 kg; 140 kg; 200 kg; 260 kg; 320 kg b. ¿Qué pesos representan para la media aritmé_ tica y para la mediana? x = 218,57 kg; me = 217 kg. c. Si el coeficiente de variación es del 53%, ¿qué peso toma la desviación estándar?

0,5 0,4 0,3 0,2 0,1

m = 115,84 kg.

34. Lean atentamente y resuelvan. La muestra A tiene coeficiente de variación 56% y una desviación estándar de 3,6, mientras que la B tiene coeficiente de variación 48% y una desviación estándar de 1,6. ¿Qué muestra tiene una media aritmética mayor? _ _ xA = 6,42 y xB =3,3. La media de la muestra A es mayor que la de B.

0 –4

–3

–2

–1

0

1

2

3

4

a. ¿A qué curva corresponde cada desviación estándar? m1: verde; m2: azul; m3: roja. m1 = 2,3; m2 = 5,6; m3 = 10,7 b. ¿Cuál distribución tiene los datos más dispersos? c. ¿Es posible calcular los coeficientes de variación? b. La roja. c. No se puede calcular el coeficiente de variación porque la media aritmética es nula.

211

53

52

54

55

56

Combinatoria ¿Para qué sirve?

INFOACTIVA

PÁGINA 15

Factorial de un número Se denomina factorial de n (n!), siendo n un número natural, al producto de todos los números naturales desde 1 hasta n. n! = n . (n – 1) . (n – 2) ... 3 . 2 . 1 ∧ 0! = 1

a. 1! = 1

b. 2! = 2

c. 3! = 3 . 2 .1 = 6

d. 4! = 4 . 3 . 2 .1 = 24

e. 5! = 5 . 4 . 3 . 2 . 1 = 120

ˆ El factorial de un número es igual al número por el factorial de su anterior. ¨ « « « « « © « « « « « ª

n! = n . (n – 1) . (n – 2) ... 3 . 2 . 1 = n . (n – 1)! (n – 1)! 5! = _____ 5 . 4! = 5 a. __ 4! 4!

3! = ________ 3! 1 b. __ = ____ 6! 6 . 5 . 4 . 3! 120

5! 5! = 5! 5 . 4 . 3! = ___ 10 ________ c. ____ 3 6! 3! 6 . 5! 3!

Número combinatorio Siendo k y n dos números pertenecientes a

o

y n ≥ k, se define como número combinatorio n sobre k a:

Número combinatorio n sobre k: 6! 6! = 6 . 5 . 4! = 15 ______ a. 6 = _________ = ____ 4! 2 4 4! (6 – 4)! 4! 2!

( )

n! ( nk ) = _________ k! (n – k)!

. 7 . 6 . 5! = 56 8! 8! = 8 ________ b. 8 = _________ = ____ 3 . 2 5! 3 3! (8 – 3)! 3! 5!

( )

Propiedades de los números combinatorios 1. ( n0 ) = 1

2. ( n1 ) = n

3. ( nn ) = 1

n! n! = ___ ( n0 ) = _________ 1 n! = 1 0! (n – 0)!

n! = _________ =n ( n1 ) = ________ 1! (n – 1)! (n – 1)!

n . (n – 1)!

n! n! = ____ ( nn ) = _________ n! . 1 = 1 n! (n – n)!

4. ( nk ) = ( n n– k ) n! n! n! n! = _________ = _________________ = __________________ = ( n n– k ) ( nk ) = _________ k! (n – k)! (n – k)! k! (n – k)! (n – n + k)! (n – k)! [n – (n – k)]!

5. ( k n– 1 ) + ( nk ) = n k+ 1

(

) n! (n – k + 1)

n! k n! n! + _________ = ___________________ + __________________ = ( k n– 1 ) + ( nk ) = ________________ (k – 1)! (n – k + 1)! k! (n – k)! k! (n – k)! (n – k + 1) k . (k – 1)! (n – k + 1)! n! k + n! (n – k + 1) (k + n – k + 1) . n! n! (n – k + 1) (n + 1)! n! k = ____________ + ____________ = _________________ = ________________ = = ____________ = n k+ 1 k! (n – k + 1)! k! (n – k + 1)! k! (n + 1 – k)! k! (n – k + 1)! k! (n + 1 – k)!

(

212

)

Test

de comprensión

1. Respondan y expliquen las respuestas. a. ¿Es cierta la siguiente igualdad? 78! = 77! . 78 b. ¿Es cierto que 53 y 52 son iguales? Verifiquen la respuesta sin hacer la cuenta.

( ) ( )

a. Sí. Por propiedad, n! = (n – 1)! . n. b. Sí, se verifica con ( nk ) = ( n n– k ).

53

ACTIVIDADES Combinatoria

38. Calculen operando con factoriales. a. 5! 2! =

240

b. 5 . 8! = 201 600

1 __ 3! 4 5 c. ____ 5! =

15! 2 730 d. ___ 12! =

39. Hallen el valor de x. 5! 21! 2 . 5! ____ a. 2x + ______ 3! 20! = 4! 1!

x = –205

6! 5! 1! _____ b. _____ 4! 5! x = (4! – 3!) . 3! 2! 1 x = ___ 20

40. Simplifiquen a su mínima expresión. 13! . (n + 2)! a. ______________ = 312 + 156n (n + 1)! . (6 + 5)!

2 6! . (2n + 3)! b. ___________ = 480n + 1 200n + 720 3! . (1 + 2n)!

41. Resuelvan los siguientes números combinatorios. 10 a. 52 =

2 a__________ + 3a + 2 2 c. ( a +a 2 ) =

b. 73 = 35

d. 42 + 64 = 21

( )

( )

( ) ( )

42. Resuelvan. a. ( x +x 1 ) = 83

39 b. 5 39 + 2z = 2z – 2

x = 55

z=9

( )

(

) (

)

213

54

53

55

56

Permutaciones, variaciones y combinaciones INFOACTIVA Permutaciones Una permutación sin repetición es cada una de las formas posibles de ordenar n elementos distintos. Pn = n!

¿De cuántas maneras se pueden ubicar 8 personas en la misma cantidad de asientos? P8 = 8! = 40 320 → Se pueden ubicar de 40 320 maneras distintas. Cantidad de veces que se repite cada elemento _,`,b, ... a

Permutación con elementos repetidos: P n

n! = ___________ _! `! b! ... a!

¿Cuántos números distintos de 8 cifras se pueden formar con las cifras 1, 2, 3, 3, 4, 4, 4, 5? 2 2,3 3! 4 . 5 . 6 . 7 . 8 = 3 360 → Se pueden formar 3 360 números distintos. 8! _____ ____________ P 8 = 2! . 3! = 2 . 3! Variaciones Variaciones sin repetición de elementos son las diversas formas que existen para agrupar m elementos distintos en grupos diferentes de n elementos. m! V n = _______ (m – n)! m

Los grupos son distintos cuando tienen los mismos elementos, pero en distinto orden o por lo menos un elemento distinto. Por lo tanto, en la variación importa el orden que tengan los elementos en el grupo.

¿Cuántos números distintos de 4 cifras distintas se pueden formar con los números 1, 2, 3, 5, 6 y 8? 6 6! = __ 6! = 6 . 5 . 4 . 3 . 2! = 360 → Se pueden formar 360 números distintos. ____________ V 4 = _______ 2! (6 – 4)! 2! m

Variaciones con elementos repetidos: V’ n = mn

¿Cuántos números distintos de 4 cifras se pueden formar con los números 1, 2, 3, 5, 6 y 8? 6 V’ 4 = 64 = 1 296 → Se pueden formar 1 296 números distintos. Combinaciones Combinaciones sin repetición son las diversas formas que existen para agrupar m elementos diferentes en grupos distintos de n elementos, de manera tal que los grupos difieran en por lo menos un elemento. m! __________ Cn = ( m n ) = n! (m – n)! m

De un grupo de 9 personas, ¿cuántos equipos distintos de voley se pueden armar? 9 9! 9! = 9 . 8 . 7 . 6! = 84 → Se pueden armar 84 equipos distintos de voley. _________ C6 = 9 = _________ = ____ 6! 3 . 2 6! (9 – 6)! 6! 3! 6

( )

m Combinaciones con repetición: C’ n = ( m + nn – 1 )

Al arrojar cinco dados simultáneamente, ¿cuántos son los resultados posibles? 6 – 1 = 10 = ____ . 9 . 8 . 7 . 6 . 5! = 252 → Existen 252 resultados posibles. 10! = 10 ________________ C’ 5 = 6 + 5 5! 5! 5! 5 . 4 . 3 . 2 5 5

(

214

) ( )

Test

de comprensión

1. Respondan y expliquen las respuestas. a. ¿En qué se diferencia una variación de una combinación? m m b. ¿Es cierta la siguiente relación? V n ≥ C n

a. En que en una variación importa el orden y en la combinación, no. b. Sí, es cierta porque la combinación se calcula igual que la variación, pero quitándole elementos.

54

ACTIVIDADES Permutaciones, variaciones y combinaciones

43. Resuelvan estas situaciones problemáticas utilizando permutaciones. a. ¿Cuántos anagramas hay de la palabra SEIS? 4! – 1 = 23. Hay 23 anagramas porque SEIS no es anagrama de sí mismo.

b. ¿De cuántas formas distintas se pueden colocar 12 lápices de colores en su caja? 12! = 479 001 600

44. Marquen las opciones correctas. a. ¿Cuántos números de 4 cifras distintas se pueden formar con 4, 8, 3 y 7? 9

12

X 24

36

b. ¿Cuántos números impares de 4 cifras distintas se pueden formar con 4, 8, 3 y 7? 9

X 12

24

36

c. ¿Cuántos números menores que 7 000 y de 4 cifras distintas se pueden formar con 4, 8, 3 y 7? 9

X 12

24

36

d. ¿Cuántos números mayores que 5 000 y de 4 cifras distintas se pueden formar con 4, 8, 3 y 7? 9

X 12

24

36

45. Resuelvan estas situaciones problemáticas utilizando variaciones. a. ¿Cuántos números de 4 cifras distintas se pueden formar con los números 2, 3, 4, 5, 6, 7, 9? 840 números distintos.

b. Un docente debe distinguir a tres de sus 14 alumnos con menciones especiales. Una por el mejor rendimiento en matemática, otro por su rendimiento en historia y otro por mejor compañero. ¿De cuántas maneras se puede realizar la distinción? 2 184 maneras distintas.

c. Una asamblea de 20 personas tiene que elegir entre sus miembros un presidente, un vice y un secretario. ¿De cuántas maneras puede realizarse la elección? 6 840 elecciones.

46. Calculen el valor de n. n+1

a. V 1 = 3 n=2

n

b. V 2 = 56 n=8

215

54

ACTIVIDADES Permutaciones, variaciones y combinaciones

47. Resuelvan. a. ¿Cuántos números distintos de 5 cifras distintas pueden formarse con los números 2, 3, 4, 7, 8 y 9? 720 números.

b. En la final de natación competirán cinco nadadores. ¿De cuántas formas distintas puede quedar conformado el podio? 60 formas.

c. ¿De cuántas maneras se puede elegir un delegado y un suplente en un curso de 25 alumnos? 600 maneras.

48. Calculen. a. C 3 =

c. C 3 . P6 – 72 =

120

7 179

10

5

( )

7

5

6

6

b. C 4 – V 4 . C 2 = –5 395

C d. ___47 + 5! – P3 =

V2 689 ____ 6

49. Lean atentamente y respondan. a. ¿Cuántas diagonales tiene un hexágono? ¿Y un polígono de n lados?

( 62 ) – 6 = 9 2 n – 3n De un polígono de n lados es C 2 – n = n_______ . 2

b. En una parada de colectivo suben 10 personas. Si solo hay 4 asientos disponibles, ¿de cuántas maneras pueden ubicarse? 10

V 4 = 5 040

c. ¿Cuántas posibilidades tiene un entrenador de básquet de elegir los jugadores que comenzarán un partido si cuenta con 8 jugadores?

( 85 ) = 56

216

54

ACTIVIDADES Permutaciones, variaciones y combinaciones

50. Lean atentamente y resuelvan. Si hay 27 letras distintas, ¿cuántos conjuntos diferentes de iniciales pueden formarse en cada caso? a. Las personas tienen exactamente dos nombres y un apellido. 273 = 19 683

b. Las personas tienen a lo sumo dos nombres y un apellido. 273 + 272 = 20 412

c. Las personas tienen a lo sumo tres nombres y un apellido. 274 + 273 + 272 = 551 853

51. Resuelvan. En un hospital se utilizan cinco símbolos para clasificar las historias clínicas de sus pacientes: los dos primeros son letras y los tres últimos son dígitos. Si utilizan 25 letras, ¿cuántas historias clínicas pueden hacerse en cada caso? a. Las letras y los números se pueden repetir. 625 000

b. Los números se pueden repetir, pero las letras no. 600 000

52. Marquen las respuestas correctas. Un centro de investigación cuenta con 5 matemáticos y 7 físicos. Se quiere formar una comisión de 2 matemáticos y 3 físicos. a. ¿Cuántas comisiones distintas se pueden armar? X 350

35 3 500 Ninguna de las anteriores. b. Si ya se eligió a un físico, ¿cuántas comisiones distintas se pueden formar? X 150

15 1 500 Ninguna de las anteriores. c. Suponiendo que hay dos matemáticos que no se llevan bien y, por lo tanto, si eligen a uno, no eligen al otro, ¿cuántas comisiones distintas se pueden formar? 300

150

35

X Ninguna de las anteriores. ( C 52 – 1 ) . C 73 = 9. 35 = 315

mente ACTIVA Tres atletas participan de una competencia. Si puede haber empate, ¿cuántos resultados posibles hay? Hay 13 resultados posibles. 217

55

54

56

Probabilidad INFOACTIVA Un experimento aleatorio es aquel cuyo resultado depende del azar; por ejemplo, arrojar un dado, sacar una carta de un mazo o una bolilla de un bolillero, etc. Un suceso S es uno de los resultados posibles y el espacio muestral E es el conjunto de todos los resultados posibles de un experimento aleatorio; por ejemplo, al arrojar un dado, el espacio muestral es: 1, 2, 3, 4, 5 y 6. de casos favorables #S _________________________ La probabilidad de que un suceso ocurra es P(S) = Número = ___ ∧ 0 ≤ P(S) ≤ 1. #E Número total de casos

En una bolsa hay 10 fichas iguales con los siguientes colores: 1 verde, 3 azules, 4 blancas y 2 rojas. El espacio muestral se puede definir como E = {verde, azul, blanca, roja}, pero al calcular el número total de casos posibles, hay que considerar la cantidad de fichas de cada color. #E = 10 La probabilidad de sacar una ficha de un color determinado es: 1; ˆ P(verde) = ___ 10

3; ˆ P(azul) = ___ 10

4; ˆ P(blanco) = ___ 10

2. ˆ P(rojo) = ___ 10

Se consideran ahora dos sucesos A y B en un espacio muestral E.

En una bolsa hay 10 bolillas iguales numeradas del 1 al 10. → E = {1; 2; 3; 4; 5; 6; 7; 8; 9; 10} ∧ #E = 10 Suceso A: sacar una bolilla con un número par. → A = {2; 4; 6; 8; 10} ∧ #A = 5 Suceso B: sacar una bolilla con un número mayor que 6. → B = {7; 8; 9; 10} ∧ #B = 4 #(A ∩ B) La probabilidad de que A y B ocurran se llama probabilidad compuesta: P(A y B) = ________ #E

¿Cuál es la probabilidad de extraer una bolilla par y mayor que 6? #(A ∩ B) 2 P(A y B) = ___ 10

#E

#(A ∪ B) La probabilidad de que A o B ocurra se llama probabilidad total: P(A o B) = ________ #E

¿Cuál es la probabilidad de extraer una bolilla par o mayor que 6? #(A ∪ B) 7 P(A o B) = ___ 10

#E

En algunos casos, los sucesos sobre un mismo espacio muestral pueden relacionarse entre sí.

En una bolsa hay 4 bolillas verdes y 4 azules y se realiza una extracción. 4 = __ 4 = __ 1 ∧ P(azul) = __ 1 , es igualmente probable sacar una bolilla verde o azul. P(verde) = __ 8 2 8 2

218

Test

de comprensión

1. Respondan y expliquen las respuestas. a. Medir el perímetro de una circunferencia de radio 2 cm ¿es un experimento aleatorio? b. La probabilidad de que ocurra un suceso ¿puede ser 1? ¿Y mayor que 1? a. El suceso no es aleatorio, es un resultado ya determinado. b. Sí, cuando el número de casos favorables coincide con el número total de casos. No puede ser mayor que 1.

55

ACTIVIDADES Probabilidad

53. Tengan en cuenta el experimento de arrojar dos dados equilibrados y anotar su suma; luego, resuelvan. a. Escriban el espacio muestral. E = {(1;1),(1;2),(1;3),(1;4),(1;5),(1;6),(2;1),(2;2),(2;3),(2;4),...,(6;6)}, #E = 36 b. Calculen las siguientes probabilidades. 3 1 ___ = ___ ˆ P(10) = 36 12

18 ___ = __1 ˆ P(que la suma sea impar) = 36 2

6 ___ = __1 ˆ P(que la suma sea un múltiplo de 2 y de 3 a la vez) 36 6

54. Lean atentamente y resuelvan. Una caja contiene 25 caramelos de frutilla y 5 de menta. Si se extraen 2 caramelos al azar, ¿cuál es la probabilidad de que ambos sean de frutilla? ¿Y de que ambos sean de menta? ¿Y de distinto gusto? 25

5

25

5

C2 C2 C . C1 125 300 10 1 ____ ___ ____ ______ P(f-f) = ___ = ____ = 0,2874. 30 = 435 = 0,6897; P(m-m) = 30 = 435 = 0,02299; P(f-m) = 30 435 C2 C2 C2

55. Resuelvan. a. Se lanzan al aire dos monedas. ¿Cuál es la probabilidad de obtener a lo sumo una vez cara? 3 __ 4

b. Se extraen simultáneamente dos cartas de una baraja española de 40. ¿Cuál es la probabilidad de que las dos sean espadas? 9 3 1 . ___ __ = ___ 52 4 39

c. Se lanzan al aire dos dados de 4 caras numeradas del 1 al 4. ¿Cuál es la probabilidad de que la suma de los números obtenidos sea mayor que 5? 3 6 ___ = __ 8 16

56. Lean atentamente y resuelvan. En una encuesta se consultó a 80 personas sobre si escuchaban radio o veían televisión antes de ir a dormirse. 60 dijeron que veían televisión y 30, que escuchaban radio. a. Si se encuestaron a 80 personas, ¿cómo es posible que se hayan obtenido 90 resultados? Al menos 10 personas tuvieron que haber elegido las 2 opciones.

b. Si todas las personas eligieron al menos una opción, ¿cuál es la probabilidad de que, al elegir una encuesta al azar, esté marcada solo la opción televisión? ¿Y de que esté marcada solo la opción radio? 50 5 20 1 ___ __ = __ P(tv) = ___ 8 ; P(r) = 80 = 4 . 80

219

56

55

Sucesos y probabilidad condicional INFOACTIVA Sucesos mutuamente excluyentes e independientes Dos sucesos son mutuamente excluyentes cuando no pueden ocurrir al mismo tiempo. En este caso: P(A y B) = 0 Si A y B son mutuamente excluyentes, P(A o B) = P(A) + P(B) Si A y B no son mutuamente excluyentes, P(A o B) = P(A) + P(B) – P(A y B) Dos sucesos son independientes cuando el hecho de que ocurra uno de los dos no altera la probabilidad de que ocurra el otro.

En una bolsa hay 4 bolillas verdes y 4 azules y se realiza una extracción. 4 = __ 4 = __ 1 ∧ P(azul) = __ 1 , es igualmente probable sacar una bolilla verde o azul. P(verde) = __ 8 2 8 2 Si se saca una bolilla y no se reintegra a la bolsa, la probabilidad de sacar verde o azul en la segunda extracción cambia, ya que cambian #V o #A y #E = 7. Se dice entonces que los sucesos son dependientes. Si se saca una bolilla y se reintegra a la bolsa, la probabilidad de sacar blanco o negro en la segunda extracción es igual que en la primera. Se dice entonces que los sucesos son independientes. Para averiguar una probabilidad compuesta, se puede utilizar la siguiente fórmula. Probabilidad de que sucedan A y B a la vez: P(A y B) = P(A) . P(B/A) Si A y B son independientes: P(A y B) = P(A) . P(B)

Probabilidad condicional Se llama probabilidad condicional a los casos donde se quiere conocer la probabilidad de un suceso sabiendo que ha ocurrido otro. P(A y B) Probabilidad de B si se conoce A: P(B/A) = ________ P(A)

Si A y B son independientes: P(B/A) = P(B) En el caso de la bolsa con bolillas:

Experimento: sacar una bolilla y luego sacar otra sin reponer la primera. ¿Cuál es la probabilidad de sacar una bolilla verde en la segunda extracción si en la primera se retiró también una verde? 4 . __ 3 __ P(A y B) _____ 8 7 __ = =3 P(B/A) = _______ 7 4 P(A) __ 8

Experimento: sacar una bolilla y luego sacar otra habiendo repuesto la primera. ¿Cuál es la probabilidad de sacar una bolilla verde en la segunda extracción si en la primera se retiró también una verde? 4 = __ 1 porque son sucesos independientes. P(B/A) = P(B) = __ 8 2

220

Test

de comprensión

1. Respondan y expliquen las respuestas. a. Se lanzó un dado seis veces y se obtuvieron seis 2. ¿Cuál es la probabilidad de que vuelva a salir 2? b. Al lanzar una moneda, los sucesos “cara” y “ceca”, ¿son mutuamente excluyentes? 1 si el dado es equilibrado. No depende de las extracciones anteriores. a. La probabilidad sigue siendo __ 6 b. Sí, pues al lanzar una moneda puede salir cara o ceca, pero nunca las dos posibilidades a la vez.

56

ACTIVIDADES Sucesos y probabilidad condicional

57. Clasifiquen estos sucesos en independientes o mutuamente excluyentes. En una urna hay dados rojos y verdes. a. Se determinan los sucesos A: sacar un dado rojo y B: sacar un dado verde. ˆ ¿A y B son sucesos mutuamente excluyentes? Sí, porque no pueden ocurrir al mismo tiempo.

ˆ ¿Cuál es la probabilidad de sacar un dado verde o un dado rojo? P(A o B) = P(A) + P(B) = 1; porque solo hay dados rojos y verdes.

b. Se hacen dos extracciones con reposición y se determinan los siguientes sucesos: C: sacar un dado verde en la primera extracción. D: sacar un dado verde en la segunda extracción. ˆ ¿C y D son sucesos independientes? Sí, porque la segunda extracción no depende de la primera.

ˆ ¿Cuál es la probabilidad de sacar dos dados verdes? P(C y D) = P(C) . P(D). No se puede saber porque no se conocen las cantidades.

58. Resuelvan. Si A y B son sucesos mutuamente excluyentes, P(A o B) = __87 y P(A) = __21 , ¿cuál es la probabilidad de B? 3 P(A o B) = P(A) + P(B) ⇒ P(B) = P(A o B) – P(A) = __ 8

59. Lean atentamente y resuelvan. a. Una caja contiene 30 lámparas, de las cuales 5 están quemadas. Si se extraen 2 lámparas al azar, en forma sucesiva y sin reposición, ¿cuál es la probabilidad de extraer dos lámparas buenas? ¿Y si luego de extraer la primera, se repone? 20 ___ ___ ; 25 29 36

b. En una biblioteca hay 5 libros de química, 4 de geografía y 3 de contabilidad. Si se extraen 2 libros al azar en forma sucesiva y sin reposición, ¿cuál es la probabilidad de que el primer libro extraído sea de contabilidad y el segundo, de química? ¿Y que los dos libros sean de geografía? 5 ____ 15 1 ____ = ___ ; 12 = __ 11 132 44 132

221

56

ACTIVIDADES Sucesos y probabilidad condicional

60. Lean atentamente y resuelvan. En una urna hay: 6 fichas verdes (3 con la letra A y 3 con la B); 2 fichas rojas (1 con la letra A y otra con la B) y 2 fichas blancas con la letra A. a. Construyan una tabla de doble entrada donde aparezcan los datos del problema. Fichas verdes

Fichas rojas

Fichas blancas

Total de fichas

Letra A

3

1

2

6

Letra B

3

1

0

4

Total

6

2

2

10

b. Si se extrae una ficha con la letra A, ¿cuál es la probabilidad de que sea verde? 1 __ 2

c. Si se extrae una ficha roja, ¿cuál es la probabilidad de que tenga la letra B? 1 __ 2

d. Si se extrae una ficha con la letra B, ¿cuál es la probabilidad de que sea blanca? 0

61. Resuelvan. a. Se encuestó a 200 personas sobre la preferencia de 4 tipos de productos y se obtuvieron los siguientes resultados. Producto

A

B

Ama de casa

14

Empleado

10

Profesional

12

Empleo

C

D

6

10

30

5

20

35

15

8

35

ˆ Si se selecciona al azar una de las personas que no es empleado, ¿cuál es la probabilidad de que prefiera el producto C?

18 ____ = 0,1385 130

ˆ Si se selecciona al azar una de las personas que prefiere el producto D, ¿cuál es la probabilidad 35 ____ = 0,35

de que sea profesional? 100 b. En un estudio realizado a un grupo de trabajadores se determinó el grado de escolaridad máximo alcanzado y el nivel de ingresos. Los resultados se muestran en la siguiente tabla. Ingresos

Altos

Medios

Bajos

Secundario

18

27

5

Universitario

26

38

16

Posgrado

9

15

9

Escolaridad

ˆ Si se selecciona al azar un trabajador que ha realizado un posgrado, ¿cuál es la probabilidad de 9 ___ = 0,2727 que tenga ingresos altos? 33

ˆ Si se selecciona al azar un trabajador que tiene ingresos medios, ¿cuál es la probabilidad de que 27 ___ = 0,3375 haya estudiado hasta el secundario? 80 222

56

ACTIVIDADES Sucesos y probabilidad condicional

62. Resuelvan. Una compañía de telefonía celular realiza llamados para ofrecer un servicio de larga distancia. Se sabe que en 4 de cada 10 llamados se encuentra al responsable de la línea y que, cuando esto ocurre, 3 de cada diez aceptan el servicio ofrecido. Al realizar un llamado, ¿cuál es la probabilidad de que el responsable de la línea esté en su casa y acepte el servicio ofrecido? P(casa) = 0,40; P(acepta/casa) = 0,30; P(casa y acepta) = P(casa) . P(acepta/casa) = (0,40) . (0,30) = 0,12

63. Lean atentamente y resuelvan. a. Se extraen cinco cartas de a una de un mazo de 52 naipes franceses. ¿Cuál es la probabilidad de obtener en todas las extracciones un as, si se reponen los naipes luego de cada extracción? ¿Y si no hay reposición? 4 P(todos ases c/r) = ___ 52

5

( )

= 0,0000027

No es posible porque solo hay 4 ases.

b. Un jugador de básquet tiene un promedio de 80% al lanzar tiros libres. Si realiza 4 intentos, ¿cuál es la probabilidad de que convierta todos? ¿Y de que enceste al menos un lanzamiento? P(convierta todos) = 0,84 = 0,4096 P(que acierte 1 o más) = 1 – P(ninguno) = 1 – 0,24 = 0,9984

64. Marquen las opciones correctas. En un grupo de amigos el 80% está casado. Entre los casados, el 90% tiene trabajo. Un 5% no está casado ni tiene trabajo. a. ¿Qué porcentaje de los amigos no tiene trabajo? X 13%

33% 80% Ninguna de las anteriores. b. Si uno de los amigos tiene trabajo, ¿qué probabilidad hay de que esté casado? X 0,83 0,72 Ninguna de las anteriores. 0,8 c. Si uno de los amigos no tiene trabajo, ¿qué probabilidad hay de que esté casado? X 0,62

0,72

0,8

Ninguna de las anteriores.

mente ACTIVA Marcos, que tiene problemas para despertarse y llegar a tiempo a clase, se pone 3 despertadores para evitarlo. La probabilidad de que cada uno funcione correctamente es de 0,95. a. ¿Cuál es la probabilidad de que solo uno de los despertadores falle? b. ¿Cuál es la probabilidad de que llegue puntualmente a clase? a. P(solo uno falle) = 3 (0,05)(0,95)(0,95) = 0,1354. b. P(Al menos 1) = 1 – P(ninguno) = 1 – 0,000125 = 0,999875 223

INTEGRACIÓN 65. Simplifiquen a la mínima expresión posible. 101! (n + 4)! (n + 3)! a. _________________________________ (98 + 2)! (1 + n)! (n2 + 9 + 6n) (n + 2)! (n – 2)! n b. C 2 . 52 _______ 3n! . P4

( )

a. 101 . (n + 2) . (n + 4). b. 40.

66. Apliquen las definiciones y propiedades necesarias y hallen el valor de la incógnita. 5 26! 4! a. 1! 3! x + ___________ 7 800 . 24! 2! – 3 = V 2

( )

[

( )]

2! . (y – 2)! y b. 3y – __________ 2 (y –1)!

3

5.V

8

3 : P2 = _____ 336

432 formas distintas.

67. Lean atentamente y resuelvan. a. Juan tiene que repartir 7 monedas iguales entre 7 amigos. ¿De cuántas formas distintas las puede repartir. ¿Cómo podrá repartirlas si hay 3 amigos que no quieren ninguna? b. El dueño de una empresa debe elegir un director, un gerente y un jefe de sección entre un grupo de 20 empleados. ¿De cuántas formas distintas puede hacer su elección? c. Un escritor visitó un curso de un colegio secundario y ofreció sortear 4 novelas de su autoría entre los 25 alumnos. ¿De cuántas formas diferentes se pueden distribuir los libros? ¿Y si las 4 novelas fuesen iguales? d. Una promotora de perfumes importados debe obsequiar 48 muestras de un perfume a 15 personas que están reunidas. Si cada una recibe al menos 3 muestras, ¿de cuántas formas puede hacer la distribución? e. Enrique prepara ramos de 12 rosas para regalar. Los arma con flores rojas, blancas y rosas, pero en cada ramo no coloca más de 6 rosas rojas. ¿Cuántos ramos distintos puede regalar? f. ¿Cuántas palabras de 13 letras pueden formarse con las letras de la palabra REPETIDAMENTE? ¿En cuántas de ellas no aparecen consecutivamente las dos T? 7

20

a. P7 = 5 040. C 3 = 35. b. V 3 = 6 840. c. V

= 303 600. C

25 4

= 12 650.

d. C = 680. e. 45. f. 129 729 600; 109 771 200.

224

324 formas distintas.

z

a. x = 2,5. b. y = 5. c. z = 0; no es solución.

25 4 17 3

a. En un programa de TV donde se debaten temas científicos, se sientan alrededor de una mesa circular 3 matemáticos, 3 físicos, 3 químicos y 2 filósofos. Si las personas que pertenecen a una misma disciplina deben sentarse juntas, ¿de cuántas maneras pueden ubicarse? 33 formas distintas.

c. z . ( 5z ) . [C 6 ]–1 = 6 . V 2 z

68. Marquen las opciones correctas.

X Ninguna de las anteriores. 10 368

b. Se lanzó 6 veces un dado con los siguientes resultados: salió dos veces el 1, una vez el 3 y el resto fueron números pares distintos. ¿De cuántas maneras puede haber ocurrido el suceso, sabiendo que en el primer tiro salió un 4? 360 X 60

30 Ninguna de las anteriores. c. ¿Cuántos grupos de cuatro, tres, dos y una letras se pueden formar con las letras de la palabra MESA? X 15

16 24 Ninguna de las anteriores. d. ¿En cuántas de las permutaciones del número 63 814 465 los dígitos impares aparecen, de izquierda a derecha, en forma creciente? 840 X 1 680

3 360 Ninguna de las anteriores.

capítulo

CONTENIDOS

9

53*54*55*56 69. Resuelvan.

74. Tengan en cuenta los datos y resuelvan.

Seis amigos llegaron juntos a realizarse un estudio médico y sortean el orden en el que serán atendidos. ¿Cuál es la probabilidad de que hayan quedado ordenados por edad de mayor a menor?

A y B son sucesos, P(A) = 0,3 y P(B) = 0,27. a. Hallen P(A yB) si A y B son mutuamente excluyentes. b. Hallen P(A o B) si A y B son mutuamente excluyentes. c. Hallen P(A y B) si A y B son independientes.

70. Lean atentamente y resuelvan. Pablo tiene una caja con 16 autitos de colección extranjera y 4 de colección nacional. Decide extraer dos al azar para regalarle a Fernando. a. ¿Cuál es la probabilidad de que ambos sean 12 de colección nacional? ____ 380 b. ¿Cuál es la probabilidad de que sean uno 64 extranjero y otro nacional? ____ 380

71. Se arrojan 3 dados equilibrados. Calculen las siguientes probabilidades. 1 ____ a. Que salgan tres 6. 216 3 b. Que salgan dos 3 y un 4. ____ 216 c. Que no salgan tres 2. 215 ____ 216

72. Tengan en cuenta el experimento y resuelvan. Se extraen tres bolas en forma sucesiva y sin reposición de una caja que contiene 6 bolas rojas, 4 blancas y 5 azules. a. Calculen la probabilidad de que la primera sea roja; la segunda, blanca y la tercera, azul. b. Calculen la probabilidad de que las tres bolas sean del mismo color. 4 34 a. ___ ; b. ____ 91 455

73. Lean atentamente y resuelvan. a. Se escriben en tarjetas las letras de la palabra CLAVE y se colocan en una fila al azar. ¿Cuál es la probabilidad de que las vocales queden juntas? __52 b. Se hornean 180 galletitas: 83 con nueces, 60 con frutas secas y el resto con trozos de chocolate. Si se extraen dos galletitas al azar sin reposición, ¿cuál es la probabilidad de sacar las dos veces una galletita con nueces? 0,2112

a. 0. b. 0,57. c. 0,081.

75. Lean atentamente y resuelvan. En una oficina hay 100 calculadoras, de las cuales 60 son científicas y 40 son graficadoras. De las calculadoras científicas hay 40 nuevas, mientras que de las graficadoras hay 10 usadas. Un empleado toma una calculadora al azar y descubre que es nueva. ¿Cuál es la probabilidad de que sea científica? ¿Y de que sea graficadora? 4 __ 3 __ 7; 7

76. Se realizó un estudio para saber si el color de pelo de las personas está asociado con el color de los ojos y se obtuvieron los siguientes resultados. Ojos Cabello

1 = ____ 1 __ 720 6!

verdes

azules

otros

castaño oscuro

70

30

20

rubio

20

110

50

a. Se selecciona una de estas personas al azar. ¿Cuál es la probabilidad de que su pelo sea castaño oscuro, si se sabe que sus ojos son verdes? b. Los sucesos tener el pelo rubio y tener los ojos azules ¿son independientes? Expliquen la respuesta. 7 a. __ = 0,78; b. No, porque P(R/A) ≠ P(R). 9

77. Se lanzan dos dados las veces necesarias hasta que salgan dos números iguales. a. ¿Cuál es la probabilidad de que esto suceda 671 _____ antes del quinto lanzamiento? 1 296 b. ¿Cuál es la probabilidad de que esto suceda 5 3 __ después del tercer lanzamiento? 6 c. Si suponemos que no sucede en el primer lanzamiento, ¿cuál es la probabilidad de que se 3 necesiten más de 4 lanzamientos? __65

( ) ( )

225

capítulo

9

AUTOEVALUACIÓN

Marquen las opciones correctas 78. En la siguiente tabla se registró la edad de las personas que viajaron en un micro que realizó un tour por la ciudad de Buenos Aires. Edad

[40;50)

[50;60)

[60;70)

[70;80)

f

15

43

25

7

a. ¿Cuál es la edad promedio de las personas que viajaron en ese micro? De 56 a 57.

X De 57 a 58.

De 58 a 59.

Ninguna de las anteriores.

60

Ninguna de las anteriores.

b. ¿Cuál es la mediana de la muestra? 50

X 57

c. ¿Qué frecuencia porcentual tiene la clase modal? 45%

46%

47%

X Ninguna de las anteriores. 47,78%

47%

Ninguna de las anteriores.

d. ¿Cuál es el coeficiente de variación? X 14,35%?

25,34%

79. ¿De cuántas formas pueden ubicarse 6 matrimonios en una mesa circular, si cada hombre debe estar rodeado por dos mujeres y los miembros de cada pareja deben estar juntos? a. 2 592

b. 2 952

c. 2 925

X d. Ninguna de las anteriores. 1 440

80. Respondan. a. ¿Cuál es la probabilidad de extraer una bolilla impar múltiplo de 3 de una caja donde hay 33 bolillas numeradas del 0 al 32? 12 ___ 33

7 ___ 33

5 X ___

Ninguna de las anteriores.

33

b. Se sabe que cuando se enfrentan los equipos de básquet Bohemios y Filósofos, el primer equipo lleva ganado el doble de los encuentros que el otro. Si se vuelven a encontrar, ¿cuál es la probabilidad de que pierda Bohemios? 1 __ 2

2 __ 3

3 __ 4

X Ninguna de las anteriores. 1 __ 3

81. En un curso de secundario hay 21 mujeres y 17 varones. De ellos, 3 mujeres y 4 varones son zurdos. a. ¿Cuál es la probabilidad de elegir una persona zurda, si se sabe que es varón? 4 X ___ 17

3 ___ 17

7 ___ 17

Ninguna de las anteriores.

b. ¿Cuál es la probabilidad de elegir una mujer, si ya se sabe que es una persona diestra? 21 ___ 38

226

18 X ___ 21

21 ___ 38

Ninguna de las anteriores.

Control de resultados capítulo

16. Por ejemplo, a. a = 5

1

1. Números reales. Intervalos 1. Por ej., a. Racional.

17. a. (–3;7) b. (–';3,5) ∪ [12,5;+') c. (–';–3) ∪ [5;+')

3.

[

10 f. – ___ 7 ;6

e. (–2;+')

Por ejemplo: 5 a. __ 2 b. 3,12 c. 2,4493 d. –2,5 __

[

__

]

2 ≤ x ≤ 3 ; __ 2; 3 . Por ej., a. __ 3 3 33

)

]

19. Solución a cargo del alumno. MENTEACTIVA Solución a cargo del alumno.

5.

2. Módulo de un número real 6. e. –b f. 0,25a

20. a. V c. V e. F g. F i. F k. V b. F d. V f. F h. V j. F l. F 21. Por ejemplo, a. 1; 8; 2; 1; 2; 4.

7. a. = b. = c. < d. = e. > f. = 8.

Por ej., a. x = 2 ∨ x = –2.

22. a. (–';0) b. [–1;+') c. (0;+') d. (–';3π) ∪ (3π;+') e. [–5;5] 5 7 __ f. –';__ 2 ∪ 2 ;+'

(

] [

)

h. [–5;–2) ∪ (2;5] j. (–';–2) ∪ [2;7]

9. Por ej., a. x = 4.

3. Ecuaciones e inecuaciones con módulo 10. a. x = 3 ∨ x = –7

(

i. (1;2)

)

27. a. F

)

b. F c. F d. V

b.

34. a. (–';–11) b. x = –4; x = –16 c. (–';3) ∪ (13;+') 35. 3 Por ej., a. x = – ___ . 16 36. a. F c. F e. F g. V i. V k. F b. V d. F f. F h. F j. F l. V 37. Por ej., a. a42. 38. Por ej., a. 5x6. 39. 3 097 a. _____ 36

164 c. – ____ 7 d. –0,4

b. 7

e. 480 81 f. ___ 2

5. Operaciones con radicales 40. 27 __ a. ___ . 2 4 3

__

__

11 b . 3 b e. __ 3 4

3 2 __ 2 1 3 4 __ __ f. __ 5 c + 2 c – 7 c . 3c

(

e. V

)

41. ___ ___ a. –3 . 3___ 15 – 12 . 333 b. 11 . 3__10 + 79 c. –1 d. 4 . 32 e. 100a7 – 36b7 4 3 2 f. 2x . y – 2x . y 42. __ Por ej., a. 30 – 12 . 36 .

13. 4 y –14. 14. Solución a cargo del alumno.

29. a. w = 0

44. a. 2

15. a. x = 0 b. ’

30. 2 a. x = 4; x = – __ 3 b. x = –7,5; x = 6,5

d. F

c. x = –2 d. x = –1

b. w = 3

__

7 b. – __ 2 . 3__3 + 16 .__35 d. 13 . 3x – 4 . 3z

28. 37 35 a. x = ___ ; x = ___ 4 4 b. No tiene solución. c. x = 0 d. x = 2a; x = 0

b. V

e. (–';1)

33. Solución a cargo del alumno.

c. 19 . 33

26. Por ej., a. |x| ≥ 1,5 ∧ |x| ≤ 3.

f. x = 4 ∨ x = 6

c. V

[ )

25. Por ej., a. |–x| < 4.

45 25 ___ e. x = ___ 2 ∨x=– 2

12. a. F

(

__

19

2 b. a > __ 9

5 g. 1;__ 2

23. Por ej., a. (4;18]; x > 4 ∧ x ≤ 18. 24. 2 ; __ Por ej., a. – __ 3 33

17 ∨ x = ___ b. x = ___ 6 6 c. No tiene solución. d. x = 0 ∨ x = 2

11. a. a = –2

32. 4 ___ a. – __ ; 16 3 3

]

4. Radicales

INTEGRACIÓN 1.2.3

Por ej., a. (–3;/].

a. 16,14 c. 6 __ __ b. 35 d. –3 + 311

) (

(

Por ej., a. 7; 1; 8; 1; 9; 2.

4.

[

18. 9 ___ 5 7 __ 3 ___ a. [–9;–3] b. – __ 2 ;– 2 c. – 14 ; 14 d. (–';0) ∪ (2;+')

2.

31. 8 a. [1,6;2,4] c. – __ 5 ;0 b. (–2;8) d.

43. ___ __ 12 12 Por ej., a. 3a9 y 3a16 . 8

__

b. 33

c. 10

3

___

d. 318

45. a. F b. V c. V d. F e. F f. F

227

63. __ a. 17 + 12 . 32 b. 6

46. ______ 12 Por ej., a. 33–1 . a–2 .

64. __ 4 Por ej., a. y2 . 3y3 .

6. Operaciones combinadas 48. __ __ __ 3 4 a. 6 + 3 . 33 d. ( 35 + 3 . 35 ) : 6 1 __ __ 2 b. 22 e. – __ 2 . 32 – 3 c. –7 . (34 ) 49. 1 a. __ 4

2

1 944 f. _____ 5 + 648 . 33

50. 2 __ Por ej., a. __ 11 . 311 . 51. __ Por ej., a. 37 – 2. 52. _________ __ a. 33 + 2 . 32

e. F f. V b. 2

15

__

b. 1 – 2 . 32 57. __ 28 a. v2 . 3v9 8 b. __ 5

11. a. 6 141

AUTOEVALUACIÓN

13. 1 39 Por ej., a. 2 . __ 2 .

68. a. |a| = –a b. |–2,3 + b| = |–2,3| + |b| y |–2,3 + b| ≤ |–2,3| + |b|

[

g. F h. F

i. V

__

)

( )

11. Análisis de sucesiones 14. a. Mon. crec. b. Mon. decr.

__

__

1 .h. h b. 4 . 3h – __ 3 2 ___ –37 + 7 . 321 d. ____________ 4 5

137 ____ 45

__

b. ( 4 – 33 ;4 + 33 )

71. __ 12 ___ a. b . 3a . 3b11 5

capítulo

c. a + x __

e. 2 + 3x

1.

d. 8

Por ej., a. 33; 39; 45. an = –3 + 6n.

c. 0 __ 10 d. a2 . 3a7

2.

59. ___ ___ Por ej., a. 36y – 36z .

4.

60. a. a = 1

9. Sucesiones aritméticas

Solución a cargo del alumno. Por ej., a. Arítmética; r = 8.

b. No existe.

a. 52,6

b. 2 416

c. 7

d. V 6. a. 1 965

62. Por ej., a. Ninguna de las ant.

18. Por ej., a. Convergente; 0. 19. Por ej., a. Conver. si –1 < t < 1; diver. si t ≥ 1 y t < –1; osc. si t = –1.

7 __ 13 Por ej., a. 1; 1; __ ; 5 ; ___ . 9 8 25 89 _______ a60 = 108 000 .

5.

b. –2 730

7. a. –2 b. a1 = 11; a2 = 9; a3 = 7

228

16. La tabla se completan con: 2; 1,732; 1,587; 1,495; 1,271; 1,047; 1,007; 1,001. a. Mon. decr. b. 1. c. 1 ≤ bn ≤ 2.

8. Sucesiones

c. 117

c. F

15. a. Esta acotada entre 0 y 1. b. Esta acotada superiormente. c. Esta acotada entre –5 y 5.

17. Por ej., a. Convergente; 3.

3.

b. V

c. Ninguna. d. Ninguna.

12. Clasificación de sucesiones

2

58. 49 __ Por ej., a. – ___ . 2. 3 3

61. a. F

b. 4 194 303

12. a. 6; 18; 54; 162 b. 8,9675 c. q = 6; s12 = 612 – 1

67. a.

55. __ 5 Por ej., a. 3x3 . 56. __ 3 89 a. ___ + 3 . 32 4

10. a. –9 216 b. 3,45 . 10–13 c. 10

c. 11 – 6 . 32

66. __ Por ej., a. 25 . 33 – 30.

70. 97 a. ___ ;+' 48

___

b. 4 . 33 ay

54. ____ 149 a. a140

10. Sucesiones geométricas __

69. a.

INTEGRACIÓN 4.5.6.7 c. F d. V

b. 745

$4 120

22 . 33 – 10 b. ___________ 13

d. 0

7. Racionalización de denominadores

53. a. F b. V

65. __ a. 4 – 8__. 32

__

1 b. –2y c. – __ 3

b

a. 55x – 135 9.

47. __ __ Por ej., a. x2 . 3x – x . 34 x .

__ 3

8.

__

c. 11 + 12 . 32 __ d. 102 + 72 . 32

20. Por ej., a. an = –3n. 21. a. F

b. V

c. F

22. Solución a cargo del alumno. 23. 1 an = 3n . __ n 24. Por ej., a. 3; 7; 15; 31; 63; 127; 255; 511; 1 023; 2 047.

25. a1 = 3;capitulo a2 = 16; an = 5an–1 + 2; para n ≥ 3.

2

25. Opuestos de un número y 26. valor absoluto. Solución a cargo del alumno.

41. 14.Por ej., a. a1 = 17; a2 = 53; a3 = 89. a. 9 42.b. 45 Solución a cargo del alumno.

26. Orden y representación AUTOEVALUACIÓN numérica

1. 27. a. 3 701 a cargo del alumno. Solución b. 45 861: c. 270 MENTEACTIVA Solución a cargo del alumno. 2. a. 9INTEGRACIÓN 8.9.10.11.12 28.b. 45 Por ej., a. an = n2 – 4. 3. 29.a. 3 701 b. 2; 45 7861: a. b. 41; 95 c. 270 30. 4. a. No. b. 10 a. 9 31.b. 45 a. 4; 1 575; 504. c. 10 5. __1 1 n–1 b. 4 . __ d. –10,0000305 a. 32 ;701 2 . b. 45 861: 32.c. 270 5; 8; 11; 14 y 17.

43. 1. a. a1 = 69,5 y a20 = –44,5. a. a9 = –704 y s = –2 814,625. b. 2 11 b. 45 44. a. 2. 45.a.c.3 y701 e. b. 45 861: 46.c. 270 a. Conv. a –5. 3. b. Diver. y acotada inferiormena. te. 9Su ínfimo es 6. b. 45

6. 33.a. a. b. b.

6. 2.

( )

9 30°; 45 60°; 90°. 24°; 48°; 96°; 192°.

7. 34.a. 3 701 __ Por45ej.,861: a. a1 =__4; a2 = 2 . 32 ; b. ac.3 = 2; a4 = 32 . 270 35. 8. crec. c. Mon. decr. a. a. Mon. 9 b. No es mon. 9. 36.a. 3 701 a. b. Sí. 45 861: b. Superiormente. c. Cotas sup: 6; 8; 9. 10.d. Divergente. a. 9 37.b. 45 Por ejemplo, a. Convergente a aco11.–1, monótona decreciente, 3 __ tada entre –1 y 2 . a. 3 701 b. 45 861: 38.c. 270 1 __ 1 Por ej., a. Osc. entre – __ 2 y 2. 12. 39.a. 9 Solución a cargo del alumno. b. 45 40. 13. 7 ___ 18 __ 10 __ 11 ___ Por a. 3 ej., 701 a. 2; 3; 3 ; 2 ; 5 ; 3 ;… b. 45 861: c. 270

4. a. 3 701 capítulo b. 45 861: c. 270

3

13. El conjunto de los números 5. complejos 1. a. 9 b. 45ej., a. ±5i; complejo. Por a. 3 701 a cargo del alumno. Solución b. 45 861: 3. c. 270 Por ej., a. –3 + 2i. 7. 4. a. 9 b. 45ej., a. (–3;1). Por 8. 5.

9.

2; y = 1 701 a. 3 x= –1; y = 2 c. x = __ 3 b. 45 861: 1 b. x = 7; y = __ 5 d. Infinit. sol.

14.a.Módulo de un complejo. 9 Complejos conjugados b. 45 6.

___

a. 313 7.

___

b. 10

c. 5

capitulo

d. 310

5

Solución a cargo del alumno.

30. Opuestos de un número y 8. valor absoluto. Por ej., a. 2 + 3i. 1. 9. a. 9 __ Por45 ej., a. 38 . (cos 45° + i. sen 45°). b. 10. 2. __ Por a. 9 ej., a. 1 + 33 i. 3. 11.a. 3 701 __ __ a. –1 b. 2 . 33 + 2i 33 b. 45 +861:

15.c.Adición y sustracción 270 12. 4. Solución a cargo del alumno. a. 9 13.b. 45 Por ej., a. (5;2). 5. 14.a. 3 701 3 __ a. i c. __ + 11 i b. 245+ 861: 3 4 c. 270 21 i b. –2 + 4 i d. 8 – ___ 5 6. 15.a. 9 a. b. 445 c. 10 – 2i e. 8 i b. –10 d. –4i f. –15 – 10i 7. INTEGRACIÓN 13.14.15 a. 3 701 16.b. 45 861: Por ej., a. ±3i. c. 270 17. 8. Por a. 9ej., a. –2 + 7i. 18. 9. Por3ej., a. 701a. (–3;5). b. 45 861: 19. Solución a cargo del alumno. 10. 20.a. 9___ ___ ___ a. 53 b. 337 c. 5 d. 318 b. 345 21. 11. Por 701z1 = 4. a. 3 ej., b. 45 861: 22.c. 270 Solución a cargo del alumno. 12. 23.a. 9 Por b. 45ej., fila 1: 3 – 5i; –3 – 5i. 24. 13. Por a. 3 ej., 701a. z = 8 + 6i o z = 8 – 6i. b. 45 861: 25.c. 270 1 d. x = 3; y = 2 a. x = 1; y = __ 3 7 14.b. x = __ e. No existe. 5 a. 9 c. No existe. b. 45 26. 14.Por ej., a. 14 . (cos 30° + i. sen 30°). a. 9 27.b. 45 __

Por ej., a. 4,5 . 33 + 4,5 i. 13. 28.a. 3 701 b. 8 Por45ej.,861: a. 7;__ . 3 c. 270

( )

29. Por ej., a. 1 – 5i. 30. Solución a cargo del alumno.

229

31. Solución a cargo del alumno. 32. a. 6 b. –2

__

c. –10i

d. 2 . 32 i

33. a. 4; –8 b. 9; –6 c. 8; –6 d. 1; 0

16. Potencias de la unidad imaginaria. Cuadrado y cubo de un complejo 34. Por ej., a. i. 35. 14 a. –8 – ___ 5 i

43. a. 6 . (cos 60° + i . sen 60°) b. cos 50° + i . sen 50° 3 c. __ 2 . (cos 30° + i . sen 30°) 44. a. a = 2 o a = –2 b. a = 3 o a = –3 45. a. 60° b. 220° c. 210° d. 30°

22 – 5i b. ___ 15

31 e. ___ – 2i 6

MENTEACTIVA Solución a cargo del alumno.

5 3 __ c. – ___ 28 – 5 i

19 __ 5 f. ___ + 2 i 4

18. Operaciones combinadas 47. – 17i a. 1______ 29 – 3i b. 6______ 5

36. __ Por ej., a. con 33 . 37. a. 16 – 30i b. –21 – 20i

3 c. __ 2 . (1 + i)

c. 26 – 18i d. 2 – 11i

38. a. 3

17. Multiplicación y división de complejos 39. a. 13

__

c. 12

e. (2 . 32 ;1) __ 39 ___ 23 ___ b. 13 + 6i d. 2 ; 3 f. (1;2 . 36 )

(

)

40. a. 21 . (cos 195° + i . sen 195°) b. 8 . (cos 345° + i . sen 345°) c. 84 . (cos 285° + i . sen 285°) d. 36 . (cos 360° + i . sen 360°) e. 24 . (cos 255° + i . sen 255°) f. 28 . (cos 15° + i . sen 15°) 41. Por ej., fila 1: 2 – 6i; 40. 42. 1 a. – __ 2 –i 3 11 i – ___ b. ___ 10 10 __

– 14i _______ d. –2 25 7 e. ___ + __1 i 20 5 7 i f. – ___ – ___ 10 10

48. 24 Por ej., a. con –2 – ___ 5 i.

b. 7

3 1 __ c. – __ 2 – 2i __ 2 – i) d. 33 . (– __ 3 __

–2 + 32 – (1 + 2 . 32 ) i e. ____________________ 3__ __ __ f. 33 – 2 + ( 36 + 32 ) i ___ 1 – 315 i 14 ___ 16 h. ________ g. ___ 5 – 5 i 4

49. 17 ____ a. ___ + 87 i 26 130 31 ___ 17 b. ___ 5 – 5 i 53 23 c. ____ – ____ i 130 130 3 1 d. __ – __ i 4 2 7 __ e. – __ + 1i 4 2

6i f. ± 3___

c. 1 ± 5i

2

54. Por ej., a. con j. 55. a. 2 – i

9

b. 3 4 __ 3 c. – __ 5 + 5i

32 d. – ___ + ___ i 13 13 5 e. 1 + __ 2i

f. –1 – 6 i

56. a. z2 – 4z + 13 = 0 b. z2 + 49 = 0 c. z2 – 2z + 7 = 0 5 d. z2 – 2z + __ =0 4 2 e. z – 4z + 5 = 0 f. z2 – 4z + 9 = 0 57. c. x = 4; y = 3 a. x = y = 2 b. x = 6; y = –2 d. x = 7; y = 3 58. a. x = –5; y = 3 b. x = –1 o x = 1 c. x = 7 o x = –7

f. –6 5 3 __ g. – __ 2 + 2i 5 __ 7 h. – __ 8 – 4i 77 __ 3 i. – ___ 12 – 4 i 24 ___ 13 j. – ___ 5 + 5 i

50. 15 a. ___ . (cos 220° + i . sen 220°) 4 b. 15 . (cos 20° + i . sen 20°) 5 c. ___ 12 . (cos 120° + i . sen 120°) 3 . (cos 150° + i . sen 150°) d. __ 4

59. 3 x = 3 ∧ y = 5 o x = –10 ∧ y = – __ 2 MENTEACTIVA Solución a cargo del alumno. INTEGRACIÓN 16.17.18.19 60. Por ej., a. 1. 61. a. 31i b. –4 – 12i c. – 66 – 15i d. –18 – 17i

e. –18 + 79i f. –16 + i g. –16 h. i

62. a. –11

c. –10i

e. 3 . (cos 180° + i . sen 180°) 27 . (cos 50° + i . sen 50°) f. ___ 20

51. 1 a. 6 – __ 8i

3 11 i d. ___ – ___ 13 13

5 3 b. – ___ + ___ i 68 68

18 __ 11 e. ___ 5 – 5i

4 1 __ c. – __ 2 + 3i

1 f. –1 – __ 2i

19. Ecuaciones 52. Por ej., a. 5i y –5i.

230

d. –1 ± 2i e. 5 ± __ 2i

d. 2 . (cos 15° + i . sen 15°)

46. a. a = –2 b. Cualquier nro. real menos el –2.

7 ___ d. – __ + 21 i 4 10

53. a. 3 ± 2i b. 2 ± 4i

b. 6

63. a. –5 – 12i b. 18i c. –15 –8i d. 60 – 32i e. –5 + 12i

1 d. – __ 8i

f. 45 – 28i g. –11 + 2i h. –46 + 9i i. 11 – 2 i j. 46 – 9 i

64. a. 4 c. 6 e. 3 b. –21 d. –4 f. –2 o 2

g. –3

65. – 20i e. 13 a. –10 capitulo b. –44 – 12i f. 74 c. 13 – i g. 17 25.d. Opuestos de h. un298 número y –9 – 5i

2

valor absoluto. 66. 1. a. 24 . (cos 45° + i . sen 45°) a. 36 .701 b. (cos 220° + i . sen 220°) b. 45. (cos 861: 130° + i . sen 130°) c. 12 1 c. 270 __ d. . (cos 335° + i . sen 335°) 6 e. 2. cos 170° + i . sen 170° 1 f. a. __ 29 . (cos 85° + i . sen 85°) b. 6 45. (cos 50° + i . sen 50°) g. h. 3 . (cos 110° + i . sen 110°) 3. 67.a. 3 701 8 1 + ___ b. 245– 861: a. 16i e. ___ i 13 13 c. 270 1 1 __ __ b. 3 + 3i f. – 3 – 3 i 4. 4 7 c. g. ___ – ___ i a. 89 – 14i 13 13 b. 45 15 3 ___ ___ d. 3 – 15i h. – 26 + 26 i 5. 68.a. 3 701 c. V a. V b. 45 861: b. F c. 270 69. 6. a. 40° b. 60° c. 50° d. 45° a. 9 70.b. 45 Solución a cargo del alumno. 7. 71.a. 3 701 19 ___ 127 ___ a. –45___ – 1 i e. – ____ + 24 i b. 20861:15 65 65 c. 270 ___ 29 44 ___ 27 ___ b. –2 + 8 i f. 13 – 13 i 8. ___ 39 122 31 ___ c. 17 – ____ g. – ___ + 23 i 17 i 20 20 a. 9 5 1 d. __ –i h. –2 + __ 2i 9. 7 a. 3 701 72. b. 45 861: 13 b. ___ a. –11 – 2i 5 73. __ 10. a. a. 19 ± 33 i b. 5 b. 45± i__ c. 1 ± 33 i 11. 74.a. 3 701 3 16 __ ___ a. b. 45 5i 5 –861: 10 ___ c. –270 ___ + 3 i b. 9

10

__

d. 1 ± __3 . 32 i__ e. { – 33 i;0;33 i }

76. 14.a. x = 2; y = 3 b. x = 8; y = 1 a. 9 77.b. 45 5 a = 4 y b = 5 o a = __ 2 y b = 8.

26. Orden y representación numérica 78. a. 9

b. –1

1. a. 9 AUTOEVALUACIÓN 79.b. 45 __ __ 4 4 a. 2 . 33 i y –2 . 33 i b. –6 2. 80.a. 3 701 13 1 – ___ b. 45 –861: a. 12 3i b. 9 – 19i c. – ___ i 10 10 c. 270 81. 3. a. a. b. 82. 4. a. a. 83.b. c. a.

–16 9 + 30i 45

b. 11 – 2i

3 1 i – ___ – ___ 20 10

3 3 b. ___ – ___ i 20 10

3 701 45 861: 270 + 12i –13

b. 3 + 312 i

16

___

capítulo

10. 12. a. 9 a. A b. 45

4

20.a. Circunferencia 3 701 b. –1; 0; 5.

2. 7. a. (x 9 – 1)2 + y2 = 16 b. (x 45 + 3)2 + (y + 3)2 = 9 c. x2 + (y – 2)2 = 4 8. 3. a. 3 701 3)2 + y2 = 6 a. (x b. 45+861: 9. b. a. 4. b. a. b.

( x – __27 )

2

9 45 x2 + y2 – 2x + 8y – 8 = 0 x2 + y2 + 4x – 10y + 13 = 0

5

10 ___ d. ___ + 62 i 29 29

12. 75.a. 9 2 2 a. b. z45– 10z + 74 d. z – 6z + 16 13 e. z2 – z + ___ b. z2 – 2z + 10 4 13. 2 ___ 466 ____ c. – 10 z + a. z3 701 3 9 b. 45 861: c. 270

X en c. de un número y 30.VaOpuestos valor absoluto. 6. 1. 16; 20; 12. a. 9 7. b. 45 b. B a. C 2. 8. a. 9 3. a. e = 0,6 a. 3 701 b. 45 861:

(y – 1)2 = 8 . (x – 2); x = 0 3 701 2 (x – 4) = 16 . (y + 3); y = –7 45 861: 270 x2 = 4 . (y – 2) (y – 2)2 = 16 . (x – 1) 9 45

c. A

15. 13. Va X en b. y d. a. 3 701 b. 45 861: 16. c. 270 b. e = 1,166 a. e = 1,18 14. 17. 2 y2 a. x9 __ a. __ – 4 =1 9 b. 45 y2

5. c. 2 – 3i

13. 11. a. a. b. b. c. 14. a. 12. b. a. b.

b. B

23. Hipérbola

+ (y + 3)2 = 16

capitulo 21. Elipse

2

– 2) (x – 3) 8. c. (y _______ + _______ =1 9 a. 9 25 2 2 (y – 3) (x + 2) d. _______ + _______ = 1 36 16 9. a. 3 701 MENTEACTIVA b. 45 861: Solución a cargo del alumno.

22. Parábola

6. 1. b. 45 861: a. 2; c. 270–3; 3

7. 11.a. 3 2701 y2 x ___ ___ = 1 a. b. 49 45 +861: 16 – 1)2 (x + 4)2 (y c. 270 _______ b. 25 + _______ = 1 2

5. a. 9 b. 45

9. c. (x 270 – 3)2 + 2)2 (y _______ a. _______ + =1 25 4 2 2 4. (x (y + 3) – 4) _______ + _______ = 1 b. a. 9 9 4 b. 45y2 x2 __ c. _____ + = 1 12,25 4 – 1)2 (x + 2)2 5. (y _______ d. + _______ =1 1 a. 3 701 4 b. 45 861: 10.c. 270 Por ej. a. Centro = (–2;3), 6. a1 = (3;3), a2 = (–7;3), a. 9 (–2;5), b = (–2;1), b1 = ___ 2 ___ b. 45 f1 = (–2 + 321 ;3), f2 = (–2 – 321 ;3).

d. D

b. e = 0,8

2

x b. __ – __ 1 =1 14. 4 2 a. (x 9 + 2)2 (y – 1) c. _______ – _______ =1 9 16 b. 45

d. 13. a. 18.b. a. c.

(y + 2)2 (x + 1)2 _______ – _______ =1 25 9

3 701 45–861: – 3)2 (x 2)2 (y _______ – _______ =1 270 4 16

(y – 1)2 (x + 4)2 b. _______ – _______ =1 25 9

231

19. __ a. Por ej. fila 1: 2 . 33 , (4;0), y2 x2 ___ __ – = 1. 12 4

y2 (x – 2)2 b. ___ – _______ =1 4 16

b. Solución gráfica. 20. ___ y2 x2 ___ a. Sí. b. ___ – = 1 c. 4 . 313 36

16

21. y2 x2 ___ a. ____ – =1 100

36 + 3)2 (x + 1)2 (y _______ b. – _______ =1 9 16

MENTEACTIVA Solución a cargo del alumno. INTEGRACIÓN 20.21.22.23 22. Por ej. fila 1: x2 + y2 = 25. 23. C1: x2 + (y + 2)2 = 9; C2: (x – 2)2 + (y – 3)2 = 25

26. Por ej. a. C: (–3;0), a1 = (–3;4), a2 = (–3;–4), b1 = (–1;0), b = (–5;0), __2 __ f1 = (–3;2 . 33 ), f2 = (–3;–2 . 33 ).

b. A

1x + 2 a. f–1(x) = __ 3 x –2 b. f–1(x) = _____ 4 –3 c. f–1(x) = x_____ 2 __ d. f–1(x) = 33 x e. f–1(x) = x2 – 1

3

c. B

AUTOEVALUACIÓN

2

(x – 1) f. f–1(x) = _______ 2

36. a.

26. Interpretación y análisis de gráficos

37. c.

8.

38. c.

Solución a cargo del alumno.

39. b.

9. Va X en a. y c.

40. b.

10. a. h(x) b. g(x) c. i(x) d. f(x)

capítulo

5

11. Por ej. a. f(x) = –0,5x + 3.

24. Funciones

12. Solución a cargo del alumno.

1. Va X en a. y d.

27. Función lineal

2.

13. y 2 x – 2; __ x + ___ a. y = __ = 1. 3 3 –2

a. Df = ; Im = b. Df = [–2;1]; Im = [–2;3] c. Df = ; Im = [2;+') d. Df = [–4;–2) ∪ [0;3]; Im = [–2;0) ∪ (2;5] e. Df = ; Im = {2} f. Df = +; Im = (–';0) 3.

1 – __ 2

{ }

a.

b.

2 f. __ 5 ;+'

(

4.

)

g.

y

1 x + 1; __ + __ x b. y = – __ 1 4 = 1. 4 y

c. y = 2; __ 2 = 1. y

1 x __ __ d. y = __ 2 x – 1; 2 + –1 = 1.

14. x + a. y = ___ –9 x b. y = ____ 0,05

– {–5;5} c.

d. (–';–3) ∪ (3;+')

e. h.

+

25 ___ 1 ; ____ a. __ ; 1 9 __ 126 30 2 1 b. – 3__ – 1; ∅; – __ 2 2 __ __ c. ∅; 0; 1 e. 2 . 35 ; 2; 35 1 1 f. 4; 1; __ d. e; __ e; 1 2

31. (x – 2)2 (y – 1)2 a. _______ – _______ =1 4 16

232

7.

35. b.

30. Va X en (5;–2), (1;–3).

(y + 3)2 (x – 1)2 b. _______ – _______ =1 1 6

Va X en a.

_____

34. a. F

– 1)2 (x – 2)2 (y b. _______ + _______ =1 25 9

29. a. x2 = –4 . (y – 3) b. (y – 2)2 = 8 . (x – 3) c. (x + 4)2 = –6 . (y – 1)

6.

33. Va X en b., c., d. y e.

27. y2 (x + 4)2 a. ___ + _______ =1 9 36

28. a. (y – 2)2 = –8 . (x – 1). D: x = 5. b. (x – 4)2 = –6 . (y – 2). D: y = 5.

25. Función inversa

– 3)2 (x – 1)2 (y _______ c. _______ – =1 1 9

24. a. (x – 1)2 + (y – 3)2 = 12 b. (x + 4)2 + (y + 2)2 = 16 c. x2 – (y – 5)2 = 9 25. Va X en b. y c.

MENTEACTIVA Solución a cargo del alumno.

32. (x – 3)2 (y + 1)2 a. _______ – _______ =1 25 9

y ___ =1 –3 y ___ + 0,2 = 1

y x ___ c. __ =1 2 + __ 4 –3 y y x + __ d. ___ = 1 e. ___ =1 0,8 2 –4

15. Por ej. a. con f. y j. 16. y y x x + ___ ___ __ a. ___ =1 –5 + __ 5 = 1 b. __ –3 6 –3 5

5. Por ej. a. Segunda opción.

17. a. y f. ⊥ ; c. y d. //.

18. –2x + 11 b. y = –5x + 9 a. y = capitulo

2

19. 1 x –de 1 25.a. Opuestos y y = – __ 1 c.uny número = – __ 2x 4

valor absoluto. b. y = 2x + 1

1 d. y = __ 2x + 2

1. a. 3 701 MENTEACTIVA Solución b. 45 861:a cargo del alumno. c. 270

53. 14.a. S: {(1;–3)} b. a. No 9 tiene solución. 15 __ b. 45 (0;1); ___ c. S: ;– 1

{

24. 5. a. y = x2 + 2x – 3 a. 3y = 701–2x2 + 6x – 4 b. b. y45= 861: c. 4x2 – 16x + 12 c. 270 __1 3 d. y = – 2 x2 + x + __ 2 6. e. y = –4x2 + 4 a. y9= 3x2 – 6x + 5 f. b. 45 25. 7. Por ejemplo, a. y = __1 x2 – x – 3; 4 a. 3 701 1 861: 2 . (x – 2) – 4; yb.=45__ 4 c. 270 1 . (x + 2) . (x – 6) y = __ 4 8. 26.a. 9 Por ej., a. Mín = (2;–4); 9. crece: (2;+'); decrece: (–';2). a. 3 701 MENTEACTIVA b. 45 861: Solución a cargo del alumno.

31. Sistemas de ecuaciones 10. mixtas a. 9 49.b. 45 La cuarta opción. 11. 50.a. 3 701 a. b. Dos. 45 861:c. Dos. b. c. Dos. 270 d. Uno.

e. Ninguno. f. Dos.

51. 12. 4 4 __ __ Por a. 9ej. a. Dos: k < 5 ; una: k = 5 ; b. 45 4 ninguna: k > __ 5. 13. 52.a. 3 701 Solución b. 45 861:a cargo del alumno. c. 270

4

)}

f. S: {(0;1);(–1;4)} 1. MENTEACTIVA a. 9 Solución a cargo del alumno. b. 45

28. Función cuadrática

20. 2. Solución a cargo del alumno. 9 a. b. 45 21. 3. Por ej. a. Tercera opción. 22.a. 3 701 b. V c. F d. V e. V a. F45 861: b. c. 270 23. 1 4. Por ej. fila 1: __ 2 ; 0; –8; (0;–8); a. x =9 0; 4 y –4; (0;–8); [–8;+') b. 45

(4

No tiene solución. 26.d. Orden y representación e. No tiene solución. numérica

INTEGRACIÓN 29.30.31 2. 54.a. 3 701 c. F a. V b. 45 861: b. V c. 270 55. 3. Solución a cargo del alumno. a. 9 56.b. 45 Por ej. a. con g. 4. 57.a. 3 701 ___ a. x45=861: 2 ± 310___ b. b. x270 = –2 ± ___ c. 310 c. x = 2 ± 310___i 5. d. x = –2 ± __ 310 i a. 9 _______ 2 ± 37 e. x = 3 b. 45 1 f. x = 1 o x = __ 3 ___ 6. ± 379 a. 3x = 701–8_______ g. 5 b. 45 861: 1 h. x = –3 o x = – __ c. 270 5 58. 7. a. b9 < –10 o b > 10 b. b 45= –10 o b = –10 c. –10 < b < 10 8. 59.a. 3 701 a. SCD b. 45 861: b. SI

61. capitulo Solución a cargo del alumno.

5

62. 30. Opuestos de un número y a. F absoluto. b. V c. F d. F e. F valor

2.

c. S:

{ ( 0;__31 ) }

d. S: a. 9 {(0;3)} 3. e. No tiene solución. f. S: {(5;24);(–1;–6)} a. 3 701 b. 45 861:

1 1 __ __ 2 5. b. y = 3 . (x + 1) + 3 ; a. 3 701 1 . (x + 2)2 + 2 yb.=45– __ 3861: c. 270 66. 21x + 13 = y 6. a. 24x – y = 8 b. x = 7; y = 160 a. 9 67.b. 45 y = –x2 + 2 3 __ 1 a. y = –x2 + 4x – 4 b. __ 2 ;– 4 7. a. 3 701 b. 45 861: c. 270

{

{

(

capítulo

8.

)

6

a. 9

32. Función polinómica 1. 9. a. f(x) = x2 . (x + 2) 3 701 b. f(x) = (x – 3)3 . (x + 3)2 45 861: c. f(x) = (x + 1) . (x + 2) . (x + 3) d. f(x) = x4 . (x – 2)3 10. 2. a. 9 Infinitas. b. 45 3. 11. Por 701a. C+ = (–3;2) ∪ (2;+'); a. 3 ej. – C = b. 45(–';3). 861: c. 270 4. 12.Va X en a. y c. a. 9 33.b.Análisis de la función poli45

nómica c. SCI

60. 9. a. Cintia $9 040 y Eduardo $5 260. 9 b. 70°; 45 70°; 40° o 80°; 50°; 50°. c. 10 y 1. d. 5 adultos y 9 niños.

63. 1. a. S: 9 {(0;3);(–4;11)} b. No 45 tiene solución.

c. 270AUTOEVALUACIÓN 64. b. 4. 65.a. 9 5 5 5 a. y45= – __ x – 1,25; y = __ x – __ b. 4 4 4

5. 13. Solución a. 3 701 a cargo del alumno. b. 45 861: 6. c. 270 Solución a cargo del alumno. 14. 7. a. 9 a. b. f(x) 45 = (x – 2) . (x + 1)2 b. f(x) = (x + 2) . (x – 3) 14.c. f(x) = (x – 1)2 . (x + 1)2 d. f(x) a. 9 = (x + 2) . (x – 1) . (x – 3) e. f(x) b. 45 = x . (x – 2) . (x + 2) __ f. f(x) = (x + __ 1)2 . (x – 1 + 2 . 32 i) . (x – 1 – 2 . 32 i) 13. a. 3 701 8. b. 45 861: Por ej. a. (2;+'). c. 270 9. Solución a cargo del alumno.

233

INTEGRACIÓN 32.33 10. Solución a cargo del alumno. 11. a. No. b. Sí.

c. Sí. d. No.

e. Sí. f. No.

12. Solución a cargo del alumno. 13. Solución a cargo del alumno. 14. Solución a cargo del alumno.

39. a.

MENTEACTIVA Solución a cargo del alumno.

b. c.

4 4 – __ ; x = __ . 3 3 – {–1}; x = –1. – {2}; x = 2.

d.

1 ; x = – __ 1. – – __ 3 3

36. Función homográfica 27. Solución a cargo del alumno. 28. Por ej. a.

– {3}.

29. 3 1 __ Por ej. a. A. V.: x = __ 2 ; A. H.: y = 2 . 30. Solución a cargo del alumno.

15. Por ej. a. Par.

31. 2 a. – __ 5 b. 0

16. a. C+: (–';a) ∪ (a;b). b. Positivo. c. Sí. 17. Solución a cargo del alumno. 18. a. F b. V c. V d. F e. F f. F g. F 19. a. f(x) = x . (x – 4) b. f(x) = x2 . (x + 3) c. f(x) = x . (x – 1) . (x + 1) d. f(x) = x2 . (x – 1)2 e. f(x) = (x + 2) . (x + 1) . (x – 3) f. f(x) = (x + 3) . (x + 2) . (x – 2)

34. Función racional 20. Solución a cargo del alumno. 21. Por ej. a. con Df =

26. Solución a cargo del alumno.

– {–2}.

22. Por ej. a. 4 ; I = – {0}; Df = – – __ 3 m 4 __ A. V.: x = – 3 ; A. H.: y = 0.

{ }

35. Representación gráfica de funciones racionales 23. Solución a cargo del alumno. 24. Solución a cargo del alumno. 25. Solución a cargo del alumno.

32. a. F b. V

4 c. __ 7

1 e. __ 2

5 d. – __ 4

1 f. – __ 2

c. V d. F

e. F f. F

g. V h. V

MENTEACTIVA Solución a cargo del alumno. 33. Solución a cargo del alumno. INTEGRACIÓN 34.35.36 34. a. b. c.

– {1} – {2;–2} – {–1}

d. e. f.

{ }

{ }

40. a. F; V; V; F; F; F. b. V; F; V; F; V; F. 41. a. a = 2; b = 6. b. a = 1; b = 4. 1 c. a = –4; b = __ 2. d. a = 3; b = 3. 42. a. 3 b. 1 c. 2 d. 3 e. 2 y 3 f. 3 43. – 2. Por ej. a. x_____ x–3 44. Solución a cargo del alumno. AUTOEVALUACIÓN 45. c. 46. a. 3 b. No tiene.

c. 2.

47. c. 48. c. 49. b.

– {–3} – {–1}

35. Solución a cargo del alumno.

50. a. (0;–2) 51. a.

3 b. __ 2

3 2 c. __ – {2} b. y = __ 2 ;0 3

( )

36. Solución a cargo del alumno. 37. a. x = –5; y = 0. b. x = 0; y = 0. 1 1 __ c. x = – __ 2; y = 2.

d. x = –4; y = 8. 2. e. x = –2; y = – __ 3

capítulo

37. Función exponencial 1. Solución a cargo del alumno. 2.

2 ; y = 0. f. x = __ 3

38. Por ej. a.

7

1 a. k = –1; a = __ 3 b. k = 1; a = 5 c. k = –1. Para a, inf. sol. 3 d. k = __ 2; a = 2

– {6}. 3.

Por ej. a. f(0) = 1; g(0) = 5; 1 1 _____ h(0) = ___ 25 ; i(0) = 3 125 .

234

4.

39. Función logarítmica

2

= 4x+2; h(x) = 4x–1 a. g(x) capitulo 1 b. g(x) = __ 3

( )

x+1

1 ; h(x) = __ 3

( )

x–3

25. Opuestos de un número y 5. valor absoluto. Solución a cargo del alumno. 1. 6. a. 3 701 Solución b. 45 861:a cargo del alumno. c. 270 7. 2. Por ej. a. con y = –3. a. 9 8. b. 45 Solución a cargo del alumno. 3. a. 3 701 MENTEACTIVA Solución b. 45 861:a cargo del alumno. c. 270

numérica

9. 4.

d. V e. V f. V

2 __ 9 19 e. – ___ 6 2 __ f. – 3

7 __ 6. b. 2 a. __ 59 c. b. 245

9

g. – ___ 10 11 h. – __ 2 21 ___ i. – 2

12. 7. a. 3 6 701 b. 3 c. 3 d. 3 e. –4 f. 5 a. b. 45 861: 13.c. 270 a. 0,60206 e. 2,7724 f. –2,7724 8. b. 2,40824 c. 3,61236 g. 4,1586 a. 9 d. –0,60206 h. 2,7724 9. 14.a. 3 701 1 a. a45+ 861: b d. __ b. 2 . (a + b) 3 e. __ 2b

b. 2a + b

1 1 __ 10.c. 3a + 2b f. __ 2a + 2b a. 9 15.b. 45 6 a. log ____ c. log 27 125 11. b. d. log 21 a. 31 701 b. 45 861: 16.c. 270 Solución a cargo del alumno. 12. 17.a. 9 5 a. b. __ 45. log5 x c. log 2x 2

13.b. a. b. c.

2

log2 ( __x1 ) 3 701 45 861: 270

20. 1. Solución a cargo del alumno. a. 9 21.b. 45 Solución a cargo del alumno. 2. 22.a. 3 701 Por fila 1: (0;+'); x = 0. b. 45ej.861: c. 270 23. 3. a. Solución a cargo del alumno. a. f9–1(x) = __1 x – 2 b. 4 b. 45 c. Solución a cargo del alumno. 4. 24.a. 3 701 a. Solución a cargo del alumno. b. 45 861: x+3 –1 b. f (x) = e c. 270 c. Solución a cargo del alumno. 5. 25.a. 9 a. b. V 45 b. F c. V d. V e. F f. F

( )

38. Logaritmos

Por9 ej. a. 3. a. b. 45 10. 5. a. F b. V c. F a. 3 701 11.b. 45 861: 7 __ a. d. c. 2270

18. 14. Solución a cargo del alumno. a. 9 b. 45 19. del alumno. 26.Solución Orden ay cargo representación

d. log5 ((x – 2) . x)

26. 6. Por a. 3 ej. 701fila 1: (0;+'); ; 1 hacia arriba; x = 0. b. 45 861: c. 270 MENTEACTIVA 7. Solución a cargo del alumno. a. 9 . . b. 45INTEGRACIÓN 37 38 39 27. 8. Solución a cargo del alumno. a. 3 701 28.b. 45 861: a. d. No tiene. e. No tiene. 9. b. (0;+') 1 a. 9 f. 0;__ c. 7 b. 45 29. a. V c. F e. F g. V b. F d. F f. V

( )

capitulo

5

30. y=2 =0 30.a. Opuestos de d. uny número y b. y = –3 e. y = 3 valor absoluto. 3 __

c. y = – 2 f. y = 4 1. 31. a. Por9 ej. a. (–';0). b. 45 32. 2. Solución a cargo del alumno. a. 9 33. 3. a. 3 f(x)701b. g(x) c. h(x) d. p(x) a. b. 45 861:

34.c. a. 4. 35. a. a. b. b.

270 y = 2x + 1 9 3 c. 4 45 2 d. 2

b. y = 2x+3 e. 3 f. 2

g. 4 i. –1 h. 3 j. –2

5. 36. a. 3 701 a cargo del alumno. Solución b. 45 861: 37.c. 270 7 8 a. 16 c. __ e. – __ 2 3 6. 8 1 b. d. – __ f. __ a. –6 9 3 6 b. 45 38. 7. Por ej. a. (1;+'). a. 3 701 39.b. 45 861: a. =3 c. (–1;+') c. a270 b. b = 1 d. Sí. 8. 40.a. Ecuaciones exponenciales 9 40. 9. Por ej. a. con x = 5. a. 3 701 41.b. 45 861: Por ej. a. 6. 42. 10. a. a92x b. ax+2 b. 45 43. 11.a. x = 3 701– __1 a. 3x = b. 2 b. 45 861: c. x = 2 c. 270 44. 12. a. a. b. b.

x9 = –1 x45= 0

c. a9x

d. a21x

d. x = –4 1 e. x = __ 4 f. x = –1 c. x = –12 d. x = –2

45. 13. Por a. 3 ej. 701a. 3. b. 45 861: 46.c. 270 a. x = –6 14.b. x = –1 c. a. x9 = 2

d. x = 4 e. x = –3 f. x = –2

b. 45 47. a. 14. x = 3 b. a. x9 = 1 c. b. x45= 3 d. No tiene. 13.e. x = 1 f. tiene. a. No 3 701 g. x = 1; x = 2 b. 45 861: h. x = c. 270 1; x = log3 2 i. x = 2; x = 3 j. x = 1 k. x = 1; x = –1 l. x = –2 m. No tiene. n. x = –1

235

41. Ecuaciones logarítmicas 48. Por ej. a. con x = 1.

60. a. x = 4 b. x = 2

49. a. x = 8 1 b. x = __ 5 c. x = 7

61. a. x = 1 d. No tiene. b. x = 2 e. x = 0 1 c. x = 2; x = 4 f. x = __ 2 g. x = 1; x = 2

d. x = 5 e. x = e2 1 f. x = – __ e

50. 43 a. x = ___ 7 b. x = 4 c. x = 3 51. 3 a. x = __ 2 b. x = 0,001 5 c. x = __ 3

d. No tiene. e. x = 2 f. x = 5

f. x = 5

65. x = 243

d. x = 25 e. x = 4 f. x = 242

54. 1 a. __ 2; 4

g. 125

e. –1 + 32

h. 16 i. 32__ __ j. –33 ; 33 11 k. __ 3

f. No tiene.

l. 729

MENTEACTIVA Solución a cargo del alumno.

236

2. a. 720° d. 270° b. 30° e. 225° c. 114° 35’ 30’’ f. 495°

5.

7.

72. a. (–1;+') b.

3 f. __ 2 π

c. F d. F

e. F f. F

g. V h. F

i. V j. V

___

ab __ . Por ej. a. ___

___

___ a. Por ej. ao = ab__. cos ^ _.

__ bc . b. Por ej. ac = ______

sen ^ _

__ __ ^ c. Por ej. bc = ac . cos `.

8.

68. 1 x = 3; x = __ 3

58. Por ej. a. Una.

5 d. __ π 9

bc

1 e. x = __ 4

g. x = 1

71. a.

5 b. __ π 4

6.

1 ; x = 64 f. x = –4 c. x = __ 4

57. Sí.

3 e. __ π 4

Por ej. a. 0,7193 1 d. x = ___ 25

b. x = 64

70. a. x = 7

7 c. __ π 4

43. Razones trigonométricas d. x = 4 e. x = 32 f. x = 9

67. 1 a. x = ___ 25

56. Por ej. a. x = 1.

2 π a. __ 3

4.

1 ; x = __ 1 g. x = __ 3 9

55. Por ej. a. 7.

59. 3 a. x = __ c. x = –4 2 b. x = –3 d. x = 1 e. x = 1; x = 2 f. x = –3; x = 1

Por ej. a. Va X en 60°.

a. V b. F

66. a. x = 6 b. x = 6 4 c. x = __ 3

69. a. x = 243 b. x = –1,37 c. x = 1,71

INTEGRACIÓN 40.41

1.

3.

63. x = –2

8

42. Sistema de medición de ángulos

6 b. x = __ 5

64. Por ej. a. x = 23.

53. a. x = 64 b. x = 8 c. x = 27

__

62. a. x = 3

11 d. x = __ 3 e. No tiene.

52. 7 Por ej. a. __ 2.

b. 5 c. 8 d. –49; 49

capítulo c. x = –3 d. No tiene.

___

3 . 310 Por ej. a. sen ^ _ = ______ ; 10 ___ 10 cos ^ _ = 3___ ; tg ^ _ = 3. 10

d. x = 1,5 e. x = 5,23

b. x = 1; x = –5

AUTOEVALUACIÓN

c. x = –1 e. (0;–3) d. (7;0)

73. b. 74. a. x = 2 c. x = –93; x = 27 b. x = 1; x = 5 d. x = 36

9.

Por ej. a. ^ _ = 66° 25’ 19’’; sen ^ _ = 0,9165; tg ^ _ = 2,2913.

10. Solución a cargo del alumno. MENTEACTIVA Solución a cargo del alumno.

44. Valores exactos y aproximados 11. 1 Por ej. a. – __ 2. 12. π Por ej. a. con sen __ 8.

13. __ 3 a. 3___ 2

2

capitulo e. 0

__

2 b. – 3__ 2

__

f. –33 __

__ 25. Opuestos de un3___ número y c. –3absoluto. 3 g. 22 valor __

2 h. – 3__ 1. d. – 1/2 2 a. 3 701 45.b.Ecuaciones trigonométricas 45 861: 14.c. 270 19 π ___ Por ej. a. ___ 12 y 12 π. 2. 15.a. 9 b. 45ej. a. con No existe. Por

3. 16. a. a. b. b. c.

5 π 701 __ 3 ; __ π 3 3861: 45 4 5 __ __ π; 3 π 270 3

5 2 __ __ 4. c. 3 π; 3 π 97 π a. ___ d. 12 b. 45

π __ e. __ ; 3π 4 4 4 5 π __ f. __ ; 2 π; __ π; __ π 3 3 3 3 g. π; 0

h. 0

INTEGRACIÓN 42.43.44.45 5. 17. a. 3 701 3 Por fila 1: __ π. b. 45ej.861: 4 c. 270 18. 1 __ 6. Por ej. a. – 2 . a. 9 19.b. 45 a. 34° 46’ 54’’ d. 84° 39’ 45’’ 7. b. 71° 8’ 35’’ e. 28° 35’ 8’’ a. 3 701 c. 75° 54’ 35’’ f. 11° 18’ 36’’ b. 45 861: 20.c. 270 c. F e. V g. F a. F 8. b. V d. F f. F a. 9 21. 9. Solución a cargo del alumno. a. 3 701 22.b. 45 861: _________ Por ej. a. ±31 – sen2 ^ _. 10. 23. a. 9 Solución a cargo del alumno. b. 45 24. π π 2 π b. –cos __ 11.a. –sen __ c. –tg __ 3 4 a. 3 7019 b. 45 861: 25. c. Por270 ej. a. π; 2π; 0. 12. 26. a. 15°94’ b. 45 27. 13.a. a. b. b. c.

F c. F 3V 701 d. F 45 861: 270

e. V f. F

f. F

28. 14.a. a. b. b.

π __ __ ; 5π 4 4 93 __ π 45 4 π 2π ; __ c. __ 3 3 y representación 26. Orden 5 13 numérica π; ___ π d. ___ 24 24 7 11 __ __ 1. e. π; 6 π; 6 π a. π9 __ ; 3 π; 0; 2π f. __ b. 2452

46. 2. Triángulos rectángulos

29.a. 3 701 __ Por a. ac = 12,73 cm; b. 45ej.861: ^ a =270 34° 26’ 20’’; ^ c = 55° 33’ 40’’. c. 30. 3. __ a. 12 b. 54,93 cm 9 . 33 cm b. 45 31. d. 2,20 m 4. a. 5,20 m b. 70122’ 37’’ e. 15,66 m a. 325° c. 33’ 16’’ b. 60° 45 861: c. 270 32. c. 3,70 m 5. a. 2,75 m b. m d. 413,54 m 9 a. 13,05 b. 45 MENTEACTIVA 6. Solución a cargo del alumno. a. 3 701 47.b.Teoremas 45 861: del seno y del coseno c. 270 33. 7. Solución a cargo del alumno. a. 9 34.b. 45 a. No. c. Sí. e. Sí. d. No. f. No. 8. b. No. a. 3 701 48.b.Triángulos oblicuángulos 45 861: 35. ___ 9. Por ej. a. ab = 54,59 cm; __ a. 9= 56,37; ^ ac b = 60°. b. 45 36. ___ __ a. = bc = 62,30 cm; ___ ad __ capitulo ab = cd = 43,71 cm. b. P = 212,5 cm c. Diag ___ = 21 cm 30.d.Opuestos ab = 35,23de un número y

5

valor absoluto.

37. 1. a. A: 3 402,75 m; B: 3 248,99 m. a. 9 b. Dist.: 16,81 m. b. 45 38. 2. a. 1 254,38 m b. 207,73 m a. 9 38’ 9,9’’ c. 79° 3. d. 491,26 m o 864,31 m a. 3 701 b. 45 861:

c. 270 MENTEACTIVA Solución a cargo del alumno. 4.

a. 9 INTEGRACIÓN 46.47.48 39.b. 45 Por ej. a. ^ n = 38° 4’ 36’’; ___ 5. ^ p = 51° 55’ 24’’; np = 19,94 cm. a. 3 701 40.b. 45 861: c. 270 51’ a. 34° c. 63° 12’ 42’’ b. 48° 21’ 59’’ d. 29° 55’ 35’’ 6. 41.a. 9 b. 45 a. 273,21 m b. 54,16 m 7. 42. a. 701 Por3ej. Per. Trián abc = 45,89 cm. b. 45 861: 43.c. 270 b. F c. F d. V a. F 8. 44.a. 9 a. No se verif. b. Se verif. 9. 45.a. 3 701 b. Por45ej.861: a. ^ c = 58°; __ __ bc = 9,29 cm; ac = 3,53 cm. 10. 46. 9 cm2. b. 67,04 cm. a. 62,35 b. 45 47. 11.Por ej. a. ^ s = 104°; __ __ a. 3 rs = 701 19,46 cm; rt = 25,82 cm. b. 45 861: 48.c. 270 b. 110,58 m. a. 15,79 m 12. 49.a. 9 b. 45ej. a. Á: 109,3 cm3; Por P: 43,84 cm. 13. a. 3 701 AUTOEVALUACIÓN 861:b. –cos 30° y cos 210°. 50.b.a.45 180°; c. 270 51. a., b., c. y d. 14. 52.a.b.9 y d. b. 45 53. a. y b. 14. 54.a.c.9 b. 45 55. b. 13. a. 3 701 b. 45 861: c. 270

237

capítulo

9

49. Estadística 1. Niv. in.: 72%; prim.: 90%; secun.: 198%. 2. a. Por ej. Básquet: 22,06%. b. Por ej. Handball: 14 alumnos. 3. a. 17 pers. b. 70,59%

c. 29,41% d. 84° 42’ 21,18’’

17. a. Solución a cargo _ del alumno. b. A, B y C tienen x = me = mo = 9. c. La muestra de A. 18._ x = $6 773,3; m = $1 218,45.

34. La muestra A.

20. a. Pachanga.

b. Balvanera.

35. Solución a cargo del alumno.

a. Por ej. fila 1: 49.

22. Por ej. a. Ning. de las ant.

5. b. 60%

6. Solución a cargo del alumno.

51. Parámetros de posición 7. 5.° año A. 8. f4 = 40; f6 = 49 9.

33. a. 80 kg; 140 kg; 200 kg; 260_ kg; 320 kg. b. x = 218,57 kg; me = 217 kg. c. m = 115,84 kg.

b. Corola.

Solución a cargo del alumno.

4.

32. Cv(A) = 16,91% y Cv(B) = 16,94%. Muestra A.

19. a. Pistilo.

21. _ a. x = 7,025; m2 = 5,9244; m = 2,4340. b. Cv = 34,65%

50. Intervalos de clase

23. a. Naranja. b. Amarilla.

36. En la muestra A. 37. a. m1: verde; m2: azul; m3: roja. b. La roja. c. No.

53. Combinatoria c. Verde. d. Azul.

24. _ a. x = 170,83 cm; m = 7,42. b. Cv = 4,34% MENTEACTIVA Solución a cargo del alumno.

38. a. 240 b. 201 600

1 c. __ 5 d. 2 730

39. a. x = –205

1 b. x = ___ 20

40. a. 312 + 156n b. 480n2 + 1 200n + 720

10. _ Por ej. a. x = 27,38 cm.

INTEGRACIÓN 49.50.51.52 25. a. Solución a cargo del alumno. b. Aprox. el 48,22%. c. 316 camisas; aprox. el 70,22%.

11. _ Por ej. b. x = 79,76 kg; me = 79,06 kg; mo = 75.

26. _ a. x= 2,82; mo = 4. b. A ≅ 16,115. c. 47,48%; 714.

12. _ Por ej. a. x = 8,0748.

27. Solución a cargo del alumno.

13. Solución a cargo del alumno.

28. No.

44. Por ej. a. 24.

14. Solución a cargo del alumno.

29. _ a. x = 27,36 b. 23 c. La original 26 y la otra, 27.

45. a. 840

15. _ Por ej. a. x = 36,433; me = 36; mo = 36.

30. Solución a cargo del alumno.

a. me = 12; mo = 15 b. me = 17,4; mo = 23,5

16. _ Por ej. a. x = 35,20. MENTEACTIVA Solución a cargo del alumno.

238

52. Parámetros de dispersión

31. _ a. x_ = 6; m2 = 9,6; m = 3,1. b. x _= 36; m2 = 9,6; m = 3,1. c. A x se le suma la constante y m no varía.

41. a. 10 b. 35

+ 3a + 2 c. a__________ 2 d. 21

42. a. x = 55

b. z = 9

2

54. Permutaciones, variaciones y combinaciones 43. a. 23

b. 479 001 600

b. 2 184

46. a. n = 2 47. a. 720 48. a. 120 b. –5 395

c. 6 840

b. n = 8 b. 60

c. 600 c. 7 179 689 d. ____ 6

49. a. 9

INTEGRACIÓN 53.54.55.56 b. 5 040

c. 56

50. a. 19 683 b. 20 412 51. a. 625 000

c. 551 853

65. a. 101 . (n + 2) . (n + 4).

b. 40.

66. a. x = 2,5. b. y = 5. c. z = 0; no es solución.

b. 600 000

52. Por ej. a. 350. MENTEACTIVA Solución a cargo del alumno.

67. a. 5 040; 35. b. 6 840. c. 303 600; 12 650. d. 680. e. 45. f. 129 729 600; 109 771 200.

55. Probabilidad 53. Solución a cargo del alumno.

68. Por ej. a. Ning. de las anteriores.

54. Por ej. P(frut-frut) = 0,6897.

69.

55. 3 a. __ 4

70. 12 a. ____ 380

3 b. ___ 52

3 c. __ 8

1 ____ 720 64 b. ____ 380

56. Solución a cargo del alumno.

71. 1 a. ____ 216

56. Sucesos y probabilidad condicional

72. 4 a. ___ 91

34 b. ____ 455

73. 2 a. __ 5

b. 0,2112

57. a. Sí; 1. b. Sí; no se puede saber. 58.

74. a. 0.

3 __ 8

59. 20 ___ a. ___ ; 25 29 36

1 b. __ 11

60. a. Solución a cargo del alumno. 1 b. __ 2

1 c. __ 2

215 c. ____ 216

3 b. ____ 216

b. 0,57.

c. 0,081.

75.

4 __ 3 __ 7; 7

76. a. 0,78

b. No.

d. 0

61. a. 0,1385; 0,35 b. 0,2727; 0,3375

77. 671 a. _____ 1 296

5 b. __ 6

( )

3

5 c. __ 6

( )

3

62. Por ej. P(casa) = 0,40.

AUTOEVALUACIÓN 78. a. De 57 a 58. c. Ning. de ant. b. 57 d. 14,35%

63. a. 0,0000027; 0 b. Por ej. P(todos) = 0,4096.

79. d.

64. Por ej. a. 13%. MENTEACTIVA Solución a cargo del alumno.

80. 5 a. ___ 33 81. 4 a. ___ 17

b. Ning. de las ant. 18 b. ___ 21

239

5 MATEMÁTICA

FIN

¿PARA QUÉ

SIRVE? Índice NÚMEROS REALES. . . . . . . . . . . . . . . . . . . . . . . . . . . 2 SUCESIONES. . . . . . . . . . . . . . . . . . . . . . . . . . . . . . . 3 SUCESIONES ARITMÉTICAS Y GEOMÉTRICAS. . . . . . . . . 4 NÚMEROS COMPLEJOS. . . . . . . . . . . . . . . . . . . . . . . . 5 CÓNICAS. . . . . . . . . . . . . . . . . . . . . . . . . . . . . . . . . . 6 FUNCIÓN INVERSA. . . . . . . . . . . . . . . . . . . . . . . . . . . 7 FUNCIONES CUADRÁTICAS. . . . . . . . . . . . . . . . . . . . . . 8 TEOREMA DE BOLZANO. . . . . . . . . . . . . . . . . . . . . . . . 9 POLINOMIOS. . . . . . . . . . . . . . . . . . . . . . . . . . . . . . . 10 FUNCIONES HOMOGRÁFICAS . . . . . . . . . . . . . . . . . . . 11 FUNCIONES EXPONENCIALES Y LOGARÍTMICAS. . . . . . . 12 TRIGONOMETRÍA. . . . . . . . . . . . . . . . . . . . . . . . . . . . 13 ESTADÍSTICA. . . . . . . . . . . . . . . . . . . . . . . . . . . . . . . 14 COMBINATORIA. . . . . . . . . . . . . . . . . . . . . . . . . . . . . 15

¿¿PPaarraaqquuééssirirvvee..?. capítulo 1 contenido

1

Números reales Si tuvieran que decir 10 números entre 0 y 9 “al azar”, ¿dirían, por ejemplo, “1, 1, 1, 1, 2, 2, 2, 2, 3, 3”? ¿O “1, 1, 1, 1, 1, 1, 1, 1, 1, 1”? ¿Y si tuvieran que decir 100 dígitos al azar? ¿Están pensando en algo para decirlos? ¿Qué significa al azar? Si buscamos en el diccionario, la definición es: “sin orden, sin planeamiento, aleatoriamente”. Con lo cual, si para decir cien números al azar pensaron en algo, lo más probable es que no hayan sido dichos al azar. ¿Qué estrategias podemos tener para ir diciendo números “al azar”? Una posibilidad es elegir un número (no entero) e ir diciendo los dígitos que están después de la coma. Ahora ¿qué pasa si el número que elegimos es un número racional? ¿Es una secuencia de números al azar? ¿Nos damos cuenta de esto enseguida? ¿De qué depende? Si un amigo les va diciendo las cifras que aparecen en el desarrollo decimal de un número racional, ¿podrían anticipar en algún momento qué va a decir, sin saber el número en cuestión? Esto dependerá un poco de qué número sea; si el núme1 = 0,3333333333..., entonces será muy fácil anticiparse. Ahora si el número ro es __ 3 1 es _________________ , la tarea será más compleja. ¿Cuántos números tendremos que 999999999999999 escuchar para poder anticiparnos correctamente? Entonces si el número que elegimos es racional, por más que tenga infinitas cifras decimales, sabemos que tienen un período y esto nos daría un ciclo. Por eso, si escuchamos atentamente, podríamos descubrir este ciclo y entonces, a partir de ahí predecir los números siguientes. Para solucionar parcialmente esto, podemos elegir un número racional cuyo período sea muy largo, con un ciclo de 1 000 dígitos, por ejemplo: al decir 100, 200 o 500 de estas cifras no será descubierto nuestro patrón. Esto es en parte lo que se conoce como números pseudoaleatorios, porque si bien con estos ciclos grandes alcanza para algunas aplicaciones, no es un número completamente al azar. De esta manera, vemos que no todos los números son racionales, pues podríamos generar números con cifras decimales al azar, que no tengan ningún ciclo. Ahora, no todos los números no racionales tienen el mismo “grado de azar”. Por ejemplo, el número 0,101001000100001000001000000100000001... no es un número racional pues no tiene período, pero uno podría predecir cómo continúa la secuencia. En cambio, es más difícil predecir la secuencia de / que es 3,141592653589793238462643383279502884197169... ¿Encuentran algún patrón en este número? Hemos encontrado entonces una estrategia para decir números al azar: los números irracionales nos sirven para esto, aunque como hemos visto, algunos funcionan mejor que otros. De todas formas, es una herramienta teórica, pues ¿cómo hacemos para saber los decimales de /? En conclusión, los números racionales no alcanzan para llenar toda la recta, pues quedarían estos agujeros de “números al azar”, ya que los racionales son esos números “con período, con ciclo, con plan, con orden” y eso no es todo lo que hay. Llenar la recta agregando los números irracionales es lo que facilita el estudio de funciones y es su verdadera utilidad.

Actividades 1. Escriban 25 números entre 0 y 9 utilizando algún patrón para generarlos. 2. Intercambien con un compañero los números que escribieron y encuentren el patrón que utilizó. Solución a cargo del alumno.

2

capítulo 2 contenido

8

Sucesiones A veces, para convencernos de que tiene sentido estudiar un objeto matemático, conviene empezar por ver dónde aparece. ¿Hay ejemplos? ¿Son muchos? ¿Nos interesan esos ejemplos? Anotar algo cada día es un buen ejemplo de sucesión; podemos anotar cuántas horas dormimos, cuánta plata gastamos, con cuánta gente conversamos, qué comimos o el precio de algo que nos queremos comprar. Para ampliar un poco el panorama, observemos el mundo que nos rodea. Cada día podríamos tomar nota de la forma de la Luna, de la temperatura mínima o máxima, del nivel de dióxido de carbono en la atmósfera, etc. O cada año podría interesarnos anotar la cantidad de habitantes de nuestra ciudad, país o del mundo entero. ¿Y para qué nos sirven todas estas anotaciones? Una de las mayores utilidades es ver si podemos encontrar alguna regularidad en esos datos que luego nos podría servir para entender el funcionamiento o predecir cómo será su futuro comportamiento e inclusive intentar modificarlo a largo plazo. Así, por ejemplo, si anotamos el estado de la Luna cada día, podríamos encontrar que se repite cada 28 días. ¡Hecho ya hoy conocido! Así, podemos saber que la Luna tarda 28 días en su giro alrededor de la Tierra, lo que nos permite predecir cuándo habrá luna llena, luna nueva, etc.

Día 1

Día 5

Día 12

Día 15

Día 19

Día 22

Día 25

Día 29

Si en cambio estudiamos las temperaturas mínima y máxima de la ciudad donde viviimos, podríamos ver que se van modificando: son más altas en los tres meses que corresponden a la estación que hoy llamamos verano, y luego empiezan a bajar durante los siguientes tres meses para finalmente bajar más en lo que hoy conocemos como invierno. También podríamos comparar qué pasa año tras año. ¿Se mantiene el ciclo? ¿Van aumentando lentamente? ¿Se hacen más caóticas? Así como la observación de estas sucesiones nos llevó a conocer el ciclo de la Luna o el de las estaciones, tomando nota de otros datos hoy en día podríamos descubrirr nuevos ciclos de la naturaleza o de lo que nos interese conocer. Por ejemplo, podríamos intentar responder preguntas tales como: ¿En qué mes me conviene comprar eso que tanto quiero? ¿Cuántos habitantes tendrá el mundo en el año 2050? ¿Pasará nuevamente las 400 ppm (partes por millón) el nivel diario de dióxido de carbono en la atmósfera?

Actividades 1. ¿Cómo continúa la siguiente sucesión de números? ¿Por qué? 2; 3; 5; 7; 11; 13; 17; 19; 23; 29; 31; 37;...

2. Encuentren algo que les gustaría estudiar: el comportamiento de algo de la naturaleza cada día, la cantidad de goles de un jugador de fútbol, la performance de un tenista, su humor o el de otra persona, la evolución de sus ahorros o gastos, el precio de algo; y tomen una cantidad suficiente de datos. Luego, realicen alguna predicción. ¿Qué tan exactas son ellas? 1. 41; 43; 47;… es la sucesión de números primos. 2. Depende de lo que elijan; la idea es que sea una tarea abierta y que vean si encuentran alguna regularidad y la puedan describir y ver cómo seguiría. Lo importante es saber que estas predicciones no son exactas.

3

¿¿PPaarraaqquuééssirirvvee..?. capítulo 2 contenido

9

Sucesiones aritméticas y geométricas Muchos alguna vez han pensado en cómo pueden invertir su dinero para ganar lo más posible. Si hoy tenemos $10 000, ¿qué hacemos? Una opción muy común es constituir un plazo fijo. Un plazo fijo nos da un interés de 1% mensual (este 1% se calcula en base al capital inicial puesto), suponiendo que ese 1% está fijo, es decir, no varía dependiendo de la cantidad de meses que se ponga ni por la cantidad que se deposite, ¿conviene ponerlo por 30 años o renovarlo todos los meses reinvirtiendo las ganancias? ¿O da lo mismo? Veamos cómo van quedando ambas sucesiones del dinero que puede obtenerse mes a mes en un año: Sin renovar: 10 000; 10 100; 10 200; 10 300; 10 400; 10 500; 10 600; 10 700; 10 800; 10 900; 11 000; 11 100; 11 200. Renovando mes a mes: 10 000; 10 100; 10 201; 10 303,01; 10 406,04; 10 510,10; 10 615,20; 10 721,35; 10 828,57; 10 936,85; 11 046,22; 11 156,68; 11 268,25. En un año la diferencia entre las dos opciones es de $68,25. ¿Valdrá la pena entonces renovarlo cada mes? ¿Qué diferencia habrá a lo largo de 30 años? Como seguir escribiendo cada opción se haría muy largo, ya que 30 años son 360 meses y nos faltarían aún 348 cuentas en cada una, conviene intentar encontrar las fórmulas generales. ¿Cómo son? La opción de no renovar (1) está descripta por 10 000 + 100 . n, donde n es el número de mes. En la renovación mes a mes (opción 2) es más difícil encontrar la fórmula general, ya que se debe aumentar el 1% de lo que tengo el mes anterior. Como para aumentarle a una cantidad el 1% se puede multiplicar por 1,01, iterando este razonamiento queda 10 000 . (1,01)n, donde n es el número de mes. Reemplazando n por 360 (meses que hay en 30 años) en cada fórmula, obtenemos la cantidad total que tendremos luego de 30 años en cada caso. Opción 1: 10 000 + 100 . 360 = 46 000

Opción 2: 10 000 . (1,01)360 = 359 496,41

Claramente nos conviene la opción 2, ya que es casi 8 veces mayor esta cantidad que la otra. Nuevamente la matemática nos sirvió para modelizar un comportamiento de la realidad. En este caso, a través de sucesiones. Una vez establecidos los modelos, pudimos compararlos y responder acerca de qué decisión nos convenía tomar. La opción de no renovar está descripta por una sucesión aritmética y la de renovación mes a mes rein reinvirtiendo las ganancias, está descripta por una sucesión geométrica. En economía, se conoce a la primera opción como interés simple, mientras que a la segunda se la llama interés compuesto. to. Con el análisis que hemos hecho podemos decir que las inversiones con interés compuesto nos pueden dar mucha ganancia a largo plazo, pero este es el momento de citar a Keynes (uno de los economistas más influyentes del siglo XX): “Los períodos largos son una guía engañosa para los temas de actualidad. A largo plazo estamos todos muertos”.

Actividades 1. ¿Qué conclusiones podemos sacar acerca del crecimiento de una sucesión aritmética y de una sucesión geométrica? 2. Investiguen quién fue John Maynard Keynes. 4

1. Si ambas crecen, el crecimiento es más rápido en las sucesiones geométricas que en las aritméticas. Para ver esto podemos pensar, por ejemplo, lo siguiente. Arrancamos del 1 y “nuestro número” es 2, la aritmética en cada paso suma 2, mientras que la geométrica en cada paso multiplica por 2; acá se ve que voy a ir más rápido multiplicando que sumando. 2. Economista británico cuyas ideas tuvieron una fuerte repercusión en las teorías y políticas económicas.

capítulo 3 contenido

13

Números complejos Uno de los pasatiempos favoritos de los matemáticos del siglo XV y XVI fue el de resolver ecuaciones. No conformes con resolver algunas ecuaciones particulares, muchos de los más reconocidos matemáticos intentaban encontrar las fórmulas generales para resolver las ecuaciones de tercer grado[1] y cuarto grado[2]. Ese era uno de los grandes desafíos de la época. Una de las complicaciones que tenían era que para hallar dichas soluciones muchas veces debían calcular raíces cuadradas de números negativos. Por ejemplo, necesitaban números que elevados al cuadrado dieran como resultado –1. Sabemos que este problema no tiene solución en los números reales, ya que todo número real al cuadrado da como resultado un número mayor o igual a 0, es decir, no puede dar –1. Después de todo, ¿por qué tendrían que diferenciarse en cuanto a su conjunto solución las ecuaciones x2 – 1 = 0 y x2 + 1 = 0? Bueno, la primera es equivalente a x2 = 1 y la segunda a x2 = –1. Si todas las ecuaciones de grado 1 —de la forma ax + b = 0 con a ≠ 0— tienen exactamente una solución real, ¿por qué habría ecuaciones de grado 2 —de la forma ax2 + bx + c = 0 con a ≠ 0— con 2 soluciones y otras con 0 solución? Lo más razonable sería que tuvieran todas 2. Para solucionar este problema nace el número imaginario, que más tarde se lo llamaría i, como i2 = –1 entonces, la ecuación x2 + 1 = 0 ahora tendría 2 soluciones i y –i. De este modo, surgieron los números complejos que tienen la forma a + bi donde a y b son números reales. Lo más llamativo es que ahora todas las ecuaciones —de grado 1, grado 2, grado 3,..., grado 80,...— tienen una solución en este nuevo conjunto de números. Es decir, para poder hallar soluciones a este tipo de ecuaciones no es necesario agregar nada... ¡nos alcanza con los números complejos! Además, si pensamos un poco más en lo anterior, la cantidad de soluciones complejas de una ecuación de grado n ¡es exactamente n! —contadas con multiplicidad, ampliaremos esto en los capítulos de función cuadrática y función polinómica—. Este es un ejemplo de cómo surge un nuevo objeto matemático —los números complejos—, cuando intentamos resolver un problema intramatemático —encontrar soluciones de ecuaciones—, que más tarde se usa en otros campos —en este caso, los números complejos se utilizan en física e ingeniería— al hablar, por ejemplo, de circuitos eléctricos, s, potencias, ondas electromagnéticas o mecánica cuántica. Y así la ciencia se retroalimenta: problemas matemáticos dan origen a nuevos objetos y teorías matemáticas que luego se pueden aplicar a otros campos, en los que surgen nuevos problemas que se traducen matemáticamente y hacen que cobren más sentido algunos objetos o teorías matemáticas o simplemente exigen nuevas creaciones, y así todo va evolucionando.

Actividades 1. ¿Por qué la afirmación de que toda ecuación de grado 1 o más tiene solución implica que una ecuación de grado n tiene n soluciones (con multiplicidad)? 2. Hallen algún método para encontrar las soluciones de cualquier ecuación de segundo grado. 1. Si tengo una ecuación de grado n, entonces (utilizando la propiedad mencionada) si n ≥ 1 tiene una solución a1, divido mi ecuación por x – a1 y me queda (x – a1) . (ec. de grado n – 1). Si n – 1 ≥ 1, entonces (utilizando la propiedad mencionada) tiene una solución a2. Divido mi ecuación por x – a2 y me queda (x – a1) . (x – a2) . (ec. de grado n – 2)... Siguiendo con este proceso se obtienen las n soluciones, eventualmente algunas repetidas (por eso contadas con multiplicidad). 2. Hay varios, si la ecuación es de la forma –b c __ ax2 + bx + c = 0, uno puede ser el de encontrar dos números que sumados den ___ a y multiplicados a . [1] [2]

ax + bx + cx + d = 0 con a ≠ 0 ax4 + bx3 + cx2 + dx + e = 0 con a ≠ 0 3

2

5

¿¿PPaarraaqquuééssirirvvee..?. capítulo 4 contenido

21

Cónicas Hay objetos matemáticos que nacen para dar respuestas a problemas de la vida cotidiana o problemas de otras ciencias. Otros objetos matemáticos simplemente nacen por el interés que algunos matemáticos tienen en estudiarlos o, eventualmente, para poder arribar a soluciones de problemas intramatemáticos. Lo curioso es que muchos de estos últimos pueden resurgir varios siglos después, dando respuestas a preguntas de otras ciencias. Es el caso de las cónicas. Las cónicas empezaron a ser estudiadas en el siglo III a. C., un poco por diversión y otro poco para intentar resolver los tres famosos problemas griegos: la duplicación del cubo, la trisección del ángulo y la cuadratura del círculo. Y reaparecieron en escena en el siglo XVII cuando Johannes Kepler, en su rol de “Matemático imperial de Rodolfo II”, se puso ycho a estudiar la órbita de los planetas. Para esto utilizó los datos que había conseguido Tycho Brahe, quien había obtenido medidas muy precisas de las posiciones de los planetas y de las estrellas. Kepler intentó encontrar un modelo que se adaptara a esos datos: empezó con la idea de ver qué circunferencias se ajustaban a la descripción de estas órbitas, pero luego se convenció de que los datos experimentales no encajaban con esta figura y fue en búsqueda de otras figuras que las describieran. Así, encontró que los planetas describían órbitas elípticas que tenían al Sol como uno de los dos focos[3], resultado que se conoce como la Primera Ley de Kepler. También, analizando estos datos, descubrió que la velocidad de los planetas no es constante, sino que se mueven más rápidamente cuando están más cerca del Sol que cuando están más alejados: este hecho se conoce como la Segunda Ley de Kepler. Si bien la órbita de los planetas tiene forma de elipse, esta elipse se acerca bastante a una circunferencia. No es así el caso de la órbita de los satélites o cometas, que es elíptica, pero está lejos de ser una circunferencia. La astronomía es una ciencia que ha atrapado a muchos científicos de todos los tiempos. Sin duda, el conocimiento del Universo, ese espacio gigante y desconocido, nos llena a todos de curiosidad. Muchos de los astrónomos de todos los siglos han sido además grandes matemáticos, como es el caso de Kepler. La matemática ha sido, como en este caso, la herramienta para describir y entender cuestiones astronómicas. También la astronomía ha ayudado a darle más sentido a algunos objetos matemáticos.

Actividades 1. Investiguen quién fue Rodolfo II. 2. ¿Cómo les parece que podemos medir qué tan cerca está una elipse de ser una circunferencia o qué tan redondeada o deforme es? 3. Cuando los aviones se desplazan a una velocidad mayor que la del sonido generan una onda de choque que tiene forma de cono, así que al intersecar la tierra (plana localmente) se forma una cónica. Este hecho se conoce como “curva del estampido supersónico”. Investiguen cuál es la cónica que se forma.

6

1. Rodolfo II fue emperador del Sacro Imperio Romano Germánico desde 1576 hasta su muerte. Le interesaban la astrología, la magia y los juguetes mecánicos, especialmente autómatas, relojes y máquinas de “movimiento perpetuo”. Muchas de las grandes maravillas de la ciudad de Praga se construyeron durante su reinado. 2. Lo medimos haciendo el cociente entre la semidistancia focal y su semieje mayor (se llama excentricidad). 3. Se forma una hipérbola. [3]

En la página 78 del libro encontrarán información sobre las características de la elipse.

capítulo 5 contenido

25

Función inversa ¿Todos los procesos se pueden deshacer? Claramente la respuesta a esta pregunta es un no rotundo. En principio, el paso del tiempo es algo que no podemos deshacer: ni las cremas antiage, ni los tratamientos estéticos ni todas las operaciones del mundo podrían revertir este hecho. Pero no es necesario pensar tan filosóficamente para darnos cuenta de que hay muchass cosas que no podemos deshacer. Una vez que pelamos una manzana, no podemos volver a ponerle la cáscara; si mezclamos tempera de dos colores, tampoco podemos reevertir este proceso. Sin embargo, hay otros procesos que sí se pueden deshacer, como por ejemplo si ponemos agua durante un tiempo en el congelador, se obtiene hielo; y luego, sacando ese hielo del congelador y esperando un rato, se vuelve a obtener el agua en estado líquido. A las funciones podemos pensarlas como procesos, y como hemos dicho, no todos los procesos pueden deshacerse. Por ejemplo, si una función toma siempre el valor 1, es decir, f(x) = 1 para todo x D , independientemente del número al que se la aplique, entonces no se puede deshacer este proceso pues, ¿adónde iría el 1? Sabemos que tiene que ir un único valor, pero ¿cuál elegir? Si pensamos que f(2) = 1, para deshacer el proceso tendríamos que mandar el 1 al 2; pero si pensamos que f(3) = 1, para deshacer el proceso tendríamos que mandar el 1 al 3, y no podemos mandarlo a 2 y a 3 a la vez, con lo cual esta función no se puede deshacer. Es diferente el caso de la función que a todo número le suma 5 pues, ¿qué sería deshacer este proceso? Efectivamente, a todo número restarle 5. Si hemos sumado 5, al restar 5, deshacemos el proceso. Otra función que también se puede deshacer es la función que a cada persona le asigna su número de DNI. Deshacerlo sería asignarle a cada número de DNI —considerando solo los números válidos— esa única persona. Para que un proceso se pueda deshacer, necesitamos que la función tenga ciertas características. En el caso de las personas y los DNI, cada persona tiene un DNI y cada número de DNI válido le corresponde a una persona. Este poder ir y volver de forma unívoca es lo que nos permite deshacerlo. Por ejemplo, Martín Pérez —hablamos de personas que son únicas y no de nombres que sí se pueden repetir— tiene DNI 43678130 y nadie más en toda la Argentina tendrá este DNI. Entonces si preguntamos quién tiene DNI 43678130, la única persona es Martín Pérez. Por otro lado, todos los números de DNI válidos corresponden a alguna persona. Esto es lo que hemos llamado inyectividad y sobreyectividad de una función y acá vemos cómo es necesario entonces pedir que la función sea biyectiva: para poder obtener este proceso que lo deshace, llamado en este caso función inversa.

Actividades 1. ¿Qué pasaría si en el ejemplo de los DNI consideramos todos los números naturales en vez de solo los números de DNI válidos?

2. Describan simbólicamente las funciones sumar 5 y restar 5 y comprueben que efectivamente son funciones inversas. 3. Las funciones multiplicar por 8 y elevar al cuadrado ¿tienen sus funciones inversas? 1. No se podría definir el proceso inverso pues habría números que no identificarían a ninguna persona. 2. Sumar 5: f(x) = x + 5; restar 5: g(x) = x – 5. f: → /y = x + 5 ⇒ y – 5 = x, cambiando de variable queda f–1: → /y = x – 5. g(x) = f–1(x). 3. La función inversa a multiplicar por 8 es dividir por 8. En cuanto a elevar al cuadrado en principio no tiene inversa, pues 2 y –2 van ambos a 4, pero si la consideramos solo con dominio en los números no negativos entonces es biyectiva y su inversa es sacar raíz cuadrada.

7

¿¿PPaarraaqquuééssirirvvee..?. capítulo 5 contenido

28

Funciones cuadráticas Las funciones cuadráticas describen fenómenos que van desde la física y la matemática hasta la medicina y la arquitectura. Un ejemplo intramatemático en el que aparecen es en el intento de buscar dos números que cumplan ciertas relaciones cuando se los suma y se los multiplica, por ejemplo, que su suma sea 10 y su producto sea 20,16. Las relaciones cuadráticas aparecen en química cuando se describe la disociación de un cierto ácido y en biología, por ejemplo, cuando se estudian los efectos nutricionales de ciertos organismos. Ahora describiremos un ejemplo que, si bien corresponde a la física, todos lo hemos experimentado y tiene que ver con nuestros pasos. Imagínense caminando. Una posibilidad es que, por ejemplo, mantengamos siempre el mismo ritmo y caminemos una cuadra por minuto. En este caso, si pasaron 15 minutos, habremos recorrido 15 cuadras; si pasaron 20, recorrimos 20 cuadras y si pasaron 30 minutos, hicimos 30 cuadras. Así, se determina una relación lineal c = t, donde t es la cantidad de minutos que pasan y c, la cantidad de cuadras que se recorren. ¿Qué sucede si en vez de tener un ritmo de caminata constante vamos acelerando? Por ejemplo, durante nuestra primera hora de caminata recorremos 16 cuadras y durante la segunda hora, 24. Supongamos, como en el ejemplo anterior, que el término constante es cero. ¿Qué ecuación describe ahora la cantidad de cuadras recorridas en cierto tiempo (t) —medido en horas—? Haciendo las cuentas obtenemos que c = 4t2 + 12t modeliza lo que queremos. Si reemplazamos t por 1, nos queda c = 16; y si reemplazamos t por 2, nos queda c = 40, lo que también es correcto pues en la primera hora se recorren 16 cuadras y en la segunda, 24, entonces 16 + 24 = 40, la cantidad de cuadras que se recorren en dos horas. Si ahora tomamos t = 3, nos queda que c = 72, lo que indica que en la tercera hora se recorren 32 cuadras (72 – 40 = 32); es decir, seguimos aumentando la velocidad. Por otro lado, si analizamos bien estos números —16, 24 y 32—, cada hora recorremos 8 cuadras más que la hora anterior; esto significa que aumenta en 8 las cuadras recorridas. Por lo tanto, si bien el ritmo de caminata no es constante, sí lo es la aceleración que justamente en este caso es 8. Si la aceleración no fuera constante, no se podría modelizar la situación con una función cuadrática.

Actividades 1. Resuelvan el primer problema presentado (ejemplo intramatemático). 2. Si c = t2 modeliza una situación de movimiento a través del tiempo, ¿cuál es la aceleración? 3. La caminata de Alexis se describe mediante c = 5t2 + 10t. ¿Qué aceleración tiene Alexis? 4. Establezcan alguna relación entre la ecuación cuadrática y la aceleración. 1. Los números son 2,8 y 7,2. 2. La aceleración es 2. 3. La aceleración de Alexis es 10. 4. Siendo a el coeficiente principal de la función cuadrática, la aceleración es siempre igual a 2a.

8

capítulo 6 contenido

32

Teorema de Bolzano El teorema de Bolzano tiene múltiples aplicaciones. Veamos qué nos dice Bolzano de una manera práctica. Las ciudades suelen tener algunas calles importantes que indican el cambio de nombre de sus calles transversales de un lado y otro. Bueno, lo que nos dice Bolzano no es otra cosa que: si estábamos de un lado de esa calle y luego estamos del otro lado, en algún momento cruzamos esa calle —considerando que no salimos de la ciudad y entramos por otro lado, no volamos ni otra cuestión por el estilo—. Con este hecho aplicado a los polinomios se obtiene un resultado muy interesante. Tomemos cualquier polinomio de grado impar y supongamos que el coeficiente principal es positivo —si es negativo sale de forma similar— evaluando el polinomio en un número positivo muy grande, nos dará mayor a 0. Si, en cambio, evaluamos el polinomio en un número negativo muy chico, nos dará menor a 0. Con lo cual, utilizando el teorema de Bolzano[4] podemos concluir que hay un número real donde el polinomio vale 0. Esto es útil pues nos garantiza que tenemos una raíz real del polinomio, cosa que ni siquiera es cierta para el polinomio x2 + 1, porque sus dos raíces son complejas; y, en general, si uno está resolviendo ecuaciones, quiere soluciones reales. Volviendo al caso de las calles, el teorema de Bolzano nos permite deducir que, para ir de una calle determinada a otra, hay que cruzar todas las que están en el medio. En términos matemáticos, si una función continua vale –2 en un punto y 8 en otro, entonces además de que en algún momento toma el valor 0, también toma todos los valores intermedios entre –2 y 8. Este hecho se conoce como teorema de los valores intermedios. Otro famoso resultado que también se deduce de utilizar el teorema de Bolzano es el siguiente. Si tenemos una función continua en el intervalo [a;b], tal que la imagen de esta función cae en el intervalo (a;b), entonces existe un punto x en [a;b] tal que f(x) vale exactamente x; es decir, no hay chances de que todos los puntos “sean movidos” por esa función f. Este hecho se conoce como teorema del punto fijo y es válido también en otros contextos y dimensiones. Se comenta que Brouwer, autor material de este teorema, se inspiró al observar su taza de café y dijo: “Cuando revolvemos el azúcar, parece siempre haber un punto inmóvil”. De esta forma dedujo que en todo momento, hay un punto de la superficie que no ha cambiado de lugar. No es del todo exacto que uno lo pueda ver mientras revuelve su taza de café, pero ¡pueden intentarlo! Obviamente, Brouwer luego lo demostró con toda rigurosidad matemática. También se comenta que otros matemáticos habían conseguido con anterioridad demostrar este resultado. De interpretar el teorema del punto fijo en otros contextos y dimensiones se puede concluir, por ejemplo, que no nos podemos peinar sin que aparezca un remolino —de hecho, este resultado se conoce como el teorema de la bola peluda—. Otra conclusión es que en todo momento debe haber al menos un punto del planeta donde no hay viento, que solo es un corolario del famoso teorema anterior.

Actividades 1. Enuncien y demuestren el teorema de los valores intermedios. 2. Enuncien y demuestren el teorema del punto fijo. 1. Sea f: [a;b] → continua y f(a) < f(b) ⇒ ∀ m ∈ [f(a);f(b)] ∃ x ∈ (a;b)/f(x) = m. Sale de utilizar el teorema de Bolzano en [a;b] para g(x) = f(x) – m para cada m en la imagen de f. (recordar que f(a) < m < f(b)). 2. Sea f: [a;b] → [a;b] continua ⇒ ∃ x ∈ (a;b)/f(x) = x. Sale de utilizar el teorema de Bolzano en [a;b] para g(x) = f(x) – x. Recordar que la imagen de f „ (a;b), es decir f(a) > a y f(b) < b. [4]

Podemos utilizarlo pues los polinomios son funciones continuas.

9

¿¿PPaarraaqquuééssirirvvee..?. capítulo 6 contenido

33

Polinomios Los polinomios son expresiones que involucran una o más variables y que se construyen utilizando únicamente sumas y multiplicaciones. Esto hace que sean expresiones “simples”, pues involucran operaciones que todos podemos hacer. Por eso, muchas veces se intenta que las descripciones de los fenómenos que estamos estudiando se modelicen a través de polinomios, ya que de esta forma resultan más sencillas de estudiar. Inclusive muchas veces se intenta q que estos polinomios sean de grado 1, 2 o 3 para que el análisis sea aún más efectivo. Los polinomios aparecen, por ejemplo, en el cálculo de alineación de antenas elec-tromagnéticas, para modelizar cómo se propaga cierta enfermedad o para estudiar el comportamiento de algún mercado. Un ejemplo muy interesante aparece en matemática financiera cuando se quiere estudiar las variaciones de las sumas de dinero a través del tiempo —lo que se conoce como ecuaciones de valor—. Por ejemplo, se puede determinar cuánto val-drá dentro de un cierto tiempo una suma de dinero de la que hoy se dispone. En estos ejemplos aparece lo que se conoce como la tasa mensual de interés (i); así, si hoy tenemos $1 000, el mes que viene tendremos $1 000 . (1 + i), y si consideramos que ese dinero nos vuelve a dar sus intereses —lo que se denomina interés compuesto—, dentro de dos meses tendremos $1 000 . (1 + i)2 y dentro de k meses, $1 000 . (1 + i)k. Supongamos que Patricio tenía que pagar $100 000 en 3 meses, $150 000 en 6 meses y $200 000 dentro de un año. Como alternativa, le propone al acreedor pagarle $472 000 en 5 meses. ¿Cuál sería en este caso la tasa de interés mensual que rendiría la deuda? La ecuación que modeliza esta situación está dada por: 100 000 . (1 + i)3 + 150 000 . (1 + i)6 + 200 000 . (1 + i)12 = 472 000 . (1 + i)5 —en el miembro izquierdo está la deuda que tenía y en el derecho cómo efectivamente ofrece pagarla—. La ecuación anterior es equivalente a: 100 000 . (1 + i)3 + 150 000 . (1 + i)6 + 200 000 . (1 + i)12 – 472 000 . (1 + i)5 = 0 Reemplazando 1 + i por x nos queda P(x) = 100 000x3 + 150 000x6 + 200 000x12 – 472 000x5 = 0; es decir, que el problema se resuelve encontrando una raíz de un polinomio de grado 12. Para encontrar esta raíz hay que tener en cuenta que el interés mensual siempre está en ciertos rangos de valores. En este ejemplo nos queda que una raíz aproximada es x = 1,01548 y como x = 1 + i nos queda 1,01548 = 1 + i lo que nos da un interés i = 0,01548 que implica un interés mensual del 1,548% y se encuentra en un rango razonable.

Actividades 1. Expliquen por qué en el mes 2 tenemos $1 000 . (1 + i)2 y en el mes k tendremos $1 000 . (1 + i)k. 2. Verifiquen que evaluando el siguiente polinomio en x = 1,01548 efectivamente nos queda un valor muy cercano a 0. 100 000x3 + 150 000x6 + 200 000x12 – 472 000x5 1. Si el interés es i entonces en el primer mes tendremos $1 000 + $1 000 . i = $1 000 . (1 + i). Es decir, cuando pasa un mes obtenemos (1 + i) veces el dinero que teníamos. Así entonces en el segundo mes vamos a tener (1 + i) veces el dinero que teníamos en el mes 1, es decir, (1 + i) . $1 000 . (1 + i) = $1 000 . (1 + i)2 y así en el mes 3 tendremos (1 + i) veces lo que tenía en el mes 2, es decir (1 + i) . $1 000 . (1 + i)2 = $1 000 . (1 + i)3. Siguiendo con este razonamiento tendremos en el mes k: $1 000 . (1 + i)k. 2. Solución a cargo del alumno. 10

capítulo 6 contenido

36

Funciones homográficas Pensemos en una bolsa llena de aire. Si ejercemos presión sobre esta, el aire ocupará menos lugar, disminuirá su volumen, con lo cual podríamos decir, a grandes rasgos, que “a mayor presión, menor volumen”. También podríamos pensarlo al revés: si un gas está “guardado” en un recipiente grande, al moverse las partículas no se chocarán tanto como si el gas estuviese guardado en una cajita chiquita, en cuyo caso se chocarían mucho y terminaría ejerciendo más presión. Entonces, podríamos sostener que “a menor volumen, mayor presión”. Ahora bien, ¿estas conclusiones se contradicen? Justamente, allá por el siglo XVII, Robert Boyle se encontraba estudiando estas relaciones. Al hacer algunos experimentos y anotar las mediciones obtenidas, descubrió que si la temperatura se mantenía constante, entonces Presión . Volumen = k, donde k es una constante. k En otras palabras, podemos decir que Volumen = _______ , lo cual nos está diciendo que, Presión si pensamos el volumen (V) de un gas en función a la presión (P), obtenemos que esta relación está dada por una función homográfica k V = f(P) = __ . Así, por ejemplo, si la presión se aumenta P al doble, el volumen se reduce a la mitad. Esta ecuación y se conoce como la Ley de Boyle-Mariotte y es tan solo un 7 ejemplo en donde aparecen estas funciones. Otro ejemplo traído de la economía es el modelo creado por William Phillips, llamado precisamente la curva de Phillips que modeliza la relación entre el desempleo y la inflación. En 1958, Phillips publicó un trabajo en donde analiza esta relación utilizando datos de Inglaterra durante el período 1861-1957. En ese entonces, pudo observar que esta es una relación inversa: cuando el desempleo es muy bajo, la inflación es alta, en cambio, cuando el desempleo es muy alto, la inflación es baja. Esta teoría ha tenido algunas modificaciones.

Tasa de inflación

6

5 La curva de Phillips

4 3 2 1 0 1

2

3 4 5 6 Tasa de desempleo

7

x

Obviamente esto es un modelo teórico simplificado que usó Phillips para intentar explicar algunas de las complejas relaciones económicas que se dan en la sociedad. Además, para obtener más precisiones al respecto se deberían tener en cuenta otras variables. De todos modos, para convencerse de por qué este hecho suena razonable, podemos pensar que, cuando baja el desempleo, los trabajadores tienen más ofertas de empleos para elegir, entonces los empleadores, para atraer y mantener a sus empleados, aumentan los salarios. Esto aumenta el poder adquisitivo de las personas y genera un aumento en la demanda de productos; los empleadores contratan más gente para satisfacer esa mayor demanda, y el ciclo se sigue repitiendo. En algún momento de esta repetición esa mayor demanda llevará a un aumento de precios. Una vez más la matemática ha servido para modelizar y describir fenómenos. ¡Busquen más ejemplos!

Actividades 1. Cuando respiramos, se produce la Ley de Boyle-Mariotte, ¿cómo les parece que sucede? 2. Supongamos que en el lejano país de Kumnchenkin la curva de Phillips está descripta por la siguiente función que relaciona el porcentaje de inflación en función de la tasa de desempleo. 9 y = __31 . x + __x . ¿Qué sucede si el desempleo es nulo? ¿Cómo interpretamos en términos económicos la asíntota horizontal de esta función? 1. Cuando inhalamos, el diafragma baja y así el volumen de los pulmones aumenta; cuando exhalamos, el diafragma sube, generando más presión y entonces el volumen de los pulmones disminuye. 2. Si el desempleo es nulo, tenemos un 9% de inflación. La asíntota se puede interpretar como que independientemente de la tasa de desempleo siempre voy a tener un porcentaje positivo de inflación 1 —. —en este caso la A. H. es y = __ 3

11

¿¿PPaarraaqquuééssirirvvee..?. capítulo 7 contenido

39

Funciones exponenciales y logarítmicas Las funciones nos permiten modelizar fenómenos. Muchos de estos se pueden describir utilizando funciones exponenciales y logarítmicas. Veamos algunos ejemplos. La radiactividad es un fenómeno físico que ocurre en el núcleo de elementos inestables capaces de transformarse espontáneamente en núcleos de elementos más estables produciendo radiación. Este fenómeno se aprovecha para la obtención de energía nuclear, se usa en medicina y en aplicaciones industriales. En general, las sustancias que no tienen un balan-ce correcto entre protones y neutrones son radiactivas. Resulta que la desintegración radiactiva se puede modelizar con la función n(t) = n0 . e–h.t donde n(t) es la cantidad de radionucleidos en un instante de tiempo t, n0 es la cantidad existente en el instante t = 0 y h es la llamada constante de desintegración radiactiva y dependerá de cada elemento. Conociendo esta función se puede analizar y predecir el comportamiento radiactivo de las diferentes sustancias, así como su velocidad de desintegración. Por otro lado, varios de nuestros sentidos tienen que ver con las funciones logarítmicas. Un ejemplo conocido es el llamado ley de Weber-Fechner, que establece la relación entre la magnitud de un estímulo físico (S) y la forma en que lo percibimos (P). Weber y Fechner propusieron que la relación S está dada por P = k . ln __ S0 donde S0 es el nivel de estímulo en el que por debajo suyo no se percibe sensación y k es una constante que dependerá de cada caso.

( )

Esta relación nos dice varias cosas. Una que seguramente todos hemos experimentado es cuando el estímulo físico es el peso. Por ejemplo, es difícil que podamos percibir si algo pesa 100 o 102 kilos, sin embargo es probable que podamos percibir algún cambio entre algo que pesa 100 y algo que pesa 110 kilos. Pero esta misma relación nos dice que probablemente no podamos percibir una diferencia entre algo que pesa 1 000 y algo que pesa 1 010 kilos, pues justamente la relación no es lineal. Lo que si nos garantiza esta relación es que los cambios exponenciales en la magnitud del estímulo los podremos percibir linealmente.

Actividades 1. Investiguen qué otros fenómenos se pueden modelizar utilizando funciones exponenciales. 2. Investiguen qué otros fenómenos se pueden modelizar utilizando funciones logarítmicas. 3. Reúnanse con un compañero y realicen la siguiente experiencia. -Uno de los dos debe tener los ojos vendados y sostener alguna cosa con determinado peso; el otro irá agregándole, de a una, cosas de igual peso. El que no puede ver, debe avisar cuando percibe un cambio en el peso y se anota el peso de inicio y el final. -Por ejemplo, pueden empezar con algo que pese unos 100 g y se van agregando otras cosas de unos 10 g hasta que el que tiene los ojos vendados perciba un cambio de peso. -Repitan varias veces la experiencia empezando con cosas de distintos pesos. ¿Cómo pueden relacionar los resultados obtenidos con la ley de Weber-Fechner?

12

1. Hay muchos; por ejemplo, el interés compuesto o el crecimiento de algunas poblaciones. 2. Hay muchos, por ejemplo ph para medir acidez y alcalinidad o la escala musical. 3. Según la ley de Weber-Fechner se debería percibir el cambio de peso en relación con el peso inicial; así, por ejemplo, se percibe cambio entre 100 y 110 gramos, entre 1 y 1,1 kilos y entre 10 y 11 kilos.

capítulo 8 contenido

47

Trigonometría Así como el teorema de Pitágoras establece una relación entre los lados de un triángulo rectángulo que nos permite conocer la medida del tercer lado una vez que conozco la medida de los otros dos, los teoremas del seno y del coseno nos permiten conocer la medida de un ángulo o un lado conociendo otros datos. Por ejemplo, el teorema del coseno, que es una generalización del teorema de Pitágoras para cualquier triángulo, establece la siguiente relación: C2 = A2 + B2 – 2 . A . B . cos ^ a donde A, B y C son las medidas de los lados de un triángulo y ^ a es el ángulo formado por los lados de medidas A y B. Esta relación nos permite, por ejemplo, si conocemos la medida de dos lados y un ángulo de un triángulo, encontrar la medida del tercer lado. El caso del teorema del seno, por otra parte, establece una relación entre ángulos y lados que nos permite, por ejemplo, si conocemos la medida de dos ángulos y un lado de un triángulo, encontrar la medida de los otros dos lados. Como vemos, la trigonometría nos sirve para calcular distancias. Uno de los principales logros en este sentido es el que le permitió a los cartógrafos realizar mapas de los diferentes países. Un proyecto de los más conocidos fue el denominado Gran Planimetría Trigonométrica allá por el siglo XIX y que, entre otras cosas, cuenta la leyenda que permitió descubrir el punto más alto sobre la Tierra en 1852, en ese momento denominado Pico XV y luego, en 1856, llamado Everest, como hoy lo conocemos, en honor a uno de los inspectores de este proyecto. La trigonometría, además de ser utilizada en áreas como la astronomía, la cartografía y la arquitectura, principalmente para “medir cosas”, también se aplica en otras disciplinas como la economía, la meteorología y la biología para estudiar objetos que tienen comportamientos periódicos. También los oceanógrafos y los músicos hacen uso de ella.

Actividades 1. ¿Cómo les parece que podemos deducir el teorema de Pitágoras utilizando el teorema del coseno? 2. ¿Podríamos utilizar estos teoremas para calcular medidas de lados de otros polígonos? ¿Cómo? / 1. Si en el teorema del coseno reemplazamos gama por __ 2 (90°) ^ entonces cos a = 0 nos queda el enunciado del teorema de Pitágoras para triángulos rectángulos. 2. Cualquier polígono se puede dividir en triángulos, entonces conociendo las medidas en los triángulos en el que queda dividido, puedo conocer todas las medidas del polígono en cuestión.

13

¿¿PPaarraaqquuééssirirvvee..?. capítulo 9 contenido

49

Estadística La estadística no solo se utiliza para estudiar las características de una población en particular o para anticipar el resultado de una elección. Se usa también, por ejemplo, para analizar la competitividad de una empresa, para estudiar cuáles son los factores que nos predisponen a cierta enfermedad, para predecir el clima y para analizar patrones en el tránsito. Otra de las aplicaciones de la estadística es la criptografía. Desde tiempos remotos ha sido de gran interés poder transmitir un mensaje sin que una tercera persona pueda interceptarlo y descubrirlo. Para esto, la idea es encriptar un mensaje y mandarlo de modo tal que el destinatario pueda desencriptarlo, aunque no así un posible interceptor. Hoy en día es de gran interés el estudio de esta rama de la matemática, pues de ella depende el tráfico seguro de información en la red, como por ejemplo, claves, números de tarjetas de crédito, etc. Un método muy simple de encriptar un mensaje es sustituyendo una letra por otra o por un nuevo símbolo. Por ejemplo, si sustituimos cada letra por la siguiente en el abecedario, la palabra “estadística” pasa a ser “ftvbejtvjdb” y si cambiamos la a por la e y dejamos igual el resto, nos queda “astedistice”. A este método se lo conoce como método de sustitución. Para poder descifrar textos encriptados con el método de sustitución, se puede recurrir al análisis estadístico de frecuencias de las diferentes letras del abecedario. Por ejemplo, el análisis de frecuencia del idioma español –que obviamente es diferente al de otros idiomas— está dado por la siguiente tabla.

Letra

%

Al tener esta información estadística, descifrar textos encriptados con el método de sustitución descripto anteriormente es bastante accesible. Hoy en día, los algoritmos de encriptación son bastante más complejos, como por ejemplo, el algoritmo RSA que debe su nombre a sus tres creadores de la Universidad de Standford: Rivest, Shamir y Adleman.

Letra

%

Letra

a

12,53

j

0,44

r

6,87

b

1,42

k

0,01

s

7,98

c

4,68

l

4,97

t

4,63

d

5,86

m

3,15

u

3,93

e

13,68

n

6,71

v

0,90

f

0,69

ñ

0,31

w

0,02

g

1,01

o

8,68

x

0,22

h

0,70

p

2,51

y

0,90

i

6,25

q

0,88

z

0,52

Actividades 1. ¿Cuáles son las seis letras más frecuentes en los textos en español? 2. Desencripten la siguiente frase sabiendo que se ha encriptado con un algoritmo de sustitución. “Pzjruzj hj rxzgxz ei ewejzix rjh dxtjs auw puwdj whrjhtzxz wh ruxoauiwz exhuxo, yx auw wo gzxh vxojz dw ox wdurxrijh hj rjhsistw wh xtibjzzxzsw dw dxtjs, sihj wh pzwpxzxz xo rwzwbzj x pwhsxz pjz su pzjpix ruwhtx y xsi oowgxz x rjhjrwz xogj auw hj figuzw wh ojs oibzjs.” Xobwzt Wihstwih

3. ¿Cómo hicieron para desencriptar el mensaje anterior? ¿La frecuencia de las letras era exactamente la descripta anteriormente? ¿Por qué les parece que pasa esto?

14

%

1. A-E-O-N-R-S. 2. “Procuro no cargar mi memoria con datos que puedo encontrar en cualquier manual, ya que el gran valor de la educación no consiste en atiborrarse de datos, sino en preparar al cerebro a pensar por su propia cuenta y así llegar a conocer algo que no figure en los libros.” Albert Einstein 3. No es exactamente la misma frecuencia. En general, en los análisis estadísticos se necesitan textos más largos para que la proporción se dé con mayor exactitud.

capítulo 9 contenido

53

Combinatoria La combinatoria es una rama de la matemática que se utiliza no solo en el estudio de probabilidades, sino también en el estudio de las formas de los espacios y en problemas de optimización. A continuación, les presentamos un ejemplo de la vida cotidiana. Juan fue al negocio de Lucio a hacer unas compras. Gastó $38,20 y pagó con $40. Lucio, que quiere gastar la menor cantidad posible de monedas, le dio de vuelto una moneda de $1, una de $0,50, una de $0,25 y una de $0,05. Al otro día Juan volvió al negocio de Lucio, gastó $23,35 y pagó con $25. Lucio le devolvió una moneda de $1, una de $0,50, una de $0,10 y una de $0,05, siempre teniendo en cuenta que quiere gastar la menor cantidad de monedas posible. Lucio utiliza un método que le permite siempre dar el vuelto a sus clientes según una premisa: gastar la menor cantidad de monedas posible. Primero, se deshace de las monedas más grandes —todas las que pueda— y va bajando ordenadamente a las más chicas. Por ejemplo, para el primer vuelto que le dio a Juan, empezó con las monedas de $1 peso (solo “entró” una); luego, siguió con las de $0,50 (una también), con las de $0,25 (una), con lass de $0,10 (ninguna) y, finalmente, con las de $0,05 (una). Un día, Lucio le contó a Juan el método que utilizaba y este le dijo que ese método no sirve en todos los contextos y le dio la siguiente explicación: “Si en vez de contar con monedas de $1, $0,50, $0,25, $0,10, $0,05 y $0,01, contáramos con monedas de $1, $0,50, $0,25, $0,12, $0,05 y $0,01, ¿cómo haríamos para dar $1,86 de vuelto? Con tu método tendríamos que dar una moneda de $1, una de $0,50, una de $0,25, dos de $0,05 y una de $0,01, o sea, 6 monedas. Sin embargo, se podría optar por dar de vuelto una moneda de $1, una de $0,50 y tres de $0,12, o sea, 5 monedas.” El método o algoritmo de Lucio es un algoritmo denominado goloso o voraz, ya que consiste en elegir la opción óptima en cada caso, a la espera de que esto resulte en una opción óptima en general. Si bien este algoritmo funciona en el caso de nuestro país y muchos otros que usan monedas de $1, $0,50, $0,25, $0,10, $0,05 y $0,01 —pueden probar por qué— no serviría en el caso de que se reemplazaran, por ejemplo, las monedas de $0,10 por las de $0,12. Para estos casos habrá que buscar otros tipos de algoritmos y encontrar la opción óptima.

Actividades 1. ¿De cuántas formas diferentes puede darle Lucio el primero de los vueltos a Juan, sin utilizar su método? 2. Tengan en cuenta el ejemplo propuesto por Juan y encuentren otros vueltos en donde el método de Lucio tampoco sirva. 3. Inventen otro ejemplo distinto al de Juan donde sea posible dar el vuelto, pero con el método de Lucio, no. 1. Hay varias opciones; por ejemplo, tres monedas de $0,50 y tres monedas de $0,10. 2. Hay muchas opciones; por ejemplo, para un vuelto de $0,48. 3. Hay muchas opciones; por ejemplo, en un país con monedas de $0,50, $0,30, $0,12 y $0,05 y un vuelto de 86 centavos. 15

¿PARA QUÉ

SIRVE?

FIN

View more...

Comments

Copyright ©2017 KUPDF Inc.
SUPPORT KUPDF